You are on page 1of 441

 

 
 
 
 
 
 

SMLE  Questions  
 
July  2017  -­‐  October  12,  2017  
 
 
 
 
 
 
 
 
 
 
 
 
 
 
 
 
 
 
 
 
 
 
 
 
 
 
 
 
 
Medicine                                                                                                                                                                                                                                                
Infectious  Diseases                                                                                                                                                                                                        3  
Allergy  and  immunology                                                                                                                                                                                        26  
Endocrinology                                                                                                                                                                                                                      28  
Pulmonology                                                                                                                                                                                                                          40  
Cardiology                                                                                                                                                                                                                                  47  
Rheumatology                                                                                                                                                                                                                      58  
Gastroenterology                                                                                                                                                                                                              62  
Hematology-­Oncology                                                                                                                                                                                              75  
Nephrology                                                                                                                                                                                                                                89  
Neurology                                                                                                                                                                                                                                    94  

Pediatrics                                                                                                                                                                                                                                            106  

Family  Medicine                                                                                                                                                                                                                        168  

Research                                                                                                                                                                                                                                              190  

Emergency  Medicine                                                                                                                                                                                                          197  

OB/GYN                                                                                                                                                                                                                                                215  

General  Surgery                                                                                                                                                                                                                        257  

Orthopedics                                                                                                                                                                                                                                      287  

ENT                                                                                                                                                                                                                                                              309  

Anesthesia                                                                                                                                                                                                                                        318  

Dermatology                                                                                                                                                                                                                                    322  

Ophthalmology                                                                                                                                                                                                                            336  

Psychiatry                                                                                                                                                                                                                                            356  

Basic  Science                                                                                                                                                                                                                                378  


Genetics                                                                                                                                                                                                                                      435  
 
Medicine    

Infectious  Diseases  
   
1.   Trichomaniasis  diseases  what  is  the  treatment?  
Answer:  metronidazole  
   
      2.    Shigella,  treatment?  
Answer:  Ampicillin  
MedScape:  Ampicillin  and  TMP-­‐SMZ  are  effective  for  susceptible  strains;  amoxicillin  is  less  effective  
than  this  because  of  its  rapid  absorption  high  in  the  GI  tract.  
First  aid:  TMP-­‐SMX  to  decrease  person-­‐person  spread.  
   
      3.    Salmonella,  treatment  ?  
Answer:  azithromycin,  cefriaxone,  ciprofloxacin  (Medscape)  
First  aid:  oral  quinolones  or  TMP-­‐SMX  
   
      4.    Enterococcus  fragilis,  treatment?  
                Answer:  clindamycin  
Medscape:  
Penicillin  G  (Pfizerpen),  Cefoxitin  (Mefoxin),  Cefotetan,  Clindamycin  (Cleocin),  Amoxicillin  and  
clavulanate  (Augmentin),  Ticarcillin  and  clavulanate  potassium  (Timentin),  Chloramphenicol,  
Meropenem  (Merrem),  Metronidazole  (Flagyl),  Tigecycline  (Tygacil),  Moxifloxacin  (Avelox),  
Entapenem.  
   
      5.    Enterococcus  feacalis,  treatment?  
Answer:  Ampicillin  is  drug  of  choice,  allergic  or  resistant  to  ampicillin:  Vancomycin  
   
      6.    Bacteroid  fragilis  Abx?  
Answer:  Ampicillin  "not  sure"  
   
      7.    Gun  shoot  with  bowel  perforation?  
Answer:  Metronidazole  +  gentamicin,  or  Metro  +  cefoxitin  
   
           8.    Rosacea,  treatment  ?  
Answe:  erythromycin,  if  Doxycyclin  "tetracycline"  wasn't  in  the  answer  
Medscape:  https://emedicine.medscape.com/article/1071429-­‐medication#3  
First  Aid:  treat  with  low-­‐potency  corticosteroids  or  topical  metronidazole.  In  more  severe  disease,  
systemic  antibiotics  may  be  used.  Extremely  severe  cases  can  be  treated  by  short-­‐term  
metronidazole.  
   
      9.    Necrotising  faschitis?  
Answer:  pipra/tazo  +  clinda  +  vancomycin,  u  can  use  carbapenem  instead  of  pipra/tazo  
   
   
    10.  UTI,  with  methicillen  sensitive,  treatment?  
                  Answer:  cloxacillin  Rx  
   
     11.  Clostridium  difficile  Rx?  
                  Answer:  metro,  if  no  metronidazole  à  vancomycin  
First  Aid:  Cessation  of  inciting  antibiotic  if  any,  PO  Metronidazole  or  PO  Vancomycin;  IV  
metronidazole  if  the  pt  can’t  tolerate  oral  medications.  
   
     12.  Animal  bite  Rx?  
Answer:  amox/clavulanate  +  tetanus  booster  
   
      13.  Best  antibiotic  for  gram  -­‐ve  bacilli?  
Answer:  cephalosporin  
   
    14.  Antibiotic  inhibits  Ca  and  aluminium  salt?  
Answer:  tetracycline  
   
    15.  Antibiotic  safe  in  pregnancy?  
Answer:  ampicillin,  penicillin,  ampi/sulbactam,  cefoxitin,  cefotetan,  cefazolin,  clinda,  erythro,  
gentamicin,  
   
   16.  Antibiotic  for  UTI,  safe  in  all  trimesters?  
Answer:  ampicillin  
   
    17.  Rheumatic  heart  disease,  allergic  to  penicillin,  prophylactic  before  surgery?  
Answer:  IV  vancomycin  and  gentamycin  
   
    18.  Two  drugs  contraindicated  with  each  others?  
Answer:  tetracycline  and  aluminum  
   
      19.  Mastoditis,  treatment?  
Answer:  vancomycin  with  ceftriaxone  
   
      20.  Nisseria  infection  risk?  
Answer:  defect  in  classical  complement  pathway  
   
           21.  Tenia  solium,  found  in  meat  of?  
Answer:  pork  
   
     22.  Tenia  saginata,  found  in  meat  of?  
Answer:  beef  
   
   
   
    23.  Chronic  granulomatous  disease?  
Answer:  recurrent  infection  with  staph,  aspergillus,  nocardia,  serratia  and  Burkholderia.  
   
    24.  Patient  comes  from  Africa  and  has  fever,  Dx?  
Answer:  depends  on  incubation  period:  
within  one  week  (7  days):  yellow,  dengue  murburg  fever  
within  3  weeks  to  1  month:  ebola,  lassa  fever,  malaria.  
   
       25.  Leshmania  treatment?  
Answer:  Miltefosine  
   
       26.  Vibrio  cholera,  treatment?  
Answer:  Doxycyclin  "30S  protein"  
   
  27.  Schistosomia,  treatment?  
Answer:  Praziquantel  
   
       28.  Inhaled  antiviral  works  by?  
Answer:  neuraminidase  inhibitor  
   
  29.  Parasite  infection  in  undercooked  food?  
Answer:  trichinellosis  
   
       30.  Bacteria  sexual  behavior?  
Answer:  conjugation  
   
       31.  GAS  throat  infection?  
Answer:  Amoxicillin,  or  erythromycin  if  allergic  to  penicillin  
   
       32.  Interstitial  lung  disease  with  small  non  necrotizing  granuloma?  
Answer:  hypersensitive  pneumonitis  
   
       33.  Septic  arthritis  resistant  to  antibiotic?  
Answer:  start  vancomycin  
   
     34.  Convulsion  after  GI  infection?  
Answer:  shigella  
   
     35.  Chemotherapy,  infection  from  venous  line?  
Answer:  pseudomonas  
   
     36.  Streptococcal  throat  infection,  Rx  duration?  
Answer:    10  days  
   
   
   37.  Ribavirin  Side  effects?  
Answer:  Anemia  
Medscape:  Decreased  Hgb  (25-­‐36%)  
   
   38.  Anti  TB,  causing  seizure?  
Answer:  isoniazid  
   
   39.  Anti  TB,  causing  parasthesia  or  nerve  problem?  
Answer:    isoniazid  
   
     40.  Anti  TB,  causing  visual  problem?  
Answer:  Ethambutol  
   
     41.  Anti  TB,  causing  vertigo  or  ear  problem?  
Answer:  streptomycin  
   
     42.  Anti  TB,  causing  red  urine?  
Answer:  Rifampicin  
   
       43.  Farmer  with  sandfly  infection,  what’s  the  treatment?  
Answer:  miltefosine    (Dx  leishmania)  
Medscape:  https://emedicine.medscape.com/article/220298-­‐medication#2  
   
       44.  HIV  with  pneumonia,  bubble  soap?  
Answer:  cryptococcus  neoformans  
   
       45.  RSV  virus  can  be  found  in?  
Answer:  Liver  
   
  46.  3  to  4  scenarios  about  TB  (all  were  direct  and  clear)  
   
       47.  PPD  positive  x  ray  negative,  next?  
Answer:    isoniazid  for  6  months  
   
       48.  A  pt  on  anti  TB  drugs  for  4  weeks  developed  needle  sensations?  
Answer:    isoniazid  
   
  49.  Patient  with  recurrent  oral  fungal  infection  and  other  things?  
Answer:  Chronic  mucocutanous  candidiasis  
   
50.  Patient  was  in  trip  in  Africa  and  had  many  symptoms.  (one  of  it  was  salamon   blenching  
something  in  the  skin).  What  is  the  investigation?  
1.   bone  marrow  
2.   blood  film  
Answer:  
Medscape:  
(Dx):  At  approximately  the  end  of  the  first  week  of  illness,  the  fever  plateaus  at  103-­‐104°F  (39-­‐40°C).  
The  patient  develops  rose  spots,  which  are  salmon-­‐colored,  blanching,  truncal,  maculopapules  
usually  1-­‐4  cm  wide  and  fewer  than  5  in  number;  these  generally  resolve  within  2-­‐5  days.  [2]  These  
are  bacterial  emboli  to  the  dermis  and  occasionally  develop  in  persons  with  shigellosis  or  
nontyphoidal  salmonellosis.  [27]  
(Invx):  The  diagnosis  of  typhoid  fever  (enteric  fever)  is  primarily  clinical.  
The  criterion  standard  for  diagnosis  of  typhoid  fever  has  long  been  culture  isolation  of  the  organism.  
Cultures  are  widely  considered  100%  specific.  
Culture  of  bone  marrow  aspirate  is  90%  sensitive  until  at  least  5  days  after  commencement  of  
antibiotics.  However,  this  technique  is  extremely  painful,  which  may  outweigh  its  benefit.  [36]  
   
   
51.  Student  with  10  of  his  classmate  develop  dry  cough  mild  SOB  and  bilateral  consolidation?  
1.   leogenialla    my  answer  
2.   Mycoplasma  ??  
3.   Staph  
4.   Strept.  
Answer:  
   
  52.  Cat  bite,  what  is  the  most  likely  organism  
1.   Staph  aureus  
2.   Pasturella  multicida  
Answer:  B  
   
     53.  associated  with  animal  bites?  
1.   Polybactrial  
2.   Pastrulla  multicodia  
Medscape:  P  multocida  often  exists  as  a  commensal  in  the  upper  respiratory  tracts  of  many  
livestock,  poultry,  and  domestic  pet  species,  especially  cats  and  dogs.  
   
   54.  Bitten  by  his  brother  and  there  is  1  cm  injury,  what  you  do?  
1.   Amoxcillin  
2.   Surgical  suture  
Answer:?    wound  cleaning  and  wound  closure  and  tetanus  prophylaxis  and  antibiotic.  Surgical  
intervention  is  frequently  necessary  and  ranges  from  simple  wound  exploration  and  debridement  to  
repair  of  complex  structures  under  magnification.  (medscape)  
https://emedicine.medscape.com/article/218901-­‐treatment  
   
   55.  Dog  bites  are?  
1.   Poly  microbial  
2.   Viral  
3.   Bacterial  
4.   Fungal  
Answer:  A  
   
 56.  Gun  shot  wound,  B  fragils.  What  to  give  the  patient?  (no  metronidazole  in  the  choices)  
Answer:  Clindamycin  and  others  
   
 57.  Patient  with  painless  penile  ulcer  with  lymphadenopathy  what  is  the  treatment?  
1.   Benazthine  penicillins  g  (correct)  
2.   penicillin  V  
Answer:  A  
   
58.  Child  present  with  malaise,  history  of  meningitis  last  week  treated  with  iv  antibiotics.  Lab;    
HGb:  low,  RBC:  low,  What  antibiotic  he  used?  
1.   Chloramphinicol  
2.   Erythromycin  
Answer:  A  
Medscape:  one  of  the  side  effects  of  chloramphenicol  is  Aplastic  anemia  <1%  
https://reference.medscape.com/drug/chloramphenicol-­‐iv-­‐chloromycetin-­‐342554#4  
   
59.  The  most  common  organism  associated  with  honeymoon  cystitis?  
A.  S.  Saprophyticus  
B.  E.  Coli  
Answer:  
Medscape:    coli  causes  70-­‐95%  of  both  upper  and  lower  UTIs.  Various  organisms  are  responsible  for  
the  remainder  of  infections,  including  S  saprophyticus,  Proteusspecies,  Klebsiella  species,  
Enterococcus  faecalis,  other  Enterobacteriaceae,  and  yeast.  Some  species  are  more  common  in  
certain  subgroups,  such  as  Staphylococcus  saprophyticus  in  young  women.  However,  S  
saprophyticus  can  produce  acute  cystitis  in  older  women  and  in  young  men  and  should  not  be  
automatically  regarded  as  a  contaminant  in  the  urine  cultures  of  these  individuals.  
   
60.  The  most  common  organism  of  meningoenchephalitis  is:  
A.  enterovirus  
B.  HSV  
Medscape:  
Primary  meningoencephalitis  
Ubiquitous  in  most  soils  in  most  environments,  N  fowleri  is  also  found  in  warm  freshwater,  
particularly  if  the  water  is  stagnant.  [7]  Exposure  to  the  this  amoeba  is  very  common.  Children  
younger  than  2  years  frequently  carry  the  organism  asymptomatically  in  their  nose  and  throat,  
especially  in  warmer  months  and  climates.    Infection  with  this  pathogen  occurs  in  both  
immunocompromised  and  immunocompetent  individuals.  [8]  
PAM  is  an  exceptionally  uncommon  occurrence  resulting  from  CNS  invasion  of  the  typically  healthy  
host  by  N  fowleri.  During  a  period  of  a  few  days  to  2  weeks  after  inoculating  a  patient  who  had  been  
swimming,  diving,  bathing,  or  playing  in  warm,  usually  stagnant,  freshwater,  the  amoebae  migrate  
through  the  cribriform  plate,  along  the  fila  olfactoria  and  blood  vessels,  and  into  the  anterior  
cerebral  fossae,  where  they  cause  extensive  inflammation,  necrosis,  and  hemorrhage  in  the  brain  
parenchyma  and  meninges.  [9]      
   
   
61.  3  yrs  old  baby  with  fever,  neck  rigidity,  culture  revealed  diplocci/Gm-­‐ve  (They  provided  a  
picture  of  diffuse  meningiococcemia  skin  rash  and  asked  about  what  to  give  to  his  Family?  

Answer:  Oral  rifampicin  

62.  Young  man  diagnosed  with  TB  wt  will  u  give  to  his  asymptomatic  family:  
1.   Bcg  
2.   rifampin  
3.   All  other  answers  were  not  related  at  all  
Answer:  B  
   
63.  A  male  patient  with  headache,  malaise,  lymphadenopathy  (and  other  non-­‐specific  symptoms).  
He  has  absolute  eosinophilia  on  CBC.  What  is  the  cause?  
A-­‐Schistosoma  
B-­‐Amoeba  
C-­‐Giardia  
Answer:  
   
64.  What  is  the  treatment  of  epidemic  vibrio  cholerae?  
 Medscape:  
●            Furazolidone  has  been  the  agent  routinely  used  in  the  treatment  of  cholera  in  children;  however,  
resistance  has  been  reported,  and  ampicillin,  erythromycin,  and  fluoroquinolones  are  potentially  
effective  alternatives.  The  use  of  quinolones  is  contraindicated  in  children  with  cholera.  
●            Travelers  to  cholera-­‐affected  regions  should  receive  a  cholera  vaccine.  The  cholera  vaccine  
Vaxchora  is  the  only  one  approved  by  the  FDA  for  cholera  prevention.  It  is  a  live,  weakened  vaccine  
administered  as  a  single,  oral  liquid  dose  of  about  three  fluid  ounces  at  least  10  days  before  travel  to  
a  cholera-­‐affected  region.  The  only  other  existing  cholera-­‐prevention  vaccines  require  2  doses,  
according  to  the  Centers  for  Disease  Control  and  Prevention  (CDC).  A  single-­‐dose  vaccine  is  
especially  beneficial  to  a  person  who  needs  to  travel  to  a  cholera-­‐affected  region  on  short  notice.  
[20]  
●            Emerging  drug  resistance  in  certain  parts  of  the  world  is  a  concern,  as  some  V  cholerae  strains  
contain  plasmids  that  confer  resistance  to  many  antibiotics.  In  areas  of  known  tetracycline  
resistance,  therapeutic  options  include  ciprofloxacin  and  erythromycin.  Strains  resistant  to  
ciprofloxacin  have  been  reported  from  Calcutta,  India.  
   

65.  About  watery  diarrhea  without  blood  and  asked  about  organism?  
-­‐  I  chose  C.  Defficile  (  Not  sure  )  
First  aid:  
C.  difficile  due  to  recent  Abx  use  (penicillin,  cephalosporin,  clindamycin),  fever,  abd  pain,  possible  
systemic  toxicity,  fecal  RBCs  and  WBCs,  most  commonly  in  large  bowel,  but  can  involve  small  bowel.  
Identify  C.  diff  toxins  in  stool.  Sigmoidoscopy  showed  pseudomembranes.  
   
66.  Scenario  about  female  sexually  active  and  came  with  symptom  I  forgot,  in  lab  results  he  
mentioned  gram  negative  diplococci  and  asked  about  the  diagnosis?  
 ANSWER:  N.  Gonorrhea  
   
67.  Scenario  typical  of  HIV  with  PCP  infection,  CMV  retinitis,  oral  thrush.  Then  asked  what  will  you  
find?  
CD  4  of  100  
ANSWER:  ??  
   
68.  Hiv  pt  before  ttt  you  must  check?  
ANSWER:  CD4  
 Medscape:  Research  data  led  to  US  guidelines  recommending  that  antiretroviral  therapy  be  initiated  
at  a  CD4  count  threshold  of  350/μL,  although  2013  guidelines  from  the  World  Health  Organization  
(WHO)  now  recommend  a  threshold  of  500/μL.  [108,  109]They  also  state  that  in  some  cases,  
antiretroviral  treatment  should  begin  immediately,  regardless  of  the  CD4  count,  including  in  HIV-­‐
positive  serodiscordant  couples,  patients  with  hepatitis-­‐B  coinfection,  pregnant  or  breastfeeding  
women,  and  children  under  age  5  years.  [108,  109]  
   
69.  Vaccine  contraindicated  in  Hiv  pt?  
 ANSWER:  varicella  vaccine  (All  live  vaccines)  
   
   
70.  IV  drug  abuser  complain  of  oral  thrush,  signs  and  symptoms  of  pneumonia  ,  broncho  alveolar  
lavage  was  done  pneumocysticitis  jiroveci  was  found  then  HIV  test  was  done  and  it  was  +ve  ,  what  
is  the  most  likely  predictor  of  her  HIV  infection?  
1.   Pneumocysticitis  jiroveci*  
2.   IV  drug  use  
3.   Candida  
 ANSWER:  A  ?  
   
71.  HIV  patint,  cough  with  bad  odour  sputum,  x  ray  attached  what  is  diagnosis?  
Answer:  PCP  pneumonia  
   
   
   
72.  Latency  period  in  HIV,cell  responsible?  
ANSWER:  CD4+  T  cells  with  a  memory  
   
73.  patient  with  HIV  and  ask  about  something  I  forget  ^_^  but  the  choices  was  :  
1.   Memory  B  cell  
2.   CD  8  T  cell  
3.   Macrophage  
Answer:  
   
 74.  Co-­‐receptor  of  HIV  ?  
ANSWER:  CXCR4  (Not  sure)  
   
75.  Man  have  one  month  sexual  relation  result  of  HIV  was  negative  when  will  repeat  the  screen  ?  
1.   2  mo  
2.   3  mo  
3.   5  mo  
4.   6  mo  
Answer:  
76.  (6-­‐2  Qs)  about  alternative  cell  reservoir  for  HIV  (CD  4)  ?  
Is  it  CD  8  ?!  I  don’t  know  
Monocytes,  other  T  cells,  macrophages,  adipose  tissue,  GALT,  genital  tract,  semen,  bone  marrow,  
and  central  nervous  system  (CNS)  cells  including  both  microglia  and  astrocytes  
   
77.  16  yrs  old  drug  abuser,  what  invx  should  be  done  to  him?  
1.   Test  for  HIV,  I  chose  this  
2.   Test  of  HBV  
ANSWER:  A  

78.  Case  of  positive  PPD,  no  symptom,  clear  x  ray,  how  to  manage  ?  (Nurse  with  10  mm  PPD  test,  
CXR  negative,  what  to  do)  
1.   Isonazide  6  months  
2.   Rifam  6  m  
ANSWER:  A?  
●            6-­‐month  or  9-­‐month  isoniazid  daily,  
●            3-­‐month  rifapentine  plus  isoniazid  weekly,  
●            3-­‐  or  4-­‐month  isoniazid  plus  rifampicin  daily,  
●            3–  or  4-­‐month  rifampicin  alone  daily.  
   
79.  Old  pt  take  antiviral  that  taken  by  inhalation?  
ANSWER:  Zanamivir  
   
80.  Man  travels  to  Sudan  2  weeks  ago,  now  he  is  presenting  with  fever,  maleas  ....  unspecific  
symptoms  (from  3  days).  How  you  will  confirm  diagnosis??  
A)  blood  culture  
B)  Serology    
C)  Blood  film  
ANSWER:  I  would  choose  C?  
https://emedicine.medscape.com/article/221134-­‐workup#c8  
   
81.  Pt  with  treated  meningiococal  meningitis  what  to  give  her  close  contacts  
Penicillin  or  similar  abx  to  remove  nasal  carriers**  
1.   Isolate  contacts  
2.   Do  nothing  
3.   Give  vaccine  
 ANSWER:  
First  aid:  close  contact  should  receive  rifampin,  ciprofloxacin  or  ceftriaxone  prophylaxis.  
   
82.  Pt  with  treated  meningiococal  meningitis  what  to  give  her  close  contacts  
1.   Penicillin  or  similar  
2.   (rifampin  or  ciprofloxacin)  
3.   abx  to  remove  nasal  carriers  Isolate  contacts  
4.   Do  nothing  Give  
ANSWER:  B  
 First  aid:  close  contact  should  receive  rifampin,  ciprofloxacin  or  ceftriaxone  prophylaxis.  
   
83.  Pictures  of  chancre  painless  ulcer  on  the  penis  how  to  treat?  Dx:  primary  syphilis?  
1.   Penicillin  V  
2.   Pencillin  G  
3.   Steroid  
ANSWER:  B  
   
   
84.  Scenario  of  patient  from  India  had  rash  hepatomegaly  Treated  and  after1  year  came  with  rash?  
What’s  the  Dx?  
1.   Dermatological  leishmaniasis  
2.   Leprosy  
ANSWER:  A  
   
85.  Patient  on  chemotherapy  developed  IV  line  sepsis,  (  bacterial  sepsis)  most  likely  organism?  
ANSWER:  Closest  option  was  pseudomonas  (correct)  
   
86.  Source  of  infection  in  venipuncture?  
ANSWER:  Site  of  insertion  ?  
   
87.  Patient  on  TB  medication  developed  numbness,  what  would  u  give  him?  
ANSWER:  Vitamin  B6  (pyridoxine)  
   
   
   
88.  safe  vaccine  to  be  given  to  immunocompromised  person?  
1.   MMR  
2.   Pnemococcal  
ANSWER:  B  
   
89.  Gun  shot  in  the  abdomen  with  bacteroid  fragilis  which  antibiotic??  
ANSWER:  Metrodinazole  
   
90.  What  would  be  helpful  in  diagnosis  of  N.gonorrhoeae  ?  
1.   Gram  stain  
2.   Culture  
3.   PCR  
4.   forgot  it  
ANSWER:  B  
Medscape:  Culture  is  the  most  common  diagnostic  test  for  gonorrhea,  followed  by  the  
deoxyribonucleic  acid  (DNA)  probe,  and  then  the  polymerase  chain  reaction  (PCR)  assay  and  ligand  
chain  reaction  (LCR).  The  DNA  probe  is  an  antigen  detection  test  that  uses  a  probe  to  detect  
gonorrhea  DNA  in  specimens.  
   
91.  HBV  commonest  virus  which  make  rejection  of  the  exparitis  or  labours  from  working  in  Saudi  
?(Community)  
1.   HBV  
2.   HCV  
3.   HIV  
Answer:  A  same  Q  from  smle  13  
ANSWER:  C  
References:    http://applications.emro.who.int/emhj/v19/07/EMHJ_2013_19_7_664_670.pdf?ua=1  
   
92.  How  to  diagnose  factitious  fever?  
1.   Blood  culture.  
2.   Urine  analysis.  
3.   Pulse  
ANSWER:  C  
   
93.  Case  scenario  about  DM  patient  with  necrotizing  fasciitis,  which  antibiotic?  
1.   penicillin/gentamycin  
2.   piperacillin/  tazobactam*  
ANSWER:  B  
   
94.  How  to  prevent  MERSA?  
1.   Hand  washing  
2.   Vaccine  
ANSWER:  A  
   
   
95.  Hepatitis  diagnosed  by?  
1.   Blood  test  (  I  choose  it  )  
2.   Imaging  
3.   Hx  
ANSWER:  A  
   
96.  Type  of  Hepatitis  B  vaccine?  
 ANSWER:  recombinant  
   
97.  20  y.  O  male    with  small  erythmatous  macules  non  planchable  ,  history  of  viral  respiratory  
infection  resolve  spontaneously  last  week,  Lab:  plt:  15  (  very  low  ),  What  TREATMENT?  
1.   IVIG  
2.   Splenectomy  
3.   Platelets  transfusion  
ANSWER:  A  
   
98.  side  effect  of  ribavirin?  
1.   Anemia.  
2.   Renal  damage  
3.   Hepatic  damage.  
ANSWER:  A  
   
99.  How  to  diagnose  enteric  fever  during  the  first  week?  
1.   Urine  and  stool  culture  
2.   Single  blood  culture  
3.   Multiple  blood  cultures  
4.   Bone  marrow  culture  
ANSWER:  
Medscape:  
●            The  diagnosis  of  typhoid  fever  (enteric  fever)  is  primarily  clinical.  
●            The  criterion  standard  for  diagnosis  of  typhoid  fever  has  long  been  culture  isolation  of  the  
organism.  Cultures  are  widely  considered  100%  specific.  
●            Culture  of  bone  marrow  aspirate  is  90%  sensitive  until  at  least  5  days  after  commencement  of  
antibiotics.  However,  this  technique  is  extremely  painful,  which  may  outweigh  its  benefit.  
●            Blood,  intestinal  secretions  (vomitus  or  duodenal  aspirate),  and  stool  culture  results  are  positive  
for  S  typhi  in  approximately  85%-­‐90%  of  patients  with  typhoid  fever  who  present  within  the  first  
week  of  onset.  They  decline  to  20%-­‐30%  later  in  the  disease  course.  In  particular,  stool  culture  may  
be  positive  for  S  typhi  several  days  after  ingestion  of  the  bacteria  secondary  to  inflammation  of  the  
intraluminal  dendritic  cells.  Later  in  the  illness,  stool  culture  results  are  positive  because  of  bacteria  
shed  through  the  gallbladder.  
   
100.  Enteric  fever  TTT?  
1.   Ciprofloxacin  
2.   Amoxicillin  
3.   Metronidazole  
ANSWER:  A  
   
102.  Enteric  fever  micro  description  dx  ?  Management  ?  
 Answer:  
   
103.  Best  Diagnosis  of  enteric  fever?  
Medscape:  
●            The  criterion  standard  for  diagnosis  of  typhoid  fever  has  long  been  culture  isolation  of  the  
organism.  Cultures  are  widely  considered  100%  specific.  
●            Culture  of  bone  marrow  aspirate  is  90%  sensitive  until  at  least  5  days  after  commencement  of  
antibiotics.  However,  this  technique  is  extremely  painful,  which  may  outweigh  its  benefit.  
●            Blood,  intestinal  secretions  (vomitus  or  duodenal  aspirate),  and  stool  culture  results  are  positive  
for  S  typhi  in  approximately  85%-­‐90%  of  patients  with  typhoid  fever  who  present  within  the  first  
week  of  onset.  They  decline  to  20%-­‐30%  later  in  the  disease  course.  In  particular,  stool  culture  may  
be  positive  for  S  typhi  several  days  after  ingestion  of  the  bacteria  secondary  to  inflammation  of  the  
intraluminal  dendritic  cells.  Later  in  the  illness,  stool  culture  results  are  positive  because  of  bacteria  
shed  through  the  gallbladder.  
●            Multiple  blood  cultures  (>3)  yield  a  sensitivity  of  73%-­‐97%.  Large-­‐volume  (10-­‐30  mL)  blood  
culture  and  clot  culture  may  increase  the  likelihood  of  detection.  [37]  
●            Stool  culture  alone  yields  a  sensitivity  of  less  than  50%,  and  urine  culture  alone  is  even  less  
sensitive.  Cultures  of  punch-­‐biopsy  samples  of  rose  spots  reportedly  yield  a  sensitivity  of  63%  and  
may  show  positive  results  even  after  administration  of  antibiotics.  A  single  rectal  swab  culture  upon  
hospital  admission  can  be  expected  to  detect  S  typhi  in  30%-­‐40%  of  patients.  S  typhi  has  also  been  
isolated  from  the  cerebrospinal  fluid,  peritoneal  fluid,  mesenteric  lymph  nodes,  resected  intestine,  
pharynx,  tonsils,  abscess,  and  bone,  among  others.  
104.  Diarrhea  (  wedding  a  couple  of  hours  later  had  diarrhea  with  micr  description)?  
ANSWER:  i  don't’  get  it  (????)  
   
105.  treatment  of  traveler's  diarrhea?  
ANSWER:  Ciprofloxacin  
   
106.  Gas  gangrene?  
ANSWER:  cl.perfeenges  
   
107.Central  line  with  fungal  inf.  What  is  the  TTT?  
ANSWER:  Fluconazole  
   
109.  which  one  of  the  following  infection  diagnosed  by  stool  analysis  by  finding  an  antigen?  
1.   Ascaris  
2.   Tenia  saginata  
3.   Schistosoma  mansoni  
4.   ………………………  
NOTE:  all  of  the  above  infections  are  diagnosed  by  stool  analysis  by  finding  eggs  or  trophozoites.  The  
infections   which   are   diagnosed   by   finding   antigens   are   H.Pylori   and   giardiasis.   SO,   MAY   BE   THE  
MISSING  ANSWER  HERE  IS  THE  CORRECT  CHOICE.      
   
110.  Treatment  of  shigellosis?  
1.   amoxicillin  
2.   ceftriaxone    
ANSWER:  B  
   
111.  INFECTIOUS  case  of  GIT  characterized  by  diarrhea  followed  by  constipation  and  give  you  
bacteriology  finding  which  is  gram  negative  and  other  features  I  cannot  remember,  then  asking  
about  treatment:  
1.   Ciprofloxacin  
2.   Chloramphenicol  
3.   Penicillin  
4.   …………………  
Note:  from  my  reading  salmonella  is  characterized  by  diarrhea  followed  by  constipation  
So  my  answer  was  (A)  but  not  sure  about  it.  
ANSWER:  A  (Dx:  S.  typhi)  
   
112.  What  would  be  helpful  in  diagnosis  of  gonorrhea?  
1.   gram  stain  
2.   culture  
3.   PCR  
4.   forgot  it  
 ANSWER:  B  
   
113.  What  is  the  parasite  usually  found  in  beef  ?  (No  choices  provided)  
ANSWER:  T.  Saginata  i  think  
   
114.  Patient  with  lymphadenopathy  &  splenomegaly?  
Answer:  EBV      
   
115.  (3  to  4  scenarios)  about  TB  (all  were  direct  and  clear)  
   
116.  T.diarrhea  >  shegella  (????)  
   
117.  The  only  organism  from  human  source?  
UpToDate:  

●            Human  bite  wound  pathogens  consist  of  both  aerobic  and  anaerobic  bacteria,  including  streptococci,  
Staphylococcus  aureus,  Eikenella,  Fusobacterium,  Peptostreptococcus,  Prevotella,  and  Porphyromonas  spp  [1-­‐
3].  In  a  study  of  50  patients  with  infected  human  bites,  the  median  number  of  isolates  per  wound  culture  was  
four  [3].  Both  aerobes  and  anaerobes  were  isolated  from  54  percent  of  wounds,  aerobes  alone  were  isolated  
from  44  percent,  and  anaerobes  alone  were  isolated  from  2  percent.  

●            Viral  pathogens,  including  hepatitis,  human  immunodeficiency  virus,  and  herpes  simplex  virus,  are  
transmissible  by  human  bites;  clinical  descriptions  are  limited  to  case  reports  [4-­‐9].  Human  bite  transmission  of  
syphilis  has  also  been  described  

   
118.  Most  common  site  for  central  line  infection?  
1.   Seeding  of  bacteria  
2.   Hospital  workers  
3.   Site  of  insertion  
ANSWER:  C  
(Up  to  date)  
   
119.    a  pt  end  stage  liver  disease  on  Central  venous  catheter  developed  sepsis  Culture  showed  
budding  yeast,  Suitable  Rx  is?  
1.   fluconazole  
2.   antiviral  
3.   Abx  
4.   cuspofungin  
ANSWER:  D  
   
120.  The  transmission  of  maternal  antibodies  to  the  fetus  in  pregnancy  is  a  way  of?  
1.   active  artificial  Immunity  
2.   passive  artificial  immunity  
3.   passive  natural  immunity  
4.   active  natural  immunity  
ANSWER:  C  
   
121.  Post  cholecystectomy  pt,  developed  unilateral  parotid  swelling,  saliva  was  cloudy,  culture  
was  negative,  What  does  he  have?  
1.   sarcoid  granuloma.  
2.   bacterial  sialadenitis  
3.   sarcoma  
4.   sojgrens  syndrome    
ANSWER:  B  
   
122.  female  her  husband  had  gonorrhea  what  best  investigation  for  her?  
1.   Gram  stain  
2.   PCR  
3.   I  forgot  others  
ANSWER:  Could  be  A  if  only  those  choices  (not  sure)  
First  aid  page  220  
   
123.  13  yrs  old  pt  with  salmonella  infection,  Resistant  to  chloramphenicol,  appropriate  Tx  is?  
1.   continue  chloramphenicol  
2.   add  cipro  
3.   give  cipro  alone  
4.   IM  ceftriaxone  
ANSWER:  C  
   
124.  I  can’t  remember  the  scenario  but  the  answer  was  Rota  Virus  
   
125.  food  poisoning  case,  4  family  members  ate  from  a  restaurant,  they  developed  diarrhea  and  
vomiting  and  remit  after  24  hrs.  Culture  showed  gram  positive  bacilli?  
1.   salmonella  
2.   shigella  
3.   SA  
4.   Bacillus  ceres  
ANSWER:  D  
   
125.  Diagnosis  of  visceral  leishmaniasis?  
1.   blood  film  
2.   bone  marrow  
ANSWER:  A  
https://emedicine.medscape.com/article/220298-­‐workup#c7  
   
126.  scenario  of  man  came  from  desert  and  got  cutaneous  leishmania  what  is  the  treatment  ??  
1.   Oral  miltefosine  
2.   Injectable  paromomycin  
3.   There  was  no  amphotrpcin  B  
Answer:  A  
https://emedicine.medscape.com/article/220298-­‐treatment#d9  
   
127.  Treatment  of  leishmania  donovani?  
●            Liposomal  amphotericin  B  alone,  given  as  a  single  dose  (currently  recommended  as  the  drug  of  
choice  by  the  Kala-­‐Azar  elimination  programme  of  India)  
●            Liposomal  amphotericin  B  in  a  single  dose,  in  combination  with  7  days  of  oral  miltefosine  or  10  
days  of  paromomycin  
●            Miltefosine  plus  paromomycin  for  10  days  
●            Amphotericin  B  deoxycholate:  0.75-­‐1  mg/kg/day  via  infusion,  daily  or  on  alternate  days  for  15-­‐20  
doses  
●            Miltefosine  orally  for  28  days  or  paromomycin  intramuscularly  for  28  days  
●            Pentavalent  antimonials:  20  mg  Sb5+/kg/day  intramuscularly  or  intravenously  for  30  days  in  
areas  where  they  remain  effective:  Bangladesh,  Nepal,  and  the  Indian  states  of  Jharkhand,  West  
Bengal,  and  Uttar  Pradesh.  
●            https://emedicine.medscape.com/article/220298-­‐treatment#d11  
 
128.  Visceral  leishmaniasis  organism?  
A.  L  donovani  
B.  L.Tropica  
ANSWER:  A  
   
129.  type  of  leishmaniasis  cause  skin  manifestation?  
1.   L.  Tropica  
ANSWER:  A  (L.  Tropica,  L.  Major,  L.  mexicana)  
https://emedicine.medscape.com/article/220298-­‐clinical  
   
130.  which  markers  of  HBV  is  present  in  window  phase?  
ANSWER:  Anti  HBc  
WiKi:  IgM  anti-­‐core  (HBc-­‐IgM)  is  the  only  detectable  antibody.  HBV  DNA  may  be  positive  as  well.  
   
131  pictures  of  chancre  painless  ulcer  on  the  penis  how  to  treat?  (Dx):  primary  syphilis?  
1.   penicillin  V  
2.   pencillin  G  and  
3.   steroid  
ANSWER:  B  
   
133.  patient  has  been  bitten  by  dog,  and  he  received  rabies  vaccine  18  months  ago  when  he  
travelled  to  some  place,  what  action  should  be  taken  immediately?  
1.   Do  nothing  as  patient  is  immunized.  
2.   Give  rabies  vaccine  and  immunoglobulin.  
3.   Give  immunoglobulin  alone.  
4.   Give  two  doses  of  rabies  vaccine.  
ANSWER:  D  (Correct)  
●            Note:  this  q  is  solved  before  in  one  of  the  questions  collection  as  B,  but  what  I  found  that  answer  
D  is  the  correct  choice.  There  are  big  details  about  this  topic.  So  you  do  not  need  to  waste  your  time.  
(previous  note)  
●            “If  bitten,  a  vaccinated  person  should  receive  two  more  doses  of  rabies  vaccine;  one  dose  
immediately  and  one  three  days  later.”  http://www.immunize.org/catg.d/p4216.pdf  
●            Previously  vaccinated  persons  include  those  who  have  received  the  3-­‐dose  preexposure  series  of  
HDCV,  rabies  adsorbed  virus  (RVA),  or  PCECV;  a  full  PEP;  or  a  previous  vaccination  with  any  rabies  
vaccine  with  a  documented  history  of  seroconversion.  HRIG  should  not  be  administered.  
●            For  the  vaccine,  administer  2  doses  (1  mL  each)  into  the  deltoid  muscle  on  day  0  and  day  3.  
https://emedicine.medscape.com/article/785543-­‐overview#a8  
134.  Patient  was  bitten  by  a  dog,  he  was  conscious,  alert.  Everything  was  normal.  He  has  a  bite  
wound  in  his  hand.  The  patient  told  you  that  he  took  rabies  vaccine  before  one  year  and  half.  
What  should  you  do?  
A.                     give  rabies  vaccine  
B.                     Give  two  dose  of  rabies  vaccine  
C.                     Observe  for  10  days  
D.                     Rabies  immuniglublin  and  vaccine  
ANSWER:  Previous  answer:  D,  but  the  answer  is  B  (according  to  medscape)  as  the  previous  Q.  
explanation.  
   
135.  Most  common  cutaneous  manifestation  of  antimalarial  medications?  
A-­‐Pruritus  
B-­‐Pigmenation  
C-­‐Photosensitivity  
D-­‐Generalized  yellow  discoloration  of  skin  
ANSWER:  (Chloroquine  A)  +  (tetracycline  D)  Medscape  
   
136.  Picture  and  they  asked  which  type  of  Malaria?  
ANSWER:  Plasmodium  falciparum  (depend  on  the  picture)  
   
137.  What  of  the  following  use  scotch  tap  in  diagnosis?  
A.        Giardia  
B.        Malaria  
C.        Schistosomiasis  
ANSWER:  Pinworm  (Enterobiasis)  Medscape  
https://emedicine.medscape.com/article/225652-­‐workup  
   
   
138.  Patient  will  go  to  endemic  area  of  malaria  (I  think  Sudan)  and  he  asked  you  about  prophylaxis  
for  malaria?  
A.        Quinine  1week  before  traveling,  until  6  week  after  returning.  
B.        Quinolone  2  days  before  traveling,  until  returned.  
Answer:  
   
139.  Treatment  of  uncomplicated  Falciparum  malaria?  
   
140.  Pt  came  from  sudan  I  guess  and  they  suspected  malaria  wt  of  these  inv  most  specific  and  
sensitive?  
A.        Malaria  rapid  test  
B.        Malaria  antibodies                                                                                                          
C.        Blood  film  
Answer:  
   
141.  Picture  of  ring  cell  stage  of  malaria.  Asked  the  stage  and  malaria  type.?  

142.  EBV  treated  by?  


1.   Paracetamol  and  rest  
2.   Acyclovir,  
3.   Amoxicillin  
Answer:  
   
143.  Patient  complain  of  hand  pain  after  bee  sting,  on  examination:  tenderness  +  Axillary  lymph  
node  enlarged,  (  +  picture:  weal  on  forearm)  what  is  dx?  
1.   -­‐  URTECARIA  
2.   -­‐  Lypmhangitis  
ANSWER:  B  
   
144.  Male  want  to  take  his  son  to  south  Asia.  Prophylaxis  for  travelers’  diarrhea?  
1.   Ciprofluxacine  
2.   Ceftrexione  
3.   Doxycycline  
 ANSWER:  A  
   
145.  how  to  diagnose  factitious  fever?  
A.                     Blood  culture  
B.                     Urine  analysis  
C.                     Heart  rate  
D.                     Rheumatoid  factor  
ANSWER:  C  
   
 146.  patient  who  is  an  IV  drug  abuser  ,  on  a  blood  smear  it  was  found  out  that  he  
developed  an  RNA  virus  of  a  Flaviviridae  family  ,  the  Virus  is?  
1.   HBV  
2.   HCV  
3.   HDV  
4.   HAV  
 ANSWER:  B  
   
   
   
   
   
 147.  Pt  with  4  recurrent  UTI,  what  is  the  dose  and  duration  of  nitrofurantoin  for  this  pt?  
1.   BID  for  3  months  
2.   TID  for  3  months  
3.   Daily  for  6  months  
Answer:  

   

148.  UTI  treated  with  100mg  nitrofurantoin  what  is  the  duration?  
A)  6h  for  one  month.  
B)once  daily  for  2  month  
C)once  daily  for  six  month  
D)  once  daily  for  one  year.  
ANSWER:  A  
   
Q.  dose  of  nitrofurantoin?  
 MEDSCAPE:  25mg,  50mg,  100mg  
https://reference.medscape.com/drug/macrobid-­‐macrodantin-­‐nitrofurantoin-­‐342567    
   
149.  UTI  case  with  gram  negative  indole  positive?  
A)  E.  coli  
ANSWER:  A  
   
150.  Patient  is  allergic  to  sulfa  drugs  and  penicillin  and  shellfish.  She  has  UTI  what  antibiotic  you  
will  give?  
A)Nitrofurantoin    
B)Trimethoprim  Sulfamethoxazole  
C)Amoxicillin  
 ANSWER:  A  (not  sure)  
   
151.  treatment  of  HSV  2?  
A)  acyclovir  
B)  ribavirin  
 ANSWER:  A  
   
152.  Syphilis:  Painless  Genital  ulcer  plus  inguinal  lymphadenopathy?  NOT  COMPLETE  
   
153.  Pt  with  painless  Genital  ulcer,  how  to  diagnose?  
ANSWER:  dark  field  microscopy  
   
154.  Painful  genital  ulcer,  bleeding,  lab  confirmed  Haemophilus  Ducreyi?  
a)treat  all  close  contacts  
b)treat  all  sexual  partners  
c)treat  symptomatic  sexual  parteners  only  
 ANSWER:  B  
http://emedicine.medscape.com/article/214737-­‐treatment  
   
   
   
   
156.  Patient  came  with  fatigue,  weight  loss,  and  diarrhea.  He  received  a  blood  transfusion  when  
he  was  in  kenya.  He  has  low  grade  fever,  vitals  are  stable,  skin  ex.  There  is  contagious  molluscum  
in  groin  and  generalized  lymphadenopathy  and  palpable  liver,  diagnosis?  
a)secondry  syphillis  
b)Persistent  chronic  hep  B  
c)HIV  
d)Acute  lymphoma  
ANSWER:  C  
   
157.  IV  drug  abuser,  what  is  the  most  important  to  test  for  >  HIV  or  HBV?  
ANSWER:  HIV  (?)  
   
158.  what  is  the  most  common  cause  of  candida  infection?  
ANSWER:  candida  albicans,  there  was  other  choices  of  candida  I  can't  remember  
   
159.  Human  bite  ..6  m(months  ?)  prior  tetanus  ..what  to  give  ..  
a)reassurance  
b)booster  tetanus  
c)suturing  
d)amoxicillin  clavulanic  
MEDSCAPE:  Although  rare,  human  bites  have  been  shown  to  transmit  Clostridium  tetani.Assess  all  
patients  for  tetanus  immune  status  and  update  as  appropriate.  According  to  the  recommendations  
of  the  US  Centers  for  Disease  Control  and  Prevention  (CDC),  [13]  tetanus  immune  globulin  and  the  3-­‐
dose  vaccine  series  should  be  administered  to  patients  with  an  unknown  tetanus  vaccine  history  or  
those  who  have  received  fewer  than  3  doses.  It  is  also  indicated  for  patients  who  received  the  
complete  tetanus  series,  but  whose  booster  administration  was  more  than  5  years  ago.  For  patients  
with  a  history  of  3  or  more  doses  of  tetanus  and  diphtheria  vaccine  who  received  a  booster  less  than  
5  years  ago,  no  tetanus  booster  is  required.  
   
160.  Pt  has  fever  and  neck  rigidity,  he  was  found  to  have  a  very  contagious  bacterial  disease.  What  
will  you  give  his  close  contacts?  
a)Prednisolone  
b)Acyclovir  
c)Loratidine  
d)rifampicin  
ANSWER:  D  
   
161.  prevention  of  brucellosis?  
ANSWER:  Pasteurization  of  milk  
   
162  Gram  -­‐ve,  lactose  non-­‐ferminting  oxidase  +ve?  
ANSWER:  Pseudomonas  
   
163.  Pic  of  lobar  pneumonia:  
ANSWER:  You  will  hear  bronchial  breathing  
   
164.  Question  about  OPV  and  IPV?  
1.   Both  has  serum  antibodies  
2.   Both  prevent  virus  to  enter  the  GI  tract  
3.   Other  options  (not  sure  about)  
ANSWER:  A  
   
165.  patient  came  complains  of  fever  and  a  sore  throat  on  examination  there  was  exudative  
tonsils  and  posterior  cervical  lymphadenopathy,  the  patient  also  had  mild  splenomegaly.  Tests  
showed  positive  EBV  antibodies  what  does  the  patient  has?  
1.   pharyngitis  
2.   infectious  mononucleosis  
ANSWER:  B  
Source:  medscape  
   
167.  DM  heavy  smoker  with  whitish  patches  in  the  mouth?  
ANSWER:  Candida  
   
168.  Bacteria  in  which  human  is  the  reservoir  and  the  infection  is  acquired?  
Answer:      
   
169.  What  is  the  cause  of  pseudomembranous  colitis?  
1.   Bacterial  
2.   Idiopathic  
3.   ……….  
4.   ……...  
Answer:  
Pseudomembranous  colitis  refers  to  swelling  or  inflammation  of  the  large  intestine  (colon)  due  to  an  
overgrowth  of  Clostridium  difficile  (C  difficile)  bacteria.  
Q.  Q  about  lyme  disease.  
   
170.  Diabetic  old  man,  using  full  dentures  has  white  oral  patches,  tx?  
ANSWER:  Miconazole  (candida)  
   
171.  Hepatitis  B  chart  with  antigens  and  antibodies.  The  question  is  about  the  window  period?  
ANSWER:  ANTI  HBc  i  think  
   
172.  Group  of  people  got  diarrhea  after  they  ate  from  outside?  
Two  choices  were  organisms  cause  food  poisoning.    (They  might  want  the  most  common)  
A.        salmonella  
B.        clostridium  perfringens  
Answer:  
   
   
   
173.  You  started  patient  on  Penicillin.  The  sensitivity  test  came  back  with  cefozlin  resistant.  
What  will  you  do  next?  
A.                     Continue  the  same  
B.                     Shift  the  patient  to  Vancomycin  
 Answer:  b  
   
174.  Giardia  micro  description  what  is  the  dx?    
 Don’t  get  it  ?  
   
175.  CSF  normal  glucose  high  protein  (Not  complete)  
A.                     Viral  
B.                     TB  
Answer:  
   
176.  Organisms  of  aseptic  meningitis  (Entroviral,,,,)  
   
177.  Managment  of  meningitis  (Look  at  the  age)  
   
178.  HBc  antibody  +  The  remaining  is  negative:  
A.        Chronic  
B.        Acute  
C.        Booster  
D.        Vaccination  
ANSWER:  recovering  from  acute  infection.  
   
179.  What’s  the  most  common  infection  done  in  pre  employment?  
A.  HAV  
B.  HBV  
C.  HIV  
ANSWER:  C  
   
180.  case  of  streptococcal  pharyngitis  ,treatment  ?  
1.   Aspirin  
2.   Penicillin  
ANSWER:  B  
   
181.  student  e  hx  of  sore  throat,headache,  mild  cough,  low  grade  fever,  chest  pain,tired,general  
malaise?  
A.        Pneumoccoci,  
B.        Mycoplasma,  
C.        aspiration  pneumonia,  
D.        Legionella  
Answer:  
   
182.  Child  with  abdominal  pain,  diarrhea,  burning  urination.  UA  showed  +ve  nitrates  +ve  
leukoestrase  +ve  protein.  How  to  treat?  
A.        suprax  for  14  days  
B.        TMX-­‐SMP  for  4  days  
C.        cephalosporins  1  high  dose  
D.        Amoxicillin  
   
183.  Gram  negative,  non  lactose,  sulfa  producing.  Tx?  
A.        Cirprofloxacin  
B.        Ceftriaxon  
C.        Chlom  
ANSWER:  A  (NOT  SURE)  
   
184.  History  of  travel  to  Africa,  came  back  to  myalgia  arthralgia.  Vitals  were  normal  (no  fever),  
what  is  the  most  likely?  
A.        Ebola  
B.        Yellow  fever                                                                                                    
C.        Lassa  fever  Chikungunya                                                                                                        
Answer  :most  likely  Yellow  fever,  however  it  is  INCOMPLETE  
   
185.  pt  has  urine  culture  MRSA  he  is  on  antibiotic  after  few  day  develop  redness  on  face,  neck?  
what  is  antibiotic?  
1-­‐pencillan  
2-­‐vancomycine  
ANSWER:  B  (REDMAN  SYNDROME)  
                                                                                                                                                                                           
186.  Which  one  of  these  vaccines  taken  by  intranasal  route?      
A.        Zanamavir  
B.        Oseltamivir  
ANSWER:  A  
                                                                   
187.  Picture  (  look  like  vesicle  )  start  as  1  only  then  spread  to  arm  legs  and  ..  with  lymph-­‐node  
enlargement  ?  
a.          herps  simplex  virus  
b.          Dermatitis  herpetic  form                                                                                                              
c.          Varicella  zoster                                          
INCOMPLETE  Q,  BUT  Verecila  zoster  is  the  closest  since  others  are  blistering  dx.                  
                                                                                                                                                                                                                   
188.  50  or  40  years  female  1  day  after  chemotherapy  and  broad  spectrum  antibiotics  developed  
painful  vesicular  rash  in  the  breast?  
A-­‐  rubella  
B-­‐  measles                                                                                                        
C-­‐  varicella  zoster        
Answer:                                                                  
https://www.ncbi.nlm.nih.gov/pmc/articles/PMC3447017/  
                                                                                           
                                                                                                                   
189.  30  y/o  lady  presents  with  productive  cough,  chest  pain  and  fever  for  the  last  one  week.  O/E  
decreased  air  entry  on  the  right  side,  presence  of  bronchial  breathing.  X-­‐ray  revealed  a  wide  
opacity  on  the  right  side.  Culture  was  methicllin-­‐sensetive,  which  of  the  following  is  the  most  
suitable  drug  for  her?                                                                                  
A)  Amoxcicillin.  
 B)  Cloxacillin.  
C)  Pipercillin.  
 D)  Penicillin  G.  
ANSWER:  https://emedicine.medscape.com/article/234240-­‐overview  
   
190.  Patient  has  lung  infection  taking  antibiotic  then  patient  develop  flushing  in  face  and  hand  
then  came  to  doctor  culture  show  methicillin  resistant  gram  +  in  cluster  
Which  antibiotic  he  took?  
A.        Vanco  
B.        Cephalo                  
ANSWER:  A  
   
191.  Human  source  infection?  
A)  salmonella
  
B)  E.  Histolytica

  
NOT  COMPLETE  
   
192.  Most  specific  for  AIDS?  
A)night  sweat
  
B)  generalized  LAP  
C)  opportunistic  infections

  
ANSWER:  B  
   
193.  Patient  with  hx  of  tick  !  classic"  bull's-­‐eye  rash:  Lyme  yme  disease  also  called  erythema  
migrans  …  NOT  COMPLETE  
Lyme  borreliosis,  is  an  infectious  disease  caused  by  bacteria  of  the  Borrelia  type  
 fever,  headache  and  feeling  tired.[1]  If  untreated,  symptoms  may  include  loss  of  the  ability  to  move  
one  or  both  sides  of  the  face,  joint  pains,  severe  headaches  with  neck  stiffness,  or  heart  palpitation.  
Tx:  early  :  erythromycin,  advanced(  CNS,arthritis)  ceftriaxone  .  
                                                                   
                                                                                           

Allergy  and  immunology  


   
 
1.  What  drug  causes  angioedema  as  a  side  effect?  
A-­‐  CCB  
B-­‐  ACEIs  
2.  Diarrhea  after  fruit  juice  
A-­‐  fructose  allergy  
Answer:  ACEI  
   
3.  Bloating  and  gases  after  eating  ice  cream  and  dairy  product.  Which  enzyme  is  deficient?  
A-­‐  Lactase  
B-­‐  Pancreatic  alpha  amylase  
C-­‐  Sacrase  
D-­‐  Isomaltose  
Answer:  Lactase  
   
4.  What  is  the  first  immune  responsible  agent  in  the  skin?  
A-­‐  Keratinocyte  
   
5.  Patient  with  tachycardia  and  SOB  after  bee  sting  what  is  the  optimal  treatment?  
A-­‐  Remove  stinger  and  watch  the  patient  
B-­‐  SC  epinephrine  and  IV  antihistamine  
Answer:  SC  epinephrine  and  IV  antihistamine  
   
6.  Patient  with  multiple  infections;  TB,  staph  ...  etc.  He  has  immune  defect  in?  
A-­‐  NADPH  oxidase  
CGD  
   
7.  The  transmission  of  maternal  antibodies  to  the  fetus  in  pregnancy  is  a  way  of?  
A-­‐  Active  artificial  Immunity
  
B-­‐  Passive  artificial  immunity  
C-­‐  Passive  natural  immunity  
D-­‐  Active  natural  immunity  
 Answer:  Passive  natural  immunity  
   
8.  Which  of  the  following  tends  to  cause  a  severe  disease  in  immunocompromised  individuals  
compared  to  immunocompetent?  
A-­‐  Cryptosporidium  
   
9.  Patient  travelled  to  Africa  where  TB  bovine  is  endemic  there,  what  prevent  him  from  receiving  
BCG  vaccine,  deficiency  in  what?  
A-­‐  IFN  gamma  
B-­‐  IFN  alpha  
C-­‐  IL  4  
D-­‐  IFN  beta  
 Answer:IFN  gamma  
   
10.  Female  patient  when  she  ate  fish  she  complained  of  mild  redness  in  limited  area  for  three  
hours  +  diarrhea  which  resolved  after  that,  what  she  has?  
A-­‐  Limited  anaphylaxis  
   
11.  What  is  the  biochemical  defect  in  x  linked  agammaglobulinemia?  
 Answer:  Mutation  in  Bruton’s  tyrosine  kinase  inhibitor  that  is  essential  for  B  cell  function  
   
12.  Women  had  insect  bite  then  developed  erythema  and  LN  enlargement  (pic  of  forearm  with  
linear  erythema).  Lab  value  =  increase  WBC.  What  the  diagnosis?  
 Answer:  Lymphangitis  
   
13.  Primary  immunodeficiency  that  expose  the  patient  to  recurrent  of  viral  and  mold  infections  
A-­‐  T  cell  
B-­‐  B  cell                                                                                                            
C-­‐  Complement  deficiency  
Answer:  T  cell    
   
14.  Old  patient  had  packed  RBC  transfusion  then  developed  fever,  chills,  irritation  at  transfusion  
site  once  it  started,  what  to  do?  
Answer:  Needs  more  details  but  if  nonhemolytic  febrile  reaction  
   
Management:  rule  out  hemolytic  reaction  or  infection  
■  if  temperature  <38ºC,  continue  with  transfusion  but  decrease  rate  and  give  antipyretics  
■  if  temperature  >38ºC,  stop  transfusion,  give  antipyretics  and  anti-­‐histamine    
   
15.  Lip  swelling  for  3  years  deficiency  in  which  enzyme?  
A-­‐  C1  esterase  inhibitor  
   
16.  Rhesus  autoimmunity  in  pregnancy.  What  type  of  hypersensitivity?  
A-­‐  Hypersensitivity  type  2  
   
17.  CD40L  defect  
A-­‐  Hyper  IgM  syndrome  
   
17.  A  question  about  x-­‐linked  immunodeficiency  
   
18.  Question  about  RAST  test  measure  ??                                                                                                                                                                              
A  radioallergosorbent  test  (RAST)  is  a  blood  test  using  radioimmunoassay  test  to  detect  specific  IgE  
antibodies,  to  determine  the  substances  a  subject  is  allergic  to.  This  is  different  from  a  skin  allergy  
test,  which  determines  allergy  by  the  reaction  of  a  person's  skin  to  different  substances.                
   
19.  What  is  the  immunoglobulin  responsible  in  HSP  disease?                                                              
A-­‐  IgG  
B-­‐  IgM  
C-­‐  IgA  
D-­‐  IgE  
Answer:  IgA  
   
20.    Allergic  to  dust,  what  is  the  test  to  confirm  asthma?                                                        
A-­‐  esinophile  something  
 Answer:  Skin  prick  test  or  Specific  IgE  testing  
 
 
 

Endocrinology  
 
   
1.Clear  Case  with  lab  results  show:  
Low  K  ,  hight  blood  preusser  (Case  of  hyperaldosteronism).  How  to  diagnose?  
A.    Renin-­‐  aldosterone  ratio  
B.    17hydroxyprogesterone  
Answer:  A  
 https://emedicine.medscape.com/article/920713-­‐workup  
   
2.Diabetic  patient,  allergic  to  sulpha  drugs,  on  metformin  and  acarbose.  Not  well  controlled,  
what  to  add!?  
A.     Novoptions  
B.     Gliptazine  
Answer:  B  
   
3.Diabetic  and  pregnant,  hba1c  control  should  be  less  than:  
A.  4  
B.  5  
C.  6  
D.  7  
 Answer:  C  
The  NICE  guidelines  for  Diabetes  in  Pregnancy  (Clinical  Guideline  63)  state  that  women  with  
diabetes  should  aim  to  achieve  an  HbA1c  result  of  43  mmol/mol  (6.1%)  or  lower.  
   
4.  scenario  of  patient  with  diabetes  and  asked  about  how  does  diabetes  affect  on  wound  
healing?  
Answer:  limit  phagocytosis  
   
5.-­‐Diabetic  pt  complain  of  (something  in  the  back)  with  multiple  discharging  sinuses  Dx:  
1.  Infected  lymphoma  
2.  Lymphangitis  
3.  Carbuncle  
4.  Furuncle  
  Answer:  C  
   
6.CASE  scenario  about  diabetic  patient  diagnosed  recently  since  6  months.  Which  one  of  the  
following  renal  investigation  we  should  do  yearly:  
A-­‐serum  creatinine  
B-­‐microalbuminuria  
C-­‐creatinine  albumin  ratio  
D-­‐24h  urine  protein  collection  
 Answer:  B  
   
   
7.Contraindicated  hypoglycemic  drug  in  diabetic  patient  with  heart  disease  
A.    pioglitazone  
B.    sitagliptin  
Answer:  A  
 https://reference.medscape.com/drug/actos-­‐pioglitazone-­‐342726#5  
   
8.Diabetic  patient  with  lower  limb  paresthesia,  what  to  give  
Answer:  Pregabalin  
   
9.What  is  the  best  diagnostic  test  for  diabetic  nephropathy  
a)Urine  dipstic  
b)Kidney  function  test  
c)Albumin/creatinine  spot  ratio.  
d)24h  urinary  protiens.  
Answer:  C  
   
10.      A  child  with  growth  retardation,  obesity  and  Stria.  What  investigation  would  u  choose?  
1)  MRI  brain  
2)  adrenal  scintigraphy  
3)  MRI  adrenals  
4)  morning  and  evening  cortisol  level  
Answer:  D  
First  you  do  biochemistry  and  confirm  the  presence  of  Cushing  before  proceeding  to  imaging  
   
11.    Pregnant  on  thyroid  medication,  how  much  should  you  increase  the  dose  
A.     20%  
B.     30%  
C.     40%  
D.   50%  
 Answer:  D  
   
12.    -­‐Obese  female,  regular  period  had  acne,  other  examination  normal  what  is  the  best  
investigation?  
Answer:  ACTH  
Cushing  syndrome:  The  right  choice  is  to  check  24h  urinary  free  cortisol  level  
Or  low  dexamethasone  suppression  test.  
ACTH  is  not  used  till  later  on.  
   
13.     Pt  with  muscle  weakness,  apart  from  hypotension  and  bradycardia,  his  examination  was  
normal.  His  lab  tests  (high  K+,  low  NA,  low  Chloride,  high  urea)  
What  is  the  etiology  behind  his  presentation?  
A.hyponatremia  
B.hyperkalemia  
C.low  chloride  
D.uremia  
Answer:  B  
   
14.    Target  HbA1c  of  DM2  
Answer:  Less  than  7%  
 http://guidelines.diabetes.ca/executivesummary/ch8  
   
15.    Patient  with  high  HgA1c  what  to  order  also?  
A.Fasting  blood  sugar  
B.CBC  
C.TSH  
Answer:  A  
   
16.    Case  of  Elderly  DM  patient  on  metformin,  did  not  eat  well  for  the  past  5  days.  He  is  
dehydrated,  Investigation  revealed  the  following:  
•  Very  high  Random  blood  glucose  levels  
•  Urine  (–ve  for  ketones,  +ve  for  Glucose).  Normal  renal  function.  Your  Assessment:  
A.  Hyperosmolar  Hyperglycemia  
B.  lactic  acidosis  
C.  Diabetic  Ketoacidosis  (DKA)  
Answer:  A  
Hyperosmolar  Hyperglycemic  Non-­‐ketotic  Syndrome  (HHNKS)  :  a  metabolic  complication  of  
DM,  especially  in  elderly  and  in  type  2  DM,    characterized  by:  
1.  Hyperglycemia  
2.  Extreme  dehydration  
3.  Hyperosmolar  plasma  
4.  Altered  consciousness  (late  sign)  
Precipitating  factors  for  this  condition  (HHNKS):  
•  Acute  infection  
•  Non-­‐adherence  to  Diabetes  Meds.  
https://www.merckmanuals.com/professional/endocrine-­‐and-­‐metabolic-­‐disorders/diabetes-­‐
mellitus-­‐and-­‐disorders-­‐of-­‐carbohydrate-­‐metabolism/nonketotic-­‐hyperosmolar-­‐syndrome-­‐nkhs  
   
17.    Metabolic  syndromes  
Diabetic  drugs  
 ??  
   
18.    The  nature  history  of  diabetic  nephropathy  (picture  provided)  when  the  changes  occur  or  
something  like  that  
A.    10  years  
B.    15  years  
C.    20  years  
D.  25years  
I  searched  and  asked  some  endocrinologist,  none  gave  me  an  answer.  
But  mostly  it's  either  15  or  20  years  
   
19.    -­‐To  differentiate  between  type  1  and  2  DM  
Answer:  Endogenous  insulin  
Or  C-­‐peptide  
   
20.    Old  patient  came  with  Na  of  123  and  serum  osmolality  of  223  ,  K  was  normal.  Urine  
osmolality  was  800.  (No  scenario  or  cause  given)  
The  probable  diagnosis  is:  
1-­‐  conns  disease  
2-­‐  addisons  
3-­‐  SIADH  
4-­‐  Cushing  
Answer:  SIADH  
In  SIADH  the  serum  osmolality  is  less  than  275  mosm  and  urine  osmolality  is  more  than  300  
mosm.  
In  conns  there's  hypernatremia  and  hypokalemia,  in  Addison  the  opposite  
https://www.slideshare.net/mobile/mhdsoud/di-­‐siadh-­‐and-­‐cerebral-­‐salt-­‐wasting-­‐syndrome  
   
21.    -­‐Type  of  the  insulin  in  DKA  
Answer:  Regular  
   
22.    Middle  aged  male  with  infertility  and  decrease  libido  and  on  examination  bilateral  
testicular  atrophy,  decrease  hair  and  musculature?  What  would  you  do  next?  (Testicular  
biopsy  or  MRI)  
 Couldn’t  find  an  answer  
   
23.    Male  student  in  high  school,  athletic,  presented  to  the  primary  clinic  for  check  up,  
increase  musculature  with  multiple  acnes  what  you  are  going  to  order  (anabolic  urine  test)  
 Answer:  Anabolic  urine  test  
   
24.    Patient  with  dm  ,  which  one  of  the  following  is  contraindicated:  
1.   losartan  
2.   Nifedipine  
3.   Hydrazine  
 Answer:  C  
   
26.    Pt  known  case  of  asthma,  his  wife  has  osteoporosis,  he  has  renal  stone  ,  he  eats  
multivitamin,  vit  D  and  calcium  .  
PTH  :  high  
What  is  the  dx?  
Answer:  Primary  Hyperparathyroidism  
https://emedicine.medscape.com/article/127351-­‐overview#a3  
   
28.    Patient  with  thyroid  storm  symptoms.  What  is  the  first  thing  to  give  the  patient?  
Answer:  Propranolol  
   
29.    Clear  case  of  hyperthyroidism.  What  is  the  most  beneficial  in  the  diagnosis?  
A.    Thyroid  scan  
B.    FNA  
C.    US  
 Once  the  diagnosis  of  hyperthyroidism  has  been  established,  the  cause  of  the  hyperthyroidism  should  be  
determined.  Graves'  hyperthyroidism  may  be  clinically  obvious  on  the  basis  of  clinical  findings  such  as  
diffuse  goiter  and  ophthalmopathy.  However,  a  24-­‐hour  thyroid  radioiodine  uptake  and  scan  are  
frequently  necessary  to  confirm  the  diagnosis  of  Graves'  hyperthyroidism  and  exclude  other  causes,  
especially  painless  thyroiditis.  
 Answer:  A  
   
30.    Smoker,  alcoholic  has  thyroid  nodule.  What  is  the  first  thing  to  do?  
Answer:  Needle  aspiration  
   
31.    Case  scenario  about  patient  with  hypertention,  labs  result  hypernatremia  hypokalemia  
and  asking  about  cell  responsible  for  that?  
-­‐              fasiculata  
-­‐              glumerulosa  
-­‐              reticularis  
Answer:  B  
   
32.    picture  of  vial  of  varl  rix  vaccine  asking  this  vaccine  for  what  
a)varicella  
b)hepatitis  b  
c)influenza  
Answer:  A  
   
33.    Case  about  DKA  ,worst  complication  ?  
Answer:  cerebral  edema  
   
34.    Water  with  low  iodine,cheer-­‐man  wants  to  treat  children  suspected  to  have  cretinism  ?  
A.Add  iodine  
B.Levothyroxine  
Answer:  B  
   
35.     ophthalmology  ex  for  DM2  patient  every  
A.  6  months  
B.  12  months  
C.  24  months  
D.  36  Months  
 Answer:  B  
   
36.    A  pt  with  Lab  findings  of  hypothyroidism,  he  has  hoarseness  and  a  large  neck  swelling  
OE  lt  lobe  is  swollen  and  is  larger  than  the  right  one  
Best  Mx  is:  
A.  FNA  
B.  Thyroid  lobectomy  
C.  Radio  isotope  scan  
 Answer:  A  
   
37.    Thyroid  mass  with  intracellular  amyloid  deposit  
1.   Medalley  
2.   Papillary  
3.   Follicular  
4.   Non-­‐hogken  
Answer:  A  
 https://oup.silverchair-­‐
cdn.com/oup/backfile/Content_public/Journal/ajcp/53/5/10.1093/ajcp/53.5.592/2/ajcpath53-­‐
0592.pdf?Expires=1509461530&Signature=Q2aCjU3tMI~IQFuoS~g67hCsnQZwVGnarLrX9s4-­‐
MEg6okBIwgGmFkccs5295SJxodv0csYSYLvJ82kIkkZnHXSMfYPPNq4SiG0GAB-­‐QkHBuXgacCsX-­‐
PcMoeOjjysophE~Qdt96UvEQLFeuSCQ4sXUpPbYc2wmWXgWT0rLbx6nyyVitu4Y7g5f3V2Zvn6G0
V1rvJ75T7J-­‐dgsyQfYQLj~wWCTFCeLwHbldDaZhdWQ-­‐05L5s-­‐3e~xWOhuevmkpPSDB04PZm0Gtd-­‐
1Mqe66QsJVmLyzwVadiRiKzVjafVsGzMbZ8cp61l2DdZyvgdhscjo-­‐jTiVCE0Ee1TF-­‐ztT~DuA__&Key-­‐
Pair-­‐Id=APKAIUCZBIA4LVPAVW3Q  
   
39.    Question  about  baby  2  weeks  old  who  is  not  active  and  mother  happy  he  is  not  cry  asking  
about  investigations  
A.    -­‐t3  level  
B.    Total.  T4  
C.    free  t  4  
D.  mother  t  level  
 Answer:  C  
   
40.    In  a  village  where  the  incidence  of  cretinism  and  iodine  was  less  than  1  microgram  the  
health  promoters  want  to  issue  a  director  for  the  for  the  management  of  those  with  
cretinism..  what  is  the  best  initial  management?  
A.    TSH  and  t4  mesruments  
B.    Start  thyroxine  medication  
C.    Iodine  supplementation  
Answer:  B  
The  mainstay  in  the  treatment  of  congenital  hypothyroidism  is  early  diagnosis  and  thyroid  
hormone  replacement.  Optimal  care  may  includes  diagnosis  before  age  10-­‐13  days  and  
normalization  of  thyroid  hormone  blood  levels  by  age  3  weeks.  
Only  levothyroxine  is  recommended  for  treatment  
http://emedicine.medscape.com/article/919758-­‐treatment#d6  
   
41.    Thyrotoxicosis  crisis,  initial  treatment?  
Answer:  B  blocker  
   
42.    Hypoparathyrodism  ,he  will  have  ?  
Answer:  Convulsions  
Tetany,  seizures,  QT  prolongation,  twitching  (Chvostek  sign),  spasm  (Trousseau  sign)  
   
43.    Propylthiouracil  (  anti  thyroid)  how  it  works?  
https://www.ncbi.nlm.nih.gov/pubmed/23883148  
   
44.    -­‐  Anti  thyroid  causing  pancytopenia:  
Answer:  Methimazole  
This  condition  is  extremely  serious  but  affects  only  one  out  of  every  200  to  500  people  who  
take  an  antithyroid  drug.  Older  people  taking  propylthiouracil  and  those  who  take  high  doses  of  
methimazole  may  be  at  higher  risk  of  this  side  effect.  
   
45.    -­‐  Antithyroid  used  in  pregnancy:  
Answer:  Propylthiouracil  
 Propylthiouracil  is  the  drug  of  choice  during  the  first  trimester  of  pregnancy  because  it  causes  
less  severe  birth  defects  than  methimazole.  Because  there  have  been  rare  cases  of  liver  
damage  in  people  taking  propylthiouracil,  some  clinicians  will  suggest  switching  to  methimazole  
after  the  first  trimester,  while  others  may  continue  propylthiouracil.  
   
46.    Cold  intolerance,  wt  gain  >  check  thyroid  Us  
The  right  answer  should  be  to  measure  TSH  and  free  T4  
   
47.    cohns  and  Addison  and  Cushing    basically  secondary  hypertension   Labs  and  
investigations  (3qs)  
   
48.    MOA  of  metformin  in  PCOS:  
Answer:  reduces  insulin  resistance.  
   
49.  Diabetic  on  sulfa  and  metformin  came  for  regular  check  up,  his  HBA1C  is  9.2  what  is  your  
management  ?  
-­‐  regular  insulin  
-­‐  aspart  
-­‐  pioglitazone\  Glitazone  
Answer:  C  
   
50.  A  45  year-­‐old  man,  who  has  diabetes  and  has  been  on  metformin  and  glimepiride  for  the  
last  four  weeks,  presents  to  the  clinic  with  poor  glycemic  control.  A  history  also  confirms  poor  
dietary  management.  
Which  of  the  following  should  be  prescribed  now?  
A.  Acarbose  
B.  Repaglinide  
C.  Tolbutamide  
D.  Pioglitazone  
Answer:  A  
   
51.  Married  man,  type2  dm,  his  wife  is  unable  to  conceive,  o/e  testicular  atrophy,  reduced  
pubic  hair,  testosterone  and  LH  are  low
Prolactin  normal
Next  step  in  dx:  

A)  karyotyping
  
B)  testicular  us
  
C)  brain  MRI  
Answer:  C  
   
52.  Type  2  dm  woman,  full  bladder,  incomplete  emptying,  the  etiology  
A)  diabetic  neuro...  
B)  detrousal...  
Answer:  A  
   
53.    What  is  the  MOA  of  glipizide  ?  
Answer:  Increase  insulin  secretions  from  pancreas  
   
54.    -­‐  What  is  insulin  regimen  that  is  similar  to  normal  physiology?  
Answer:  ASPART  AND  GLARGINE  
   
55.    Many  Q  about  thyroid  in  medicine  I  don’t  remember,  review  what  you  should  do  if  
there’s  mass  and  about  hypo  &  hyperthyroidism,  there’s  investigation  in  the  Questions  
   
56.    Patients  have  D.M.  On  metformin  1  g  and  another  anti  dm,  present  with  increasing  
blood  glucose  at  morning,  what  you  will  give  :  
A.  NPH  
B.  Lispro  
C.  Regular  
 Answer:  A  
   
57.    pt  with  hyperthyroidism,  with  palpitation  ,  what  is  the  most  likely  finding  in  his  ECG?  
1.   SVT  
2.   VT  
3.   A  FIB  
Answer:  C  
   
58.    Pt  k/c  of  hypothyroidism  ,  use  Levothyroxin  175  mg  ,  switch  to  200  mg  .  
Labs  :    Tsh  :  high.        T4  :  normal.    What  is  the  cause  ?  
A.    ectopic  thyroid  
B.    Primary  hypo  
C.    Secondary  hypo  
D.  Small  dose  .  
Answer:  D  
   
60.    How  hyperglycemia  affect  wound  healing:  
A.    dec  imuune  system  
B.    Dec  phagocytosis  
C.    Stimulates  bacteria  growth  
Answer:  B  
   
61.  case  of  hyperthyroidism  what  will  be  associated:  
A)AFib  
B)  VT  
C)  SVT  
D)  WPW  
 Answer:  A  
   
62.    Multi-­‐nodular  goiter  treatment:  
A.    1-­‐surgery  
B.    2-­‐radioidonie  
C.    3-­‐beta-­‐blockers  
D.  4-­‐antithyriod  
 Answer:  A  
   
63.    Case  of  hypothroidism  lipid  screening  after  
a.6  months  
b.1year  
c.3years  
d.5years  
 Answer:  A  
   
64.    GLipizide  mode  of  action?  
1.   increases  insulin  release  from  the  pancreas  
2.   Increase  peripheral  cells  sensitivity  to  insulin  
 Answer:  A  
   
65.    A  known  case  of  Addison  disease  presented  with  weakness,  cold  and  clammy  skin,  
dizziness  and  weak  pulses.  Treatment?  
1.   0.9%  N/S  over  1hour+hydrocortisone  IV  
2.   0.9%  N/S  over  8hours+hydrocoriesone  IV  
3.   0.9%  N/S  over  1  hour  +  fludrocortisone  orally  
4.   0.9%  N/S  over  8hrs  +fludrocortisone  orally  
 Answer:  A  
https://emedicine.medscape.com/article/765753-­‐
treatment?pa=Rev1p9JVxdupXJ3%2FGoHsV3tvHK9mpdTiE1lxb0CSsSQ8YUkKorinsY0X%2F6L3clY
qX8MwC0EECwzp432Skuf9qw%3D%3D  
   
67.    8  year  with  T1DM  dx  before  1  month  when  to  start  ophtha  screening?  
A)    After  3  years  
B)    After  5  years  
 Answer:  B  
   
68.    Patient  is  diabetic  on  glyburide  she  gained  7kgs  in  a  year.  HBA1C  is  6.6  what  is  your  
management?  
A)    Add  metformin  
b)  Stop  glyburide  and  start  her  on  metformin  
 Answer:  B  
   
69.    23  year  old  man  is  complaining  of  polydepsia  and  polyurea  and  he  is  worry  and  afraid  
about  getting  dm  what  is  the  best  value  for  diagnosis  ?  
1-­‐  7.7  
2-­‐8  
3-­‐  9  
4-­‐  12  
No  enough  information  
NOTE:  
Table  2.1—Criteria  for  the  diagnosis  of  diabetes
FPG  126  mg/dL  (7.0  mmol/L).  Fasting  is  
defined  as  no  caloric  intake  for  at  least  8  h.*  
OR  
2-­‐h  PG  200  mg/dL  (11.1  mmol/L)  during  an  OGTT.  The  test  should  be  performed  as  described  by  
the  WHO,  using  a  glucose  load  containing  the  equivalent  of  75  g  anhydrous  glucose  dissolved  in  
water.*  
OR  
A1C  6.5%  (48  mmol/mol).  The  test  should  be  performed  in  a  laboratory  using  a  method  that  is  
NGSP  certified  and  standardized  to  the  DCCT  assay.*  
OR  
In  a  patient  with  classic  symptoms  of  hyperglycemia  or  hyperglycemic  crisis,  a  random  plasma  
glucose  200  mg/dL  (11.1  mmol/L).  
*In  the  absence  of  unequivocal  hyperglycemia,  results  should  be  confirmed  by  repeat  testing.  
reference:  American  diabetes  association  2016  
   
Q.    Scenario  about  female  with  hypothyroidism  and  take  levothyroxine  for  something  
duration,  she  is  asymptomatic  now.  
The  lab  result  :  T4  within  range  ,  TSH  still  high  1  above  normal  range  ,  what  will  you  do:  
-­‐              continue  with  the  same  dose  
-­‐              increase  dose  by  ….  
-­‐              Increase  dose  by  …  
-­‐              Increase  dose  by  …  
Answer:  Increase  dose  by  25  mcg  
   
Q.  Hypothyroidism  pt  after  2  weeks  of  treatment  lab  show  normal  t4  +  high  tsh:  
Same  dose  &  f\u  1-­‐2  months  
Decrese  dose  &  f\u  1-­‐2  months  
Decrease  dose  &  f\u  1-­‐2  weeks  
Answer:  continue  same  dose  
T4  takes  one  week  only,  while  TSH  needs  2  months  to  stabilize  
   
70.    Lady  who  is  taking  thyroxin  75  mg.  She  stopped  taking  the  drug  3  days  back  and  came  
now  for  refill  of  the  drug.  Labs  show  normal  T4  and  high  TSH  (85).  What  is  the  dose  you’re  
going  to  give  her?  
A.   50  
B.   75  
C.   100  
D.  150  
Answer:  C  
   
71.    Side  effect  of  hyperthyroidism  drugs  à  choices  where  medication  
   
72.    most  reliable  marker  for  diagnosing  hypothyroidism;  
1.   -­‐  Serum  T4  
2.   -­‐  Total  T4  
3.   -­‐  TSH  
4.   -­‐  T3  
 Answer:  C  
   
73.    Case  of  hyperthyroidism  what  will  be  associated:  
A)AF  
B)  VT  
C)  SVT  
D)  WPW  
 Answer:  A  
   
74.    Scenario  of  Cushing  syndrome  without  the  diagnosis  asking  which  one  will  be  present?  
Telangiectasia  
In  some  rare  cases  you  find  telangiectasia  
   
75.    Incretin  mode  of  action?  
Answer:  Increase  insulin  secretion  
   
77.    HbA1c  target  for  dm2  àless  than  7  
   
78.    Female  trying  to  lose  weight  but  she  couldn’t,  you  ordered  thyroid  panel  and  it  came  out:  
TSH:  high  
T4  normal.  What’s  the  most  likely  diagnosis?  
1.   hyperthyroidism  
2.   Subclinical  hypothyroidism  
Answer:  B  
   
79.    Patient  with  hyperthyroidism.  Which  drug  more  rapid  action?  
1.   Propylthiouracil  
  B.  radioactive  iodine
Answer:  A  
   
80.    What  is  the  mechanism  of  insulin  resistance  in  obesity?  
https://www.ncbi.nlm.nih.gov/pmc/articles/PMC3936017/  
   
81.  Patient  with  hyperthyroidism  on  medication.  Came  with  red  eye.  What  is  the  medication  
responsible  for  that?  
 Iodine  can  cause  exacerbation  of  eye  disease.  
That's  the  only  explanation  for  red  eye  
   
82.    Diabetic  on  Metformin.  She  got  pregnant,  what  will  you  do?  
Answer:  Shift  her  on  insulin.  
   
83.    Patient  is  having  seizure  episodes  since  3  days.  Blood  osmolality  is  low.  Urine  osmolality  
is  normal.  What  is  the  most  likely  cause?  
A.    Cons  syndrome  
B.    Addison  syndrome  
C.    Cushing  
D.  Inappropriate  ADH  secretion  
Answer:  D  
   
Patient  came  to  PHC  complaining  of  fatigue  and  polyuria  ,  investigation  :  RBS  450  ,  positive  
  ketone  body  ,  what  is  the  predominant  type  of  ketone  body  is  present  in  the  urine?
  Answer:  Acetoacetate
   
85.  32  female  c/o  breast  milk  discharge  and  irregular  menses,  ask  about  investigation:  
A.  TSH  
B.  brain  MRI                                                                                                                                                        
C.  biopsy                                                                                              
Answer  :  TSH  (Uptodate  mentions  TSH  as  first  initial  test  before  brain  MRI!)  
   
86.  Amenorrhea,  low  prolactin,  what  to  see  in  CT  or  MRI:                                                                                            
Answer  :  EMPTY  SELLA  TRUCICA  
   
87.  Mother  developed  gestational  dm  in  pregnancy  then  come  after  giving  birth  at  6  month  
the  HA1C  is  6.9  what  is  the  managment  ?  
1.   insulin  
2.   metformin  
3.   sulfenourea  
4.   pigletazone  
Answer:  B  
   
88.  Long  scenario  for  30  years  female  patient  has  manifestation  of  hypothyroidism  
(tirdness.....)  Lab  and  vital  sign  were  provided  and  it  shows,  low  T4,  low  24hours  urine  
cortison,  low  testesteron  ,  low  somatomedin  ,  low  IGF  .  Beside  thyroid  replacement  what  you  
will  give:  
1.   GH  
2.   Steroid  
Answer:  B  
   
89.  Female  34  years  old  presented  with  breast  milk  discharge  and  9  period  a  year,  which  of  
the  following  most  likely  she  had:  
 Answer:  prolactinoma  
   
90.  Case  of  women  with  hyperprolactenemia  and  they  found  pituitary  adenoma  .7  cm  what  is  
the  management  :  
1.   Cabergoline  
2.   bromocroptine  
3.   surgery  
Answer:  A  
   
91.  Case  of  SIADH  ,  they  wrote  lab  results  and  they  want  the  diagnosis:  hyponatremia  and  
hypo-­‐osmolality  ,  urine  osmalirty  hhigh  
   
92.  Diabetic  patient  on  NPH  and  regular  insulin  twice  before  breakfast  and  dinner  ,  glucose  
was  high  at  4pm  ,  how  to  manage  :  
1.   Increase  NPH  before  breakfast  
2.   Increase  regular  before  breakfast  
3.   Increase  NPH  before  dinner  
4.   Increase  regular  before  dinner  
Answer:  A  
   
93.  Patient  with  dm  ,  which  one  of  the  following  is  contraindicated:  
 a)  losartan  
b)  Nifedipine  
c)  Hydrazine  sulfate  
Answer:  C  
   
   
 

Pulmonology  
   
1.  Acute  asthma  in  ER  take  SABA?  Next?  
A.  Theophylline  
B.  Inhaler  ipratropium  
Answer:  B  
 Reference:  http://cursoenarm.net/UPTODATE/contents/mobipreview.htm?19/39/20081  
   
2.  Obstructive  Sleep  Apnea  breathing?  
Answer:  Cheyne–Stokes  respiration.  
 Reference:  https://en.wikipedia.org/wiki/Cheyne%E2%80%93Stokes_respiration  
   
3.  Lung  infiltration,  decrease  air  entry,  greenish  sputum  foul  smell.  
A.  Bronchiolitis  
B.  Bronchiectasis    
Answer:  B  
 Reference:  
https://www.nhs.uk/Conditions/Bronchiectasis/Pages/Symptoms.aspx  
https://www.emedicinehealth.com/bronchiectasis/page6_em.htm  
 
4.  Anti-­‐cancer  drug  causes  reactions,  skin  changes  and  lung  fibrosis:  
A.  Bleomycin  
B.  Cisplatin  
C.  Methotrexate  
Answer:  A    
 
Both  medication  can  cause  
all  of  these  side  effects  but  
bleomycin  more  common  
Reference:  
https://www.drugs.com/sfx/
bleomycin-­‐side-­‐effects.html  
https://www.drugs.com/sfx/
methotrexate-­‐side-­‐
effects.html  
   
   
5.  Patient  complaining  of  
face  flushing  and  head  
heaviness  in  the  morning  
that's  relieved  during  the  
day,  CXR  shows  bilateral  
hilar  mass,  he  is  smoker  for  
30  years?  
A.  Hodgkin's  lymphoma  
B.  Lung  cancer  
C.  COPD  
Answer:  B  
Symptoms  of  Superior  Vena  Cava  Syndrome  
Reference:  https://www.cancer.net/navigating-­‐cancer-­‐care/side-­‐effects/superior-­‐vena-­‐cava-­‐
syndrome  
   
6.  Old  patient  with  facial  swelling  when  waking  up  &  decrease  during  the  day,  he  is  smoker.  What  
is  the  diagnosis?  
A.  COPD  
B.  TB  
C.  Lung  cancer  
Answer:  C  
Symptoms  of  Superior  Vena  Cava  Syndrome  
Reference:  https://www.cancer.net/navigating-­‐cancer-­‐care/side-­‐effects/superior-­‐vena-­‐cava-­‐
syndrome  
   
7.  Patient  exposures  to  asbestos  what  will  present  in  chest  x  ray:  
A.  Hyperinflation  
B.  Bilateral  lungs  fibrosis  
C.  Interstitial  lung  disease  
 Answer:  B  
Hyperinflation  may  also  be  present,  so  could  be  A  as  well  
Reference:  
For  A:  http://www.clinicaladvisor.com/pulmonary-­‐medicine/asbestos-­‐related-­‐pulmonary-­‐
disorders/article/625653/  
For  B:  https://www.asbestos.com/asbestosis/diagnosis/  
   
8.  1  year,  fine  crackles,  ?  
All  choices  were  not  related  looks  look  bronchopneumonia  
   
9.    Female  with  history  of  pulmonary  embolism,  3  abortion.  Have  positive  lupus:  
I  chose  Antiphospholipid  
   
10.    Man  works  in  the  new  office  and  he  experienced  cough  and  wheezing  and  respiratory  
symptoms  I  don't  remember  exactly.  Otherwise  he  is  fine  and  the  condition  return  once  he  back  to  
this  office:  
A.  Asthma  
B.  Allergic  pneumonitis  
 Answer:  B  
 Reference:  https://medlineplus.gov/ency/article/000109.htm  
   
   
11.    Which  of  the  following  is  the  typical  picture  of  primary  TB:  
A.  Caseation  of  lymph  node  
B.  Cavitation  in  lung  
C.  Can’t  remember  other  choices  
 Answer:  A  
 Cavitation  is  uncommon  in  primary  TB,  seen  only  in  10-­‐30%  of  cases  
Reference:  https://www.ncbi.nlm.nih.gov/pmc/articles/PMC3215852/  
   
12.    Lung  abscess,  best  antibiotics?  
A.  Cipro  
B.  Macrolides  
C.  Third  generation  
D.  Piperacillin  
 Answer:  A  
 Standard  therapy  for  lung  abscess  with  anaerobic  bacteria  is  clindamycin  but  not  in  the  choices  
Reference:  https://emedicine.medscape.com/article/299425-­‐medication#2  
   
13.    Treatment  for  bilateral  pneumonia  
A.  Ampicillin  
B.  Ciprofloxacin  
C.  Vancomycin  
 Answer:  B  
 Depend  on  the  organism,  but  atypical  pneumonia  is  often  diffuse  and  bilateral.  
The  first  line  antibiotic  in  atypical  pneumonia  is  macrolide  or  doxycycline  but  both  are  not  in  the  
choices,  and  the  second  line  is  fluoroquinolone  
Reference:  
http://bestpractice.bmj.com/best-­‐practice/monograph/18/treatment/details.html  
https://emedicine.medscape.com/article/300157-­‐treatment#d6  
 
14.    Most  common  cough  in  adults  
Answer:  post  nasal  drip  
 Reference:  http://www.aafp.org/afp/2003/0101/p169a.html  
   
15.    Middle  age  man  with  acute  onset  of  fever  malaise  anorexia  x-­‐ray  show  bilateral  hilar  and  left  
paratracheal  lymphadenopathy  what  is  the  diagnosis:  
A.  HIV  
B.  sarcoidosis  
C.  TB  
D.  rheumatic  fever  
 Answer:  B    
The  most  frequent  radiological  abnormality  involves  enlarged  bilateral  hilar  and  right  paratracheal  
lymph  nodes  in  sarcoidosis  
Reference:  https://www.ncbi.nlm.nih.gov/pmc/articles/PMC3789928/  
   
16.    Smoker  with  CXR  result  of  calcification:  
A.  Squamous  cell  carcinoma  
B.  Adenocarcinoma  
Answer:  A  
 Most  types  of  lung  cancer  linked  to  smoking  cigarette  but  the  strongest  association  is  with  
squamous  cell  carcinoma  and  small  cell  carcinoma  
Reference:  Rubin's  Pathology:  Clinicopathologic  Foundations  of  Medicine  page  539,  5th  Edition  
   
   
17.    Student  with  history  of  sore  throat,  headache,  mild  cough,  low-­‐grade  fever,  chest  pain,  tired,  
general  malaise?  
A.  Pneumococci  
B.  Mycoplasma  
C.  Aspiration  pneumonia  
D.  Legionella  
Answer:  B  
Picture  of  atypical  pneumonia  symptoms.  Mycoplasma  and  Legionella  are  organisms  of  atypical  
pneumonia,  but  Legionella  common  has  GI  symptoms  with  diarrhea  
Reference:  http://cursoenarm.net/UPTODATE/contents/mobipreview.htm?39/18/40239  
   
18.    Drug  for  asthma  decreases  secretion  more  than  bronchodilation:  
Answer:  ipratropium  promide  
 
19.    28  years  pregnant  at  20  weeks  of  pregnancy.  Developed  dyspnea  and  resp.  distress  of  sudden  
onset,  what  is  the  diagnostic  test:  
A.  2  chest  x-­‐rays  
B.  Echocardiogram  
C.  CT  scan  
D.  V/Q  ventilation    
Answer:  D    
Because  of  the  superior  accuracy  of  V/Q  scanning  and  lower  maternal  breast  irradiation,  there  has  
been  a  movement  away  from  CT  angiography  toward  V/Q  scanning  as  the  diagnostic  test  of  choice  in  
the  workup  of  pulmonary  embolism  in  pregnancy  in  women  with  a  negative  CXR.  In  those  patients  
with  an  abnormal  CXR,  CT  angiography  remains  the  test  of  choice.  
Reference:  http://cursoenarm.net/UPTODATE/contents/mobipreview.htm?5/7/5233#H13  
   
20.    2  Q  about  squamous  cell  carcinoma  of  lung  
First  was  “x-­‐  ray“,  Second  I  don’t  remember  
   
21.    Exacerbation  of  asthma  >  inhaled  steroids  
   
22.    Old  Pt  have  recurrent  DVT  and  obstruction  SVC,  what  the  cause?  
Lung  carcinoma  
   
23.    Cancer  in  the  middle  of  esophagus,  which  type?  
A.  Adenocarcinoma  
B.  Scc  
 Answer:  B  
 Reference:  https://www.mayoclinic.org/diseases-­‐conditions/esophageal-­‐cancer/symptoms-­‐
causes/syc-­‐20356084  
     
24.    Old  male  present  with  mid  clavicle  mass  (smoker  for  20y  and  I  think  he  drinks  Alcohol)  What  is  
your  initial  management?  
A.  Core  biopsy  
B.  Fine  needle  biopsy  
C.  bronchoscopy  
   
Answer:  B  
 If  the  question  mean  thyroid  mass,  so  it  is  B  
Reference:  https://emedicine.medscape.com/article/127491-­‐overview#a4  
   
25.  40  years  old,  non-­‐smoker,  present  with  weight  loss  and  dyspnea,  he  works  in  electricity  
generation,  what  is  the  cause?  
A.  Asbestose  
B.  Inhaled  coal  dust  
C.  Passive  smoking  
Answer:  A  
Reference:  https://academic.oup.com/annweh/article-­‐
abstract/37/6/645/130162?redirectedFrom=PDF  
     
26.    Patient  with  obstructive  sleep  apnea  CPAP  not  effective  what  to  do?  
A.  Promethazine  
B.  Beclomethasone  
C.  Ipratropium  
D.  Modafinil  
 Answer:  D  
 Reference:  https://emedicine.medscape.com/article/295807-­‐medication  
   
27.    34  years  old  man  has  an  intermittent  epigastric  pain  for  3wks.  It  is  worse  by  food  but  helped  
by  some  tablets  he  obtained  from  the  pharmacy.  He  had  a  similar  episode  3yrs  ago  and  his  doctor  
gave  him  a  course  of  3  types  of  tablets  at  the  time.  What  is  the  most  appropriate  next  
investigation?  
A.  Abdomen  US  
B.  Barium  meal  
C.  Urea  breath  test  
 Answer:  C  
 Reference:  https://emedicine.medscape.com/article/176938-­‐workup  
   
28.    Lung  cancer  of  stage  1  small  cell  mixed  with  non  small  cell  and  asked  about  management?  
A.  Surgery  and  chemotherapy  
B.  Chemotherapy  and  radiation  
C.  Chemotherapy  plus  something  
 Answer:  A  
Both  of  Small  Cell  Lung  Cancer  &  Non-­‐Small  Cell  Lung  Cancer  the  standard  management  is  surgical  
resection  
Reference:  https://emedicine.medscape.com/article/279960-­‐treatment  
https://emedicine.medscape.com/article/280104-­‐treatment  
   
29.    Which  lung  cancer  causes  SIADH  syndrome?  
While  the  association  with  small  cell  lung  cancer  (SCLC)  is  well  known,  that  with  non  small  cell  lung  
cancer  (NSCLC)  has  been  rarely  reported  
 Reference:  http://www.sciencedirect.com/science/article/pii/S2213007117302381  
   
30.    Long  scenario  about  lung  cancer  on  pathology  (polygonal  cells)  and…?  
Answer:  squamous-­‐cell  lung  cancer  
   
Reference:  https://www.pathologystudent.com/?p=4616  
   
   
31.    Known  asthma  has  whitish  rash  easily  removed  in  mouth.  Which  antiasthmatic  cause  this?  
A.  Cromolyn  sodium  
B.  Betamethasone  
C.  Albutmerol  
 Answer:  B  
Reference:  https://www.nhs.uk/Conditions/Oral-­‐thrush-­‐-­‐-­‐adults/Pages/Introduction.aspx  
   
32.    COPD  exacerbation  by  Infection,  patient  has  fever  and  greenish  sputum,  what  is  the  most  
likely  microorganism?  (no  pseudomonas  in  the  answers)  
A.  staph.  Aureus  
B.  streptococcus  pneumonie  
C.  mycoplasma  pneumonia  
D.  homophiles  influenza  
 Answer:  D  
 Haemophilus  influenzae;  Moraxella  catarrhalis  more  common  in  pre-­‐existing  structural  lung  disease  
(CF,  bronchiectasis,  COPD)  and  the  elderly  
Pseudomonas,  Haemophilus,  and  pneumococcal  species:  May  produce  green  sputum
  
   
Reference:  Kumar  and  Medscape  
https://emedicine.medscape.com/article/300157-­‐overview  
     
33.    Old  asthmatic  patient  came  with  urinary  retention  diagnosed  with  BPH  and  he  was  found  to  
have  high  blood  pressure  180/110  (exactly  it  was  that  high)  what  to  give  now?  
A.  IV  Labetalol  
B.  propranolol  
C.  prazosin  
D.  something  ends  with  lol  ,  BBlocker  ?  
Answer:  C  
Reference:  http://cursoenarm.net/UPTODATE/contents/mobipreview.htm?19/60/20417#H5  
   
34.      Patient  with  recurrent  hemoptysis  since  3  years.  What’s  the  most  likely  cause?  
Missing  details  
A.   Bronchiactasis  
B.   Tb  
 Answer:  
   
35.  Treatment  of  bronchiolitis  
Answer:  Conservative  
   
Reference:  https://emedicine.medscape.com/article/961963-­‐treatment  
   
36.  Old  pt  in  ICU  with  pneumonia,  ask  about  organism:  
Answer:  Pseudomonas  
   
37.  What  is  the  best  investigation  for  determine  severity  of  asthma?  
A.  PEFR  
B.  ABG  
       Answer:  A  
   
Reference:  http://bestpractice.bmj.com/best-­‐practice/monograph/44/diagnosis/tests.html  
   
   
38.  Atypical  pneumonia  management?  
Azithromycin  is  the  drug  of  choice  for  children  with  suspected  or  confirmed  Legionella  disease  
   
39.  About  asthma  management  she  is  on  leukotrienes  since  6  month  then  come  with  every  day  
coughing  relieved  by  sulbutamol  in  ER.  What  you  will  give  her  as  maintenance?  
A.  Oral  steroid  and  SABB  on  needed                      
B.  Inhaled  2  doses  steroid  and  SABB  on  needed.  
C.  Leukotrienes  and  SABB  on  needed.  
D.  Long  acting  beta  blocker  and  SABB  on  needed.  
 Answer:  B  
Reference:  https://emedicine.medscape.com/article/296301-­‐guidelines#g3  
                                                                                             
40.  Asthmatic  exacerbation  came  to  the  ER  give  him  sulbutamol  the  relieved  and  the  PEF  70%  
what  you  will  do  next  for  maintenance?  
A.  Ipratraupium  
B.  Inhaled  steroid  
C.  Thyphelen  
   
Answer:  B  
Reference:  https://emedicine.medscape.com/article/296301-­‐guidelines#g3  
   
41.  Pt  inhaling  3  nanogram  of  Cotton  in  a  factory.  Eventually  the  cotton  will  end  up  
A.  Engulfed  by  alveolar  macrophages  
B.  Trapped  in  mucocilliary  system  without  reaching  alveoli  
C.  Pass  without  any  damage  
D.  trapped  in  distal  airway  leading  to  fibrosis  
                                                                   
Answer:  D  
Reference:  http://thorax.bmj.com/content/59/12/1095  
     
42.  Bronchospastic  symptoms  (eg,  wheeze  and  cough),  usually  responsive  to  bronchodilator  
therapy  
   
43.  Cavernous  hemangioma  +  pleural  effusion                                                                                                  
Answer:  Pleural  hemangioma  
 Reference:  https://www.ncbi.nlm.nih.gov/pmc/articles/PMC4821329/  
                   
44.  What  is  the  most  clinical  sign  associated  with  pulmonary  hypertension  
A.  Central  cyanosis  
B.  Pulmonary  edema                                                                  
C.  Stroke  
D.  Lower  limb  edema  and  ascites  
   
Answer:  D  
 Reference:  http://www.aafp.org/afp/2001/0501/p1789.html  
   
 45.  X-­‐ray  of  patient  with  pleural  effusion,  what  U  gonna  hear  during  auscultation:  
A.  Decrease  breath  sounds  
B.  Decrease  vocal  resonant  
C.  Bronchial  breath  sounds  
   
Answer:  A  
Reference:  
https://www.medicinenet.com/pleural_effusion_fluid_in_the_chest_or_on_lung/article.htm  
   
46.  Woman  had  +  pulmonary  TB  since  2  week  her  husband  come  for  tuberculin  test,  minimal  
induration  to  be  positive:  
A.  5  
B.  9  
C.  13  
D.  17  
Answer:  A  
Reference:  https://www.cdc.gov/tb/publications/factsheets/testing/skintesting.htm  
   
47.  Definition  of  cheyne  stokes  breathing?  
Abnormal  pattern  of  breathing  characterized  by  progressively  deeper  and  sometimes  faster  
breathing,  followed  by  a  gradual  decrease  that  results  in  a  temporary  stop  in  breathing  called  an  
apnea.  The  pattern  repeats,  with  each  cycle  usually  taking  30  seconds  to  2  minute  
   
48.  Patient  complaining  of  dry  cough  and  weight  loss.  Nodular  infiltrate  and  hepatomegaly.  
A.  Miliary  TB  
B.  Blastomycosis  
 Answer:  A  
 Reference:  http://cursoenarm.net/UPTODATE/contents/mobipreview.htm?33/53/34640  
   
 49.  Loss  of  sensation  over  shoulder  which  part  of  pleura  will  be  affected?  
A.  Mediastinal  
B.  Pleural  
C.  Vesceral  
Answer  :A  
   
Mediastinal  and  central  diaphragmatic  pleural  pain  referred  to  root  of  neck  and  over  shoulder  
(Dermatomes  C3-­‐C5)  
 Reference:  Lippincott's  Concise  Illustrated  Anatomy:  Thorax,  Abdomen  &  Pelvis    
“The  reference  about  referred  pain  to  the  shoulder  not  loss  of  sensation”  
   
   
                                                                           

Cardiology  
 
1.  Patient  presented  with  SOB.  On  examination  there  was  ejection  systolic  murmur  in  the  aortic  area  
radiating  to  neck.  What  is  the  pathophysiology  of  SOB?  
A-­‐  Pumonary  venous  congestion  
B-­‐  systemic  venous  congestion  
Answer:  Pulmonary  venous  congestion  
   
2.  Patient  with  mitral  stenosis  and  increased  pressure  in  the  left  atrium,  and  ECG  was  provided.  
What  is  the  complication?  
A-­‐  Right  ventricular  hypertrophy  
B-­‐  Pulmonary  HTN  
C-­‐  Dilation  of  the  left  atrium  
Answer:  Dilation  of  the  left  atrium  
   
3.  Patient  presented  with  palpitation.  Normal  ECG  &  examination.  What  is  the  treatment?  
A-­‐  Verapamil  
B-­‐  Propranolol  
C-­‐  Digoxin  
Answer:  Can  not  decide  based  on  messing  information.  
   
4.  Treatment  of  pericarditis  
A-­‐  Aspirin  
B-­‐  Corticosteroid  
C-­‐  Antiviral  
D-­‐  Antibiotics  
Answer:  Aspirin    
   
5.  ECG  pic  showing  anterior  MI.  Upon  history,  he  took  sildenafil  (viagra)  what  drug  is  contraindicated  
to  be  taken  in  this  case?  
A-­‐  Morphine  
B-­‐  Aspirin  
C-­‐  Nitrate  
 Answer:  Nitrate  
   
6.  Patient  complains  of  chest  pain,  describes  it  as  close  fist  crushing  his  sternum.  What  is  the  
diagnosis?  
A-­‐  Myocardial  ischemia  
B-­‐  Pulmonary  embolism  
C-­‐  Pericarditis  
Answer:  MI  
   
7.  Mid  diastolic  murmur  on  auscultation,  located  at  the  left  sternal  border.  What  is  the  diagnosis?  
A-­‐    Mitral  stenosis  
   
8.  Long  scenario,  positive  findings  were  radiofemoral  delay  and  on  CXR  there  shows  rib  notching.  
What  is  the  diagnosis?  
A-­‐  Coarctation  of  the  aorta  
   
9.  Murmur  that  changes  with  changing  the  position?  
A-­‐  Innocent  murmur  
   
10.  Patient  with  atrial  fibrillation  on  warfarin  12  mg  had  hematuria,  rectal  bleeding,  and  bruises.  His  
INR  is  6.2  what’s  next?  
A-­‐  Reduce  warfarin  
B-­‐  Give  whole  blood,  
C-­‐  Give  vitamin  k  
D-­‐  Replace  warfarin  with  aspirin  
 Answer:  Give  Vitamin  K  
   
11.  Patient  with  mitral  stenosis  (0.7  mm),  with  moderate  MR,  RVH  and  Afib,  what’s  the  
management?  
A-­‐  Ballon  
B-­‐  Valve  replacement  
C-­‐  Valve  commissurotomy  
D-­‐  Medical  treatment  
Answer:  Valve  replacement  
Patient  can  not  have  both  MR  and  MS  
However  looking  at  the  size  of  0.7  mm,  it  indicates  severe  stenosis.  
   
   
12.  Patient  with  aortic  stenosis,  what’s  the  management?  
 Answer:  
Asymptomatic:  serial  echos,  avoid  exertion  
Symptomatic:  avoid  nitrates/arterial  dilators  and  ACEI  in  severe  AS  
Surgery  if:  symptomatic  or  LV  dysfunction  
Surgical  Options  
Valve  replacement:  aortic  rheumatic  valve  disease  and  trileaflet  valve  
–  prior  to  pregnancy  (if  AS  significant)  
–  balloon  valvuloplasty  (in  very  young)  
Interventional  Options  
Percutaneous  valve  replacement  (transfemoral  or  transapical  approach)  
is  an  option  in  selected  patients  who  are  not  considered  good  candidates  for  surgery  
   
13.  Congestive  heart  failure  with  systolic  dysfunction,  what’s  the  treatment?  
 Answer:  if  Acute  -­‐>  lasix  ACEI  or  ARBs  and  Correcting  underlying  cause  
Chronic  -­‐>  B  blockers  ACEI  or  ARBs  diurteics  ie  lasix  or  spironolaxtin  
Aspirin  and  statins  if  Previous  MI  
   
   
14.  ECG  showing  ST  elevation  MI,  what  is  the  most  appropriate  management?  
A-­‐  LMWH  
B-­‐  Streptokinase  
C-­‐  Thrombolysis  
D-­‐  Stenting  
Answer:  depends  on  duration.  If  within  90  minutes  stenting  
   
15.  19  years  old  female  with  arthralgia  and  pansystolic  murmur  radiating  to  the  axilla.  ESR  is  high.  
Which  one  of  the  following  is  the  most  likely  diagnosis?  
A-­‐  SLE  
B-­‐  Miliary  TB  
C-­‐  Hodgkin's  lymphoma  
 Answer:  SLE  
   
16.  Severe  aortic  regurgitation.  Other  findings?  
A-­‐  Collapsing  pulse  
B-­‐  Low  raising  pulse  
C-­‐  Pulsus  paradoxus  
D-­‐  Pulsus  bisferiens  
Answer:  Pulsus  bisferiens  
   
17.  Right  bundle  branch  block?  
   
18.  Case  of  cardiac  patient,  he  started  to  develop  sign  and  symptoms  of  heart  failure                                                  
which  one  of  the  following  will  prolong  patient  life?  
A-­‐  ACEIs  
B-­‐digoxin  
C-­‐diltiazem  
D-­‐diuretic  
Answer:  ACEI  
   
19.  ECG  showing  inferior  MI,  which  one  of  the  following  arteries  blocked?  
A-­‐  Right  coronary  artery  
B-­‐  Left  coronary  artery  
C-­‐  Circumflex  
 Answer:  RCA  
   
   
20.  Patient  with  anterolateral  MI  (ECG  was  provided),  presented  within  45  minutes  and  is  going  for  
cardiac  catheterization.  He  was  given  aspirin  and  oxygen  at  the  ER.  What  are  you  going  to  give  him  
before  the  procedure?  
A-­‐  Morphine  and  nitroglycine  
B-­‐  Small  dose  of  beta  blocker  in  the  IV  infusion  
C-­‐  Normal  saline  with  pacemaker  
D-­‐  Dopamine  and  non-­‐epinephrine  
 Answer:  Morphine  and  nitroglycine  
   
21.  Systolic  ejection  murmur  in  aortic  area  radiating  to  the  neck,  and  the  patient  has  SOB,  what  is  
the  cause?  
A-­‐  Congestion  in  systemic  venous  
B-­‐  Pulmonary  congestion  
C-­‐  Pulmonary  stenosis  
 Answer:  Pulmonary  congestion  
   
22.  Patient  thin  and  tall  and  has  chest  deformity.  He  came  to  the  doctor  because  2  of  his  brothers  
died  at  a  young  age,  you  are  afraid  of  which  of  the  following?  
A-­‐  Hypertrophic  cardiomyopathy  
B-­‐  Aortic  root  aneurism  
Answer:  Aortic  root  aneurysm  (Marphan)  
   
23.  Case  of  chest  pain  since  1  hour.  What  enzyme  will  confirm  MI?  
A-­‐  Troponin  
B-­‐  CK  
C-­‐  Lactate  dehydrogenase  
D-­‐  Aspartate  dehydrogenase  
E-­‐  Myoglobin  
Answer:  Troponin  (confirm  MI  but  can  take  up  to  6  Hrs  to  rise)  
So  Myoglobin.  
   
24.  Which  of  the  following  breaks  the  thrombus  in  MI  
A-­‐  Warfarin  
B-­‐  Heparin  
C-­‐  Aspirin  
D-­‐  Streptokinase  
 Answer:  Streptokinase  
   
25.  Murmur  heard  best  in  the  2nd  right  left  sternal  border.  Which  valve  is  stenosed  ?  
A-­‐  Aortic  
B-­‐  Pulmonic  
C-­‐  Tricusped  
D-­‐  Mitral  
Answer:  if  right-­‐>  aortiv  
left-­‐>  pulmonary  
   
26.  Scenario  with  obese,  heavy  smoker  (20  cigarettes  daily).  Greatest  CVS  risk  factor?  
A-­‐  Age  
B-­‐  Smoking  
C-­‐  Obesity  
Answer:  Obesity  
   
27.Best  treatment  for  symptomatic  acute  rheumatic  fever?  
A-­‐  Penicillin  Q6  hours  
B-­‐  Acetylsalicylic  acid  
C-­‐  Steroids  
D-­‐  Single  dose  of  benzathine  penicillin  
Answer:    Single  dose  of  benzathine  penicillin  
   
28.  MI  treatment  
   
29.  Case  of  AF  long  term  prophylaxis  
A-­‐  Warfarin  
B-­‐  Beta  blocker  
 Answer:  Warfarin  
   
30.  Patient  came  to  ER,  retrosternal  chest  pain,  lasting  for  15  min,  relieved  by  rest,  not  compliant  on  
his  medications.  Vitally  stable,  mild  HTN.  What  is  best?  
A-­‐  ECG  stress  test  
B-­‐  Cardiac  enzymes  and  follow  up  in  clinic  
Answer:  none  of  the  provided  choices    (ECG  and  Cardiac  enzymes)  
   
31.  Patient  with  only  palpation,  examination  normal  and  ECG  shows  heart  block  what  is  the  
treatment  
A-­‐  CCB  
B-­‐  Beta  blocker  
C-­‐  Digoxin  
Answer:  depending  on  degree  of  block.  if  second  type  II  treatment  is  Pacemaker  
   
32.  Common  cardiac  anomaly  associated  with  infective  endocarditis?  
A-­‐  ASD  
B-­‐  VSD  
C  -­‐  TOF  
D-­‐  Truncus  arteriosus  
Answer:  TOF  
   
33.  Long  scenario  Of  patient  with  coronary  artery  disease,  what  is  the  appropriate  level  of  LDL  for  
him?  
A-­‐  3.5  -­‐  4.1  
B-­‐  2.4  -­‐  3.4  
Answer:  2.4  -­‐  3.4  
   
34.  Which  of  the  following  is  protective  from  heart  diseases?  
A-­‐  HDL  
B-­‐  LDL  
C-­‐  ILD  
D-­‐  HLDL  
Answer:  HDL  
   
35.  Which  one  of  the  following  has  more  risk  for  CAD?  
A-­‐  LDL  
B-­‐  HDL  
C-­‐  Triglycerides  
D-­‐  Total  cholesterol  
Answer:  LDL  
   
36.  Patient  with  mitral  regurgitation,  what  will  delay  the  need  for  surgery?  
A-­‐  Nifedipine  
B-­‐  ACEIs  
C-­‐  BB  
Answer:  ACEI  
   
37.  Man  with  aortic  stenosis  developed  syncope  what  is  the  cause?  
A-­‐  Systemic  hypotension  
   
38.  Coarctation  of  aorta  associated  with  which  of  the  following?  
A-­‐  Turner  syndrome  
   
39.  Patient  is  hypertensive  in  upper  extremities  and  absent  pulse  in  lower  extremities.  What  is  the  
diagnosis?  
A-­‐  Coarctation  of  aorta  
   
40.  ECG  showing  second  degree  heart  block,  and  asked  about  the  diagnosis  
   
41.  ECG  showing  MI  in  one  of  the  leads,  and  asking  about  the  location  of  the  infarction  
   
42.  Patient  with  heart  failure  on  ACEI,  furosemide,  spironolactone,  and  other  drugs.  Now  stable  and  
clinically  free,  what  drug  modification  is  appropriate?  
A-­‐  Stop  spironolactone  
   
50.  Best  treatment  plan  for  Infective  endocarditis?  
A-­‐  Multiple  Abx  high  dose  for  a  short  time  
B-­‐  Multiple  Abx  extended  over  a  long  period  of  time  
Answer:  Multiple  Abx  extended  over  a  long  period  of  time  
   
   
51.  Young  patient  with  BP  160/110,  high  creatinine,  ++  proteinuria.  What  is  the  drug  of  choice  for  
HTN?  
A-­‐  ACEI  
   
52.  VT  on  ECG  +  hypotensive.  What  is  the  management?  
A-­‐  Amiodarone  
B-­‐  Adenosine  
C-­‐  DC  shock  
Answer:  DC  shock  
   
53.  Man  with  chest  pain  and  transient  LOC  x4  times,  CXR  given.  What  is  the  management?  
   
   
54.  A  31  year-­‐old  female  T1DM  presented  to  ED  with  syncope  one  hour  before.  She  has  had  
shortness  of  breath  and  chest  pain.  
Vital  signs:  hypotension,  tachycardia.  
ECG  picture  attached  showing  V3-­‐V4  STE.  
What  is  the  diagnosis?  
A-­‐  Pulmonary  embolism  
B-­‐  Lateral  MI  
C-­‐  Wolf  parkinson  white  syndrome  
Answer:  Anterior  MI  
   
   
55.  Which  of  the  following  is  wide  complex  tachycardia?  
A-­‐  Afib  with  high  ventricular  rate  
B-­‐  Atrial  flutter  2:1  
C-­‐  Monomorphic  ventricular  tachycardia  
D-­‐  Paroxysmal  supraventricular  tachycardia  
Answer:  Monomorphic  ventricular  tachycardia  
   
56.  Obese  patient  with  chest  pain  on  exertion,  what  to  do?  
A-­‐  Treadmill  
B-­‐  Myocardial  perfusion  test  
C-­‐  Coronary  angiography  
 Answer:  Myocardial  perfusion  test  
   
57.  ECG  of  2nd  degree  AV  block,  next  step?  
A-­‐  Lidocaine  
B-­‐  Atropine  (if  pacemaker  not  in  the  choices)  
C-­‐  Electro  cardioversion  shock  
 Answer:  Atropine  (if  pacemaker  not  in  the  choices)  
   
58.    HTN  patient  on  ACEIs,  not  controlled,  what  to  add?  (No  CCB  in  choices)  
A-­‐  Furosemide  
B-­‐  Beta  blocker  
C-­‐  Thiazide  
Answer:  Thiazide  
   
59.  Drug  that  prolongs  survival  in  CAD.  
 ACEI  
   
60.  What  is  the  drug  that  increases  survival  rate  in  aortic  stenosis?  
A-­‐  Digoxin  
B-­‐  Captopril  
C-­‐  Calcium  channel  blocker  
Answer:  Could  not  find  a  definite  answer  
   
61.  Attacks  of  migraine  and  syncope  (not  together).  Family  history  mother  died  due  to  cardiovascular  
cause  at  age  of  35  years.  Decreased  sounds  over  carotids  on  auscultation.  Vitals  show  hypertension.  
A-­‐  Carotid  Doppler  
B-­‐  Blood  pressure  monitoring  
Answer:  Carotid  Doppler  (fibromuscular  dysplasia)    
   
62.  Treatment  of  AF  
A-­‐  Beta  blocker  
   
63.  Most  likely  scenario  in  rheumatic  fever?  
A-­‐  History  of  URTI  followed  by  knee  joint  swelling  
B-­‐  Painful  finger  joints  and  fever  
Answer:  History  of  URTI  followed  by  knee  joint  swelling  
   
64.  Heart  failure  what  is  the  treatment?  
A-­‐  digoxin  
   
65.  In  MI  which  lab  test  what  will  be  increased?  
A-­‐  ALP  
B-­‐  CK  
Answer:  CK-­‐MB  
   
66.  case  of  old  man  with  chest  pain  and  transient  loss  of  consciousness  (3-­‐4  episodes).  CXR  and  
asked  about  management?  
   
 
67.  Cardiac  syncope  what  is  characteristic  of  it?  
A-­‐  Rapid  recovery  
B-­‐  Aura  
C-­‐  Neurological  defect  
Answer:  Rapid  recovery  
   
68.  Patient  has  history  of  transit  angina.  Now  he  is  asymptomatic  and  doing  well  but  his  ECG  shows  
AF.  What  is  your  action?  
A-­‐  Reassure  
B-­‐  Give  digoxin  
C-­‐  Give  anti-­‐coagulation  
Answer:  Give  anticoagulation  after  calculating  CHADSVASc  score  
   
69.  Long  history  of  a  man  with  systolic  murmur  at  ...  sternal  border.  Recently  develop  syncope.  What  
is  the  diagnosis?  
A-­‐  AS  
   
70.  77  years  old  male  with  early  onset  of  dyspnea  when  moving  50  meters,  has  left  apical  heave  and  
loud  audible  systolic  murmur  most  intense  to  hear  in  left  sternal  border.  What  is  the  diagnosis?                                                                                
   
A-­‐  Sever  mitral  regurgitate  
B-­‐  Calcified  aortic  stenosis  
C-­‐  Mitral  stenosis  
 Answer:  Calcified  aortic  stenosis  
   
71.  Old  patient  present  with  angina  symptoms,  presyncope  and  plural  effusion.  Echo  shows  AS,  valve  
is  .75  mm.  What  is  your  management?  
A-­‐  Medical  management                                                                                                  
B-­‐  Surgical  repair  
C-­‐  Ballon  valvoplasty  
D-­‐  Observation  
Answer:  Surgical  repair  
   
   
72.  Patient  complaining  of  SOB  on  exertion  and  chronic  cough,  on  examination  there  is  
hepatomegaly.  Best  test  to  determine  the  cause?  
A-­‐  Alpha  1  antitrypsin  level                                                                                                                                
B-­‐  ECHO  
Answer:  needs  more  details  it  can  be  heart  failure  or  Alpha  1  antitrypsin  
   
73.  History  of  angina  that  become  more  frequent  and  lasted  longer  time  without  elevation  of  cardiac  
enzymes?  
A-­‐  Unstable  angina  
   
   
74.  Young  patient  came,  tell  you  2  of  his  brothers  died  when  they  running,  you  are  afraid  of  which  of  
the  following?  
A-­‐  Hypertrophic  cardiomyopathy  
   
   
75.  Patient  with  MI  and  in  the  5th  day  he  deteriorated?  
A-­‐  Acute  mitral  regurgitation  
   
76.  Patient  with  atrial  fibrillation  and  you  described  for  him  a  medication  what  is  the  mechanism  of  
action  of  that  medication?  
Warfarin:  vitamin  K  carboxylase  inhibitor  
   
77.  Which  type  of  congenital  heart  disease  is  least  associated  with  infective  endocarditis?  
A-­‐  ASD  
   
78.  Patient  has  chest  pain  on  left  side.  Examination  showed  pleuritic  chest  pain  and  friction  rub  on  
left  side.  What  is  the  next  step?  
A-­‐  Chest  x-­‐ray  
B-­‐  12  lead  ecg  
C-­‐  Echo  
D-­‐  Refer  to  cardio  
Answer:  ECG  
   
79.  Patient  with  symptoms  of  endocarditis  but  cultures  are  negative.  What  is  the  cause?  
A-­‐  SLE  
   
80.  A  33  years  old  man,  smoker,  overweight,  inactive  present  with  heart  problem,  which  of  the  
following  is  most  commonly  associated  with  heart  disease?                                                                                                                  
A-­‐  Smoking  
B-­‐  Obesity  
C-­‐  Inactivity  
 Answer:  Obesity  
   
81.  Diabetic  patient  with  chest  pain  for  3  days,  dyspnea  with  exertion,  history  of  URTI.  ECG:  T  
depression.  Labs:  high  troponin.  What  is  the  diagnosis?  
A-­‐  Myocarditis  
B-­‐  MI  
C-­‐  Constrictive  pericarditis  
 Answer:  Myocarditis  
   
82.  Which  drug  if  you  stop  abruptly  will  cause  hypertensive  crisis?  (No  beta  blocker  in  choices)  
A-­‐  Clonidine  
   
83.  What  type  of  heart  failure  occurs  with  high  cardiac  output?  
   
84.  Patient  with  central  chest  pain,  CXR  showed  pleural  effusion  and  ECG  showed  ST  elevation  in  4  
leads.  What  is  the  diagnosis?  
A-­‐  MI  
B-­‐  Pericarditis                                        
C-­‐  Pneumothorax  
D-­‐  Pneumonia  
Answer:  MI    
   
85.  Young  girl  come  with  HTN  crisis,  antihypertensives  are  not  effective,  She  has  history  of  
hypotension  during  appendectomy?  
A-­‐  Pheochromocytoma  
B-­‐  Renal  artery  stenosis  
C-­‐  Renal  vein  thrombosis  
D-­‐  Essential  HTN  
Answer:Pheochromocytoma  
   
86.  Post  MI  patient  wants  to  drive.  When  will  he  be  able  to  drive?  
A-­‐  4  weeks  
B-­‐  1  week  
Answer:  6  weeks    
   
87.  History  of  Palpitation  &  SVT  (ECG).  What  is  the  treatment?  
A-­‐  Deltiazim  
   
88.  Picture  of  ECG:  bradycardia,  on  history  he  has  only  palpitation.  What  is  the  treatment?  
A-­‐  Digoxin  
B-­‐  Propranolol  
C-­‐  ACEI  
Asnwer:  none  
   
89.  clear  scenario  about  essential  hypertension  
   
90.  After  ER  management  of  MI,  what’s  the  next  step?  
A.  primary  stenting  
B.  give  tPA  
C.  give  streptokinase  
Answer:  depends  on  duration  
   
   
91.  A  patient  after  MI  when  can  he  go  back  to  his  regular  routine  or  exercise?  
1)  1  week  
2)  3  weeks  
3)  5  weeks  
Answer:  6  weeks  
   
   
92.  Right  sided  heart  failure  symptoms.  Echo  done  which  was  normal.  
1)  coxsackie  B  
2)  amyloidosis  
3)  Constrictive    pericarditis  
Answer:  Coxackie  B  virus  causing  myocarditis  
   
93.  ECG  shows  inferior  MI,  what  artery  is  occluded  
-­‐  right  coronary  
-­‐  left  coronary  
-­‐  circumflex  
-­‐  conus  
 Answer:  RCA  
   
94.  Which  of  those  are  in  diagnosis  of  infective  endocarditis  criteria  considered  major?  
   
95.  Best  treatment  for  acute  myocarditis?  
A-­‐  Penicillin  
B-­‐  Corticosteroids  
 Answer:  Supportive  
   
96.  Most  common  electrolyte  disturbance  in  a  patient  with  digoxin?  
A-­‐  K  
B-­‐  Na  
Answer:  K  
   
97.  Drug  which  decreases  HR  and  Pre  and  after  load?  
A-­‐  Carvidilol  
B-­‐  Nifidepine  or  Amlodepine  
Anwer:  CCB  
   
   
98.  Most  common  cause  of  secondary  HTN?  
A-­‐  Renal  disease  
   
99.  Diabetic  patient  with  chest  pain  for  3  days,  dyspnea  with  exertion,  and  history  of  URTI.  
ECG  shows  T  depression  
Labs:  high  troponin  
What  is  the  diagnosis?  
A-­‐  Myocarditis  
B  -­‐  MI  
C  -­‐  Constrictive  pericarditis  
Answer:  Myocarditis    
   
100.  Patient  present  to  the  ER  with  typical  symptoms  of  Ischemic  heart  disease,  he  had  history  of  
erectile  dysfunction.  He  is  on  PDE-­‐5  inhibitor  (sildenafil),  which  of  the  following  drug  should  not  be  
given?  
A-­‐  Nitroglycerine  
B-­‐  Metopralol  
C-­‐  Dilitazem  
 Answer:  Nitroglycerine  
   
   
101.  Which  one  of  the  following  carries  more  risk  for  CAD?  
A-­‐  55,  male,  DM  
B-­‐  50,  male,  hyperlipidemia  
C-­‐  HTN,  obese  
 Answer:55,  male,  DM  
   
   
102.  52  Years  old  female  k/c  of  DM,  recently  diagnosed  with  HTN.  Examination  was  normal  except  
for  mild  lower  limb  edema.  What  you  will  give?  
A-­‐  Propranolol  
B-­‐  Perindopril  
C-­‐  Atenolol  
D-­‐  Hydrochlorothiazide  
 Answer:  Perindopril    
 
103.  Malignant  hypertension  is?                    
   
104.  Adult  patient  presented  with  chest  pain  and  palpitation,  ECG  showed  atrial  fibrillation.  Vitals:  
HR:  140  BP:  80/50.  How  to  treat?  
A-­‐  Digoxin  
B-­‐  Synchronized  Cardioversion                                                                                              
C-­‐  Beta  blocker  
   
105.  Patient  with  hyperthyroidism,  with  palpitation,  what  is  the  most  likely  finding  in  his  ECG?    
a)  SVT                                                                                                  
b)  VT  
c)  A  FIB  
 

Rheumatology  
   
 1.  Symptoms  of  diffuse  systemic  sclerosis.  What  is  the  antibody?  
A-­‐  Anticentromere  antibody  
B-­‐  Anti-­‐topoisomerase  I  (anti-­‐Scl-­‐70)  antibodies  
Answer:  B  
   
2.  Tall  &  thin  boy  with  joint  laxity,  also  his  mother  has  the  same  features.  What  is  mode  of  
inheritance?  
Answer:  Autosomal  dominant  (Marfan  syndrome).  
   
3.  Boutonniere  deformity  description?  
Answer:  Flexed  PIP  and  hyperextended  DIP.  
   
4.  (REPEATED)  Ehler-­‐Danlos  mode  of  inheritance?  
Answer:  Autosomal  dominant  
   
5.  Case  of  RA  asking  about  genetics?  
Answer:  HLA  B27  
HLA  B27  is  associated  with  seronegative  spondyloarthropathies  (ex:  ankylosing  spondylitis).  
   
6.  Clear  scenario  of  Raynaud's  phenomenon  
   
7.  SLE  prognosis  
Answer:  Degree  of  Renal  involvement  
Medscape:  The  disease  course  is  milder  and  survival  rate  higher  in  persons  with  isolated  skin  and  
musculoskeletal  involvement  than  in  those  with  renal  disease  
   
8.  Polymyalgia  rheumatica  is  associated  with?  
Answer:  Proximal  stiffness  
Toronto  notes:  characterized  by  pain  and  stiffness  of  the  proximal  extremities  
   
9.  Antiphospholipid  syndrome  
   
10.  Which  of  the  following  is  diagnostic  of  lupus?  
A-­‐  Anti-­‐RNP  antibodies  
B-­‐  Cystoid  bodies  on  fundoscopy  
C-­‐  Severe  Raynaud's  phenomena  
Answer:  Anti-­‐RNP  associated  with  both  mixed  connective  tissue  disease  and  SLE.  
“Cytoid”  bodies  (AKA  cotton  whool  spots)  associated  with  inflammatory  and  ischemic  conditions.  
   
11.  Dermatomyositis  associated  with  what?  
Answer:  Malignancy  
Toronto  notes:  malignancy  screening  part  of  management.  
   
12.  Polymyalgia  rheumatica,  what  are  the  x-­‐Ray  findings?  
A-­‐  Calcification  in  joints  
B-­‐  Normal                                                                                                                                    
Answer:    B.  
Uptodate:  While  subdeltoid/subacromial  bursitis  is  a  characteristic  imaging  feature  of  PMR,  it  is  not  
specific  and  is  seen  in  patients  with  rheumatoid  arthritis  (RA)  and  other  shoulder  pathology.  
   
13.  Most  specific  test  for  rheumatoid  arthritis?  
A-­‐  CRP  
B-­‐  RF  
C-­‐  HLA  b27  Antidnase  
D-­‐  anti-­‐CCP                  
Answer:  D  
Toronto  notes:  Anti-­‐CCP  specific  for  RA  (94-­‐98%)  
   
14.  Female  with  new  onset  Raynaud's  phenomena  and  upper  GI  endoscope  showed  gastric  
vascular  ectasia  (something  like  that)  what  is  positive  in  blood?  
A-­‐  Anticentromere  
B-­‐  Anti  Scl70  
Answer:  B  
Anti-­‐Scl70  (=anti-­‐topoisomerase)  >  systemic  scleroderma.  
Anti-­‐centromete>  CREST.  
   
15.  Patient  with  gout,  what  should  be  measured?  
Answer:  Uric  acid  level  
   
16.  Polymalegia  +  polymyositis  
   
17.  Patient  with  recurrent  renal  stones  on  HTN  medication  to  reduced  calcium  excretion  and  cause  
gout?  
A-­‐  Hydrochlorithiazide  
B-­‐  Furosemide  
 Answer:  A  
   
18.  Case  of  gout  and  the  answer  was  indomethacin  
   
19.  Patient  develop  back  pain  in  morning,  pain  improve  with  walking  and  movement.  He  had  the  
same  complain  in  past  1  year.  What  is  the  diagnosis?  
A-­‐  Ankylosing  spondylitis  
B-­‐  Recurrent  gout  
Answer:  A  
   
20.  Patient  with  gout,  which  of  the  following  will  be  inhibited  by  the  drug  you  will  give?  
A-­‐  Xanthine  oxidase  
B-­‐  PRPP  
Answer:  A  
   
21.  Behçet  disease  HLA  association  
Answer:  HLA  B51  
 Medscape:  Carriers  of  HLA-­‐B51/HLA-­‐B5  have  an  increased  risk  of  developing  Behçet  disease  
compared  with  noncarriers.  
   
22.  Symmetrical  MCP  &  PIP  pain  increase  at  morning  
Answer:  Rheumatoid  arthritis  
   
23.  Patient  with  oral  and  genital  ulcer  and  arthritis.  What  is  the  diagnosis?  
Answer:  Bechet  disease  
   
24.  Patient  with  bilateral  shoulder  and  hip  stiffness  and  ask  about  the  diagnosis?  
Answer:  Polymyalgia  rheumatica  
 Depends  on  rest  of  scenario.  Most  important  characteristic  is  :stiffness  and  pain  without  weakness.  
   
25.  Case  about  RA  came  with  bilateral  swelling  in  DIP  due  to?  
Answer:  Synovial  thickening    
   
26.  Scenario  about  a  little  girl  who  had  high  ESR  and  high  ANA  and  no  obvious  symptoms.  What  is  
the  diagnosis?  
Answer:  Latent  lupus  
   
27.  Patient  with  uric  acid  calculi,  with  normal  uric  acid  level,  how  to  prevent  recurrence  of  calculi:  
A-­‐  Allopurinol  
B-­‐  Probenecid  
C-­‐  Disodium  citrate  
Answer:  
Toronto  notes:  Treatment>  increase  fluid  intake,  alkalinization  of  urine  ±  allopurinol.  Hyperuricosuria  
not  necessarily  present.  
   
28.  (REPEATED)  Most  specific  test  for  rheumatoid  arthritis?  
A.  CRP  
B.  RF  
C.  HLA  27  
D.  Anti-­‐DNAse  
 Answer:  
   
29.  Patient  with  sudden  severe  shoulder  and  neck  pain  with  stiffness,  also  hip  and  lumbar  spine  
pain,  also  complain  of  fatigue  and  increase  sleepiness,  ESR  was  90.  
A-­‐  Cartilaginous  tissue  degeneration  
B-­‐  Antigen  antibody  mediated  arthritis  
C-­‐  Soft  tissue  with  giant  cell  arteritis  
D-­‐  Connective  tissue  disease  something  
 Answer:C  
   
30.  Case  of  Wegener’s  but  asked  about  the  basic  pathology  of  disease  
   
31.  Marfan  syndrome  features  of  the  mother  and  her  boy:  what  is  the  probability  of  the  children  
to  have  it?  
A-­‐  25%  
B-­‐  50%  
C-­‐  75%  
D-­‐  100%  
Answer:  B  
Autosomal  dominant.  
   
32.  Female  presented  with  hand  joints  swelling,  she  was  informed  by  physician  that  she  has  bone  
loss,  she  is  angry  about  her  permanent  bone  loss.  What  is  the  explanation  of  bone  loss?  
A-­‐  Increase  pressure  in  joint  space  
B-­‐  Material  secreted  from  synovial  fluid  
C-­‐  Drug  induced  menopause  
Answer:  B  
 
   
33.  Patient  with  osteoarthritis  and  they  ask  about  the  name  of  nodes  over  the  PIP  joint?  
A-­‐  Heberden  nodes  
B-­‐  Bouchards  nodes  
Answer:  B  
 Heberden’s  >  DIP.  
   
34.  Case  about  back  pain  and  its  management  
 
35.  Male  patient  with  oral  ulcers,  abdominal  pain,  nausea,  diarrhea,  severe  headache  and  polyarthritis.  (Attached  
a  pictures  of  penile  ulcer  and  ulcers  over  the  buccal  mucosa).  What  is  the  diagnosis?    
A-­‐  Behcet    
B-­‐  Syphilis  
C-­‐  Ulcerative  colitis  
   
36.  Patient  presented  with  right  knee  pain  and  swelling,  arthrocentesis  done:  
Color:  yellow  
Viscosity:  low                                                                                                                          
WBC:  15.2  
Clarity:  opaque  (Didn't  mention  anything  about  crystals  in  the  aspiration)  
What  is  the  diagnosis?  
A-­‐  Gout  
B-­‐  Septic  arthritis  
C-­‐  Rheumatoid  arthritis  
D-­‐  Pseudo  gout  
Answer:    
 
37.  Somethings  about  dermatomysitis.  Rash  +  
A.   Proximal  muscle  weakness  
B.   Tenderness  of  muscle  
 Answer:  A.  
Heliotrope  rash  (periorbital),  Gottron’s  papules  (violaceous  papules  over  knuckles  and  IP  joints)  ±  poikiloderma.  
Shawl  sign:  macular  erythema  over  chest  and  shoulder.  Proximal  muscle  weakness  ±  pain.  Dyspnea  on  exertion.  
Toronto  notes.  
 
 

Gastroenterology    
   
1.  how  to  differentiate  a  huge  ovarian  mass  from  an  ascites?  
Answer:  Anterior  dullness  and  lateral  Tympani  
   
2.                      Vinson  Plummer  syndrome?  
   
Plummer-­‐Vinson  Syndrome  Triad  
•  Iron  deficiency  anemia  
•  Dysphagia  
•  Esophageal  webs  
*  rare  (prevalence  <1  in  1,000,000)  but  good  prognosis  when  treated  with  iron  and  esophageal  
dilatation.  Toronto  
   
3.                      Patient  complaining  of  abdominal  pain  I  think  with  diarrhea  ,  also  his  wife  noticed  he  had  
SOB  and  tightness.  Doctor  order  5-­‐hydroxyindoleacetic  acid  in  urine  Which  cell  responsible?  
A.chromaffin  cell  
B.Enterocell  
C.Lympho  cell  
D.Goblet  cell  
Answer:  A

 
 
4.                      which  markers  of  HBV  is  present  in  window  phase:  
Answer:  HBcAb        
First  aid  step  2  p157  
   
5.                      what's  the  most  common  cause  of  gastroenteritis?  
1)  Noro  virus  
2)  rotavirus  
Answer??  
For  acute    viral  gastroenteritis:  
In  infants,  most  cases  are  due  to  rotavirus.  
In  adults,  the  most  common  cause  is  norovirus.  
Medscape  and  ACG  
   
6.                      Causes  of  aphthus  ulcer:  
Celiac  disease  
Ibs  
Bilirubin  retention  
Answer:  A  
Malabsorption  in  gastrointestinal  disorders:  About  3%  of  patients  experience  these  disorders,  
particularly  celiac  disease  (gluten-­‐sensitive  enteropathy)  but,  occasionally,  Crohn  disease,  pernicious  
anemia,  and  dermatitis  herpetiformis.  HLA  DRW10  and  DQW1  may  predispose  patients  with  celiac  
disease  to  oral  ulceration.  
medscape  
   
7.                      Which  drug  will  decrease  baseline  and  acid  secretion  from  stomach:  
-­‐              Ranitidine  (decreases  secretion)  
-­‐              PPI  
-­‐              Other  choices  that  I  don’t  remember.  
Answer:  B  
   
8.                      Tumor  marker  for  pancreatic  cancer?  
Ca  19  
   
9.                      Patient,  with  diarrhea,  flagellated  protozoa,  how  it  cause  diarrhea?  
A.                      increase  secretion  of  fluids  
B.                      kills  normal  flora  
C.                      coats  the  small  bowl  
D.                     absorption  is  prevented  
   
Seems  like  he’s  talking  about  giardia...so:    A  or  D  not  sure  
The  mechanisms  by  which  Giardia  causes  diarrhea  and  intestinal  malabsorption  are  probably  
16]  
multifactorial  and  not  yet  fully  elucidated.  [ Postulated  mechanisms  include  damage  to  the  
endothelial  brush  border,  enterotoxins,  immunologic  reactions,  and  altered  gut  motility  and  fluid  
hypersecretion  via  increased  adenylate  cyclase  activity.  
   
10.    knee  pain  old  age  with  NSAID  3  weeks.  Come  with  bloody  vomitting.  Reason?  
PUD  caused  by  NSAID  
   
11.    Young  Male  have  epigastric  pain,  fullness,  nausea,  postprandial  bloating  for  three  years.  His  
symptoms  appeared  after  he  stayed  in  India  for  one  year  for  studying.  Symptoms  are  worse  with  
food.  What  is  the  diagnosis?  
A.  Functional  dyspepsia  
B.  H.  Pylori  Infection  
C.  Esophageal  ulcer.  
Answer:  B  most  likely  
   
12.    Pt  treated  for  peptic  ulcer  with  triple  what  u'll  do?  
A.                      -­‐nothing  
B.                      -­‐H  pylori  antigen  test  
C.                      –endoscopy  
(not  sure)    If  in  the  scenario  there’s  symptoms  refractory  to  medication.  C  to  rule  out  gastric  cancer  
or  cancer  transformation.  All  gastric  ulcers  must  be  biobsied.        First  aid  2  
   
13.    Peptic  ulcer  treated  with  triple  therapy  after  3  weeks.  therapy  now  she  is  free  of  sx.  what  to  
do?  
A.                      Endoscopy  
B.                      h.pylori  antigen  
Answer:  B  
American  college  of  gastroenterology  2017:  
Should  we  test  for  teatment  success  after  H.  pylori  eradication  therapy?  

Whenever  H.  pylori  infection  is  identified  and  treated,  testing  to  prove  eradication  should  be  
performed  using  a  urea  breath  test,  fecal  antigen  test  or  biopsy-­‐based  testing  at  least  4  weeks  
after  the  completion  of  antibiotic  therapy  and  after  PPI  therapy  has  been  withheld  for  1–2  
weeks.  (Strong  recommendation;  Low  quality  of  evidence  (for  the  choice  of  methods  to  test  for  
eradication:  Moderate  quality  of  evidence)).  

   

What  are  the  indications  to  test  for,  and  to  treat,  H.  pylori  infection?  

Since  all  patients  with  a  positive  test  of  active  infection  with  H.  pylori  should  be  offered  
treatment,  the  critical  issue  is  which  patients  should  be  tested  for  the  infection  (strong  
recommendation;  quality  of  evidence  not  applicable).  
All  patients  with  active  peptic  ulcer  disease  (PUD),  a  past  history  of  PUD  (unless  previous  cure  
of  H.  pylori  infection  has  been  documented),  low-­‐grade  gastric  mucosa-­‐associated  lymphoid  
tissue  (MALT)  lymphoma,  or  a  history  of  endoscopic  resection  of  early  gastric  cancer  (EGC)  
should  be  tested  for  H.  pylori  infection.  Those  who  test  positive  should  be  offered  treatment  
for  the  infection  (Strong  recommendation;  quality  of  evidence:  high  for  active  or  history  of  
PUD,  low  for  MALT  lymphoma,  low  for  history  of  endoscopic  resection  of  EGC).  

In  patients  with  uninvestigated  dyspepsia  who  are  under  the  age  of  60  years  and  without  alarm  
features,  non-­‐endoscopic  testing  for  H.  pylori  infection  is  a  consideration.  Those  who  test  
positive  should  be  offered  eradication  therapy  (conditional  recommendation;  quality  of  
evidence:  high  for  efficacy,  low  for  the  age  threshold).  

When  upper  endoscopy  is  undertaken  in  patients  with  dyspepsia,  gastric  biopsies  should  be  
taken  to  evaluate  for  H.  pylori  infection.  Infected  patients  should  be  offered  eradication  
therapy  (strong  recommendation;  high  quality  of  evidence).  

   

15.    Sudden  severe  generalized  abdominal  pain  in  a  patient  known  to  have  A  Fib:  
Answer:  Mesenteric  ischemia  
   
16.    Treatment  of  Wilson  disease  
penicilamine  or  trientine    and  zinc  but  should  space  the  doses  
   
18.    Healthy  women  asymptomatic  on  regular  check  up  found  to  have  HB  antigen  positive  ?  
1-­‐acute  hepatitis  
2-­‐chronic  hepatitis  
3-­‐acute  carrier  
4-­‐chronic  carrier  
Answer:  A  Not  sure.  Depends  what  other  ones  they  did  or  didn’t  do  
   
 
 
 
 
 
 
 
 
 
 
 
 
 
19.    What  is  the  mechanism  of  diarrhea  
A.                      Decrease  fluid  intake  
B.                      Decrease  fluid  absorption  
C.                      Increase  fluid  intake  
D.                     Increase  fluid  absorption  
Answer:  B  
   
Q.  What  is  the  mechanism  of  diarrhea  with  patient  has  gardia  infection  ?  
1-­‐  reduction  of  absorption  
2-­‐  increase  secretion  of  mucosal                                                                                                                                              
Answer:  reduction  of  absorption  most  likely  but  2  could  be  right  
   
20.    Case  of  achalasia  clear  sentence  of  bird  like  appearance  on  xray  
   
21.    U  want  to  start  statin  for  a  patient  what  test  u  should  order?  
A-­‐LFT  
B-­‐GGT  
Amswer:  A  
Medscape  
   
22.    Most  common  site  for  crohns?  
Ileocecal  area  
   
23.    Best  test  to  diagnose  hepatitis  B?  
A-­‐  US  
B-­‐blood  test  
C-­‐blood  culture  
D-­‐liver  biopsy  
Answer:  B  
   
24.    Retrosternal  pain  and  barium  swallow  showed  esophageal  corkscrew  appearance?  
A-­‐achalasia  
B-­‐  Esophageal  cancer  
C-­‐diffuse  esophageal  spasm  
Answer:  C  
   
25.    BRUNNER's  gland  location?  
Answer:  submucosal  duodenum  
   
26.    What  is  the  drug  that  given  with  analgesia  to  decrease  side  effect:"  ‫ ﺑﻬﮭﺬﺍا ﺍاﻟﻨﺺ‬ "  
A)  cimetidine  
B)  Metoclopramide.  
C)  misoprostol  
Answer:  depends  what  they  think  of  as  analgesia…  if  it’s  NSAIDS,  choice  C  would  be  good  prophylaxis  
for  ulcer  
   
27.    Which  one  of  the  following  GIT  cell  secret  defensins:  
A:  paratial  cell  
B:  chief  cell  
C:  paneth  cell  
D:  plasma  cell  
Answer:  C  
   
28.    Scenario  of  hepatorenal  syndrome:  
renal  failure  in  cirrhosis  
Classifications  
◆  pre-­‐renal  (usually  due  to  over-­‐diuresis)  
◆  acute  tubular  necrosis  
◆  HRS  
–  Type  I:  sudden  and  acute  renal  failure  (rapid  doubling  of  creatinine  over  2  wk)  
–  Type  II:  gradual  increase  in  creatinine  with  worsening  liver  function  (creatinine  doubling  
over  years)  
HRS  can  occur  at  any  time  in  severe  liver  disease,  especially  after  
–  overdiuresis  or  dehydration,  such  as  diarrhea,  vomiting,  etc.  
–  GI  bleed  
–  sepsis  
Treatment  for  hepatorenal  syndrome  (generally  unsuccessful  at  improving  long-­‐term  survival)  
–  for  type  I  HRS:  octreotide  +  midodrine  +  albumin  (increases  renal  blood  flow  by  increasing  
systemic  vascular  resistance)  
–  definitive  treatment  is  liver  transplant  
TORONTO  
   
29.    Scenario  of  zollinger  ellison  syndrome  
Gastrin-­‐producing  neuroendocrine  tumour  that  causes  gastric  acid  hypersecretion  and  peptic  ulcer  
disease.  
·∙              Can  be  sporadic  or  associated  with  multiple  endocrine  neoplasia  syndrome  type  1.  
·∙              Common  presentation  includes  refractory  peptic  ulcer  disease,  abdominal  pain,  
diarrhoea,  and  gastro-­‐oesophageal  disease.  
·∙              The  main  goal  of  treatment  is  control  of  gastric  hypersecretion  with  proton-­‐pump  
inhibitors.  
·∙              The  most  common  cause  of  morbidity  and  mortality  is  metastatic  gastrinoma.  
   
30.    Orilstat  Mechanism?  
Inhibits  gastric  and  pancreatic  lipases,  prevents  triglyceride  hydrolysis  resulting  in  decreased  
absorption  of  dietary  fats  
   
31.    About  watery  diarrhea  without  blood  and  asked  about  organism?  
Answer:  C.  Defficile  (  Not  sure  )  
Depends  what  other  options  were  there.  Most  cases  of  acute,  watery  diarrhea  are  caused  by  viruses  
(viral  gastroenteritis).  The  most  common  ones  in  children  are  rotavirus  and  in  adults  are  norovirus.  
The  ACG  
   
32.    Vit.c  and  iron  absorption?  
Iron  absorption  from  diet  is  enhanced  in  the  presence  of  vitamin  C  and  diminished  by  excess  
calcium,  zinc,  or  manganese  
   
34.    pt  complain  of  constipation  on  abdominal  examination  there  is  a  mass  in  the  lower  abdomen  
you  did  US  which  confirmed  the  presence  of  a  mass  in  which  you  decided  to  do  colonoscopy  which  
find  2  polyps  with  high  grade  hyperplasia  ,  when  you  will  follow  up  the  pt  by  colonoscopy?  
A-­‐after  1  year,  
B-­‐3  yrs  ,  
C-­‐8  yrs  
Answer:  A  
Toronto,  after  1  yr  then  every  3-­‐5  yrs  
   
36.    End  stage  liver  disease  with  fungal  infection  ttt  
Answer:  Capsofungin  
   
37.    most  specific  sign  for  perforated  duodenal  ulcer,  
 Answer:  Sudden  epigastric  pain  
   
38.    Patient  diagnosed  with  Barrett's  esophagus  will  have:  
A.  Adenocarcioma  
B.  Squamous  cell  carcinoma  
Answer:  A  
   
39.    Most  common  cancer  mets  to  stomach:  
A.          lung  
B.          Prostate  
C.          Melanoma  
D.          -­‐Brain  
Answer:  A  
If  ‘from’  then  liver,  lung,  bone  and  lymph  nodes  
   
40.    Scenario  :  pt  develop  bloody  diarrhea,  abd  pain  and  vomiting,  after  eating  from  restaurant  ,  
history  if  hematuria,  
Lab  :  Forgot  !  
HUS:  
   
-­‐  ...  
If  there’s  conservative,  choose  it.  
   
41.    upper  epigastric  pain  +  fever  ,  Dx?  (Incomplete  Q)  
A.                      perforated  peptic  ulcer  
B.                      cholecystitis  
C.                      appendicitis  
D.                     pancreatitis  
   
42.    Side  effect  of  ribavirin:  
A)                      anemia.  
B)                      Renal  damage  
C)                      Hepatic  damage.  
Answer:  A  
   
43.    Sign  for  low  intestinal  obstruction:  
A)                      Altered  bowel  movement.  
B)                      Absolute  constipation  
C)                      Flatulence  
D)                     Diarrhea.  
Answer:  B  not  sure  
 
 
 
Also,  revise  functional  obstruction  
   
44.    Patient  with  deoudnal  ulcer  ,  what  ttt?  
Answer:    triple  therapy  
   
45.    Clear  case  about  crohn  dis:  child  with  abd  cramps  ,  diarhea  ....  etc  ,  there  is  skip  lesion  ,  
transmural  :  
A.                      -­‐  crohn✅  
B.                      -­‐  Uc  
C.                      -­‐  Celiac  .  
Answer:  A  
GALS  are  crohns:  Granuloma,  All  Layers,  Skip  lesions  
   
46.    Clear  scenario  about  Crohn's  disease  (mentioned  that  she  has  post  meal  periumbilical  pain  
plus  other  symptoms:  
Answer:  Crohn's  disease  
   
47.    Similar  case:  child  with  abd  pain  and  tenderness  ,  bloody  diarhea  ,  wt  loss.  (  No  lab  results  ).  
What’s  the  diagnosis:  
A.                      crohn  
B.                      Celiac  
C.                      UC  
Answer:  C  
   
48.    Celiac  disease  common  site:  
A.  Proximal  small  bowl  
B.  Distal  small  bowl  
C.  Proximal  large  bowl  
D.  Distal  large  bowl  
Answer:A  
   
49.    Many  Qs  about  different  type  of  jaundice  easy  just  look  for  indirect  and  direct  bilirubin  levels  
and  liver  enzymes.  
   
50.    Patient  lost  weight  and  has  constipation.  Which  one  has  the  highest  diagnostic  value  ?  
Colonoscopy      
   
51.    Patient  is  taking  penicillin  (or  medication  from  penicillin  family).  After  the  second  day  he  
developed  diarrhea.  What  is  the  most  likely  cause?  
A.                      Psudomembranus  colitis  
B.                      Something  colitis  
C.                      Ulcerative  colitis  
Answer:  A  caused  by  C.  diff  
   
52.  Within  hours  diarrhea?  
A  Staph  
B  Bacillus  
C  Salmonella  
Answer:  A  
   
53.  Lady  presented  with  jaundice  and  confusion,  her  husband  is  positive  for  hepatitis  B.  Her  labs  
were  attached  and  she  had  high  bilirubin  but  negative  hepatitis  screen,  which  one  of  the  following  
will  you  order?  
A.          Serum  copper  level  
B.          Serum  ceruloplasmin  level  
   
Copper  would  be  more  specific,  ceruloplasmin  might  be  normal  in  pts  then  it’s  useless  if  it  comes  
back  normal.  urinary  copper  24  hrs.    BMJ  
54.  male  patient  with  symptoms  of  pancreatitis,  k/c  of  hyperlipidemia  on  treatment  
Labs  
Low  Hgb  
High  MCV  
High  Amylase  
High  AST  
Normal  lipid  profile.  What  is  the  cause?  
A:hyperlipidemia  
B:  Alcohol  consumption  
Answer:  B  i  think.  
Alcoholic  Liver  disease  causes  macrocytic  anemia  and  can  cause  pancreatitis.  Also,  normal  lipid  
profile  
   
55.  Long  scenario  pt  on  atrovastatin  since  1  yr,increase  LFT:  
Drug  induced  hepatitis  
   
56.  Pt  in  40s  with  obs  jaundice,  labs  showed  direct  bilirubin  40,  ferritin  high??,Alkaline  
phosphatase    high,  AST  elevated,  what  dx?  gallbladder  Stone?  

Could  be  hemochromatosis…  high  ferritin,  not  sure  

57.  Which  of  the  following  will  reduce  both  spontaneous  and  contact  induced  acid  secretion  in  the  
stomach?  
PPI  
   
58.Hemochromatosis  has  cirrhosis  before  5  years  came  with  pain  and  jaundice.  What  is  the  next  
step:  
A.  US  
B.  CEA  
Answer:  A  
   
59.  Most  common  symptom  with  hepatitis  C?                                                                                                                                                    
A:Loss  of  appetite  
B:  Flapping  tremor  
C:  Jaundice  
Answer:  A  
Vague  constitutional  symptoms  are  the  most  common…  Toronto  
   
60.  Marker  for  acute  liver  failure?  
This  a  huge  topic  to  summarize  pls  check  the  link:  
http://emedicine.medscape.com/article/177354-­‐workup#c8  
   
61:  60  yrs  old  patient  with  missed  teeth  and  white  patches  in  the  marginal  of  tongue  not  relived  by  
??  also  he  had  rash  with  ulcer  (Incomplete  Q)  
A-­‐  nurofibramatosis  
B-­‐  dysplasia  
C-­‐  vascular  malformation                                                                                                                                                                            
                                                                                                                                                                               
62.  What  is  the  early  sign  of  acute  hepatic  failure?                                                                                                                                  
A:Hepatic  encephalopathy  
B:Palmar  erythema  
C:Spider  naevi                                                                            
Answer:  A  
BMJ  
   
Acute  liver  failure  is  a  broad  term  that  encompasses  both  fulminant  hepatic  failure  and  subfulminant  
hepatic  failure  (or  late-­‐onset  hepatic  failure).  Fulminant  hepatic  failure  is  generally  used  to  describe  
the  development  of  encephalopathy  within  8  weeks  of  the  onset  of  symptoms  in  a  patient  with  a  
previously  healthy  liver.  Subfulminant  hepatic  failure  is  reserved  for  patients  with  liver  disease  for  up  
to  26  weeks  before  the  development  of  hepatic  encephalopathy.  
Signs  and  symptoms  of  acute  failure  may  include  the  following:  
·∙              Encephalopathy  
·∙              Cerebral  edema:  May  lead  to  signs  of  increased  intracranial  pressure  (ICP)  (eg,  
papilledema,  hypertension,  bradycardia)  
·∙              Jaundice:  Often  present  but  not  always  
·∙              Ascites:  Potential  for  hepatic  vein  thrombosis  (budd  chiari)  with  rapid  development  
in  the  presence  of  fulminant  hepatic  failure  accompanied  by  abdominal  pain  
·∙              Right  upper  quadrant  tenderness:  Variably  present  
·∙              Change  in  liver  span:  May  be  small  due  to  hepatic  necrosis  or  may  be  enlarged  due  
to  heart  failure,  viral  hepatitis,  or  Budd-­‐Chiari  syndrome  
·∙              Hematemesis  or  melena:  Due  to  upper  gastrointestinal  (GI)  bleeding  
·∙              Hypotension  and  tachycardia:  Due  to  reduced  systemic  vascular  resistance  
medscape  
   
63.  Patient  has  Wilson  disease  with  signs  of  liver  failure,  tremor,  prominent  upper  vessel,  ascites.  
Treatment  for  this  stage:  
A:Pencllamine  
B:Zinc  
C:Liver  transplant  

D:  Tips  (Transjugular  intrahepatic  portosystemic  shunt)  


Answer:  C  
●   Patients  should  be  transplanted  if  they  present  with  severe  hepatic  failure  (Nazer  scores  
≥10)  or  if  they  develop  serious  further  deterioration  in  liver  function  while  on  anti-­‐copper  
medical  therapy.  
●   Hepatic  transplant  should  not  be  offered  to  patients  with  neurological  symptoms,  who  
should  be  treated  with  medical  therapy.  
   

                                                         
 
 

Hematology-­Oncology  
   
1.  When  you  give  FFP  with  heparin  
Answer:  DIC  
   
Disseminated  intravascular  coagulation  (DIC)  involves  abnormal,  excessive  generation  of  thrombin  and  fibrin  
in  the  circulating  blood.  During  the  process,  increased  platelet  aggregation  and  coagulation  factor  
consumption  occur.  DIC  that  evolves  slowly  (over  weeks  or  months)  causes  primarily  venous  thrombotic  and  
embolic  manifestations;  DIC  that  evolves  rapidly  (over  hours  or  days)  causes  primarily  bleeding.  Severe,  
rapidly  evolving  DIC  is  diagnosed  by  demonstrating  thrombocytopenia,  an  elevated  PTT  and  PT,  increased  
levels  of  plasma  d-­‐dimer  (or  serum  fibrin  degradation  products),  and  a  decreasing  plasma  fibrinogen  level.  
Treatment  includes  correction  of  the  cause  and  replacement  of  platelets,  coagulation  factors  (in  fresh  frozen  
plasma),  and  fibrinogen  (in  cryoprecipitate)  to  control  severe  bleeding.  Heparin  is  used  as  therapy  (or  
prophylaxis)  in  patients  with  slowly  evolving  DIC  who  have  (or  are  at  risk  of)  venous  thromboembolism .  
(http://www.merckmanuals.com/professional/hematology-­‐and-­‐oncology/coagulation-­‐
disorders/disseminated-­‐intravascular-­‐coagulation-­‐dic)  
Platelet  and  factor  replacement  should  be  directed  not  at  simply  correcting  laboratory  abnormalities  but  at  
addressing  clinically  relevant  bleeding  or  meeting  procedural  needs.  Heparin  should  be  provided  to  those  
patients  who  demonstrate  extensive  fibrin  deposition  without  evidence  of  substantial  hemorrhage;  it  is  
usually  reserved  for  cases  of  chronic  DIC.  Heparin  is  appropriate  to  treat  the  thrombosis  that  occurs  with  
DIC.  It  also  has  a  limited  use  in  acute  hemorrhagic  DIC  in  a  patient  with  a  self-­‐limited  condition  of  acral  
cyanosis  and  digital  ischemia.  
It  is  generally  considered  that  cryoprecipitate  and  coagulation  factor  concentrates  should  not  routinely  be  
used  as  replacement  therapy  in  DIC,  because  they  lack  several  specific  factors  (eg,  factor  V).  Additionally,  
worsening  of  the  coagulopathy  via  the  presence  of  small  amounts  of  activated  factors  is  a  theoretical  risk.  
Specific  deficiencies  in  coagulation  factors,  such  as  fibrinogen,  can  be  corrected  by  administration  of  
cryoprecipitate  or  purified  fibrinogen  concentrate  in  conjuction  with  fresh  frozen  plasma  (FFP)  
administration.  
Data  suggest  that  the  consumption-­‐induced  deficiency  of  coagulation  factors  can  be  partially  rectified  by  
administering  large  quantities  of  FFP,  particularly  in  patients  with  an  international  normalized  ratio  (INR)  
 [ 65]  
higher  than  2.0,  a  2-­‐fold  or  greater  prolongation  of  the  aPTT,  or  a  fibrinogen  level  below  100  mg/dL. The  
suggested  starting  dose  is  15  mg/kg  (https://emedicine.medscape.com/article/199627-­‐treatment#d1)  
   
   
2.  Anemic  patient  on  treatment,  he  came  with  black  stool,  what  is  the  cause  
A-­‐  Ferrous  sulfate  
B-­‐  Iron  dextran  
Answer:  A  
 Ferrous  sulfate  side  effects:  
 (Constipation,  upset  stomach,  black  or  dark-­‐colored  stools,  and  temporary  staining  of  the  teeth.)  
(https://www.drugs.com/ferrous_sulfate.html)  
   
   
3.  Young  patient  with  sickle  cell  anemia,  he  has  penile  pain  and  edematous  shaft  of  penis  (with  
picture),  what  the  diagnosis?  
A-­‐  priapism  
B-­‐  peyronie's  disease  
C-­‐  Peronism  
D-­‐  Paraphimosis  
Answer:  A  
priapism  is  a  well-­‐recognized  complication  of  SCD  (https://emedicine.medscape.com/article/205926-­‐
overview)  Low-­‐flow  or  ischemic  priapism:  This  type  happens  when  blood  gets  trapped  in  the  erection  
chambers.  Most  of  the  time,  there’s  no  clear  cause,  but  it  may  affect  men  with  sickle-­‐cell  disease,  leukemia  
(cancerof  the  blood),  or  malaria.  If  you  don’t  get  treatment  right  away,  it  can  lead  to  scarring  and  permanent  
erectile  dysfunction  (ED).  (https://www.webmd.com/erectile-­‐dysfunction/erectile-­‐dysfunction-­‐priapism#1)  
   
   
4.  Father  with  hemophilia  A  and  mother  carrier.  What  is  the  percentage  that  their  child  will  have  
hemophilia?  
A-­‐  25  %  
B-­‐  50  %  
C-­‐  75  %  
D-­‐  100  %  
Answer:  B  
   
   
50%  will  have  hemophilia  
25%  carrier  
25%  normal  
   
 
 
 
Hemophilia  A  and  hemophilia  B  are  inherited  in  an  X-­‐linked  recessive  pattern.  The  genes  associated  with  
these  conditions  are  located  on  the  X  chromosome,  which  is  one  of  the  two  sex  chromosomes.  In  males  
(who  have  only  one  X  chromosome),  one  altered  copy  of  the  gene  in  each  cell  is  sufficient  to  cause  the  
condition.  In  females  (who  have  two  X  chromosomes),  a  mutation  would  have  to  occur  in  both  copies  of  the  
gene  to  cause  the  disorder.  Because  it  is  unlikely  that  females  will  have  two  altered  copies  of  this  gene,  it  is  
very  rare  for  females  to  have  hemophilia.  A  characteristic  of  X-­‐linked  inheritance  is  that  fathers  cannot  pass  
X-­‐linked  traits  to  their  sons.  In  X-­‐linked  recessive  inheritance,  a  female  with  one  altered  copy  of  the  gene  in  
each  cell  is  called  a  carrier.  (https://ghr.nlm.nih.gov/condition/hemophilia#resources)  

   
5.  Patient  with  multiple  blood  transfusions  and  splenomegaly,  what's  the  cause?  
Answer:  Ankyrin  deficiency  
   
Differential  for  multiple  blood  transfusions  and  splenomegaly:  
Answer:  beta  thalassemia  major  
Patients  with  the  beta  thalassemia  trait  generally  have  no  unusual  physical  findings.  In  patients  with  beta  
thalassemia  major,  the  physical  findings  are  related  to  severe  anemia,  ineffective  erythropoiesis,  
extramedullary  hematopoiesis,  and  iron  overload  resulting  from  transfusion  and  increased  iron  absorption  
Abdominal  examination  may  reveal  changes  in  the  liver,  gallbladder,  and  spleen.  Hepatomegaly  related  to  
significant  extramedullary  hematopoiesis  is  typically  found.  Patients  who  have  received  blood  transfusions  
may  have  hepatomegaly  or  chronic  hepatitis  due  to  iron  overload.  
(https://emedicine.medscape.com/article/206490-­‐clinical?src=refgatesrc1)  
   
6.  Case  about  hematology  all  description  and  investigations  were  consistent  with  hereditary  
spherocytosis,  positive  osmotic  fragility  test  and  jaundice  with  splenomegaly:  
Which  one  of  the  following  is  correct  regarding  this  scenario?  
A-­‐  G6PD  
B-­‐  SPECTRIN  –ANKIRIN  DEFICINCY  
Answer:  B  
most  patients  with  dominant  HS  have  combined  ankyrin  and  spectrin  deficiency  and  that  the  two  proteins  
are  usually  about  equally  deficient.  (https://www.ncbi.nlm.nih.gov/pubmed/8219186)  
(http://www.bloodjournal.org/content/bloodjournal/82/10/2953.full.pdf?sso-­‐checked=true)  
   
   
7.  SCA  mode  of  inheritance?  
Answer:  Autosomal  recessive  
Reference:  STEP  UP  TO  MEDICINE,  pg  333.  
   
8.  Fanconi  anemia  mode  of  inheritance?  
A-­‐  Autosomal  recessive  
B-­‐  Autosomal  dominant  
Answer:  A  
https://ghr.nlm.nih.gov/condition/fanconi-­‐anemia#inheritance  
   
9.  Absorption  of  non-­‐heme  iron  is  enhanced  by  
Answer:  Vitamin  C  
http://ajcn.nutrition.org/content/73/1/93.full  
   
10.  Which  type  of  Hodgkin  lymphoma  has  Reed–Sternberg  cells  and  band  of  collagen  or  fibrosis.  
Which  one  of  the  following  is  the  diagnosis?  
A-­‐  Mixed  cellularity  
B-­‐  Lymphocyte  -­‐predominate  
C-­‐  Lymphocyte  –depleted  
D-­‐  Nodular  sclerosis  
Answer:  D  
Reference:  STEP  UP  TO  MEDICINE,  pg  352  
   
11.  You  gave  a  patient  a  drug  and  asked  him  to  come  back  to  check  PT  and  INR  what  was  the  drug?  
A-­‐  Aspirin  
B-­‐  Warfarin  
C-­‐  Enoxaparin  
D-­‐  UF  heparin    
Answer:  B  
Reference:  STEP  UP  TO  MEDICINE,  pg  348  
   
12.  Factor  7  affects  which  lab  result?  
A  normal  aPTT  and  a  prolonged  PT  in  a  patient  with  a  lifelong  history  of  a  tendency  for  mild  or  severe  
bleeding  is  consistent  with  the  diagnosis  of  factor  VII  deficiency  or  the  presence  of  an  inhibitor  to  factor  VII.  
https://emedicine.medscape.com/article/209585-­‐workup  
   
13.  Scenario  with  a  patient  who  came  with  fever  and  altered  level  of  consciousness  for  5  days.  His  
condition  deteriorated  and  developed  body  rash,  and  became  jaundiced.  Labs  show  high  urea  and  
creatinine,  low  platelets  and  anemia  (Scenario  of  TTP).  What's  the  gene  mutation  responsible?  
Answer:  ADAMTS13  
Mutations  in  the  ADAMTS13  gene  cause  the  familial  form  of  thrombotic  thrombocytopenic  purpura.  The  
ADAMTS13  gene  provides  instructions  for  making  an  enzyme  that  is  involved  in  the  normal  process  of  blood  
clotting.  Mutations  in  this  gene  lead  to  a  severe  reduction  in  the  activity  of  this  enzyme.  The  acquired  form  
of  thrombotic  thrombocytopenic  purpura  also  results  from  a  reduction  in  ADAMTS13  enzyme  activity;  
however,  people  with  the  acquired  form  do  not  have  mutations  in  the  ADAMTS13  gene.  Instead,  their  
immune  systems  often  produce  specific  proteins  called  autoantibodies  that  block  the  activity  of  the  enzyme.  
https://ghr.nlm.nih.gov/condition/thrombotic-­‐thrombocytopenic-­‐purpura#genes  
   
14.  Typical  presentation  of  multiple  myeloma,  what  is  the  diagnostic  test?  
A-­‐  Protein  electrophoresis  
B-­‐  CT  head  
Answer:  A  
The  diagnosis  of  multiple  myeloma  is  determined  by  a  number  of  different  diagnostic  tests,  because  
multiple  myeloma  is  difficult  to  diagnose  on  the  basis  of  any  single  laboratory  test  result.    
-­‐  Serum  and  urine  Protein  electrophoresis:  detect  the  presence  of  M  protein.  
-­‐  Skeletal  survey  (x-­‐ray):  detect  lytic  lesions.  
-­‐  MRI  spine:  detect  spinal  cord  compression.  
-­‐  Bone  marrow  aspiration:  presence  of  plasma  cells  in  the  bone  marrow.    Reference:  Davidson’s,  pg  1047  
   
15.  Which  one  of  the  following  follows  a  Mendelian  inheritance?  
A-­‐  Thalassemia  
Answer:  A  
   
Inheritance   Disease  Examples  
Pattern  

Autosomal   Huntington’s  disease,  neurofibromatosis,  achondroplasia,  familial  


Dominant   hypercholesterolemia,  Marfan  syndrome,  von  Willbrand.  

Autosomal   Tay-­‐sachs  disease,  sickle  cell  anemia,  cystic  fibrosis,  phenylketonuria  (PKU),  
Recessive   Thalassemia,  congenital  adrenal  hyperplasia.  

X-­‐linked   Hypophatemic  rickets  (vitamin  D-­‐resistant  rickets),  


Dominant  

X-­‐linked   Hemophilia  A,  Duchenne  muscular  dystrophy,  G6PD  


Recessive  

Reference:  https://www.ncbi.nlm.nih.gov/books/NBK132145/  
https://ghr.nlm.nih.gov/condition/beta-­‐thalassemia  
   
   
16.  Excessive  ice  taking.  What  do  we  call  it?  
Answer:    Pagophagia  
Pagophagia:  A  craving  to  eat  ice,  often  associated  with  anemia  resulting  from  iron  deficiency  
https://medical-­‐dictionary.thefreedictionary.com/pagophagia  
   
18.  75  years  old  male,  asymptomatic,  with  high  WBC,  90%  lymphocytes.  Immunohistochemistry  
(positive  CD19,  CD23,  CD56).  What’s  the  Treatment?  
A-­‐  No  treatment  
B-­‐  Rituximab  +  CVB  
C-­‐  Rituximab  +  Prednisolone  
D-­‐  Cyclophosphamide  
Answer:  A  
Reference:  (Hematology  resident):  the  patient  has  CLL  (  indolent  type),  he’s  asymptomatic  and  he’s  old,  
there  is  no  need  to  subjecting  the  patient  to  chemotherapy  and  it’s  complication.  
If  the  patient  was  symptomatic,  CD20  positive,  and  has  agood  life  expectancy  we  can  give  him  rituximab,  
and  if  the  patient  is  suffering  from  AIH  we  can  add  steroid  to  the  rituximab.  
   
19.  Treatment  of  von-­‐willbrand  disease?  
A-­‐  FFP  
B-­‐  Cryptoprecipitate  
   
Treatment  of  von-­‐Willbrand:  
-­‐DDAVP  (desmopressin).  
-­‐Factor  VIII  concentrates.  
Cryoprecipitate  is  not  recommended  as  treatment  for  vWD.  
Reference:  STEP  UP  TP  MEDICINE,  pg  342  
DAVIDSON’S,  pg  1055  
                                                                                           
20.  Pencil  cell  in  blood  smear?  
A-­‐  Sickle  cell  anemia  
B-­‐  Thalassemia  
Answer:  Iron  deficiency  anemia  
http://onlinelibrary.wiley.com/store/10.1002/ajh.21283/asset/21283_ftp.pdf;jsessionid=E4B26FAA449B5D
9EA7C4B538AD19756A.f01t03?v=1&t=j9b7kjl0&s=4f19d363b4f2210f0c451fa46d4af0a066430d51  
Cigar  cells  (also  referred  to  as  pencil  cells)  are  red  blood  cells  that  are  cigar  or  pencil  shaped  on  
Peripheral  blood  smear.  Cigar  cells  are  commonly  associated  with  hereditary  elliptocytosis.  However,  
they  may  also  be  seen  in  iron  deficiency  anemia  and  other  pathological  states  that  decrease  red  blood  
[1]
cell  turnover  and  or  production.  In  the  case  of  iron  deficiency  anemia,  microcytosis  and  
hypochromia  would  also  be  expected.  http://www.wikiwand.com/en/Cigar_cell  
   
21.  Recurrent  hemarthrosis  what  is  the  diagnosis?  
Answer:  Hemophilia  
Hemarthrosis  is  the  most  common  musculoskeletal  manifestation  of  hemophilia.  
http://cursoenarm.net/UPTODATE/contents/mobipreview.htm?14/6/14433  
http://cursoenarm.net/UPTODATE/contents/mobipreview.htm?1/34/1568  
   
22.  Long  scenario,  biopsy  showed  starry  sky  appearance.  What  is  the  gene  affected?  
Answer:  C-­‐MYC  (8;14)  
https://emedicine.medscape.com/article/1447602-­‐overview#a4  
   
23.  Picture  of  sickle  cell  anemia  diagnosis  
   
24.  Two  couple  carrier  of  thalassemia,  probability  of  child  to  have  
thalassemia  
A-­‐  25%  
B-­‐  50%  
C-­‐  100%  
D-­‐  75%  
Answer:  A  
   
   
 
 
 
 
 
25.  Adult  patient  with  thrombocytopenia,  high  creatinine  &  Bun  with  purpura.  What  is  the  
diagnosis?  
Answer:  TTP  (correct)  
Thrombotic  thrombocytopenic  purpura  (TTP)  is  a  rare  blood  disorder  characterized  by  clotting  in  small  blood  
 
vessels  of  the  body  (thromboses),  resulting  in  a  low  platelet  count. In  its  full-­‐blown  form,  the  disease  
consists  of  the  following  pentad:  
·∙              Microangiopathic  hemolytic  anemia  
·∙              Thrombocytopenic  purpura  
·∙              Neurologic  abnormalities  
·∙              Fever  
·∙              Renal  disease  
Clinical  differentiation  of  hemolytic-­‐uremic  syndrome  (HUS)  and  TTP  can  be  problematic.  Differentiation  is  
often  based  on  the  presence  of  central  nervous  system  involvement  in  TTP  and  the  more  severe  renal  
involvement  in  HUS.  In  HUS,  an  antecedent  history  of  diarrheal  illness  is  more  often  present.  In  fact,  some  
investigators  suggest  a  clinical  classification  of  HUS  based  on  the  presence  or  absence  of  diarrhea.  
   
26.  ITP  case  
-­‐  Immune  thrombocytopenic  purpura  (ITP)  is  a  clinical  syndrome  in  which  a  decreased  number  of  circulating  
platelets  (thrombocytopenia)  manifests  as  a  bleeding  tendency,  easy  bruising  (purpura),  or  extravasation  of  
blood  from  capillaries  into  skin  and  mucous  membranes  (petechiae).  Although  most  cases  of  acute  ITP,  
particularly  in  children,  are  mild  and  self-­‐limited,  intracranial  hemorrhage  may  occur  when  the  platelet  
9 3    
count  drops  below  10  ×  10 /L  (<10  ×  10 /µL); this  occurs  in  0.5-­‐1%  of  children,  and  half  of  these  cases  are  
fatal.  
   
-­‐ITP  is  a  primary  illness  occurring  in  an  otherwise  healthy  person.  Signs  of  chronic  disease,  infection,  
wasting,  or  poor  nutrition  indicate  that  the  patient  has  another  illness.  Splenomegaly  excludes  the  diagnosis  
of  ITP.  
   
-­‐On  complete  blood  cell  count,  isolated  thrombocytopenia  is  the  hallmark  of  ITP.  Anemia  and/or  
neutropenia  may  indicate  other  diseases.  Findings  on  peripheral  blood  smear  are  as  follows:  
·∙              The  morphology  of  red  blood  cells  (RBCs)  and  leukocytes  is  normal  
·∙              The  morphology  of  platelets  is  typically  normal,  with  varying  numbers  of  large  platelets  
·∙              If  most  of  the  platelets  are  large,  approximating  the  diameter  of  red  blood  cells,  or  if  they  lack  
granules  or  have  an  abnormal  color,  consider  an  inherited  platelet  disorder  
Many  children  with  acute  ITP  have  an  increased  number  of  normal  or  atypical  lymphocytes  on  the  
peripheral  smear,  reflecting  a  recent  viral  illness.  
   
27.  Most  common  cause  of  iron  deficiency  anemia?  
Causes  of  iron  deficiency:  
-­‐  Lack  of  iron  in  your  diet.  
-­‐  Chronic  blood  loss.  
-­‐  Pregnancy  
-­‐  Inability  to  absorb  iron  
https://www.mayoclinic.org/diseases-­‐conditions/iron-­‐deficiency-­‐anemia/symptoms-­‐causes/syc-­‐20355034  
https://emedicine.medscape.com/article/202333-­‐overview#a3  
   
28.  Patient  on  warfarin  came  with  melena  and  PT  high  what  is  the  treatment?  
A-­‐  IV  vitamin  k  
B-­‐  Fresh  frozen  plasma  
C-­‐  Protamine  sulphate  
Answer:  A  
   
29.  Vitamin  k  dependent  factors  
Vitamin  K  serves  as  an  essential  cofactor  for  a  carboxylase  that  catalyzes  carboxylation  of  glutamic  acid  
residues  on  vitamin  K-­‐dependent  proteins.  The  key  vitamin  K-­‐dependent  proteins  include:  
●   Coagulation  proteins:  factors  II  (prothrombin),  VII,  IX  and  X  
●   Anticoagulation  proteins:  proteins  C,  S  and  Z  
   
30.  Patient  with  symptoms  of  gastritis  and  has  anemia.  How  would  you  treat?  
A-­‐  Oral  iron  
B-­‐  IM  iron  
C-­‐  IV  iron  
Answer:  C  
   
31.  What  determines  the  prognosis  in  CLL?  
A-­‐  Stage  of  the  disease  
B-­‐  Age  at  the  time  of  diagnosis  
C-­‐  Bone  marrow  aspiration  
ANSWER:Rai/binet  staging  
The  prognosis  (chance  of  recovery)  depends  on:  
·∙         Whether  there  is  a  change  in  the  DNA  and  the  type  of  change,  if  there  is  one.  
·∙              Whether  lymphocytes  are  spread  throughout  the  bone  marrow.  
·∙              The  stage  of  the  disease.  
·∙              Whether  the  CLL  gets  better  with  treatment  or  has  recurred  (come  back).  
·∙         Whether  the  CLL  progresses  to  lymphoma  or  prolymphocytic  leukemia.  
·∙              The  patient's  general  health.  
   
   
32.  Picture  of  histopathology  of  tumor  with  starry  sky  pattern  
Answer:  EBV  
https://emedicine.medscape.com/article/1447602-­‐overview#a4  
   
33.  K/C  of  Sickle  cell  anemia  presented  with  chest/back/hip  pain.  He  reports  having  previous  
episodes  and  was  hospitalized  for  them.  Now  in  severe  pain.  What  to  do?  
A.  Give  oral  narcotics  and  follow  up.                                                                                              
B.  Admission  with  pain  management.                                                                                                                
C.  Refer  to  tertiary  hospital  
Did  not  find  a  specific  answer.  
Answer:  B  
Oral  medication  is  not  enough  in  such  case.  Referral  is  right  thing  to  do  but  not  for  acute  management.  
So  you  admit  and  manage  first,  then  refer.  (Opinion)  
For  sickle  cell  crisis,  when  the  severity  of  the  episode  is  assessable,  self-­‐treatment  at  home  with  bed  rest,  
oral  analgesia,  and  hydration  is  possible.  Individuals  with  SCD  often  present  to  the  emergency  department  
(ED)  after  self-­‐treatment  fails.  Do  not  underestimate  the  patient's  pain.  Failure  to  treat  acute  pain  
aggressively  and  promptly  may  lead  to  chronic  pain  syndrome.  According  to  the  2003  BCHS  acute  painful  
crisis  guidelines,  these  patients  should  receive  analgesia  within  30  minutes  of  entering  the  hospital,  with  the  
goal  of  achieving  effective  pain  control  by  60  minutes.  
If  patients  with  SCD  crisis  are  being  transported  by  emergency  medical  services  (EMS),  they  should  receive  
supplemental  oxygen  and  intravenous  hydration  en  route  to  the  hospital.  Some  areas  have  specialized  
facilities  that  offer  emergency  care  of  acute  pain  associated  with  SCD;  many  EDs  have  a  standardized  
treatment  plan  in  place.  https://emedicine.medscape.com/article/205926-­‐treatment  
   
34.  What  is  the  characteristic  for  von-­‐willbrand  disease?  
A-­‐  Prolonged  PT  
B-­‐  Prolonged  PTT  
C-­‐  Factor  12  deficiency  
D-­‐  Factor  VIII  deficiency  
Answer:  D  
https://emedicine.medscape.com/article/206996-­‐overview  
   
35.  Patient  on  warfarin  5  mg,  his  INR  7.  No  signs  of  active  bleeding,  what  to  do?  
A-­‐  Increase  warfarin  dose  to  7.5  mg  
B-­‐  Decrease  warfarin  dose  to  2.5  mg  
C-­‐  Hold  warfarin  for  one  day  and  measure  INR  again  
D-­‐  Continue  the  same  dose.  
Answer:  C  
https://emedicine.medscape.com/article/2172018-­‐overview  
   
36.  Sickler  patient  with  splenic  sequestration?  
Splenic  sequestration  occurs  with  highest  frequency  during  the  first  5  years  of  life  in  children  with  sickle  cell  
anemia.  Splenic  sequestration  can  occur  at  any  age  in  individuals  with  other  sickle  syndromes.  This  
complication  is  characterized  by  the  onset  of  life-­‐threatening  anemia  with  rapid  enlargement  of  the  spleen  
and  high  reticulocyte  count.  
Splenic  sequestration  is  a  medical  emergency  that  demands  prompt  and  appropriate  treatment.  Parents  
should  be  familiar  with  the  signs  and  symptoms  of  splenic  sequestration  crises.  Children  should  be  seen  as  
rapidly  as  possible  in  the  emergency  room.  Treatment  of  the  acute  episode  requires  early  recognition,  
careful  monitoring,  and  aggressive  transfusion  support.  Because  these  episodes  tend  to  recur,  many  
advocate  long-­‐term  transfusion  in  young  children  and  splenectomy  in  older  children.  
https://emedicine.medscape.com/article/205926-­‐clinical  
   
37.  How  to  confirm  the  diagnosis  of  patient  with  thalassemia?  
A-­‐  Level  of  hemoglobin  A2  
B-­‐  Genetic  profiling  
Answer:  A  
Elevation  of  the  Hb  A2  level,  demonstrated  by  electrophoresis  or  column  chromatography,  confirms  the  
diagnosis  of  beta  thalassemia  trait.  https://emedicine.medscape.com/article/206490-­‐workup#c7  
   
38.  Patient  with  polycythemia  vera  with  all  values  elevated  Hgb,  WBC  and  platelets.  what's  the  
treatment?  
A-­‐  Phlebotomy  
B-­‐  Meylosuppressive  treatment  
Answer:  A  
All  patients  with  PV  should  undergo  phlebotomy  to  keep  their  hematocrit  below  45%.  Lower  hematocrit  
targets  have  been  proposed  for  women  with  PV,  but  no  empiric  evidence  supports  that  
recommendation.https://emedicine.medscape.com/article/205114-­‐treatment  
   
39.  What  type  of  anemia  that  those  who  are  on  chemotherapy  get?  
Answer:  Aplastic  
·∙        Radiation  and  chemotherapy  treatments.  While  these  cancer-­‐fighting  therapies  kill  cancer  cells,  
they  can  also  damage  healthy  cells,  including  stem  cells  in  bone  marrow.  Aplastic  anemia  can  be  a  
temporary  side  effect  of  these  treatments.  https://www.mayoclinic.org/diseases-­‐conditions/aplastic-­‐
anemia/symptoms-­‐causes/syc-­‐20355015  

   
40.  Patient  presented  with  fatigue  and  weight  loss.  Lab  show  Hgb  12,  platelets  200,  WBCs  19000  
and  peripheral  blood  film  showed  neutrophilia,  basophilia,  promyelcytes  and  myelocytes.  What's  
the  next  step?  
A-­‐  Flow  cytometry  
B-­‐  Bone  marrow  biopsy  and  aspiration  
C-­‐  Repeat  CBC  
Answer:  A  
CML,  so  you  do  flow  cytometry  to  check  for  Philadilphia  chromosome  (Opinion)  
   
50.  How  to  monitor  the  response  of  iron  treatment?  
A-­‐  Ferritin  
B-­‐  Reticulocyte  count  
C-­‐  Hematocrit  
Answer:  B  
Response  to  iron  therapy  can  be  documented  by  an  increase  in  reticulocytes  5-­‐10  days  after  the  initiation  of  
iron  therapy.  The  hemoglobin  concentration  increases  by  about  1  g/dL  weekly  until  normal  values  are  
restored.  https://emedicine.medscape.com/article/202333-­‐treatment#d13  
In  addition  to  hemolysis,  increased  reticulocytes  may  be  a  response  to  blood  loss  or  the  treatment  of  iron,  
vitamin  B-­‐12,  or  folate  deficiencies.  https://emedicine.medscape.com/article/201066-­‐workup#c8  
   
51.  Patient  with  cut  that  is  continuously  bleeding.  Prolonged  BT  and  thrombocytopenia.  Bleeding  
time  normalizes  with  platelet  transfusion  but  not  FFP.  
A-­‐  Von  Willebrand  
B-­‐  Bernard  Soulier  
C-­‐  Hemophilia  
Answer:  A  
   
52-­‐  Most  important  test  in  Von  Willebrand?  
A-­‐  PT  
B-­‐  PTT  
C-­‐  BT  
Answer:  C  
   
53.  Patient  on  heparin  what  should  be  monitored?                        
A-­‐  PT  
B-­‐  INR                      
C-­‐  PTT      
Answer:  C  
   
54.  Patient  has  night  sweating,  weight  loss,  fever,  and  lymph  node  enlargement  in  the  left  
subclavicular  lymph  node.  What  is  the  diagnosis?  
A-­‐  Burkitt  lymphoma                                                
B-­‐  Hodgkin  lymphoma                                                                    
C-­‐  Nonhodgkin  lymphoma  
I  believe  the  question  is  incomplete,  we  can  not  differentiate  between  Hodgkin  and  non  Hodgkin  lymphoma  
without  examining  the  cell  under  the  microscope.  I  would  go  with  Hodgkin  lymphoma  since  they  have  
higher  tendency  to  present  with  B  symptoms.                    
   
55.  Patient  has  pallor  and  dyspnea  
CBC:  macrocytic  anemia  
 
AST:  High.  Blood  film:  no  megablast.  What  is  the  cause?  
A-­‐  B12  deficiency  
B-­‐  Folate  deficiency  
C-­‐  Drug  induced  
D-­‐  Alcohol  abuse                                                              
Answer:  Alcohol  abuse  
Although  the  mean  corpuscular  volume  is  increased  by  recognized  causes  that  include  vitamin  B12  deficiency  
or  folic  acid  deficiency,  it  is  also  raised  by  high  alcohol  intake  alone.  
https://www.medscape.com/viewarticle/562546_2  
   
56.  Scenario  for  ITP,  what  you  see  in  bone  marrow?  
Answer:  Increase  Megakaryocytes  
The  number  of  megakaryocytes  may  be  increased.  Because  the  peripheral  destruction  of  platelets  is  
increased,  megakaryocytes  may  be  large  and  immature,  although  in  many  cases  the  megakaryocyte  
morphology  is  normal  https://emedicine.medscape.com/article/202158-­‐workup#c7  
   
57.  Patient  with  knee  trauma,  bleeding  in  cut  with  increased  bleeding  time.  He  was  given  FFP  with  
not  improved  but  improved  with  platelet  transfusion?  
A-­‐  VWD  
B-­‐  TTP                                              
C-­‐  DIC  
Answer:  A  
   
58.  Most  common  type  of  Hodgkin  lymphoma?  
A-­‐  Lymphocytes  predominant  
B-­‐  Lymphocytes  depleted  
C-­‐  Mixed  cellularity  
D-­‐  Nodular  sclerosing  
Answer:  D  
Reference:  DAVIDSON’S,  pg  1042  
 
59.  A  girl  presented  with  fatigue  malaise  after  doing  full  investigation  she  had  +ve  sickling  test  
A-­‐  Aplastic  crisis  
B-­‐  Vasoocclusive  crisis  
Answer:  A  
A  serious  complication  is  the  aplastic  crisis.  This  is  caused  by  infection  with  Parvovirus  B-­‐19  (B19V).  This  
virus  causes  fifth  disease,  a  normally  benign  childhood  disorder  associated  with  fever,  malaise,  and  a  mild  
rash.  https://emedicine.medscape.com/article/205926-­‐clinical  
 
60.  Scenario  of  patient  with  anemia,  blood  film  showed  brown  color.  What  is  the  diagnosis?  
A-­‐  Aplastic  anemia  
B-­‐  Hemolytic  anemia  
Not  enough  information  !  
 
61.  34  years  old  female  presented  with  fever  and  decrease  consciousness  for  6  hours.  
Investigations  shows  low  platelets  only.  What  is  the  diagnosis?  
A-­‐  ITP  
B-­‐  TTP  
C-­‐  Antiphospholipd  syndrome  
Answer:  B  
62.  20  years  old  male,  with  small  erythematous  non-­‐planchable  macules,  history  of  viral  
respiratory  infection  resolve  spontaneously  last  week,  Lab:  platelets  15  (very  low).  What  is  the  
treatment?                                          
A-­‐  IVIG  
B-­‐  Splenectomy  
C-­‐  Platelets  transfusion  
-­‐the  answer  is  corticosteroids  (ie,  oral  prednisone,  intravenous  [IV]  methylprednisolone,  or  high-­‐dose  
 
dexamethasone) should  remain  the  drugs  of  choice  for  the  initial  management  of  acute  ITP  
-­‐Treatment  with  corticosteroids  may  not  only  reduce  the  rate  of  platelet  destruction  but  may  also  rapidly  
alter  endothelial  cell  integrity  to  facilitate  primary  hemostasis  and  to  reduce  bleeding  and  bruising.  
 
-­‐IV  immunoglobulin  (IVIG)  has  been  the  drug  of  second  choice  (after  corticosteroids)  for  many  years.  
https://emedicine.medscape.com/article/202158-­‐treatment  
63.  Patient  with  features  of  leukemia,  Lab  shows:  blast  cell;  high.  Aure  rodes  in  blood  film.  What  is  
diagnosis?                                                                    
A-­‐  AML                                                                                                              
B-­‐  Aplastic  anemia  
C-­‐  CML  
Answer:  A  
Reference:  STEP  UP  TO  MEDICINE,  pg  356  
64.  Management  of  HSP?  
Answer:  Supportive  
To  date,  no  form  of  therapy  has  been  found  to  shorten  the  duration  of  Henoch-­‐Schönlein  purpura  (HSP)  to  
any  significant  degree.  Therefore,  treatment  remains  primarily  supportive  in  most  cases.  
Management  of  HSP  includes  adequate  hydration;  immediate  discontinuance  of  any  exposure  to  antigenic  
stimulants  (eg,  drugs);  and  follow-­‐up  each  week  for  the  first  month,  every  other  week  for  the  second  month,  
and  monthly  thereafter  until  abnormal  urinary  findings  subside.  
https://emedicine.medscape.com/article/984105-­‐treatment  
   
65.  What  causes  false  positive  sickling  test?  
A-­‐  Anemia  
B-­‐  High  protein  
Answer:  B  
   
66.  What  is  the  cause  of  hyposeplenism?  
A-­‐  Sickle  cell  
B-­‐  Spherocytosis  
Answer:A  
   
67.  A  case  mentioning  presence  of  Reed–Sternberg  cells.  What  is  the  diagnosis?  
 A-­‐  Hodgkin  lymphoma  
B-­‐  Non-­‐Hodgkin  lymphoma  
Answer:  A  
Reference:  DAVIDSON’S,  pg  1042  
   
68.  Patient  known  case  of  sarcoidosis  for  2  years,  labs  showing  anemia.  What  type  of  anemia  he  
most  likely  has?  
Answer:  Anemia  of  chronic  disease  
Anemia  of  chronic  disease:  
This  type  of  anemia  occurs  as  part  of  a  chronic  inflammatory  disorder,  most  often  chronic  infection,  
autoimmune  disease  (especially  RA),  kidney  disease,  or  cancer;  however,  the  same  process  appears  to  begin  
acutely  during  virtually  any  infection  or  inflammation,  including  trauma  or  post-­‐surgery.  (See  also  Anemia  of  
Renal  Disease.)  http://www.msdmanuals.com/professional/hematology-­‐and-­‐oncology/anemias-­‐caused-­‐by-­‐
deficient-­‐erythropoiesis/anemia-­‐of-­‐chronic-­‐disease  
Another  answer  I  found  was  a  rare  association  between  sarcoidosis  and  cold  autoimmune  hemolytic  
anemia.  
https://www.omicsonline.org/open-­‐access/sarcoidosis-­‐and-­‐cold-­‐autoimmune-­‐hemolytic-­‐anemia-­‐a-­‐rare-­‐
association-­‐2329-­‐8790-­‐1000255.php?aid=80787&view=mobile  
   
69.  Patient  started  to  bleed  from  IV  line  and  other  orifices  
Answer:    DIC  
clinical  features  of  DIC:  
-­‐bleeding  tendency:  
-­‐                superficial  hemorrhage  (ecchymosis,  petechiae,  purpura).  
-­‐                bleeding  from  GI  tract,  Urinary  tract,  gingival  or  oral  mucosa.  
-­‐                oozing  from  site  of  procedures  and  incisions.  
-­‐thrombosis:  
1.          occur  more  often  in  chronic  cases.  
Reference:  STEP  UP  TP  MEDICINE,  pg  344.  
   
70.  Splenomegaly,  high  hgb,  plt?  
Answer:  polycythemia  rubra  vera  
Reference:  STEP  UP  TO  MEDICINE,  pg  358-­‐359  
   
71.  Scenario  about  patient  came  from  Ghana  
Answer:  Burkitt  lymphoma  
   
72.  What  type  of  anemia  is  associated  with  RA?  
Answer:  Normocytic  normochromic  
Patients  with  rheumatoid  arthritis  (RA)  may  exhibit  a  variety  of  hematologic  abnormalities.  Common  
changes  associated  with  active  disease  include  anemia  of  chronic  disease  
http://www.uptodate.com/contents/hematologic-­‐manifestations-­‐of-­‐rheumatoid-­‐arthritis#H3  
anemia  of  chronic  illness  and  chronic  kidney  disease  usually  fall  under  the  classification  of  normochromic,  
normocytic  anemia.  https://emedicine.medscape.com/article/1389854-­‐overview  
   
73.  Patient  on  anticoagulant,  INR:1,  PTT  &  platelet  within  normal,  and  was  given  appointment  
after  2  weeks  to  reevaluate  the  result,  what  is  the  drug?  
A-­‐  Aspirin  
B-­‐  Warfarin  (correct)  
C-­‐  LMWH  
D-­‐  Unfractionated  heparin  
Answer:  B  
   
75.  Case  scenario  of  patient  with  history  of  URTI.  lab  results:  low  Hbg,  high  WBC,  I  think  
reticulocyte  was  within  range  or  decrease,  what  is  the  investigation  you  want  to  do?  
A-­‐  Electrophoresis  
B-­‐  Bone  marrow  biopsy  
I  would  say  it’s  Cold  agglutinin  disease,  so  we  should  do  Coombs  test.  
   
76.  Case  scenario  of  patient  with  history  of  URTI.  lab  results:  low  Hbg,  high  WBC,  I  think  
reticulocyte  was  within  range  or  decrease,  what  is  the  diagnosis?  
A-­‐  Aplastic  anemia  
B-­‐  Hypoplastic  
C-­‐  Immune  hemolytic  
Answer:  C  (Opinion)  
   
78.  asking  about  Ann  Arbor  staging  of  burkitt’s  lymphoma  (multiple  lymph  node  in  same  side  of  
diaphragm)  
A-­‐  Stage  1  
B-­‐  Stage  2  
C-­‐  Stage  3  
D-­‐  Stage  4  
Answer:  B  
 

Stage   Area  of  Involvement  

I   Single  lymph  node  group  


II   Multiple  lymph  node  groups  on  same  side  of  diaphragm  

II   Multiple  lymph  node  groups  on  both  sides  of  diaphragm  

IV   Multiple  extranodal  sites  or  lymph  nodes  and  extranodal  disease  

https://emedicine.medscape.com/article/2007081-­‐overview  
   
   
79.  60  years  old  man  coming  with  tachypnea.  Found  to  have  low  Hg  and  high  LDH  all  other  labs  
were  normal.  What  is  the  diagnosis?  
A-­‐  Aplastic  anemia  
B-­‐  Hemolytic  anemia  
Answer:  B  
Serum  LDH  elevation  is  a  criterion  for  hemolysis.  
LDH  elevation  is  sensitive  for  hemolysis,  but  is  not  specific  since  LDH  is  ubiquitous  and  can  be  released  from  
neoplastic  cells,  the  liver,  or  from  other  damaged  organs.  https://emedicine.medscape.com/article/201066-­‐
workup#c10  
   
80.  Question  about  pernicious  anemia  
Answer:  B12  deficiency  due  to  intrinsic  factor  deficiency  
   
81.  Hook  worm,  how  to  cause  anemia?  
A-­‐  Destruction  of  RBC  
B-­‐  Send  toxins  to  bone  marrow  
C-­‐  Compete  with  host  on  b  12  
Answer:  A  
·∙              Classic  hookworm  disease  -­‐  This  is  a  gastrointestinal  (GI)  infection  characterized  by  chronic  blood  loss  
that  leads  to  iron-­‐deficiency  anemia  and  protein  malnutrition;  it  is  caused  primarily  by  N  americanus  and  A  
duodenaleand  less  commonly  by  the  zoonotic  species  A  ceylonicum  
 [ 6]  
·∙              Intestinal  blood  loss  secondary  is  the  major  clinical  manifestation  of  hookworm  infection. In  fact,  
hookworm  disease  historically  refers  to  the  childhood  syndrome  of  iron  deficiency  anemia,  protein  
malnutrition,  growth  and  mental  retardation  with  lethargy  resulting  from  chronic  intestinal  blood  loss  
secondary  to  hookworm  infection  in  the  face  of  an  iron  deficient  diet.  
·∙              Hookworms  ingest  and  digested  some  of  the  blood  from  the  injured  mucosa  by  means  of  a  multienzyme  
cascade  of  metallohemoglobinases.  Each  Necator  worm  ingests  0.03  mL  of  blood  daily,  whereas  each  
Ancylostoma  worm  ingests  0.15-­‐0.2  mL  of  blood  daily.  Inhibited  host  coagulation  due  to  a  series  of  
anticoagulants  directed  against  factor  Xa  and  the  factor  VIIa–tissue  factor  (TF)  complex,  as  well  as  against  
platelet  aggregation,  further  exacerbates  blood  loss.  
   
   
82.  Known  case  of  sarcoidosis,  presented  with  signs  of  anemia.  From  investigation  he  has  iron  
deficiency  anemia.  What  is  the  pathophysiology?  
A-­‐  Decreased  hepcidin  
B-­‐  Increased  hepcidin  
Answer:  B  
The  hepatic  peptide  hepcidin  was  identified  as  the  systemic  iron-­‐regulatory  hormone.  In  the  efferent  arc,  
hepcidin  regulates  intestinal  iron  absorption,  plasma  iron  concentrations,  and  tissue  iron  distribution  by  
inducing  degradation  of  its  receptor.  Increased  hepcidin  concentrations  in  plasma  are  pathogenic  in  iron-­‐
restrictive  anemias  including  anemias  associated  with  inflammation,  chronic  kidney  disease  and  some  
cancers.  Hepcidin  deficiency  causes  iron  overload  in  hereditary  hemochromatosis  and  ineffective  
erythropoiesis.  
https://www.ncbi.nlm.nih.gov/pmc/articles/PMC4048856/  
 
 83.  Most  common  presentation  in  hemophilia  
A-­‐  Hemarthrosis  
B-­‐  Petechiae  
 Answer:    
 
84.  Case  of  patient  tired,  his  peripheral  blood  show  spherocytes.  
Labs:  low  Hgb  70,  low  platelets  <  15.  What  is  the  treatment?  
A-­‐  Platelet  infusion  
B-­‐  Plasma  exchange                                                                                                        
C-­‐  IV  acyclovir  
   Answer:    
 
85.  Poor  prognostic  factors  of  MM?  
A-­‐  High  calcium  
B-­‐  Low  IgA  
Answer:    
   
86.  Wiskott-­‐Aldrich  Syndrome  typical  symptoms  (diagnosis)?  
A-­‐  Eczema,  thrombocytopenia  and  pyogenic  infections.  
                                                                                                                                                 
87.  Wiskott-­‐Aldrich  Syndrome  mode  of  inheritance?  
A-­‐  X-­‐linked  recessive  
   
88.  DIC  
A-­‐  Prolong  APTT  and  PT  ,  improve  by  platelet  
   
89.  VWD  
A-­‐  Prolong  bleeding  time  and  improve  by  FFP  and  platelet  but  FFP  should  be  in  larger  amount  
   
90.  Male  patient  complaining  of  back  pain.  Imaging  shows  bony  lytic  lesions  on  multiple  levels  
with  moth  eaten  appearance.  Diagnosis  by?  
A-­‐  Serum  protein  electrophoresis  
B-­‐  Bone  scan  
 Answer:    
 
91.  What  are  you  going  to  find  in  the  blood  film  of  sickle  cell  anemia  patient?    
A-­‐  Spherocytosis  &  shictocytes  
B-­‐  Pencile  cells  
C-­‐  Howell-­‐Jolly  body  
D-­‐  Target  cells  
 Answer:    
 
92.  Characteristic  feature  of  NHL?  
A-­‐  Extranodal  involvement  
 Answer:    
 
93.  Sickle  cell  disease  triggering  to  occlusive  crisis?  
A-­‐  Parvo  virus  B19  
B-­‐  EBV  
Answer:    

Nephrology  
 
1.  Which  one  of  the  following  present  with  positive  anti–glomerular  basement  membrane  
antibody?  
A-­‐  IgA  nephropathy  
B-­‐  Post  streptococcal  glomerulonephritis  
C-­‐  SLE  
D-­‐  Goodpasture  syndrome  
Answer:  D  
Reference:  Toronto  Notes  2017  +  Medscape  
Goodpasture  Syndrome  associated  with  antibodies  against  type  IV  collagen  present  in  lungs  and  
GBM  
   
2.  Case  scenario  of  nephrotic  syndrome  and  asking  about  treatment?  
A-­‐  Prednisone  
B-­‐  Azathioprine  
Answer:  A  
Reference:  Kaplan  Lecture  Notes  
Treatment  of  nephrotic  syndrome  is  to  control  underlying  disease.  In  addition,  steroids  are  used  to  
treat  all  forms  of  idiopathic  primary  renal  causes  of  nephrotic  syndrome,  Such  as  membranous,  
minimal  change  disease,  membranoproliferative,  mesangial,  and  focal  segmental  disease.  
If  steroids  do  not  work,  the  next  best  step  in  therapy  is  to  add  cyclophosphamide  or  mycophenolate.  
Azathioprine  is  sometimes  useful.  ACEIs  or  ARBs  are  used  for  all  patients  with  proteinuria,  but  they  
do  not  reserve  the  underlying  disease.  
   
3.  Patient  presented  with  hematuria,  on  examination  he  had  bilateral  mass.  What  is  the  most  
likely  diagnosis?  
A-­‐  Polycystic  kidney  disease  
Answer:  A  
Reference:  Toronto  Notes  2017  
Polycystic  changes  are  always  bilateral.  It  is  often  asymptomatic,  but  patients  may  present  with  
abdominal  flank  pain/dull  lumbar  back  pain,  hematuria,  nocturia,  HTN  +/-­‐  palpable  kidneys.  
   
4.  Athlete  man  with  hematuria  and  calf  muscle  pain?  
A-­‐  Rhabdomyolysis  
Answer:  A  
Reference:  WebMD  
The  “classic  triad”  of  rhabdomyolysis:  muscle  pain  in  the  shoulders,  thighs,  or  lower  back;  muscle  
weakness  or  trouble  moving  arms  and  legs;  and  dark  red  or  brown  urine  or  decreased  urination.  
   
5.  Patient  drank  an  ethylene  glycol  containing  product,  what  renal  damage  is  suspected?  
A-­‐  Acute  tubular  necrosis  
Answer:  A  
Reference:  Toronto  Notes  2017  
Etiology  of  ATN  
1.  Toxins  
    a.  Exogenous:  antibiotics  (aminoglycosides,  cephalosporins,  amphotericin  B),  antiviral  
(cidofovir),  antineoplastics  (cisplatin,  methotrexate),  contrast  media,  heavy  metals,  other  
(fluorinated  anesthetic,  ethylene  glycol)  
    b.  Endogenous:  endotoxins  (bacterial),  myoglobin,  hemoglobin    
2.  Ischemia  
    a.  Decreased  circulating  volume  
    b.  Decreased  effective  circulating  volume  
    c.  Vessel  occlusion  
   
6.  Patient  with  vomiting  and  hyperventilation,  what  acid  base  disturbance  will  the  patient  have?  
A-­‐  Metabolic  acidosis  and  respiratory  alkalosis  
B-­‐  Metabolic  acidosis  and  respiratory  acidosis  
C-­‐  Metabolic  alkalosis  and  respiratory  acidosis  
D-­‐  Metabolic  alkalosis  and  respiratory  alkalosis  
Answer:  D  
Reference:  Toronto  Notes  2017  
   
7.  Minimal  change  glomerulonephritis  
A-­‐  LM:  normal,  EM:  fusion  of  foot  process  
Answer:  A  
Reference:  Kaplan  Lecture  Notes  
   
8.  Low  bicarb,  low  ph.  What  is  the  diagnosis?  
A-­‐  Metabolic  acidosis  
B-­‐  Metabolic  alkalosis  
C-­‐  Respiratory  acidosis  
D-­‐  Respiratory  alkalosis  
Answer:  A  
   
9.  Patient  with  tender  flank  and  mutation  in  chromosome  16.  What  is  the  mode  of  inheritance?  
A-­‐  Autosomal  dominant  
B-­‐  Autosomal  recessive  
 Answer:  A  
Reference:  Toronto  Notes  2017  
Adult  polycystic  kidney  disease  is  an  autosomal  dominant  disease;  
at  least  2  genes:  PKD1  (chr  16p)  and  PKD2  (chr  4q)  
   
10.  Young  Patient  with  hemoptysis,  week  later  he  developed  hematuria.  What  is  the  diagnosis?  
A-­‐  Goodpasture  syndrome  
B-­‐  HSP  
C-­‐  PSGN  
Answer:  A  
Reference:  Kaplan  Lecture  Notes  
Goodpasture  syndrome  is  idiopathic  disorder  of  renal  and  lung  disease.  Patients  may  present  with  
hematuria,  proteinuria,  hemoptysis,  cough,  and  SOB.  
 
11.  Clear  case  of  post-­‐streptococcal  glomerulonephritis.  What  is  the  most  important  step  in  
management?  
A-­‐  High  dose  corticosteroids  
B-­‐  Diuretics  for  high  blood  pressure  
C-­‐  Diuretics  for  edema  
D-­‐  Antibiotics  
Answer:  C  
Reference:  Nephrology  consultant  
If  the  patient  has  active  infection,  the  answer  would  be  antibiotics.  If  not,  then  choose  diuretics.  
   
12.  Patient  with  recurrent  UTI,  bilateral  hydronephrosis.  Renal  biopsy  showed  multiple  cysts  with  
some  description.  What  is  the  diagnosis?  
A-­‐  Renal  cell  carcinoma  
B-­‐  Polycystic  kidney  disease  
C-­‐  Renal  dysplasia  
Answer:  B  
   
13.  20  years  old  male  with  history  of  hematuria  and  proteinuria  6  gm.  What  you  will  see  in  renal  
biopsy  (history  of  post-­‐streptococcal  glomerulonephritis)?  
A-­‐  Membrane  
B-­‐  Minimal  
C-­‐  Post  infection  
Answer:  Renal  biopsy  will  show  humps  on  EM.  IgG  and  C3  will  be  deposited  in  the  mesangium  as  
subepithelial  humps  
Reference:  Kaplan  Lecture  Notes  
For  more  details  about  the  histologic  findings  -­‐  https://emedicine.medscape.com/article/240337-­‐
workup#c7  
   
14.  Patient  woke  up  with  edema  under  eye  +  oral  dehydrated.  Labs:  hypoalbuminemia,  
hyperlipidemia.  What  is  the  treatment?  
A-­‐  Aspirin  
B-­‐  Prednisone  
C-­‐  Cyclophosphamide  
D-­‐  Cyclosporine  
Answer:  B  
Reference:  Kaplan  Lecture  Notes  
   
15.  Picture  of  patient  with  macular  rash  in  buttocks,  associated  with  bilateral  knee  pain  (Henoch-­‐
Schonlein  Purpura).  How  would  you  treat  this  disease?  
A-­‐  Antibiotics  
B-­‐  IVIG  
C-­‐  Corticosteroids  
Answer:  C  
Reference:  Kaplan  Lecture  Notes  
HSP  most  often  resolves  spontaneously.  Treatment  is  supportive.  Steroids  can  be  used  if  the  disease  
is  progressive  and  severe.  
   
16.  Case  of  glomerulonephritis,  which  of  these  considered  a  strong  support  of  your  diagnosis?      
A-­‐  Streptozyme  test  
B-­‐  Low  C3  
Answer:  It  depends  on  the  case.  However,  the  single  most  important  test  is  renal  biopsy.  
Reference:  Nephrology  consultant  
                                                                   
17.  Typical  history  of  SLE  with  urine  casts.  What  is  the  diagnosis?  
A-­‐  SLE  nephropathy  
Answer:  A  
   
18.  Patient  taking  diuretics,  he  has  muscle  weakness,  diarrhea,  irritability.  What  is  the  cause?  
A-­‐  Hypokalemia  
B-­‐  Hyperkalemia  
C-­‐  Hyponatremia  
D-­‐  Hypocalcemia  
Answer:  A  
Reference:  Nephrology  consultant  
   
19.  Patient  have  bilateral  thin  walled  parenchyma  of  kidney?  
A-­‐  PKD  
Answer:  A?  
In  general,  thin-­‐walled  parenchyma  of  kidney  indicates  the  presence  of  cysts.  
Upon  searching  online  I  found  the  following  possible  answers  (if  not  in  choices  I  would  choose  A)  
1.  Simple  renal  cysts  
2.  Lymphangioma  in  adults  is  usually  characterized  by  peripelvic  and  perirenal  thin-­‐walled  cysts  
   
20.  Patient  with  hypokalemia,  Hypochloremia,  hyponatremia  and  HyperHCO3.  I  think  with  some  
symptoms,  what's  the  1ry  defect?  
A-­‐  NaCl  
B-­‐  H  excretion                                                                                                                                                      
C-­‐  H  absorption  
D-­‐  K  
Answer:  incomplete  question,  but  the  above  labs  could  be  caused  by  vomiting  
Reference:  Nephrology  consultant  
   
21.  Patient  with  muscle  weakness,  apart  from  hypotension  and  bradycardia,  his  examination  was  
normal.  His  lab  tests  (high  K+,  low  Na,  low  Chloride,  high  urea).  What  is  the  etiology  behind  his  
presentation?  
A.  Hyponatremia  
B.  Hyperkalemia  
C.  Low  chloride  
D.  Uremia  
Answer:  the  question  is  incorrect  (nephrology  consultant)  
   
22.  Patient  presented  with  hematuria,  weight  loss,  loin  pain  (scenario  of  RCC).  What  is  the  gene  
affected?  
A-­‐  VHL  
Answer:  A  
Reference:  
1-­‐  https://www.ncbi.nlm.nih.gov/pmc/articles/PMC2873025/  
2-­‐  https://www.ncbi.nlm.nih.gov/pubmed/22125026  
Alteration  of  the  VHL  gene  by  mutation,  loss  of  heterozygosity,  and  promoter  methylation  has  been  
found  to  be  important  to  renal  cell  cancer  pathogenesis.  
   
23.  Scenario  of  a  patient  whose  thirsty  with  low  urine  output  because  of  ADH  secretion.  (They  
attached  a  picture  of  a  nephrons  with  number  labels  on  each  segment  and  asked  to  choose  which  
area  does  ADH  work  on).  
A-­‐  Collecting  duct  (was  labeled  by  number  5  in  the  picture)  
Answer:  A  
Reference:  Toronto  Notes  2017  
   
   
   
   
   
 

 
   
   
   
23.  Patient  with  controlled  DM  and  HTN  he  is  on  ACEI,  furosemide  and  spironolactone.  Electrolyte  
were  normal,  what  should  you  do?  
A-­‐  Stop  spironolactone  
B-­‐  Stop  furosemide  
Answer:  A  
Reference:  https://www.rxlist.com/aldactone-­‐drug.htm  
   
24.  Renal  cell  carcinoma  marker?  
A-­‐  M2-­‐PK  
Answer:  ?  
Reference:  
1-­‐  https://www.ncbi.nlm.nih.gov/pubmed/3776772  
ESR  and  fibrinogen  were  thought  to  be  useful  for  screening  renal  cell  carcinoma  patients  ESR,  CEA,  
haptoglobin,  fibrinogen  and  C3  for  estimation  of  staging,  and  ESR,  CEA,  haptoglobin  and  C3  for  
estimation  of  prognosis.  
 
2-­‐  https://www.ncbi.nlm.nih.gov/pubmed/23113677  
α-­‐enolase  could  be  used  in  the  postoperative  follow  up  of  renal  cell  carcinoma  patients,  whereas  the  
combined  use  of  galectin-­‐1  and  galectin-­‐3  might  represent  a  useful  tool  for  primary  detection.  
 
3-­‐  https://www.hindawi.com/journals/dm/2015/251403/  
Unfortunately,  there  is  no  existing  biomarker  for  kidney  cancer  diagnosis.  The  currently  available  
biomarkers  appear  to  have  the  most  utility  as  diagnostic  adjuncts,  as  prognostic  indicators,  and  in  
following  up  patients.  
   
25.  Case  scenario  regarding  HUS  and  asking  about  treatment?  
Answer:  Plasmapheresis  is  used  to  treat  severe  cases  of  HUS.  Mild  disease  resolve  spontaneously.  
Reference:  Kaplan  Lecture  Notes  
   
26.  Minimal  change  GN  happens  at  which  age?  
Answer:  The  incidence  of  minimal  change  disease  peaks  in  children  aged  2  years,  with  approximately  
80%  being  younger  than  6  years  at  the  time  of  diagnosis.  In  adults,  the  mean  age  of  onset  is  40  years  
Reference:  Medscape  
   
27.  Goodpasture's  syndrome  diagnosis?  
Answer:  the  best  initial  test  to  confirm  the  diagnosis  is  the  level  of  anti-­‐basement  membrane  
antibodies  to  type  IV  collagen.  The  single  most  accurate  test  is  a  lung  or  kidney  biopsy.  
Reference:  Kaplan  Lecture  Notes  
   
28.  Calculate  anion  gap  
Answer:  AG  =  Na  –  (HCO3  +  Cl)  
Reference:  Toronto  Notes  2017  
Normal  range  10-­‐14  mEq/L  
 

Neurology  
   
1.                      Patient  with  epilepsy  presented  complaining  of  gum  bleeding  and  increase  hair  in  face  
which  of  the  following  drugs  would  be  the  cause:  
A.          Phenytoin  
B.          Valporic  acid  
C.          Carbamazepine  
   Answer:  A  
   
2.                      Treatment  of  trigeminal  neuralgia?  
A.          -­‐carbamazepine.  
B.          -­‐naproxen.  
answer:  A  
NOTE:  
Carbamazepine  and  oxcarbazepine  are  considered  first  line  therapy  in  trigeminal  neuralgia  (TN).  
Reference:  Toronto  notes  http://emedicine.medscape.com/article/1145144treatment#d9  
   
3.                      Dental  procedure,  then  pt  developed  numbness  on  lower  or  upper  lip  what  is  the  nerve:  
Inferior  orbital  nerve  
   Answer:  A.  Infraorbital  nerve  supplies  upper  lip.  
   
4.                      Loss  of  forehead  muscle  action  what  is  the  nerve:  
Answer:  Facial  nerve  
   
5.                      Loss  of  anterior  2/3  tongue  taste:  
Answer:  Fascial  nerve  
Anterior  ⅔:  (taste>  facial),  (sensation>  trigeminal).  Posterior  ⅔:  glossopharyngeal.  
   
6.                      Loss  of  posterior  1/3  tongue  sensation  what  is  the  nerve:  
Answer:  Glossopharyngeal  
   
7.                      Loss  of  gag  reflex  but  normal  uvula:  
1.  glossopharyngeal  
2.  Vagus  
 Answer:  A  
CN  IX  carries  sensory  information  (afferent  limb  of  reflex).  CN  X  carries  motor  (efferent).    
   
8.                      Pt  had  occipitofrontalis  paralysis  which  branch  of  facial  nerve  is  affected:  
A.          Temporal  
B.          Buccal  
 Answer:  A  
Frontal  part  from  temporal  branches.  Occipital  part  from  posterior  auricular  branch.  
   
9.                      Old  man  with  abnormal  hand  and  head  movement  he  took  drug  1  day  before  what  is  the  
drug:  
A.                      Chloramphenicol  
B.                      Digoxin  
C.                      ethambutol  
D.                     Metoclopromide  
 Answer:  D  
ChlorPROMAZINE  Antidopaminergic  may  also  cause  dystonia..  
   
10.    Child  with  rapid  blinking  of  the  eye  communicate  with  his  parents  during  it  what  is  the  
diagnosis  all  are  normal:  
A)TIC  
B)  petit  mal  seizure  
C)  tourette  syndrome.  
 Answer:  A  most  likely.  
Tics:  sudden  rhythmic  movement.  
Tourette  syndrome:  2  or  more  tics.  
Petit  mal  (absence)  seizure  characterized  by  staring  off  into  the  distance,  without  communication.  
   
11.    Case  of  myasthenia  gravis.  What  to  give?  
A.          Pyridostigimine  
B.          Rivastigimine  
C.          Other  cholinergic  
 Answer:  A  
Toronto  notes:  symptomatic  treatment  (pyridostygmine).  
   
12.    which  of  the  following  drugs  used  in  Alzheimer's  disease  causes  heptotoxicity?  
1)  tacrine  
2)  rivastigmine  
3)  galantamine  
4)  donepezil  
Answer:  A  
 https://www.livertox.nih.gov/AlzheimersDrugs.htm  
   
13.    Which  of  the  following  is  common  cause  of  aseptic  meningitis?  
A)    Enteroviruse  
b)  HSV  
Answer:  A  
viral  infection  is  the  most  common  form  of  aseptic  meningitis,  and  enteroviruses  are  the  most  
common  viral  cause.  http://emedicine.medscape.com/article/1169489-­‐
overview?pa=cGMbL4akzepNjcC0iphPLqQmWAHFzPxik4mBwCZXH5UtGqvCm5qPNND3MD3gTIsJNFs
YxDuz%2Fz2hge3aAwEFsw%3D%3D  
   
15.    12  yrs  old  wake  up  with  ptosis  and  diplopia  at  morning  then  symptoms  resolves?  
A.          Botilism  
B.          Myasthenia  gravis  
 Answer:  
Myasthenia  gravis  symptoms  usually  improve  throughout  the  day.  
Botulism  starts  6-­‐8  hrs  after  ingestion.  Starts  with  paralysis  of  EOM  then  spreads  to  trunk  and  limbs.  
Toronto  notes.  
   
16.    Treatment  of  acute  migraine?  
Answer:  
1st  line:  acetaminophen,  NSAIDs,  ASA  ±  caffeine  2nd  line:  NSAIDs  3rd  line:  5-­‐HT  agonists  ±  
antiemetic.  
Toronto  notes.  
   
17.    25  years  old  male  presented  after  acute  syncopal  attack  when  he  was  standing  in  a  waiting  
line.  He  recalls  feeling  of  dizziness,  blurring  of  vision  and  yawning  four  minutes  before  he  lost  his  
conscious  level.  Diagnosis:  
a)TIA  
b)Ischaemic  attack  
c)Fainting  
b)Out  of  body  experience  
   
18.    Old  man  presented  by  his  children,  they  suspect  his  cognitive  function  is  impaired..  what  can  you  
tell  them  about  trying  to  improve  his  condition  
A)  Daily  exercise  will  help  
B)  Brain  exercise  will  help  prevent  dementia  
C)  New  measure  of  conducting  his  daily  activities  will  now  be  required  ?  
   
19.      Duchenne  muscular  dystrophy  percentage  of  recurrence  
a)50%  
b)25%  
C)2%  
d)7%  
Answer:  A  
For  a  carrier  female,  with  each  pregnancy  there  is  a  one  in  two  (50%)  chance  her  sons  will  inherit  the  
disease  allele  and  a  one  in  two  (50%)  chance  her  daughters  will  be  carriers.  
http://hihg.med.miami.edu/code/http/modules/education/Design/Print.asp?CourseNum=1&Lesson
Num=3  
 
20.    A  middle  age  woman  with  multiple  sclerosis,  complaining  of  urinary  incontinence..she  doesn't  
feel  the  urge  to  empty  her  bladder  but  urine  incontinence  occurs..  
a)Reflex  incontinence  like  urge,  but  no  urgency  
b)Stress  incontinence  
c)Overflow  incontinence  
d)Urge  incontinence  
Answer:C  
   
21.    Prophylaxis  of  migraine  in  asthmatic  pt:  
A)  propanol  
B)  amitriptyline  
Answer:  B  
   
22.    Adolescent  with  migraines;  what  is  the  long  term  complications:  
A.          Hearing  loss  
B.          Reflux  esophagitis  
C.          Depression  
 Answer:  C  
   
23.    what  is  the  cause  of  ischemic  stroke  in  females:  
A)  hypercoagulable  state  
B)  previous  DVT  
Answer:  A.  Intracranial  atherosclerosis  may  be  the  cause  of  thrombotic  stroke  in  patients  with  
widespread  atherosclerosis.  Causes  include  hypercoagulable  state.  
https://emedicine.medscape.com/article/1916852-­‐overview#a5  
   
24.    pt  complain  of  proximal  muscle  weakness  and  ptosis  there  is  improvement  after  
administration  of  anticholinesterase  what  is  the  cause:  
A)  autoimmune  
B)  viral  induced  
 C)  drug  induced  
 D)  nutritional.  
 Answer:  A.  Myasthenia  gravis.  
   
25.    What  organism  can  cause  paralysis?  
Answer:  botolinium  
   
26.    Danlos  ehler  what  is  the  type  of  transmission?  
Answer:  Autosomal  dominant  
   Most  common  form  of  Ehlers  Danlos  is  AD.  
   
27.    pt  k/c  of  MS  and  he  has  new  MS  attack,  treated  by:  
A.          interferon,  
B.          IVIG  ,  
C.          Glucocorticoids  
 Answer:  C  
Acute  treatment:  methylprednisolone  1,000  mg  IV  daily  x  3-­‐7  d  (no  taper  required);  if  poor  response  
to  corticosteroids  may  consider  plasma  exchange  
Toronto  notes.  
   
28.    Most  common  cause  cerebral  abscess:  
A.          head  trauma  
B.          something  in  ear  i  think  OM  ,  
C.          some  thing  in  nose  
   
29.    Male  present  with  multiple  maculs  5-­‐15  mm  ,  and  axilary  frickling  ,  his  wife  is  pregnant  ,  what  
is  the  mood  of  inheritance?  
Autosomal  
X-­‐linked  
Mitochondrial  
Answer:  A.  Neurofibromatosis  is  autosomal  dominant.  
   
30.  Alcoholic  patient  presented  withdrawal  manifestation,  mainly  CNS  (confusion,  wide  gate,  etc.)  
What  would  you  give  him?  
A.     thiamine  
B.     lorazepam  
Answer:  A.  Basic  protocol  includes  thiamine  and  diazepam.  
Toronto  notes.  
   
31.    Case  of  MG  take  neostigmine  then  present  with  worse  symptoms    to  ER  what  is  the  Rx?  (No  
IVIG  in  answers)  
Answer:  Plasmapheresis  
 For  myasthenic  crisis  IVIG  and  plasmapharesis.  
   
32.  Patient  with  facial  Bells  palsy  (UMNL)  which  of  the  following  muscles  will  be  intact?  
A-­‐  orbicularis  oris  
B-­‐  orbicularis  oculi  
C-­‐  masseter  
D-­‐  buccinator  
Answer:  B  if  UMNL.  (Bell’s  palsy  is  a  LOWER  motor  neuron  lesion  affecting  whole  side  of  the  face).  
   
33.    Extradural  hematoma  source  of  bleeding:  
A-­‐middle  cerebral  artery  
B-­‐middle  meningeal  artery  
C-­‐anterior  cerebral  artery  
 Answer:  B.  Subdural>  bridging  veins.  
   
34.  What's  true  about  dyslexia:  
1.   Prevalence  <1%  
2.   Associated  with  hearing  or  visual  impairment  
3.   Problem  in  brainstem  
Answer:  
Prevalence  10-­‐20%.  Poor  vision  is  NOT  a  cause.  Neuroimaging  showed  variations  in  the  right  
temporoparietal-­‐occipital  region.  
Medscape  and  Toronto  notes.  
   
35.  Male  patient  presented  with  painful  lump  in  his  arm  that  developed  after  some  weight  lifting.  
What  would  be  the  appropriate  next  step?  
A-­‐  Give  diclofenac  
   
36.    Treatment  of  MS  attack  in  ER:  
A-­‐  Oral  antibiotic  
B-­‐IV  antibiotic  
C-­‐  Oral  steroid  
D-­‐  IV  steroid  
 Answer:  D.  Toronto  notes.  
   
37.    pt  ē  DM  suddenly  develop  weakness  in  half  of  his  body  one  hour  ago,  no  dysarthria  ,  no  
headache,  what’s  the  management?  
A.          aspirin  
B.          anti  plasminogen  
Answer:  B  (Given  that  the  patient  is  a  candidate  +  after  ruling  out  hemorrhage  with  CT)  
•  Acute  ischemic  stroke:  thrombolytics  (rt-­‐PA,  e.g.  alteplase)  if  within  4.5  h  of  symptom  onset  with  
no  evidence  of  hemorrhage  on  CT  scan  
•  Antiplatelet  agents:  prevent  recurrent  stroke  or  stroke  a  er  TIAs,  e.g.  Aspirin®  (1st  line);  
clopidogrel,  Aggrenox®  (2nd  line).    Toronto  notes.  
   
38.    Epileptic  drug  that  causes  hair  growth?  
 Phenytoin.  
   
39.    old  pt  with  neck  stiffness  and  parasthesia  on  morning  exam  there  was  paraspinal  muscle  
spasm  ,  ttt?  
Physiotherapy  
   
40.    S&S  of  stroke  ,  what  is  the  first  step  in  the  management?  
Answer:  Brain  CT  
 Must  determine  whether  it  is  a  hemorrhagic  or  ischemic  type  of  stroke.  
   
41.    pt  with  dm  &  htn  on  medication  had  tremor  with  movement?  (No  cerebellar  lesion  on  choices)  
A.                      Essential  tremor  
B.                      Psychological  
C.                      Parkinsonian  
D.                     Physiological  
   
   
42.    Patient  with  meningitis,  culture  showed  gram  positive  bacilli,  what’s  the  organism?  
Listeria  
Answer:  A  
   
43.      CSF  circulates  in  which  space?  
A.     1/epidural  
B.     2/subdural  
C.          3/Subarchnoid  
Answer:  C  
   
   
44.    Drug  of  choice  in  absence  seizures?  
Answer:  Ethosuxamide  
   
45.    Pt  complain  of  proximal  muscle  weakness  and  ptosis  there  is  improvement  after  
administration  of  anticholinesterase  what  is  the  cause:  
A)                      autoimmune  
B)                      viral  induced  
C)                      drug  induced  
D)                     nutritional.  
Answer:  A  
   
46.      Another  scenario  about  Mysthenia  graves.  Part  affected?  
Acetylcholine  receptors  
Answer:  A  
Antibodies    against  Ach  receptors  
   
47.    Women  with  bilateral  breast  nipple  white  discharge  and  vision  disturbance  
Prolactin  level  is  high,  where  is  the  lesion:  
There  was  no  pituitary  lesion  in  choices  BUT  THERE  WAS  Sella  turcica  lesion  
Answer:  sella  turcica  
   
48.      female,  K/c  of  epilepsy  on  phenytoin,  she  has  been  using  it  for  a  long  time,  she  came  e  a  
complaint  of  hairy  growth  in  her  body,  and  stated  that  she  is  symptom  free  for  about  6  months.  
What  is  the  correct  action?  
1/stop  phenytoin  
2/lower  the  dose  
3/shift  to  another  antiepileptic  (I  chose  this)  
4/continue  the  drug.  
   
49.  58  years  old  hypertensive  had  hemiplegia  for  10  hours  CT  show  no  hemorrhage  start  
physiotherapy,  examinations  show  hemiplegia,  take  ACIE,  thiazide,  had  acute  peptic  ulcer  before  2  
years  ,  patient  need  which  drug:  
A.     Noting  
B.     Aspirin  
C.     Anticoagulant  
D.   Tpa  
Answer:  B  
   
50.    -­‐Parkinson  case  à  low  dopamine  
   
51.    -­‐Another  case  of  parkinson  à  problem  in  substantia  nigra  
   
52.  CSF    readings  were  with  high    both  lymphcytes  &  neutrophils  high  protien  normal  glusose  à  
Cryptococcal  meningitis    my  answer  by  exclusion  
   
53.    -­‐Toxoplasmosis  encephalitis  in  hiv  scenario  (scenario  of  HIV  pt  with  neurological  symptomes  )  
   
54.    -­‐  Stroke  presentation  (unilateral  body  weakness  &  slurred  speach  in  1  hr  )  what's  next  step  ?  
CT  of  the  brain  
   
 55.  What  is  the  early  sign  of  increase  intracranial  pressure?  
A.          Hypertension  
B.          Unconscious  (Change  in  LOC  is  the  first  earliest  sign  in  increased  ICP)  
C.          Epailateral  papillary  ...  
D.        Contralateral  papillary  ..  
Answer:  Change  in  LOC  
https://amy47.files.wordpress.com/2010/04/increased-­‐intracranial-­‐pressure-­‐pp.pptx  
   
56.    6hrs  post  cerebral  infarction  what  will  u  give?  
1-­‐Aspirin  
2-­‐TPA  
3-­‐Warfarin  
4-­‐Forgot  it  
 Answer:A  
57.    Most  serious  cause  of  stroke  
  -­‐  HTN
  -­‐  Arrhythmias
  -­‐  Smoking
Answer:  A?  
   
58.    Active  seizure  pt  what  to  give  ;  
A.          Phenotin  
B.          Diazepam  
 Answer:  B  
   
59.  Patient  with  guillain  barre,  what  cell  is  affected?  
Answer:  schwan  cell  
   
60.    -­‐Pt  have  medial  squent  the  lesion  is  where  ?  
Pone  
Medulla  ablngate  
Midbrain  
 Answer:  A  
CN  VI  nucleus  located  in  pons.  Exits  at  pontomedullary  junction.  
https://emedicine.medscape.com/article/1198383-­‐overview  
   
61.    Scenario  of  megis  syndrome  (triad),  what  is  the  appropriate  investigation  for  diagnosis  
MRI  pelvis  
Histopatholgy  
Tumor  markers  
 Meig’s  syndrome  (benign  ovarian,  tumour  and  ascites  and  pleural  effusion)  
   
62.    Case  scenario  child  with  sign  and  symptoms  of  meningitis  gram  stain  and  culture  result  was  
gram  +ve  cocci  what  is  the  treatment:  
Answer:  ceftriaxone  and  vancomycin  
 Answer:  A  
   
63.    Sign  in  Duchene  muscular  dystrophy?  
Answer:    gowers  sign  
   
64.  Patient  with  left  side  paralysis  (stroke)  what  to  do?  
A.          CT  is  first  to  know  type  
B.     aspirin  
Answer:  A  
   
65.  CP  patient  all  limbs  affected  but  the  lower  limbs  less  spastic?  
Quadriplegia  
   
66.  Long  scenario  about  patient  come  to  ER  with    chest  pain  radiate  to  left  arm  then  get  coma  and  
death  what  you  will  find  in  the  brain  ?  
1-­‐  necrosis  because  of  left  middle  artery  
2-­‐red  neuron  degeneration  in  hippocampus  
   
67.  Man  lost  smell,  which  lobe  is  affected  
A)  parietal  
B)  temporal  
C)  frontal  
D)  occipital  
Answer:  B  .  
   
68.  Man  with  stroke  and  vision  loss,  affected  lobe:  
A)  parietal  
B)  temporal  
C)  frontal  
D)  occipital  
Answer:  D  
   
68.    Patient  with  pain  in  lips  and  left  cheek,  with  tenderness,  what  to  give?  
A.     Carbamezapine  
B.     Amytriptaline  
C.     Propanolol  
Answer:  A  
First  line  treatment  of  trigeminal  neuralgia  is  carbamazepine.  Toronto  notes.  
   
69.  Migraine  +  HTN  ?  (  I  got  the  Q  without  B  blocker  in  the  choices)  
B  blocker  
CCB  
Answer:  A  
   
70.    Year  old  male,  newly  married,  axillary  freckles  pigmentations  around  5-­‐15  cm,  his  cousin  has  
the  same  condition.  His  wife  is  pregnant  what  is  the  mode  of  inheritance  of  disease  to  child?  
A.          autosomal  linked  (  I  think  this  is  the  correct  answer  cause  the  father  might  have  NF)  
B.          mitrochondrial  
C.          x-­‐  linked  
Answer:  C  
   
71.    Asthmatic  lady  with  recurrent  severe  headache  and  photophobia.  She  gets  the  attacks  3-­‐4  
times  per  week.  She  is  unable  to  sleep  and  has  poor  appetite.  What  is  the  prevention?  
A-­‐Amitryptalin  
B-­‐Propranolol  
C-­‐  Verapamil  
Answer:  A  
Prophylaxis:  1.  TCA  2.  Anticonvulsants  3.  Propranolol.  Toronto  notes.  
   
72.    Teacher  with  band  like  headache?  
Stress  headache  
Answer:  tension  headache.  
   
73.    Case  about  cluster  headache.  What  is  the  treatment  ?  (‫ )ﻣﺎ ﻛﺎﻥن ﻓﻲ ﺍاﺩدﻭوﻳﯾﺔ ﺍاﻋﺮﻓﻬﮭﺎ ﻟﻠﻜﻠﺴﺘﺮ ﻫﮬﮪھﻴﯿﺪﻳﯾﻚ‬ 
Ergotamine  
Paracetamol  
Answer:  
Acute  Rx:  •  O2  •  Sumatriptan  (nasal  or  injection).  
Prophylaxis:  •  Verapamil  •  Lithium  •  Methysergide  •  Prednisolone  
Toronto  notes.  
Acute:  sumatriptan,  100%  O2  
   
74.    Prevention  of  cluster  headache?  
Verapamil  
   
Q.  Scenario  typical  of  cluster  headache  in  a  male  with  eye  pain,  tearing,  comes  clusters  during  the  
day.  What  prophylactic  treatment?  
A.          propanol  
B.          valporic  acid  
C.          amytriptalin  
D.        verapamil  
Answer:  D.  
   
 75.  Pt  with  signs  of  neck  rigidity  and  headache  came  with  seizures,  they  gave  her  lorazepam.  
What’s  the  next  step?  
A.  Lumbar  puncture  
B.  another  dose  of  lorazepam  
Answer:  A.  Do  not  delay  antibiotics  for  lumbar  puncture  or  blood  culture.  
Toronto  notes.  
   
76.  Pt  with  neck  rigidity  and  abdominal  muscle  rigidity  (clear  extrapyramidal  manifestations),  after  
administering  some  drug.  What  is  the  drug?  
Metochlopramide  
   
77.    -­‐Patient  with  increase  ICP  what  nerve  would  u  examine  before  the  CT  scan  
A.          II  
B.          III  
C.          IV  
D.        X  
Answer:  A  
Increased  ICP  can  cause  CN  VI  palsy.  CN  II  will  show  papilledema.  
   
78.    To  decrease  stroke  
Answer:  HTN  screening  
   
79.    Elderly  take  haloperidol  and  came  to  ER  afebrile  rolled  eye  
A.          Neuroleptic  syndrome  
B.          Dyskinesia  
C.          Antonia  
 Answer:  B  
   
80.    Amenorrhea,  low  prolactin,  what  to  see  in  CT  or  MRI:  
Answer:  Sella  turcica  
   
81.  60  year  old  male  known  to  have  DM  and  HTN  came  complaining  of  tremors  of  the  upper  limbs  
that  have  been  progressing  during  the  last  6  months  especially  when  the  patient  is  stressed.  It  
increases  with  movement  and  disappears  at  rest.  What  kind  of  tremor  does  he  have?  
1)  essential  
2)  psychological  
3)  Parkinsonian  
4)  aggravated  physiological  
 Answer:A  
Parkinsonian>  resting.  Essential>  Postural,  worsened  with  sustained  posture.  
   
82.    -­‐Pt  came  with  his  daughter  she  noticed  his  father  starting  to  forget  and  recently  started  to  
have  problems  with  his  daily  activity.  What  drug  would  u  start  
A.          Pyridostigmine  
B.          Rivastigmine  
C.          Neostigmine  
 Answer:  B  
Management  of  dementia.  Toronto  notes.  
   
83.      16  yo  with  GI  symptom  &  bilateral  symmetric  muscle  weakness  
Botulism  
Answer:  
If  preceded  by  exposure,  followed  by  EOM  disturbance  then  spasms  spreading  to  trunk  and  limbs>  
botulism.  
If  preceded  by  history  of  URTI  or  GI,  followed  by  ascending  weakness>  Guillian  Barre  syndrome.  
   
84.    Female  c/o  breast  milk  discharge  and  irregular  menses,  ask  about  investigation:  
A  tsh  
B  brain  mri  
C  biopsy  
 Answer:  B  
   
85.  Scenario  snake  bite.  What  should  you  do?  
1-­‐  evacuate  venom  
2-­‐  apply  tourniquet  
3-­‐  immobilize  limb  
Answer:C  
 Answer:  Supportive.  Wound  irrigation/debridement.  Compression  bands  NOT  useful.  Antivenom  if  
appropriate.  
Toronto  notes.  
   
86.    -­‐  Parents  of  child  noticed  that  he's  sitting  alone,  not  developing  well,  playing  with  his  toy,  and  
have  repetitive  behavior  what's  ur  dx:  
A-­‐  Global  developmental  delay  
B-­‐  Autism  
Answer:  B  
Ref:  https://www.cdc.gov/ncbddd/autism/hcp-­‐dsm.html  
   
88.    -­‐What  is  the  pathology  in  Huntington  syndrome?    What  exactly  increase  and  decrease  and  
where  it’s  located?  
1-­‐something  related  to  stratum  whith  nerotransmitter  deffect  have  look  for  it  *  
Huntington's  primarily  affect  the  striatum.  
Dopamin  level  is  normal  (  in  contrast  with  Parkinson).  
GABA  is  decrease  (  causing  chorea  because  there  is  no  GABA[inhibitory  neurotransmiter]  &  continue  
move)  .  
The  most  striking  neuropathology  in  HD  occurs  within  the  neostriatum,  in  which  gross  atrophy  of  the  
caudate  nucleus  and  putamen  is  accompanied  by  selective  neuronal  loss  and  astrogliosis.  Marked  
neuronal  loss  also  is  seen  in  deep  layers  of  the  cerebral  cortex.  Other  regions,  including  the  globus  
pallidus,  thalamus,  subthalamic  nucleus,  substantia  nigra,  and  cerebellum,  show  varying  degrees  of  
atrophy  depending  on  the  pathologic  grade.  
   
89.  Patient  with  abnormal  movements.  What  is  the  etiological  drug:  
-­‐         Digoxin  
-­‐         Can’t  remember  other  choices  
 (Usually  this  scenario  gives  metoclopramide  as  an  option).  
   
90.  Wernicke's  area  injured,  which  type  of  aphasia?                                                                                                    
A.          can  understand  but  can't  speech                                                                                                                                            
B.          2-­‐can't  understand  or  speech  or  write  
Answer:  Can’t  comprehend/understand  but  speaks  fluently  (receptive  aphasia).  
Toronto  notes.  
   
91.    Phyenytoin  side  effect?  
Gum  hyperplasia  
   
93.    What  is  a  relative  contraindication  of  Methylergometrine?  
A-­‐Asthma  
B-­‐DM  
C-­‐HTN  
 Answer:  C  
   
95.                              Prevention  of  postherpatic  neuralgia  
Answer:  Antivirals.  
•  early  treatment  of  acute  herpes  zoster  with  antivirals  (acyclovir;  longer-­‐acting  famciclovir  
and  valacyclovir  more  effective).  
•  treatment  of  herpes  zoster  with  corticosteroids  DOES  NOT  decrease  PHN.  
Toronto  notes.  
   
96.  female,  K/c  of  epilepsy  on  phenytoin,  she  has  been  using  it  for  a  long  time,  she  came  e  a  
complaint  of  hairy  growth  in  her  body,  and  stated  that  she  is  symptom  free  for  about  6  
months.  What  is  the  correct  action  ?  
A.          stop  phenytoin  

B.  lower  the  dose
  

C.        Shift  to  another  antiepileptic  (  I  chose  this)  

D.        Continue  the  drug.  

 
 

Pediatrics  
 
Part  1  
●       A  child  presented  with  painless  neck  mass,  cough,  sore  throat  and  generalized  pruritus?  
1.   Streptococcal  pharyngitis  
2.   Hodgkin’s  lymphoma  
3.   Infectious  mononucleosis  
4.   Lyme  disease    
Answer:  C  
https://emedicine.medscape.com/article/963894-­‐clinical  
   
 
●       2  month  old  baby  with  creamy  white  plaques  in  the  mouth.  When  he  was  1  week  old  he  
developed  conjunctivitis  treated  with  erythromycin.  What’s  the  best  drug?  
1.   Oral  nystatin  
2.   Oral  tetracycline  
Answer:  A  
http://cursoenarm.net/UPTODATE/contents/mobipreview.htm?4/7/4209  
This  is  a  case  of  oral  thrush  caused  by  candida  albicans  due  to  the  use  of  antibiotics  (erythromycin)  
   
●       Mechanism  of  polio  vaccine?  
http://cursoenarm.net/UPTODATE/contents/mobipreview.htm?13/63/14335  
   

 
●       Child  with  Iron  toxicity  several  hours  ago,  what  will  you  do?  
1.   Gastric  lavage  
2.   Activated  charcoal  
3.   IV  deferoxamine  
Answer:  C  
I  think  its  IV  deferoxamine.  
The  Q  lack  a  lot  of  information,  below  is  what  I  could  gather  about  Iron  toxicity.  
Acute  Iron  posining:  http://cursoenarm.net/UPTODATE/contents/mobipreview.htm?16/16/16641  
Uptodate:   The   decision   to   perform   GI   decontamination   is   based   upon   the   specific   poison(s)  
ingested,  the  time  from  ingestion  to  presentation,  presenting  symptoms,  and  the  predicted  severity  
of  poisoning.  GI  decontamination  is  most  likely  to  benefit  patients  who:  Present  for  care  soon  after  
ingestion  (usually  within  one  to  two  hours)  

Pediatric  Iron  Toxicity  Treatment  &  Management:  

https://emedicine.medscape.com/article/1011689-­‐treatment  
Indications   for   deferoxamine   include   shock,   altered   mental   status,   persistent   GI   symptoms,  
metabolic  acidosis,  pills  visible  on  radiographs,  serum  iron  level  greater  than  500  µg/dL,  or  estimated  
dose  greater  than  60  mg/kg  of  elemental  iron.  
   
●       Scenario  of  a  child  presenting  with  bilious  vomiting.  What’s  the  diagnosis?  
1.   Pyloric  stenosis  
2.   Duodenal  atresia  
3.   Meckel’s  diverticulum  
Answer:  B  
   
https://emedicine.medscape.com/article/1011689-­‐treatment  
   
●       17   year   old   athlete   male   gained   7   kg   lately   and   he   has   all   the   characteristics   of   normal  
puberty,  he  also  has  a  foul  smelling  breath?  
1.   Anabolic  steroid  
2.   puberty  
   
I  couldn’t  find  a  good  explanation.  
But,  Athlete  +  weight  gain  +  bad  breath  all  favors  the  use  of  steroid  
   
●       3  year  old  girl  ingested  20  pills  of  isoniazid.  What’s  the  treatment?  
   
http://cursoenarm.net/UPTODATE/contents/mobipreview.htm?10/12/10447  
https://emedicine.medscape.com/article/180554-­‐treatment#d8  
   
●       A  9  or  12  month  old  asthmatic  child  on  corticosteroids  which  vaccine  to  give?  
   
●       A   child   who’s   always   alone,   doesn’t   own   any   toys   and   doesn’t   play   with   others.   What   is  
the  relation  cause  or  something?  
1.   Intelligence  
2.   Interpersonally  
Answer:  B  
   
●       Scenario  of  a  child  who  swallowed  batteries.  How  would  you  manage  this  case?  
1.   Observation  for  12  hours  
2.   IM  glucagon  
3.   Immediate  endoscopy  
4.   Remove  it  using  a  foley’s  catheter  
Answer:  C  
https://emedicine.medscape.com/article/774838-­‐treatment  
   
●       Which  of  the  following  organisms  cause  acute  epiglottitis?  
1.   Influenza  
2.   Hemophilus  influenza  
Answer:  B  
   
●       Neonate   with   umbilical   swelling   and   yellow   discoloration   over   it.   The   patient   is   vitally  
stable.  What’s  the  diagnosis?  There  was  a  picture.    
1.   Urachal  cyst  
2.   Umbilical  granuloma  
3.   Omphalomesenteric  cyst  
https://emedicine.medscape.com/article/935618-­‐overview#a12  
   
●       An  infant  starts  smiling  at  which  age?  
1.   4  weeks  
2.   8  weeks  
3.   16  weeks  
Answer:  B  
 
Toronto  notes:  

 
Illustrated  textbook  of  pediatric:  

 
●       A  child  who  can  ride  a  tricycle,  what’s  his  age?  
Answer:  3  years  
   
●       Which  one  is  considered  as  delayed  milestone  in  a  13  month  old  child?  
Answer:  Sit  with  support  
   
●       Question  about  Rickets  vs  hypophosphatemia  (know  how  to  differentiate  between  the  lab  
investigations).  
   
●       Diaper  rash  with  satellite  lesion?  
DX:  Candidal  diaper  dermatitis  
Irritant  diaper  dermatitis:  Shiny,  red  macules/patches,  no  skin  fold  involvement.  
Candidate   dermatitis:   Erythematous   macerated,   papule/plaques,   Satellite   lesions,  
involvement  of  skin  folds.  
http://bestpractice.bmj.com/best-­‐practice/monograph/676/diagnosis/history-­‐and-­‐examination.html  
   
●       8   and   a   half   year   old   child   with   breast   bud,   sparse   pubic   hair   and   mild   pigmentation   of  
labia?  
1.   Normal  development  
2.   Precocious  puberty  
Answer:  A  
Precocious   puberty   refers   to   the   appearance   of   physical   and   hormonal   signs   of   pubertal  
development   at   an   earlier   age   than   is   considered   normal.   For   many   years,   puberty   was  
considered  precocious  in  girls  younger  than  8  years;  however,  recent  studies  indicate  that  
signs   of   early   puberty   (breasts   and   pubic   hair)   are   often   present   in   girls   (particularly   black  
girls)  aged  6-­‐8  years.  For  boys,  onset  of  puberty  before  age  9  years  is  considered  precocious.  
https://emedicine.medscape.com/article/924002-­‐overview  
   
●       UTI  treatment  in  a  child?  
Answer:  Augmentin  for  10  days.  
https://emedicine.medscape.com/article/969643-­‐treatment  
   
●       Child  with  DKA  what’s  the  initial  treatment?  
1.   Bolus  of  normal  saline  
2.   SC  insulin  
3.   IV  insulin  infusion  
Answer:  A  
It   is   important   to   pay   close   attention   to   the   correction   of   fluid   and   electrolyte   loss   during   the   first  
hour   of   treatment.   This   always   should   be   followed   by   gradual   correction   of   hyperglycemia   and  
acidosis.   Correction   of   fluid   loss   makes   the   clinical   picture   clearer   and   may   be   sufficient   to   correct  
acidosis.   The   presence   of   even   mild   signs   of   dehydration   indicates   that   at   least   3   L   of   fluid   has  
already  been  lost.  
https://emedicine.medscape.com/article/907111-­‐treatment#d1  
   
●       A  child  with  constipation  since  birth  which  investigation  will  help  establish  diagnosis?  
1.   Plain  x  ray  
2.   Manometry  
3.   Rectal  biopsy  
4.   US  
Answer:  A  
It  is  often  very  difficult  to  differentiate  a  distended  colon  from  small  bowel  on  the  basis  of  a  plain  
abdominal   x-­‐ray   of   a   neonate   with   intestinal   obstruction.   The   presence   of   air-­‐fluid   levels   is   evidence  
of   obstruction,   but   is   non-­‐specific.   There   may   be   an   absence   of   air   in   the   lower   pelvis.   A   normal   film  
does   not   exclude   the   possibility   of   Hirschsprung's   disease,   but   without   any   evidence   of   distended  
colon  it  is  unlikely  to  be  this  disease,  so  plain  abdominal  x-­‐ray  should  always  be  performed  initially.  
http://bestpractice.bmj.com/best-­‐practice/monograph/750/diagnosis/step-­‐by-­‐step.html  
   
●       3  day  old  neonate  with  normal  perinatal  history.  Appears  tired,  with  burned  sugar  smell  in  
the  urine?  
1.   Maple  syrup  sugar  disease  
2.   Phenylketonuria  
Answer:  A  
https://emedicine.medscape.com/article/946234-­‐overview  
   
●       A   child   came   with   symptoms   of   meningitis,   and   culture   showed   gram   -­‐ve   diplococci,   his  
brother  had  contact  with  him.  What  to  do  for  his  brother?  
Answer:  rifampicin.  
   
●       Picture  of  CTG,  what’s  the  diagnosis?  
   
●       Newborn   has   tetany   after   delivery.   Calcium:   1   (normal   2-­‐3).   One   year   later,   he   has  
recurrent   upper   respiratory   infection   with:   HiB,   HSV,   Pneumocystis   jirovecii,   aspergillus.   What   is  
his  underlying  condition?  (Digeorge  syndrome)  
1.   22q11.2  deletion  
2.   Depletion  of  cd4  lymphocytes  
3.   Wiskott  Aldrich  syndrome  
4.   NADPH  oxidation  
Answer:  A  
   
●       Child  with  hip  pain,  x-­‐ray  was  normal  but  US  showed  fluid.  Lab  revealed  high  ESR,  and  CRP,  
otherwise  normal.  What  to  do?  
1.   MRI  
2.   CT  Hip  and  pelvis.  
3.   Aspiration  
Answer:  A  
The  Q  lacks  some  information.  
It  could  be  a  case  of  Transient  synovitis  vs  Septic  arthritis  
Medscape:  
Adding  in  the  CRP  as  a  predictive  factor,  Jung  et  al  found  that  patients  with  4  of  5  predictors  (body  
temperature   >37ºC,   ESR   >20   mm/h,   CRP   >1   mg/dL,   WBC   >11,000/mL,   and   an   increased   hip   joint  
space   of   >2   mm)   had   a   high   probability   of   having   septic   arthritis   and   were   candidates   for   further  
study  by  MRI  or  joint  aspiration.  
In   settings   in   which   routine   aspirations   of   effusions   is   not   performed,   an   MRI   may   help   physicians  
differentiate  transient  synovitis  from  septic  arthritis.  
https://emedicine.medscape.com/article/1007186-­‐workup#c5  
   
●       Typical  case  seborrheic  dermatitis:  greasy  lesions  on  a  baby’s  head.  
   
●       What  are  the  findings  upon  auscultating  a  child  diagnosed  with  croup?  
1.   Inspiratory  wheeze  because  of  the  secretions  
2.   Inspiratory  wheeze  because  of  narrowing  of  the  subglottic  area  
Answer:  B  
http://bestpractice.bmj.com/best-­‐practice/monograph/681/basics/pathophysiology.html  
https://emedicine.medscape.com/article/962972-­‐overview  
Croup  manifests  as  hoarseness,  a  seal-­‐like  barking  cough,  inspiratory  stridor,  and  a  variable  degree  
of   respiratory   distress.   However,   morbidity   is   secondary   to   narrowing   of   the   larynx   and   trachea  
below   the   level   of   the   glottis   (subglottic   region),   causing   the   characteristic   audible   inspiratory  
stridor.  
   
   
●       Scenario   of   a   child   with   meningitis.   CSF   findings   are   as   follows:   High   cellular   count,   normal  
glucose  and  high  protein.  What’s  the  diagnosis?  
1.   Tuberculous  meningitis  
2.   Viral  meningitis  
Answer:  A  
It  also  could  be  GBS,  as  they  have  elevated  protein  
   
●       A   child   with   fever   for   3   days,   then   he   developed   a   rash.   There’s   also   a   picture   showing  
small  spots  in  the  buccal  mucosa  (koplik  spots).  What’s  the  diagnosis?  
1.   Measles  
2.   Kawasaki  
Answer:  A  
   
●       Which  of  the  following  is  part  of  Tetralogy  of  fallot?  
1.   Pulmonary  stenosis  
2.   Left  ventricular  enlargement  
Answer:  A  
   
●       A  baby  with  pansystolic  murmur  in  the  3rd  left  parasternal  area.  He  also  turns  blue  when  
he  cries.  What’s  the  diagnosis?  
Answer:  VSD  
VSD  is  acyanotic  heart  disease,  unless  Eisenmenger’s  syndrome  has  developed  
   
●       13   month   old   baby   with   a   pan   systolic   murmur.   His   echo   shows   a   2   ml   opening   in   the  
muscular  interventricular  septum.  How  will  you  manage  this  baby?  
1.   Watchful  waiting  
2.   Surgical  repair  
Answer:  A  
Small  muscular  ventricular  septal  defects  (VSDs)  have  a  high  spontaneous  closure  rate  (80-­‐
90%)  within  the  first  2  years  of  life  and  often  require  no  medical  or  surgical  management.  
Larger  defects  may  not  close  but  may  become  smaller  with  time.  
https://emedicine.medscape.com/article/899873-­‐treatment  
   
●       A  14  year  old  boy  presented  to  the  clinic  to  take  his  second  dose  of  varicella  vaccine  (His  
1st  one  was  taken  a  year  ago).  Which  of  the  following  is  the  correct  management?  
1.   Give  the  second  dose  
2.   Check  his  varicella  titer  then  give  the  second  dose  
3.   Start  from  the  beginning  and  give  first  and  second  dose  
Answer:  A  
People  13  years  of  age  and  older  (who  have  never  had  chickenpox  or  received  chickenpox  vaccine)  
should  get  two  doses  at  least  28  days  apart.  
http://www.immunize.org/vis/varicella.pdf  
   
●       A  baby  who  can  sit  in  a  tripod  position,  roll  over  and  reach  out  for  objects.  How  old  is  he?  
Answer:  6  months  
   
●       A  child  who  can  tell  a  story,  draws  his  brother  and  pretends  to  be  a  father.  How  old  is  he?      
1.   4  years  old  
2.   5  years  old  
Answer:  B  
   
●       A  mother  presented  to  the  clinic  with  her  3  year  old  child  asking  if  he  has  developmental  
delay?  (Read  about  the  milestones  of  a  3  year-­‐old)  
   
●       A  child  who  can  obey  2  step  commands  and  strangers  can  understand  75%  of  his  words.  
He  asks  questions  frequently.  Which  of  the  following  is  true?  
1.   The  child  has  normal  development  
2.   Delayed  speech  
3.   Delayed  response  
I  couldn’t  find  anything  related  to  (asks  questions  frequently)  
Speech  and  language:  
2  years  old:  obey  2  step  commands  
3  years  old:  strangers  can  understand  75%  of  his  words.  
   
●       A   child   presented   to   the   ER   after   swallowing   caustic   material   and   he’s   drooling.   What’s  
you  immediate  management?  
1.   Protect  the  airways  
2.   Activated  charcoal  
3.   Emergency  endoscopy  
Answer:  A  
Always  start  with  Airway,  Breathing,  Circulation  
   
●       Scenario  of  a  child  with  recurrent  fractures  and  a  blue  sclera.  What’s  the  diagnosis?  
Answer:  Osteogenesis  imperfecta  
   
●       Child  with  frequent  vomiting  and  GERD.  What’s  the  pathophysiology?  
Answer:  Weak  lower  esophageal  sphincter  pressure  
   
●       In  a  female,  the  spine  stops  growing  after  the  onset  of  menarche  by  how  many  months?  
1.   6  months  
2.   12  months  
3.   24  months  
Answer:  C  
In  93  %  of  girls,  the  first  physical  sign  of  puberty  occurs  about  2  years  before  menarche,  and  
final  height  is  usually  achieved  2.5–3  years  after  menarche  
https://www.ncbi.nlm.nih.gov/pmc/articles/PMC3566248/  
   
●       A   mother   brought   her   13   month   old   baby   o   the   clinic.   He   was   found   to   have  
developmental  delay.  Which  of  the  following  is  considered  delayed  in  a  baby  his  age?  
1.   Crawling  
2.   Walks  using  furniture  
3.   Can  walk  alone  
Answer:  A  
   
●       Child  presented  with  breath  that  smells  like  garlic.  What’s  the  diagnosis?  
I  couldn’t  find  anything  related  
   
●       Parents   of   a   diabetic   child   called   you   to   tell   you   that   he   lost   consciousness.   What   would  
you  advise  as  part  of  the  initial  management?  
Answer:  IM  glucagon  
   
●       13  year  old  boy  with  delayed  puberty.  His  growth  parameters  are  normal  and  he  has  fair  
hair  in  the  groin.  How  will  you  manage  him?  
1.   Wait  
2.   Free  testosterone  
3.   Testicular  US  
Answer:  A  
In   most   patients,   however,   the   distinction   between   congenital   GnRH   deficiency   and   constitutional   delay   of  
puberty   remains   uncertain,   and   can   be   resolved   only   with   serial   observations.   In   view   of   these   diagnostic  
difficulties,  the  initial  therapeutic  approach  is  similar  for  both  disorders  [14,15].  The  two  major  options  are:  

●            "Watchful  waiting"  with  reassurance  and  psychological  support  for  the  patient  and  family  

●            Administration  of  gonadal  steroids.  

Short-­‐term  hormonal  therapy  with  testosterone  in  boys  and  with  estrogen  in  girls  may  be  appropriate  when  
the  pubertal  delay  is  severe  or  the  patient's  psychosocial  concerns  about  the  delay  play  a  prominent  role  that  
cannot  be  addressed  by  reassurance  and  education  alone.  

Except  under  unusual  circumstances,  therapy  should  be  restricted  to  boys  older  than  14  years  and  girls  older  
than   12   years   who   show   few   or   no   signs   of   puberty   and   are   expressing   considerable   anxiety   about   their   delay.  
The  short-­‐term  use  of  exogenous  testosterone  in  boys  or  estrogen  in  girls  does  not  appear  to  have  any  long-­‐
term  sequelae  except  for  the  potential  of  skeletal  maturation  that  might  result  in  some  loss  of  adult  height.  

http://cursoenarm.net/UPTODATE/contents/mobipreview.htm?40/20/41295  

                               
●       Female  with  delayed  puberty.  She  has  a  wide  neck,  no  breast  buds.  What’s  her  diagnosis?  
1.   Turner  syndrome  
2.   Noonan  syndrome  
3.   Fragile  X  syndrome  
Answer:  A  
   
●       46   year   old   mother   presented   with   her   baby.   He   has   ASD   and   other   features   of   Down  
syndrome?  
Answer:  Trisomy  21  
   
●       A  case  of  precocious  puberty  
http://bestpractice.bmj.com/best-­‐practice/monograph/1127/diagnosis/step-­‐by-­‐step.html  
   
●       Antibiotics   used   in   neonatal   sepsis:   either   3rd   generation   cephalosporins   (cefotaxime)   or  
combination  (ampicillin  +  amoxicillin  +  gentamicin).  
The   current   approach   to   the   treatment   of   early-­‐onset   neonatal   sepsis   includes   combined   IV  
aminoglycoside  and  expanded-­‐spectrum  penicillin  antibiotic  therapy.  
Cephalosporins  are  attractive  in  the  treatment  of  nosocomial  infection  because  of  their  lack  of  dose-­‐
related   toxicity   and   their   ability   to   reach   adequate   serum   and   cerebrospinal   fluid   (CSF)  
concentrations;  however,  their  use  has  led  to  resistance  in  gram-­‐negative  organisms.  
https://emedicine.medscape.com/article/978352-­‐treatment#d6  
   
●       Face  cellulitis  in  peds:  group  A  streptococcus  
   
●       Aspirin   in   kids   after   viral   infection:   Reye’s   syndrome   (fulminant   hepatitis   and   cerebral  
edema).  
https://emedicine.medscape.com/article/803683-­‐overview  
   
●       Qs  about  APGAR  score.  
   
●       Milestone  of  a  baby  who  can  hold  his  head  and  when  he  looks  at  his  flying  hand  he  laughs?  
Answer:  4  months  
   
●       Scenario   of   a   child   who   went   to   a   camp   and   developed   fever   and   vomiting   after   coming  
home?  
Answer:  brucellosis  (not  sure)  
The  presentation  doesn’t  go  with  brucellosis  or  tick-­‐borne  infections.  
It  could  be  simple  gastroenteritis  
   
●       Scenario  of  a  child  who  was  born  pre-­‐term  and  has  shortness  of  breath?  
Answer:  Surfactant  
RDS  caused  by  surfactant  deficiency  
https://emedicine.medscape.com/article/976034-­‐overview  
   
●       Long   scenario   the   summary   is   a   child   who   was   normal   when   he   was   breast   feeding   and  
now   his   mother   feeds   him   juice   and   he   developed   symptoms   (I   only   remember   jaundice).   What  
should  be  avoided?  
Answer:  Fructose  
   
●       A  child  with  episodes  of  cyanosis  in  lips  and  peripheries  +  precordial  murmur.  What  is  best  
to  reach  a  diagnosis?  
1.   CXR  
2.   ECG  
3.   echocardiography  
Answer:  C  
Echocardiography    (TOF)  
   
●       Pediatrics  cases  (rubella  infection  "fever  plus  rash  started  on  the  face  and  spread  into  the  
trunk  and  LL)  
   
●             Milestone   of   a   child   at   hospital   laughing   and   cooing,   when   the   doctor   approaches   him,   he   felt  
afraid  and  was  looking  for  his  mother?  
Answer:  6  months  
   
●             Pediatric  with  purulent  eye  discharge.  Culture  showed  gram  negative  diplococci.  What’s  the  
treatment?  
1.   IV  cephalosporin  
2.   Steroids  
3.   Topical  antibiotics  
Answer:  A  
DX:  Gonococcal  Conjunctivitis  
   
●       A  child  with  cutaneous  hemangioma,  those  hemangiomas  could  be  found  in  which  organ?  
1.   Liver  
2.   Spleen  
3.   Kidneys  
Answer:  A  
https://emedicine.medscape.com/article/1083849-­‐overview  
Spleen  could  be  involved,  but  most  commonly  the  liver  
   
●             3   year   old   baby   with   fever   and   neck   rigidity.   Culture   revealed   gram   –ve   diplococci   (They  
provided   a   picture   of   diffuse   meningococcemia   skin   rash)   and   asked   about   what   to   give   to   his  
Family?  
Answer:    Oral  rifampicin  
   
●       A  child  brought  to  ER  with  barking  cough,  red  epiglottis,  and  thumb  sign  on  x  ray.  What’s  
the  best  initial  management?  
1.   Examination  of  epiglottis  
2.   Endotracheal  intubation.  
3.   Emergent  tracheostomy  
4.   Nasopharyngeal  Tube  
Answer:  B  
DX:  Acute  epiglottitis  
Illustrated:   The   child   should   be   intubated   under   controlled   conditions   with   a   general   anaesthetic.  
Rarely,  this  is  impossible  and  urgent  tracheostomy  is  life-­‐saving.  
   
●       A   child   with   diarrhea,   abdominal   bloating,   diagnosed   with   giardiasis.   What   is   the   most  
sensitive  test  to  diagnose  his  case?  
1.   3  consecutive  stool  analysis  
2.   3  separate  stool  analysis  
3.   Stool  immunoassay  
Answer:  C  
Because   Giardia   cysts   can   be   excreted   intermittently,   multiple   stool   collections   (i.e.,   three   stool  
specimens  collected  on  separate  days)  increase  test  sensitivity  1.  The  use  of  concentration  methods  
and   trichrome   staining   might   not   be   sufficient   to   identify   Giardia   because   variability   in   the  
concentration   of   organisms   in   the   stool   can   make   this   infection   difficult   to   diagnose.   For   this   reason,  
fecal  immunoassays  that  are  more  sensitive  and  specific  should  be  used  
https://www.cdc.gov/parasites/giardia/diagnosis.html  
   
●       Child   from   Ghana   with   a   neck   mass.   Biopsy   showed   starry   sky   appearance,   KI   67   was  
positive.  Which  genetic  change  is  true?  
Answer:  Over  expression  of  C  myc  gene  
DX:  Burkitt  lymphoma  
   
●       A   child   developed   fever   headache   and   malaise,   after   traveling   to   a   southern   area   on   a   trip  
with   his   friends,   his   mother   thought   that   it   is   normal   after   this   tired   trip,   his   condition   became  
worse   and   he   developed   skin   rash,   sore   throat,   lymphadenopathy,   and   hepatosplenomegaly   on  
examination.  What  test  do  you  want  to  order  to  reach  a  diagnosis?  
Answer:  Monospot  test  
(Infectious  Mononucleosis)  
   
●             A  child  who  has  episodes  of  vomiting  after  feeds,  although  his  appetite  is  good  and  wants  to  
eat  despite  the  vomiting.  His  labs  show  hypokalemia  and  low  chloride.  Most  likely  diagnosis  is?  
Answer:  Hypertrophic  pyloric  stenosis  
The   classic   presentation   of   IHPS   is   the   3-­‐   to   6-­‐week-­‐old   baby   who   develops   immediate   postprandial,  
non-­‐bilious,   often   projectile   vomiting   and   demands   to   be   re-­‐fed   soon   afterwards   (a   "hungry  
vomiter").  
http://cursoenarm.net/UPTODATE/contents/mobipreview.htm?21/52/22336  
   
●       A   child   with   different   types   of   infections   including   aspergillosis,   Tb,   and   staph   ...   etc.  
Immune  defect  in?  
Answer:  NADPH  oxidase  
(chronic  granulomatous  disease)  
Pathogens   such   as   Aspergillus   spp.,   Nocardia   spp.,   Serratia   marcescens,   and   Burkholderia  
cepacia,  or  recurrent  infections  with  Staphylococcus  aureus,  may  suggest  CGD  
http://bestpractice.bmj.com/best-­‐practice/monograph/703/diagnosis/history-­‐and-­‐examination.html  
   
●       A   child,   known   case   of   thalassemia,   had   fever   after   blood   transfusion.   What   is   the   best  
measure  to  take  for  prevention  of  fever?  
1.   Antipyretic  
2.   Pre-­‐transfusion  medications  
3.   Leukocyte  depleted  (prior  storage)  
4.   Leukocyte  filters  at  transfusion  session.  
Answer:  C  
When   red   cells   and   plasma   are   separated   from   whole   blood,   small   amounts   of   residual  
plasma   and/or   leukocyte   debris   may   remain   in   the   red   cell   concentrate.   During   blood  
storage,   these   leukocytes   release   cytokines   that   when   transfused   can   cause   transient  
fevers,  chills,  and  malaise  (without  hemolysis)  within  1-­‐6  hours  of  transfusion.  This  state  is  
called  febrile  non-­‐hemolytic  transfusion  reaction  (FNHTR).  
Although  prophylactic  paracetamol  is  often  administered  to  prevent  FNHTRs,  there  is  little  
evidence  in  the  literature  to  support  this  practice.  
Cytokine  accumulation  during  storage  of  cellular  components  (especially  in  platelet  units)  is  
thought   to   be   the   most   common   event   leading   to   symptoms   of   FNHTRs.   Cytokines   are  
released  by  white  cells  and  pre-­‐storage  leucodepletion  has  reduced  this  risk.  
   
●       Cellulitis   in   the   face   of   infants   between   6   and   24   months   with   purple   discoloration   is  
caused  by?  
1.   GAS  
2.   Staph  aureus  
3.   Hemophilus  influenzae  
4.   Strep  pneumonia  
Answer:  C  
Haemophilus   influenzae   cellulitis   is   a   cutaneous   condition   characterized   by   a   distinctive   bluish   or  
purplish-­‐red  cellulitis  of  the  face.  
Hib  cellulitis  usually  involves  the  face,  head,  or  neck.  
Most  cases  occur  in  children  aged  2  years  or  younger.  
   
●       Treatment  of  GERD  in  a  child?  
Answer:  Give  antacid  
Pharmacological  therapy  used  in  GERD  include:  
-­‐Histamine  blocker  
-­‐PPI  
-­‐Antacid  
https://www.uptodate.com/contents/acid-­‐reflux-­‐gastroesophageal-­‐reflux-­‐disease-­‐in-­‐children-­‐and-­‐
adolescents-­‐beyond-­‐the-­‐basics  
   
●       Child  with  DM1  got  multiple  hypoglycemic  attacks?  
Answer:  Due  to  honeymoon  period  
http://bestpractice.bmj.com/best-­‐practice/monograph/25/treatment/step-­‐by-­‐step.html  
Honeymoon  period  is  one  of  the  causes  of  hypoglycemia  in  DM1  
   
●       Milestone  of  a  child  who  can  draw  a  line?  
Answer:  18  month  (other  options  were  all  younger  than  18  months)  
   
●       Intussusception  age  group?  
Intussusception   is   the   most   common   cause   of   intestinal   obstruction   in   infants   between   6   and   36  
months  of  age.  Approximately  60  percent  of  children  are  younger  than  one  year  old,  and  80  to  90  
percent   are   younger   than   two   years   [3].   Intussusception   is   less   common   before   three   months   and  
after  six  years  of  age.  When  it  does  occur  in  these  younger  or  older  age  groups  it  is  more  likely  to  be  
associated  with  a  lead  point.  
https://www.uptodate.com/contents/intussusception-­‐in-­‐children  
   
●       Which  vaccines  are  given  in  a  9  month  old  baby?  
1.   Measles  &  meningococcal  vaccine  
2.   MMR  
Answer:  A  
 

 
   
   
●       Nevus  crepitus  something  like  this  (child  with  scalp  swelling  with  hair  loss  above  it).  
   
●      
Vaccine  that  is  contraindicated  in  eczema,  psoriasis  and  contact  dermatitis?  
1.   Rubella  (German  measles)  
2.   Measels  
3.   Polio  
4.   Smallpox  
Answer:  D  
Eczema  or  atopic  dermatitis  (in  the  past,  even  if  not  currently  active).  
Patients  with  these  diseases  or  a  history  of  these  diseases  should  not  
be  vaccinated.  

Acute,   chronic,   or   exfoliative   skin   conditions,   including   burns,  


impetigo,   chickenpox,   contact   dermatitis,   shingles,   herpes,   severe  
acne,  Darier's  disease  (keratosis  follicularis),  and  psoriasis.  Until  these  
conditions  clear,  patients  should  not  be  vaccinated.  

http://www.aafp.org/afp/2003/0901/p889.html  
   
●       Pediatric  patient  with  enuresis.    Which  method  is  affective  for  long  time?  
1.   Voiding  before  sleep  
2.   Decrease  fluid  intake  
3.   Alarm  
Answer:  C  
Enuresis  alarms  are  the  most  effective  long-­‐term  therapy  and  have  few  adverse  effects.  
https://www.uptodate.com/contents/nocturnal-­‐enuresis-­‐in-­‐children-­‐management  
   
●       What  a  3  year  old  can  do?  
1.   Climb  stair  
2.   Know  the  names  of  the  days  
3.   Trace  a  triangle  
4.   Catch  a  ball  with  alternative  feet  
Answer:  A  
   
●       3  year  old  baby  have  a  watery  discharge  from  his  eyes  since  birth  and  redness  what  is  the  
cause?  
1.   Bacterial  conjunctivitis  
2.   Lacrimal  duct  obstruction    
Answer:  B  
http://cursoenarm.net/UPTODATE/contents/mobipreview.htm?35/16/36110  
   
●       Still  birth  what  is  the  right  sentence  (thalassemia  case)?  
1.   Two  normal  beta  and  4  abnormal  alpha  
2.   Two  abnormal  beta  and  4  normal  alpha  
3.   4  normal  beta  and  2  abnormal  alpha  
4.   4  abnormal  beta  and  2  normal  alpha  
Answer:  A  
   
●       A  mother  brought  her  7  year  old  girl  to  the  clinic  because  she  has  pubic  hair  growth.  Her  
height   and   weight   are   normal   for   her   age.   On   examination,   no   breast   enlargement   or   other  
secondary  sexual  characteristics.  Clitoris  size  is  normal.  What  do  you  tell  her  mother?  
1.   Early  puberty  
2.   Normal  
3.   Turner  syndrome  
Answer:  A  
https://emedicine.medscape.com/article/924002-­‐clinical  
   
●       One  year  old  baby  was  breast  feed  till  6  months  of  age  and  was  developing  normally,  then  
the   mother   started   to   give   him   fruit   juices   and   he   started   to   have   developmental   delays.   Which   of  
the  following  substance  he  should  avoid?  
1.   Galactose  
2.   Fructose  
3.   Lactose  
Answer:  B  
Fructose   intolerance;   Affected   individuals   are   completely   asymptomatic   until   they   ingest   fructose.  
Thus,  homozygous  neonates  remain  clinically  well  until  confronted  with  dietary  sources  of  fructose.  
http://reference.medscape.com/article/944548-­‐overview#a5  
                                                                                                                                       
●       Doctor  asked  a  child  to  bend  forward  with  the  feet  together,  arms  hanging  and  the  knees  
in  extension  and  the  doctor  is  inspecting  the  patient  from  the  back.  What  is  the  test?  
1.   Adam's  test  
2.   Sexual  abuse  
Answer:  A  
The  patient  takes  off  his/her  t-­‐shirt  so  that  the  spine  is  visible.  The  patient  needs  to  bend  forward,  starting  at  
the  waist  until  the  back  comes  in  the  horizontal  plane,  with  the  feet  together,  arms  hanging  and  the  knees  in  
extension.  The  palms  are  hold  together.  The  examiner  stands  at  the  back  of  the  patient  and  looks  along  the  
horizontal   plane   of   the   spine,   searching   for   abnormalities   of   the   spinal   curve,   like   increased   or   decreased  
lordosis/  kyphosis,  and  an  asymmetry  of  the  trunk.  
   
●       Baby  with  only  mild  jaundice  and  everything  else  is  normal.  Indirect  bilirubin  was  elevated  
about  28  maybe  (but  in  the  twenties).  What’s  the  management?  
1.   Observation  
2.   Phototherapy  
Question  lack  some  information,  but  most  probably  it’s  PHOTOTHERAPY  
The  initiation  of  phototherapy  based  on  total  serum  bilirubin  levels,  gestational  age,  age  of  the  infant  in  hours,  
and   individual   risk   factors.A   commonly   used   rule   of   thumb   in   the   NICU   is   to   start   phototherapy   when   the   total  
serum   bilirubin   level   is   greater   than   5   times   the   birth   weight.   Thus,   in   a   1-­‐kg   infant,   phototherapy   is   started   at  
a  bilirubin  level  of  5  mg/dL;  in  a  2-­‐kg  infant,  phototherapy  is  started  at  a  bilirubin  level  of  10mg/dL  and  so  on.  
https://emedicine.medscape.com/article/1894477-­‐overview#a2  
   
●       Dehydrated  child  (I  think  moderate)  with  severe  vomiting.  What’s  the  management  ?  
1.   Pedialyte  10  ml  over  5  min  
2.   Normal  saline  50ml/kg  
3.   ORS  with  ratio  of  1:1  glucose  to  sodium  
Answer:  B  
   
●       3  years  old  baby  with  dipper  rash  with  no  satellite  lesion.  What’s  the  treatment?  
1.   Barrier  cream  with  frequent  change  of  dipper  
2.   Topical  antibiotics  
3.   Oral  antibiotics  
4.   Topical  steroid  
Answer:  A  
Good  nappy-­‐changing  hygiene  practices  should  be  ensured.  No  matter  which  type  of  nappy  is  used,  it  should  
be  changed  every  2  hours.  
If  the  child  is  prone  to  frequent  nappy  rash,  empirically  apply  a  topical  barrier  containing  zinc  oxide,  white  soft  
paraffin,  glycerin,  lanolin,  sucralfate,  or  mineral  oil  at  each  nappy  change.  
http://bestpractice.bmj.com/best-­‐practice/monograph/676/treatment/details.html  
   
●       Baby  can  sit  without  support  and  turns  his  head  to  the  doctor  and  smiles.  How  old  is  he?  
●       6  months  
●            9  months  
Answer:  B  
   
●       Tumor   in   the   pediatric   age   group   that   arises   from   kidney   or   adrenals   and   displace   the  
collecting  system?  
●            Wilm’s  
●            Neuroblastoma  
Answer:  B  
Abdominal   neuroblastomas   usually   develop   in   the   retroperitoneum.   Most   arise   from   the   adrenal   gland   and  
displace   the   kidney   inferomedially.   In   rare   cases,   a   neuroblastoma   may   mimic   a   Wilms   tumor,   arising   from  
tissues   in   the   kidney   or   invading   the   kidney.   To   make   diagnosis   complicated,   rare   neuroblastomas   possess  
other  features  more  typical  of  Wilms  tumor  than  of  intrarenal  neuroblastomas.  
USMLE-­‐13:  A,  but  I’m  not  sure  of  the  answer  (I  choose  it  because  it  says  displace  displace  the  collecting  system  
and  displacement  goes  with  Wilms-­‐  read  the  explanation  bellow)  I  also  asked  a  oncologist  he  said  he  should  go  
with  B  BUT  the  question  is  not  complete  and  he  needs  more  information.  
                                                                                           
●       A  mother  came  with  her  4  month  old  child  to  vaccinate  him  but  she  reported  that  her  child  
is  having  severe  diarrhea  for  3  days.  What  are  you  going  to  do?  
1.   Refer  all  vaccination  
2.   Give  hepatitis  B  vaccine  
3.   Give  vaccination  without  DTP  
                                                                                                                                                                                                                                                 
-­‐                 There   is   no   evidence   that   acute   illness   reduces   vaccine   e   cacy   or   increases   vaccine   adverse   events.1,2  
However,  as  a  precaution  with  moderate  or  severe  acute  illness,  all  vaccines  should  be  delayed  until  the  illness  
has   improved.   Mild   illnesses   (such   as   otitis   media,   upper   respiratory   infections,   and   diarrhea)   are   NOT  
contraindications  to  vaccination.  Do  not  withhold  vaccination  if  a  person  is  taking  antibiotics.  
   
-­‐                 A  "moderate  or  severe  acute  illness"  is  a  precaution  for  administering  any  vaccine.  A  mild  acute  illness  
(e.g.,  diarrhea  or  mild  upper-­‐respiratory  tract  infection)  with  or  without  fever  is  not  a  precaution,  and  vaccines  
may  be  given.  The  concern  in  vaccinating  someone  with  moderate  or  severe  illness  is  that  a  fever  following  the  
vaccine   could   complicate   management   of   the   concurrent   illness  -­‐   it   could   be   difficult   to   determine   if   the   fever  
was   from   the   vaccine   or   due   to   the   concurrent   illness.   In   deciding   whether   to   vaccinate   a   patient   with  
moderate  or  severe  illness,  the  clinician  needs  to  determine  if  deferring  vaccination  will  increase  the  patient's  
risk   of   vaccine-­‐preventable   diseases,   as   is   the   case   if   the   patient   is   unlikely   to   return   for   vaccination   or   to   seek  
vaccination  elsewhere.  
   
   
●       Case  of  cystic  fibrosis  (+ve  chloride  sweat  test).  They  asked  about  the  gene:  
1.   Chromosome  7  
2.   Chromosome  15  
3.   Chromosome  16  
Answer:  A  
   
●       Child  with  epiglottitis,  x-­‐ray  shows  a  swollen  epiglottis.  What  is  the  first  thing  you’re  going  
to  do?  
1.   Cricothyroidotomy  
2.   Endotracheal  intubation  
3.   Nasopharyngeal  tube  
Answer:  B  
Note:  I  had  the  same  scenario  but  with  different  choices,  they  were:  
1.   Admission  &  refer  to  ENT  
2.   Abx  
3.   Steroids  
Answer:  B  
Appropriate  antibiotics  include  ceftriaxone,  cefotaxime,  and  cefuroxime.  
Corticosteroid   administration,   although   advocated   in   the   past   based   on   anecdotal   reports,   remains  
controversial.  These  agents  have  no  proven  efficacy  for  treating  epiglottitis.  
   
●       13   year   old   boy   brought   to   the   clinic   by   his   parents   for   delayed   puberty.   He   has   sparse  
pubic  hair,  has  always  been  in  the  25th  percentile.  He’s  healthy.  What  are  you  going  to  do?  
1.   Watchful  waiting  
2.   Give  him  testosterone  
Answer:  A  
In   most   patients,   however,   the   distinction   between   congenital   GnRH   deficiency   and   constitutional   delay   of  
puberty   remains   uncertain,   and   can   be   resolved   only   with   serial   observations.   In   view   of   these   diagnostic  
difficulties,  the  initial  therapeutic  approach  is  similar  for  both  disorders  [14,15].  The  two  major  options  are:  

●            "Watchful  waiting"  with  reassurance  and  psychological  support  for  the  patient  and  family  

●            Administration  of  gonadal  steroids.  

Short-­‐term  hormonal  therapy  with  testosterone  in  boys  and  with  estrogen  in  girls  may  be  appropriate  when  
the  pubertal  delay  is  severe  or  the  patient's  psychosocial  concerns  about  the  delay  play  a  prominent  role  that  
cannot  be  addressed  by  reassurance  and  education  alone.  

Except  under  unusual  circumstances,  therapy  should  be  restricted  to  boys  older  than  14  years  and  girls  older  
than   12   years   who   show   few   or   no   signs   of   puberty   and   are   expressing   considerable   anxiety   about   their   delay.  
The  short-­‐term  use  of  exogenous  testosterone  in  boys  or  estrogen  in  girls  does  not  appear  to  have  any  long-­‐
term  sequelae  except  for  the  potential  of  skeletal  maturation  that  might  result  in  some  loss  of  adult  height.  

http://cursoenarm.net/UPTODATE/contents/mobipreview.htm?40/20/41295  
   
   
●            Patient  with  barking  cough.  What  is  the  causative  organism?  
Answer:  Parainfluenza  virus  
   
●       A   hypotensive   baby   with   severe   vomiting   and   watery   diarrhea.   What   is   the   electrolyte  
imbalance?  
1.   Low  na  
2.   High  na  
3.   High  k  
4.   Hyperglycemia  
Answer:  A  
It  could  be  either  hyper  or  hypo  Na  
Since  there  is  hypotension,  I  think  the  answer  is  Low  NA  
●       Blood   in   diaper   with   vomiting   and   constipation.   US   shows   a   donut   sign.   What   is   the  
diagnosis?  
Answer:  Intussusception  
   
●       3  year  old  child  presented  with  burn  involving  her  whole  right  arm,  with  blisters.  What  is  
the  stage  and  percentage?  
   
   

   
   
   
   
●       A  child  diagnosed  with  UTI.  What  to  do  before  the  treatment?  
1.   US  
2.   Cystoscopy.  
Answer:  A  
Indications  for  renal  and  bladder  ultrasonography  are  as  follows:  
●       Febrile  UTI  in  infants  aged  2-­‐24  months  
●       Delayed  or  unsatisfactory  response  to  treatment  of  a  first  febrile  UTI  
●       An  abdominal  mass  or  abnormal  voiding  (dribbling  of  urine)  
●       Recurrence  of  febrile  UTI  after  a  satisfactory  response  to  treatment  
Cystourethroscopy  is  contraindicated  in  febrile  patients  with  urinary  tract  infections  (UTIs)  and  those  
with  severe  coagulopathy.  
https://emedicine.medscape.com/article/1829911-­‐overview#a5  
   
●       Child   with   malignancy   after   bone   marrow   transplant,   by   3   weeks,   his   father   came   because  
his   other   healthy   son   got   chicken   box.   What   would   you   give   the   child   with   the   malignancy   as  
prophylaxis?  
1.   Varicella  vaccine  
2.   Varicella  Ig  
3.   Both  A&B  
4.   Give  acyclovir.  
Answer:  B  
The   most   important   use   of   VZIG   is   for   passive   immunization   of   susceptible,   immunocompromised  
children   after   significant   exposure   to   chickenpox   or   zoster.   This   includes   children   with   primary  
immunodeficiency   disorders   and   neoplastic   diseases   and   children   currently   receiving  
immunosuppressive  treatment.  
https://www.cdc.gov/Mmwr/Preview/Mmwrhtml/00022690.htm  
   
●       5  year  old  child  with  hand  and  foot  pain.  What  is  the  diagnosis?  
Answer:  Sickle  cell  anemia  
   
●       What  causes  cyanotic  heart  disease?  
1.   Tetralogy  of  fallot  
2.   Patent  ductus  arteriosus  
3.   VSD  
Answer:  A  
   
●       Missed  vaccination  (study  the  symptom  of  the  disease  Diphtheria,  pertussis,  mumps,  and  
measles).  
   
●       What   is   the   treatment   of   post-­‐streptococcus   glomerulonephritis   in   children   with   edema  
and  HTN?  
1.   High  dose  of  antibiotics  
2.   Diuretic  for  edema  
3.   Diuretic  for  HTN  
4.   High  dose  of  steroids  
Answer:  B  
https://emedicine.medscape.com/article/980685-­‐medication#1  
Administer   antibiotics   (penicillin   or   erythromycin)   for   10   days   to   ensure   eradication   of   the  
streptococcus  if  the  disease  is  believed  to  be  acute  poststreptococcal  glomerulonephritis  and  if  risk  
of  contamination  is  present  
Furosemide   is   a   loop   diuretic   that   is   useful   in   patients   with   acute   glomerulonephritis   who   are  
edematous.  This  agent  also  has  some  BP-­‐lowering  effect  by  increasing  excretion  of  salt  and  water  via  
interfering   with   the   chloride-­‐binding   cotransport   system   in   the   ascending   loop   of   Henle.   In   acute  
hypertensive  states,  administer  furosemide  intravenously  (IV).  
   
http://cursoenarm.net/UPTODATE/contents/mobipreview.htm?17/10/17568  
Loop  diuretics  generally  provide  a  prompt  diuresis  with  reduction  of  blood  pressure  and  edema.  In  
our  practice,  intravenous  furosemide  is  given  at  an  initial  dose  of  1  mg/kg  (maximum  40  mg).  
Patients   with   evidence   of   persistent   group   A   streptococcal   infection   should   be   given   a   course   of  
antibiotic  therapy.  
   
●       What  is  the  most  common  cause  of  nephrotic  syndrome  in  children?  
A.   ●       Membranous  G  
B.   ●            IgA  nephropathy  
C.   ●            Minimal  change  disease  
D.   ●            Mesangioproliferative  GN  
Answer:  C  
   
●       NICU  child  suddenly  developed  distress  with  absent  breath  sounds.  Where  will  you  place  
the  butterfly  needle?  
●       2nd  intercostal  space  
●       3rd  intercostal  space  
●       4th  intercostal  space  
●       5th  intercostal  space  

Insert  needle  into  the  pleural  space  (directly  over  the  top  of  the  rib  in  the  second  or  third  intercostal  
space  in  the  midclavicular  line)  until  air  is  aspirated  into  the  syringe.  Expel  air  through  the  three-­‐way  
stopcock.  
http://www.asph.mobi/Guidelines_Neonatal/Pneumothorax.pdf  
https://www2.health.vic.gov.au/hospitals-­‐and-­‐health-­‐services/patient-­‐care/perinatal-­‐
reproductive/neonatal-­‐ehandbook/procedures/pneumothorax-­‐drainage  
   
●       6  month  old  infant  brought  by  his  parents  with  history  of  repeated  vomiting;  his  pulse  was  
(190).   He   had   dry   mucous   membranes   and   sunken   anterior   fontanel.   What   is   the   appropriate  
volume  of  fluid  that  should  be  given  initially?  
1.   Bolus  10  ml/kg  of  body  weight  
2.   Bolus  20  ml/kg  of  body  weight  
3.   Slow  infusion  10  ml/kg  of  body  weight  
4.   Slow  infusion  20  ml/kg  of  body  weight  
Answer:  B  
https://emedicine.medscape.com/article/801012-­‐treatment#d11  
   
●       Child   unvaccinated   developed   fever,   SOB   and   stridor.   Lateral   X-­‐ray   shows   a   thumb   sign.  
What’s  the  treatment?  
1.   Erythromycin  
2.   Vancomycin  
3.   Ceftriaxone  
4.   Metoconazole  
Answer:  C  
https://emedicine.medscape.com/article/763612-­‐medication#2  
   
●       10   year   old   child   brought   by   his   parents   because   they   were   concerned   about   his   weight.  
He  eats  a  lot  of  fast  food  and  french  fries.  Your  main  concern  in  managing  this  patient  is?  
A.     His  parents  concern  about  his  weight  
B.     His  BMI  >  33  
C.          Family  history  of  heart  disease    
D.        Eating  habits  (fast  food  and  french  fries)  
Answer:  D  
   
●       A  4  year  old  child  that  doesn’t  speak  well  except  for  saying  mama  and  papa.  He  responds  
well  to  verbal  commands  and  his  older  brother  had  the  same  problem  when  he  was  in  the  same  
age.  He  is  also  shy.  What  is  the  diagnosis?  
A.          Autism  
B.          Development  language  disorder.  
Answer:  B  
   
●       Parents   brought   their   son   with   cystic   fibrosis,   asking   about   the   chance   of   his   sister   of  
becoming  a  carrier:  
A.     1:2  
B.     2:4  
C.     1:25  
Answer:  B  
https://www.cff.org/What-­‐is-­‐CF/Genetics/CF-­‐Genetics-­‐Basics/  
https://rarediseases.info.nih.gov/diseases/6233/cystic-­‐fibrosis  
 
When  two  carriers  of  an  autosomal  recessive  condition  have  children,  each  child  has  a:  

-­‐                25%  (1  in  4)  chance  to  have  CF  


-­‐                50%  (1  in  2)  chance  to  be  a  carrier  of  CF  like  each  parent  
-­‐                25%  chance  to  not  have  CF  and  not  be  a  carrier  CF  
When  a  carrier  of  CF  has  a  child  with  a  person  with  CF,  each  child  has  a:  
-­‐                50%  (1  in  2)  chance  to  have  CF  
-­‐                50%  (1  in  2)  chance  to  be  a  carrier  of  CF  
   
●       What  age  in  months  can  a  child  say  a  few  words?  
A.     6  
B.     9  
C.          12  
D.        24  
Answer:  D  
   
●       A  mother  presented  to  the  clinic  with  her  5  month  old  baby  worried  that  he  might  have  
developmental  delay.  Which  of  the  following  milestones  is  appropriate  for  his  age?  
A.          Pincer  grip  
B.          Reach  objects  
C.          Wave  goodbye  
D.        Sit  
Answer:  B  
   
●       A  baby  who  can  run  and  laugh  when  he  sees  the  doctor.  What’s  his  age?  
A.     6  months  
B.     9  months  
C.     12  months  
D.        24  months  
Answer:  D  
   
●       4  month  old  girl  that  refuses  feeding.  She  becomes  cyanotic  and  cries  whenever  she  feeds.  
On  auscultation:  harsh  ejection  systolic  murmur.  What  is  the  diagnosis?  
A.          Aortic  stenosis  
B.          Ventricular  defect  
Answer:  A  
Aortic  stenosis:  Usually  asymptomatic;  symptoms  may  include  dyspnea,  easy  fatigue,  chest  pain,  or  
syncope;  newborns  and  infants  may  present  with  CHF  
Systolic   ejection   murmur   (grade   2   to   5)   best   heard   at   upper   right   sternal   border   with   radiation   to  
carotid  arteries;  left  ventricular  heave;  thrill  at  ULSB  or  suprasternal  notch  
http://www.aafp.org/afp/2011/1001/p793.html  
   
●       A   child   presented   with   a   barking   cough.   What   is   the   most   likely   finding   in   the   physical  
examination?  
A.          Stridor  
B.          Wheeze  
Answer:  A  
http://www.aafp.org/afp/2011/0501/p1067.html  
   
●            A  scenario  of  a  child  with  frontal  bossing,  bowed  legs  and  a  chest  x-­‐ray  showing  rachitic  rosary.  
Which  of  the  following  is  most  likely  to  be  deficient  in  this  child?  
Answer:  Vitamin  D  
   
●       A  4  year  old  girl  presenting  with  generalized  malaise  and  bony  aches  especially  in  the  hand  
and  feet.  Her  labs  show  high  ANA  and  the  ESR  is  high.  What  is  the  cause?  
A.     Latent  lupus  
B.     Growing  pain  
No  enough  information.  
   
●       Pediatric   case   scenario   about   respiratory   distress   syndrome.   Which   of   the   following   is  
deficient?  
1.   Sphingomyelin  
2.   Dipalmitoylphosphatidylcholine  
Answer:  B  
Surfactant   is   a   complex   lipoprotein   (see   the   image   below)   composed   of   6   phospholipids   and   4  
apoproteins.   Surfactant   recovered   by   alveolar   wash   from   most   mammals   contains   70-­‐80%  
phospholipids,   8-­‐10%   protein,   and   10%   neutral   lipids,   primarily   cholesterol.   Dipalmitoyl  
phosphatidylcholine   (DPPC),   or   lecithin,   is   functionally   the   principle   phospholipid.  
Phosphatidylglycerol  makes  up  4-­‐15%  of  the  phospholipids;  although  it  is  a  marker  for  lung  maturity,  
it  is  not  necessary  for  normal  lung  function.  
https://emedicine.medscape.com/article/976034-­‐overview  
   
   
●       Pediatric  case  of  a  baby  who  vomits  after  feeds  but  has  a  good  appetite  and  wants  to  eat  
despite  vomiting.  During  examination  you  found  an  epigastric  swelling.  What’s  the  next  step?  
1.   Abdominal  US  
2.   X-­‐ray  
3.   Barium  enema  
Answer:  A  
   
●       A   case   scenario   about   a   pediatric   patient   with   abdominal   pain   and   vomiting   (case   of  
intussusception).  What  is  the  best  initial  treatment?  
1.   IV  fluid  and  bowel  rest  
2.   Surgery  
3.   Barium  enema  
4.   CT  abdomen  
Answer:  A  
   
●       A   pediatric   case.   Asymptomatic   patient   with   a   picture   of   an   ECG   (I   just   noticed   irregular  
rhythm,   bradycardia   and   PR   interval   was   unequal   some   lead   was   3   little   squares   and   another   5  
little   squares).   I   think   it   was   a   case   of   1st   degree   heart   block.   Then   they   asked   about   the  
consequences?  
1.   Syncope  
2.   Needs  pacemaker  
Answer:  A  
No  enough  information.  
First-­‐degree  AV  block  in  and  of  itself  does  not  produce  any  symptoms,  nor  need  pacemaker.  
It  could  be  2nd  degree,  Mobitz  I  heart  block  which  is  explained  by  irregular  rhythm  and  unequal  PR  
interval.  
I  don’t  know  if  they  mean  temporary  or  permanent  pacemaker.  
I  choose  syncope  because  it  can  happen  and  Permanent  pacing  is  rarely  required.  
https://emedicine.medscape.com/article/161919-­‐overview  
   
●       Child  with  rapid  blinking  of  the  eyes.  Can  communicate  with  his  parents  during  it.  What  is  
the  diagnosis?  
A)  TIC  
B)  Petit  mal  seizure  
C)  Tourette  syndrome.  
Answer:  A  
http://www.schoolbehavior.com/disorders/tourettes-­‐syndrome/tics-­‐and-­‐tourettes-­‐syndrome-­‐
overview/  
   
 
....  
Part  2  
1)   What  can  a  3  year  old  do?  
A)   Climb  stairs  
B)   Know  the  names  of  the  days    
C)   Trace  a  triangle    
D)   Catch  a  ball  with  alternative  feet    
 
Answer  is  A  
 
2)   A   3   year   old   baby   has   watery   discharge   from   his   eyes   since   birth   associated   with  
redness.  What  is  the  cause?  
A)   Bacterial  conjunctivitis    
B)   Lacrimal  duct  obstruction    
 
Answer  is  B  
 
3)   A   Researcher   wants   to   measure   obesity   in   children.   He   took   their   BMI.   What   else   he  
should  he  take?  
A)   Girth  measurement      
B)   HDL/LDL  level    
C)   Dietary  habits    
D)   Skin  fold  thickness  
 
Answer  is  D  
 
4)   An  18  month  old  child  presented  with  leg  bowing  and  inability  to  walk.  His  labs  show  
high  ca,  low  phosphate,  and  high  ALP.  What’s  the  diagnosis?  
A)   Vitamin  D  deficiency  rickets  
B)   Hypophosphatemia  
C)   Familial  hypophosphatemia    
 
Answer  is  D  
 
Source  is:  
 https://emedicine.medscape.com/article/922305-­‐overview  
 
familial   hypophosphatemia   also   known   by   X-­‐linked   hypophosphatemia,   or   vitamin   D  ̶   resistant  
rickets.   In   XLH   patients,   the   severe   hypophosphatemia   (<   2.5   mg/dL)   is   associated   with   elevated  
serum   alkaline   phosphatase.   However,   despite   the   severe   hypophosphatemia,   the   serum   calcium,  
PTH,  and  25-­‐hydroxyvitamin  D  concentrations  are  normal.  with  high  alkaline  phosphatase.  
 
 
 
5)   Scenario  of  a  child  with  cafe  au  lait  spots  and  short  stature.  What’s  the  diagnosis?    
A)   Fanconi  anemia  
 
Answer  is  A    

6)   A   14   month   old   infant   presented   with   n/v.   Upon   examination,   he   was   found   to   have  
depressed   anterior   fontanelle,   decreased   skin   turgor   and   sunken   eyes.   What   is   the  
degree  of  dehydration?  
A)   5  
B)   10  
C)   15  
D)   20  
 
Answer  is  C  

 
 
7)   A  mother  complains  that  her  child  who  is  6  months  old  is  crying  too  much  for  the  past  2  
weeks.   The   mother   said   that   she   was   regularly   feeding   her   child   then   recently  
introduced  cow  milk.  Upon  examination,  there  was  abdominal  distention.  What  is  the  
diagnosis?  
A)   Cow  milk  allergy    
B)   Infantile  colic  
 
Answer  is  A  
 
8)   A   child   complains   of   barking   cough   with   inspiratory   stridor.   What’s   the   best   way   to  
diagnose  him?  
A)   Clinically  
B)   X-­‐ray  
C)   Laryngeal  swab  
 
Answer  is  A  
 
Source:  emedicine  
https://emedicine.medscape.com/article/962972-­‐workup  
Most  importantly,  croup  is  a  clinical  diagnosis.  Radiographs  can  be  used  as  a  tool  to  help  confirm  this  
diagnosis,  but  are  not  required  in  uncomplicated  cases.   [20]    The  anteroposterior  (AP)  radiograph  of  
the  soft  tissues  of  the  neck  classically  reveals  a  steeple  sign  (also  known  as  a  pencil-­‐point  sign),  which  
signifies   subglottic   narrowing,   whereas   the   lateral   neck   view   may   reveal   a   distended   hypopharynx  
(ballooning)  during  inspiration  (see  the  images  below).  [21]   However,  these  x-­‐ray  findings  may  not  be  
seen  in  up  to  50%  of  children  with  clinical  symptoms  of  croup.  

9)   A   scenario   of   a   patient   with   wide   spaced   nipples   and   a   short   neck.   What’s   her  
diagnosis?    
A)   Turner  syndrome  
B)   Down  syndrome  
 
Answer  is  A  

 
10)  Scenario  of  a  female  patient  that  developed  secondary  sexual  characteristics  but  has  no  
vagina.  What  is  the  diagnosis?    
A)   Mullerian  agenesis  
B)   Ovarian  atrophy  
 
Answer  is  A  

11)  A   mother   is   complaining   that   her   baby   has   a   rash   in   the   diaper   area,   that   rash   is  
associated  with  satellite  lesions  around  it.  What  is  the  treatment?  
A)   Metronidazole  
B)    
Source:  emedicine  
https://emedicine.medscape.com/article/801222-­‐treatment  
 
This   is   yeast   diaper   infection,   due   to   the   presence   of   satellite   lesions.   treatment   is   with   topical  
antifungal   medication.   If   candidal   infection   is   suspected,   topical   ointments   or   creams,   such   as  
nystatin,   clotrimazole,   miconazole,   or   ketoconazole   can   be   applied   to   the   rash   with   every   diaper  
change  
 
12)  Pediatric  patient  presented  with  a  centripetal  rash  distribution,  more  on  the  trunk  and  
gradually  less  towards  extremities.  What  is  the  incubation  period  of  this  disease?    
A)   0-­‐10  
B)   11-­‐20  
C)   21-­‐30  
 
If  the  rash  is  vesicle  and  pustules  this  varicella,  incubation  period  is  10-­‐21  days  
Source:  https://www.webmd.boots.com/children/guide/chickenpox-­‐incubation-­‐period  
 
The  incubation  period  is  seven  to  21  (usually  10  to  21)  days  after  exposure  to  the  herpes  varicella-­‐
zoster   virus   to   the   development   of   the   symptoms.   The   disease   is   most   contagious   a   day   or   two  
before  the  rash  appears  and  until  the  rash  is  completely  dry  and  scabbed  over,  about  five  to  six  days  
after  onset  of  the  rash.  
 
13)  Pediatric  patient  being  treated  for  H  pylori.  He  is  on  metronidazole.  What  to  add?  
A)   Tetracycline  
B)   Clarithromycin  
 
Answer  is  B  
Source:  http://gi.org/guideline/treatment-­‐of-­‐helicobacter-­‐pylori-­‐infection/  
 
Clarithromycin  triple  therapy  consisting  of  a  PPI,  clarithromycin,  and  amoxicillin  or  metronidazole  for  14  days  
remains  a  recommended  treatment  in  regions  where  H.  pylori  clarithromycin  resistance  is  known  to  be  <15%  
and  in  patients  with  no  previous  history  of  macrolide  exposure  for  any  reason    
 
14)  A   Pediatric   patient   is   complaining   of:   Intermittent   dysphagia,   retrosternal   pain   and  
heartburn.  Barium  swallow  showed:  dilated  esophagus.  Synchronous  contractions  and  
gradual  distal  narrowing.  What  is  the  diagnosis?  
A)   Achalasia  
B)   Diffuse  esophageal  spasm  
C)   GERD  
 
Answer  is  A  
Source:  wikipedia  
 
There   is   acute   tapering   at   the   lower   esophageal   sphincter   and   narrowing   at   the   gastro-­‐esophageal  
junction,  producing  a  "bird's  beak"  or  "rat's  tail"  appearance.  The  esophagus  above  the  narrowing  is  often  
dilated  (enlarged)  to  varying  degrees  as  the  esophagus  is  gradually  stretched  over  time.  
 
Characteristic  manometric  findings  are:
●   Lower  esophageal  sphincter  (LES)  fails  to  relax  upon  wet  swallow  (<75%  relaxation)  
●   Pressure   of   LES   <26   mm   Hg   is   normal,>100   is   considered   achalasia,   >   200   is   nutcracker  
achalasia.  
●   Aperistalsis  in  esophageal  body  
●   Relative  increase  in  intra-­‐esophageal  pressure  as  compared  with  intra-­‐gastric  pressure  
 
15)  A   pediatric   patient   complaining   of   daily   fever,   fatigue   and   knee   swelling   for   two  
months.   Also,   recently   developed   epistaxis.   Labs:   Low   platelet,   Low   hemoglobin,  
Normal  WBC.  What  is  your  next  step?  
A)   Bone  marrow  aspirate  
B)   ANA  
 
16)  A  child  with  swelling.  Which  organ  is  mostly  affected  by  mumps?  
A)   Heart  
B)   Lung  
C)   Testes  
D)   Parotid  
 
Answer  is  D  
 
17)  A   pediatric   patient   with   sore   throat,   migrating   arthralgia   and   fever.   What’s   the  
diagnosis?  
A)   Rheumatic  fever  
 
Answer  is  A  

 
18)  A   15-­‐year-­‐old   girl   brought   by   her   mother   because   she   did   not   get   her   period   yet.   On  
examination  she  has  breast  buds,  normal  pubic  hair  and  her  height  has  increased  during  
the  last  year.  Which  one  of  the  following  will  support  your  diagnosis?  
A)   Onset  of  menstruation  
B)   Bone  age  estimation    
 
Answer  A?  
Source:  https://www.girlology.com/puberty-­‐timeline-­‐girls  

Puberty  timeline  in  girls  

●   Breast  buds  are  first  to  develop  for  girls.  


●   Shortly   after   breasts   bud   (a   sign   of   circulating   estrogen),   she   will   begin   to   have   vaginal  
discharge  (also  estrogen-­‐driven).  
●   Pubic  hair  is  second  for  most  girls.  
●   The  biggest,  most  rapid  growth  spurt  follows.  
●   About   6   months   after   she   has   grown   the   fastest   (sometimes   2-­‐3   inches   in   a   matter   of  
months),  she  will  start  her  period.  
●   Once  the  period  begins,  most  of  her  growth  is  finished,  but  she  will  continue  to  grow  in  her  
trunk  and  there  may  still  be  some  lengthening  in  her  legs.  
 
19)  A   school   boy   with   an   itchy   scalp,   10   other   classmates   are   affected.   What   is   the  
diagnosis?  
A)   Pediculosis  capitis  
 
Answer  is  A  
 
20)  An   infant   with   bilateral   absent   red   reflex,   both   pupils   are   reactive.   What   is   the   most  
likely  diagnosis?  
A)   Congenital  cataract  
 
Answer  is  A  
 
21)  Scenario  about  pediatric  herpetic  gingivostomatitis.  What  is  the  causative  organism?  
A)   HSV  1  
 
Answer  is  A  
 
22)  An  18  month  old  baby  diagnosed  with  meningitis.  What  is  the  management?  
A)   Gentamicin  and  penicillin  
B)   Cefotaxime  and  vancomycin  
 
Answer  is  B  
 

 
 
23)  Treatment  of  baby  with  iron  deficiency  anemia  (no  options).    
A)   Oral  route  
 
Answer  is  A  
 
24)  Child   with   repeated   polymicrobial   chest   infections.   Skin   test   was   positive   for   candida  
antigen  and  all  blood  test  are  normal  except:    high  IgG  or  IgM?  And  low  lymphocytes.  
Lymph   node   biopsy   showed:   rudimentary   germinal   centers.   What   is   the  
pathophysiology  of  this  disease?  No  options.    
 
I  think  it’s  a  case  of  X-­‐linked  agammaglobulinemia  (XLA),  or  Bruton  agammaglobulinemia  
 
25)  What  is  the  treatment  of  typhoid  fever  in  children?  
A)   Chloramphenicol  
 
Source:  emedicine  
https://emedicine.medscape.com/article/968672-­‐treatment  
 
Enteric  fever  caused  by  S.  typhi  infection  
●   For  S.  typhi  infection,  initial  empiric  therapy  with  ceftriaxone  is  recommended  due  to  
widespread  resistance.  If  susceptible,  chloramphenicol,  ampicillin,  or  TMP-­‐SMZ  may  be  
used.  Duration  of  therapy  should  be  14  days.  
 
26)  A  milestone  question:  A  child  who  knows  the  names  of  colors  and  can  ride  a  tricycle  but  
has  difficulty  in  drawing  a  square?    
A)   3  years  old  
 
Answer  is  A  
 
27)  A   4   year   old   child   with   decreased   in   head   growth,   has   weird   hand   movements  
(wringing),  lost  expressive  end  receptive  language  skills  and  lost  his  interest  in  his  social  
environment.  What  is  the  diagnosis?  
A)   Autism  
B)   Mental  retardation  
C)   Rett  syndrome  
 
Answer  is  C  
 
Source:  wikipedia  
https://en.wikipedia.org/wiki/Rett_syndrome  
 
Symptoms   include   small   hands   and   feet   and   a   deceleration   of   the   rate   of   head   growth   (including  
microcephaly).   Repetitive   stereotyped   hand   movements,   such   as   wringing   and/or   repeatedly   putting  
hands   into   the   mouth,   are   also   noted.   People   with   Rett   syndrome   are   prone   to   gastrointestinal   disorders  
and  up  to  80%  have  seizures.  
 
28)  A   child   with   a   red   mass   growing   in   his   shoulder   since   birth.   What’s   the   most   likely  
diagnosis?  
A)   Strawberry  nevus    
 
Answer  is  A  
 
29)  A   child   with   developmental   delay,   mild   low   calcium,   pigeon   chest,   bulging   of   frontal  
bone,  mild  high  alkaline  phosphates,  normal  phosphate.  What’s  the  diagnosis?  
A)   Rickets    
B)   Osteoporosis  
C)   Paget  disease  
 
Answer  is  A  
 
30)  Anxious  mother  brings  her  2  month  old  baby  having  history  of  diarrhea  and  crying  too  
much  since  2  weeks.  He  was  born  on  the  36th  week,  breast  fed  but  the  mother  started  
bottled  milk  2  weeks  ago.  What  is  the  cause  of  his  abdominal  pain?  
A)   Increased  gases  
B)   Neuro  system  still  not  fully  developed  
C)   Decreased  bowel  peristalsis    
 
31)  Child   with   scaly   erythematous   plaques   with   follicular   hyperkeratosis   over   elbows   and  
knees.  What  is  the  other  area  in  the  body  most  likely  to  be  affected?  
A)   Eye  
B)   Adrenal  
C)   Heart    
D)   Kidney  
 
Answer  is  A  
 
Source:    
Blepharitis   involves   chronic   inflammation   of   the   eyelids   and   is   one   of   the   more   common   eye  
problems   associated   with   psoriasis.   Another   common   ocular   finding   is   anterior   uveitis,   which   has  
been  reported  to  occur  in  7%  to  20%  of  patients  with  psoriasis.  
   
32)  A   4   week   old   baby   with   his   head   tilted   to   one   side.   Normal   neuro   examination   and  
normal  labs.  What  is  the  diagnosis?    
A)   Congenital  torticollis  
B)   Cretinism  
C)   Erb’s  palsy    
 
Answer  is  A  
 
33)  Boy   (neither   BMI   or   weight   were   mentioned)   eats   fast   food   and   high   fat   diet   (French  
fries)   his   parents   are   concerned   about   his   cholesterol,   both   parents   have   cholesterol  
disease,  you  will  do  cholesterol  test  because:  
A)   BMI  above  30  
B)   Family  Hx  of  cardiovascular  disease  
C)   High  cholesterol  diet  
D)   Parents  concern  
 
34)  Child   patient   came   with   scenario   of   chest   infection,   first   day   of   admission   he   was  
treated   with   cefotaxime.   On   the   next   day,   the   patient’s   state   became   bad   with  
decreased  perfusion  and  x-­‐ray  shows  complete  right  sided  opacification  +  hydrothorax.  
What  is  the  causative  organism?  
A)   Strept  Pneumoniae    
B)   Staph.  Aureus      
C)   Hemophilus  influenzae  type  b  
D)   Pseudomonas  
 
Answer  is  B  
asked  a  pediatric  consultant  
 
35)  Long  scenario  of  a  pediatric  patient  with  the  following  labs:  K:  3,  Na:  124.    How  would  
you  correct  the  electrolyte  imbalance?    
A)   Normal  saline  and  Kcl  5  over  20  cc/hr  
B)   Normal  saline  and  Kcl  40  over  80  cc/hr  
C)   1/2  saline  and  Kcl  5  over  20cc/hr  
D)   1/2  saline  and  Kcl  5  over  20cc/hr  
 
36)  Long  scenario  of  a  boy  with  diarrhea  (gastroenteritis  and  dehydration  symptoms)  with  
no   urine   output   (acute   kidney   injury).   And   they   gave   you   labs   (k:   high,   urea:   high,  
sodium  bicarbonate:  low  and  high  creatinine).  What  is  your  immediate  action?  
A)   Dialysis  
B)   Normal  saline    
C)   NaHCO3  
D)   Antibiotics  
 
Answer  is  B  
 
37)  A   child   presented   with   fatigue   and   lethargy.   Labs:   Hb   10.5,   MCV   40.   What's   your  
management?  
A)   Oral  iron  
B)   IM  iron  
C)   Blood  transfusion  
 
Answer  is  A  
 
38)  Calculate  the  fluid  replacement  of  a  baby  who  weighs  15  Kgs.    >  1250  ml  
●   Frist  10  KG  multiply  100  
●   Second  10  KG  multiply  50  
●   So  10*100=1000  
●   5*50=250  
●   Total:  1000+250=1250ml  
 
39)  A   child   with   short   stature   and   no   family   history.   What’s   the   most   likely   underlying  
etiology?  
A)   Constitutional  growth  delay  
B)   Genetic    
C)   Hormonal  
 
Answer  is  A  
 
40)  A  clear  scenario  about  croup  (brassy  cough  and  hoarseness).  What’s  the  treatment?  
A)   Racemic  epinephrine  and  oral  steroid  
Source:    http://www.aafp.org/afp/2011/0501/p1067.html  
 
Treatment  of  coup    
A  single  dose  of  dexamethasone  (0.15  to  0.60  mg  per  kg  usually  given  orally)  is  recommended  in  all  
patients   with   croup,   including   those   with   mild   disease.   Nebulized   epinephrine   is   an   accepted  
treatment  in  patients  with  moderate  to  severe  croup  
 
41)  A  clear  scenario  about  whooping  cough.  What’s  the  causative  organism?  
A)   Pertussis  
B)   Psittacosis  
C)   HiB  
 
Answer  is  A  
 
42)  An   8   year-­‐old   girl   who’s   tired   and   looks   pale.   Her   liver   is   palpable   and   spleen   is   6   cm  
below  costal  margin.  Her  blood  pressure  is  93/55  and  Hb  is  (low)  and  she  has  positive  
sickle  test  screening.  What  is  the  next  appropriate  investigation  (I  don't  recall  if  she  was  
febrile  or  not)?  
A)   Bone  marrow  aspiration  
B)   Abdominal  US  
C)   Reticulocyte  count  
 
Answer  is  C?  
 
 
 
43)  Foul  smelling  urine  in  a  7  year-­‐old  child  with  fever  and  lower  abdominal  pain.  What’s  
the  causative  organism?  
A)   E.  Coli  
B)   Proteus  
C)   Klebsiella  
 
Answer  is  A  
 
44)  A  child  had  diarrhea  and  vomited  once  with  mild  dehydration:  
A)   Oral  rehydration  solution    
 
45)  A  baby  who  can  elevate  his  head  45°  when.  He's  also  cooing  and  smiling.  How  old  is  he?  
A)   1  month  
B)   3  months    
C)   6  months  
 
Answer  is  B  
 
46)  Child  who  took  20  pills  of  aspirin.  How  are  you  going  to  manage  him?  
A)   Urine  alkalization    
 
Answer  is  A  
Source:  wikipedia  
Initial   treatment   of   an   acute   overdose   involves  resuscitation   followed   by   gastric   decontamination   by  
administering   activated   charcoal,   which   adsorbs   the   aspirin   in   the   gastrointestinal   tract.   Stomach  
pumping  is  no  longer  routinely  used  in  the  treatment  of  poisonings  but  is  sometimes  considered  if  
the   patient   has   ingested   a   potentially   lethal   amount   less   than   one   hour   before   presentation.[15]  
Inducing   vomiting   with   syrup   of   ipecac   is   not   recommended.[10]   Repeated   doses   of   charcoal   have  
been  proposed  to  be  beneficial  in  cases  of  aspirin  overdosing,[16]  although  one  study  found  that  they  
might  not  be  of  significant  value.[17]  Regardless,  most  clinical  toxicologists  will  administer  additional  
charcoal  if  serum  salicylate  levels  are  increasing.  

Intravenous  fluids  
Intravenous   fluids   containing   dextrose   such   as   D5W   are   recommended   to   keep   a   urinary   output  

between  2  and  3  ml/kg/h.  

Sodium  bicarbonate  is  given  in  a  significant  aspirin  overdose  (salicylate  level  greater  than  35  mg/dl  6  
hours  after  ingestion)  regardless  of  the  serum  pH,  as  it  enhances  elimination  of  aspirin  in  the  urine.  It  

is  given  until  a  urine  pH  between  7.5  and  8.0  is  achieved.  
 
47)  In  a  well-­‐baby  clinic,  the  mother  put  her  child  on  bed  and  he  was  laughing  loudly,  when  
the  doctor  came  to  examine  him,  he  pulled  away  and  wanted  to  reach  his  mother.  How  
old  is  he?    
A)   4  months  
B)   6  months  
C)   9  months    
 
Answer  is  B  
 
48)  A  child  who  is  laughing  and  cooing,  can  support  his  head  but  can’t  move  a  toy  from  one  
hand  to  the  other?  
A)   4  months  
B)   6  months  
C)   8  months  
D)   2  months    
 
Answer  is  A  
 
49)  Complication  of  mumps  in  a  child?  
A)   Meningitis  
B)   Encephalitis  
C)   Orchitis  
 
Answer  is  A  
Source:  http://www.who.int/biologicals/areas/vaccines/mmr/mumps/en/  
The   most   common   complication   of   mumps   in   children   is   meningitis,   sometimes   associated   with  
encephalitis,   and   in   young   adults   orchitis.   Most   complications   due   to   mumps   infection   resolve  
without   permanent   damage.   Death   following   mumps   is   rare   and   is   mostly   due   to   mumps  
encephalitis.  
 
50)  A  few  hours  after  a  neonate  was  born,  he  started  to  become  jaundiced  and  he  is  getting  
worse  with  time.  What  is  the  diagnosis?  
A)   G6PD    
B)   Pyruvate  kinase  
 
Answer  is  A  
 
51)  A   baby   has   constipation   since   birth.   Which   of   the   following   investigations   will   help   in  
diagnosing  his  condition?  
A)   Rectal  biopsy    
B)   Rectal  exam  
C)   Anal  exam  
D)   X-­‐ray  
 
Answer  is  A  
Source:  https://emedicine.medscape.com/article/178493-­‐workup#c6  
 
The   definitive   diagnosis   of   Hirschsprung   disease   is   confirmed   by   a   full-­‐thickness   rectal   biopsy  
demonstrating  absence  of  ganglion  cells.  The  specimen  must  be  obtained  at  least  1.5  cm  above  the  
dentate  line  because  aganglionosis  may  normally  be  present  below  this  level.  Disadvantages  of  full-­‐
thickness  rectal  biopsy  include  the  necessity  of  general  anesthesia  and  risks  of  bleeding  and  scarring.  
 

52)  What  is  the  treatment  of  typhoid  fever  in  children?  
Answer  is:  Ceftriaxone  
Source:  https://emedicine.medscape.com/article/968672-­‐treatment  
 
Enteric  fever  caused  by  S.  typhi  infection  
●   For  S.  typhi  infection,  initial  empiric  therapy  with  ceftriaxone  is  recommended  due  to  
widespread  resistance.  If  susceptible,  chloramphenicol,  ampicillin,  or  TMP-­‐SMZ  may  be  
used.  Duration  of  therapy  should  be  14  days.  
 
53)  A  child  with  a  honey  colored  crust  rash.  What’s  the  diagnosis?    
Source:  http://www.innerbody.com/diseases-­‐conditions/impetigo  
 
Impetigo  is  a  skin  infection  caused  by  bacteria,  resulting  in  a  red  rash,  blisters,  and  a  characteristic  
honey-­‐colored  crust  on  the  skin.  Impetigo  most  commonly  occurs  in  infants  and  children  age  2  to  6.  
   
54)  A  child  with  coca  cola  colored  urine?  
 
55)  A  neonate  with  poor  APGAR  score  and  cyanosis.  How  would  you  manage?    
 
56)  13  year  old  patient  with  salmonella  infection  resistant  to  chloramphenicol.  What  is  the  
appropriate  choice  of  therapy?  
A)   Continue  chloramphenicol  
B)   Add  ciprofloxacin  
C)   Give  cipro  alone  
D)   IM  ceftriaxone  
 
Answer  is  D  
 
57)  Child   born   from   a   mother   who   has   gram   negative   streptococcus   positive   at   time   of  
delivery.  After  birth,  child  has  retraction  cyanosis  and  difficulty  breathing.  What’s  your  
next  step  in  management?  
A)   X-­‐ray  of  chest  
B)   Blood  culture  and  look  for  infection
  
C)   2  antibiotics  names  were  given    
 
58)  A  child  has  spot  on  teeth.  What  supplement  to  add?    
A)   Fluoride  
 
Answer  is  A  
 
59)  Congenital  adrenal  hyperplasia  test?  
 
Source:  emedicine  
21-­‐hydroxylase   deficiency:   High   serum   concentration   of   17-­‐hydroxyprogesterone   (usually   >1000  
ng/dL)  and  urinary  pregnanetriol  (metabolite  of  17-­‐hydroxyprogesterone)  in  the  presence  of  clinical  
features  suggestive  of  the  disease;  24-­‐hour  urinary  17-­‐ketosteroid  levels  are  elevated  
 
60)  A  child  swallowed  a  bottle  filled  with  iron  pills,  presented  to  ER  with  abdominal  pain,  
nausea  and  confusion.  What’s  the  best  management?  
A)    IV  deferoxamine  
 
Answer  is  A  
Source:  https://emedicine.medscape.com/article/815213-­‐treatment#d10  
 
In  acute  or  chronic  iron  toxicity,  chelation  therapy  with  deferoxamine  is  indicated  for  patients  with  
serum  iron  levels  >350  mcg/dL  who  have  evidence  of  toxicity,  or  levels  of  >500  mcg/dL  regardless  of  
signs  or  symptoms.  In  patients  with  significant  clinical  manifestations  of  toxicity,  chelation  therapy  
should  not  be  delayed  while  one  awaits  serum  iron  levels.  

 
61)  A  child  with  SCA  presented  with  pain  in  his  penis.  Upon  physical  examination  you  saw  
this  sign.  What  is  the  diagnosis?  

A)   Paraphimosis    

B)   Priapism  

C)   Pyroni's  disease  

Answer  is  B    

62)  A  question  about  comparing  breast  milk  with  cow’s  milk?    

(iron  is  more  in  cow’s  milk)  

63)  8  month  old  infant  with  meningitis.  What  is  the  causative  organism  in  this  age  group?    

                                   A)  Strep  pneumoniae  

Answer  is  A  
 

64)  Neonate  after  delivery  had  bleeding.  What  would  you  give?    

Source:  emedicine  

Immediately  administer  vitamin  K  subcutaneously  (hold  pressure  on  the  site)  for  any  infant  in  whom  
vitamin  K  deficiency  bleeding  is  suspected  or  who  has  serious,  unexplained  neonatal  bleeding.    

65)  Baby  born  with  1  artery  in  umbilical  cord?  

66)  A   3   year   old   came   to   ER   with   cough   and   SOB,   rhinorrhea   for   three   days.   The   cough   is  
worse  at  night.  What  is  the  most  appropriate  nebulizer  medication?  

A)   Salbutamol  
B)   Ipratropium  bromide    
C)   Chromolyn  sodium    
D)   Racemic  epinephrine    
 
Answer  is  A?  
Since   the   cough   is   worse   at   night   it   goes   more   with   asthma   that   is   triggered   by   respiratory   tract  
infection.  
Croup  will  have  inspiratory  stridor  and  barking  cough.  
 
 
67)  Newborn   with   severe   jaundice   at   birth.   He   was   found   to   be   O+   and   his   mother   is   A-­‐.  
What  is  the  mechanism?  
A)   Fetal  antibody  agglutinating  fetal  antigen  
B)   Fetal  antibody  agglutinating  maternal  antigen    
C)   Maternal  antibody  agglutination  fetal  antigen    
 
Answer  is  C  
Source:  https://en.wikipedia.org/wiki/Rh_disease  
 
Rh  isoimmunization  
The   disease   ranges   from   mild   to   severe,   and   typically   occurs   only   in   some   second   or   subsequent  
pregnancies   of   Rh   negative   women   where   the   fetus's   father   is   Rh   positive,   leading   to   a   Rh+  
pregnancy.  If  the  mother  is  Rh  negative  and  the  baby  is  Rh  positive,  the  mother  produces  antibodies  
(including   IgG)   against   the   rhesus   D   antigen   on   her   baby's   red   blood   cells.   During   this   and  
subsequent  pregnancies  the  IgG  is  able  to  pass  through  the  placenta  into  the  fetus  and  if  the  level  of  
it   is   sufficient,   it   will   cause   destruction   of   rhesus   D   positive   fetal   red   blood   cells   leading   to   the  
development  of  Rh  disease.  
 
68)  A  boy  who  took  primaquine  developed  symptoms  of  anemia.  Deficiency  of  which  of  the  
following  caused  his  symptoms?    
A)   B-­‐  G-­‐6-­‐P  dehydrogenase    
B)   C-­‐  Glucose  6  phosphatase  
C)   D-­‐  Pyruvate  kinase    
 
Answer  is  A  
Source:  https://en.wikipedia.org/wiki/Glucose-­‐6-­‐phosphate_dehydrogenase_deficiency#Triggers  
 
Antimalarial   drugs   that   can   cause   acute   hemolysis   in   people   with   G6PD   deficiency   include  
primaquine,  pamaquine,  and  chloroquine.    
 
69)  A  mother  brought  her  child  to  ED  with  itchy  skin  lesions  that  appeared  after  intake  of  
some   food.   The   lesions   appear   in   a   certain   area   and   stay   for   2   hours   then   disappear  
then  appear  in  another  area.  What  is  it  called?  
A)   Burrow    
B)   Wheal    
Answer  is  B  
Source:   Wheal:   A   wheal   is   a   rounded   or   flat-­‐topped,   pale   red   papule   or   plaque   that   is  
characteristically  evanescent,  disappearing  within  24  to  48  hours.  
 
70)  Long   scenario   of   a   child   who   had   a   high   fever   and   developed   a   tonic-­‐clonic   seizure.  
What  is  management?    
A)   Fever  control    
B)   Diazepam    
C)   Phenytoin    
Answer  is  A  
Source:  http://www.aafp.org/afp/2006/0515/p1761.html  
 
Current   recommendations   do   not   support   the   use   of   continuing   or   intermittent   neuroleptic   or  
benzodiazepine  suppressive  therapies  after  a  simple  febrile  seizure.  
For  patients  who  have  an  ongoing  seizure  at  the  time  of  assessment  (i.e.,  febrile  status  epilepticus),  
intravenous   diazepam   (0.2   to   0.5   mg   per   kg   of   weight   intravenously   every   15   minutes   for   a  
cumulative  dosage  of  5  mg  in  children  one  month  to  five  years  of  age)  often  is  effective.  
 
71)  A   baby   who   was   recently   started   on   cow’s   milk,   presenting   with   wheezing   and   SOB.  
Sputum  test  shows  there  is  blood.  Labs  show:  microcytic  hypochromic  anemia.  What’s  
the  treatment?  
A)   Steroids  
B)   Cytotoxic  drugs  
C)   Cromolyn  sodium  
D)   Stop  cow  milk    
 
Answer  is  D  
 
72)  A  young  patient  was  playing  presented  with  severe  epigastric  pain,  retching  and  unable  
to  vomit.  Unable  to  pass  NG  tube.  What  is  the  diagnosis?  
A)   Hiatal  hernia  
B)   Volvulus  
   
73)  12  year  old  presented  with  growth  retardation  and  abdominal  striae.  What  is  next?  
A)   Morning  and  night  cortisol  level  
B)   MRI  brain  
 
Answer  is  B  
 
74)  8   and   half   year   old   brought   by   her   older   sister   for   early   puberty.   Her   sister   menarche  
was  at  14  years  old.  She  presented  with  breast  buds  and  no  pubic  hair  growth.  What  is  
the  cause?  
A)   Precious  puberty  
B)   Normal  development  
C)   Early  thelarche  
 
Answer  is  B  
 
75)  Neonate   with   cyanosis   and   shortness   of   breath   while   feeding   with   diaphoresis.  
Presented  with  ejection  systolic  murmur,  3/6,  normal  S1  loud  S2,  best  heard  on  left  side  
of  chest.  What  is  the  cause?  
A)   Large  ASD  
B)   Large  VSD  
C)   PDA  
D)   Mitral  Regurgitation  
 
76)  Diarrhea  after  introducing  juices  to  baby  with  positive  reducing  substances  test.  What  
kind  of  intolerance  the  baby  has?    
A)   Galactose  intolerance  
B)   Fructose  intolerance  
C)   Lactose  intolerance  
 
Answer  is  C  
 
77)  One   year   old   baby   with   a   prominent   forehead.   Hemoglobin   electrophoresis   show:   Hb  
A2  2-­‐3%  and  Hb  F  2%.  What’s  the  diagnosis?    
A)   Alpha  thalassemia  
B)   Beta  thalassemia  major  
C)   Beta  thalassemia  minor  
 
78)  A  baby  with  recurrent  jaundice  and  multiple  blood  transfusions,  what  causes  the  RBC  to  
be  destructed,  blood  results  show  low  RBC,  DLT  test  normal  and  some  other  labs  and  
blood  smear  (spirometric??,  nucleated...)  
A)   G6PD  
           
79)  Child   with   repeated   polymicrobial   chest   infections.   Skin   test   positive   for   candida  
antigen.  Blood  test  all  normal  except  high  IgG  or  IgM!?,  Low  lymphocytes.  Lymph  node  
biopsy   showed:   rudimentary   germinal   centers.   What   is   the   pathophysiology   of   this  
disease?  Answer  :  the  diagnosis  is  x-­‐linked  Bruton’s  Agammaglobulinemia  
 
80)  A   child   presented   with   difficulty   in   breathing   and   mild   fever.   The   parent   said   that   he  
had  2  similar  episodes  two  weeks  ago.  What's  your  first  step?  
A)   Observe  
B)   Fluids  and  supportive  care    
C)   Inhaled  steroids    
D)   Ventilatory  support  
 
Answer  is  B?  
 
81)  A  scenario  of  a  boy  with  recurrent  infections.  His  brother  died  when  he  was  3  years  old.  
He  has  2  sisters  who  are  alive  and  well.  What  does  he  have?    
A)   Combined  immunodeficiency  
B)   X-­‐linked  immunodeficiency  
 
Answer  is  B  
 
82)  Child   had   URTI   2   weeks   ago,   presented   with   right   upper   quadrant   pain.   Labs   show   high  
WBC,  low  hg  and  other  labs  not  sure  of  them.  Electrophoresis  shows  inclusion  bodies.  
What's  the  diagnosis?  
A)   Acute  appendicitis  
B)   Sickle  cell  disease  
C)   Lithium  
D)   GI  infarct    
 
83)  Child  presented  with  rash  that  started  in  the  back  then  it  was  spread  to  all  his  body.  The  
rash   was   pustular   with   other   areas   vesicular.   The   rash   had   truncal   distribution   more  
than  the  limbs.  What  is  the  incubation  period  for  this  organism?    
A)   5-­‐11  days    
B)   11-­‐  21  days    
C)   23  -­‐  30  days  
 
Answer  is  B  
Source:  https://www.webmd.boots.com/children/guide/chickenpox-­‐incubation-­‐period  
 
The  incubation  period  is  seven  to  21  (usually  10  to  21)  days  after  exposure  to  the  herpes  varicella-­‐
zoster   virus   to   the   development   of   the   symptoms.   The   disease   is   most   contagious   a   day   or   two  
before  the  rash  appears  and  until  the  rash  is  completely  dry  and  scabbed  over,  about  five  to  six  days  
after  onset  of  the  rash.  
           
84)  Child   came   with   knee   swelling   or   bleeding   after   mild   trauma.   Bleeding   time   was  
abnormal  and  it  was  NOT  corrected  after  we  gave  fresh  frozen  plasma.  Then  bleeding  
time   got   back   to   normal   after   giving   platelet   transfusion   (his   platelet   level   was   50).  
What  does  the  patient  have  (PT  and  PTT  were  not  included  in  the  question)?  
A)   VWF  deficiency  
B)   HSP  
C)   Thrombotic  thrombocytopenic  purpura  (Not  ITP)  
D)   Weird  syndrome  name,  can't  remember  
 
85)  New   born   totally   healthy   with   left   thigh   bruise.   All   examination   normal.   Prolonged   pt  
and  ptt.  What’s  the  diagnosis?    
A)   Hemophilia  
B)   Factor  10  deficiency          
C)   Idiopathic  thrombocytopenic  purpura  
Answer  is  B  
Source:  https://emedicine.medscape.com/article/209867-­‐workup  
Coagulation  study  findings  in  patients  with  factor  X  deficiency  include  the  following:  
●   The  prothrombin  time  (PT)  is  prolonged  
●   The  activated  partial  thromboplastin  time  (aPTT)  is  prolonged  
●   The  Russell  viper  venom  time  (RVVT)  is  prolonged;  Russell  viper  venom  cleaves  factor  
X  to  produce  active  factor  Xa  
●   Bleeding  time  is  within  the  reference  range  
 
86)  A   child   with   multiple   plaques   on   his   face,   abdomen   and   feet.   What   is   the   Diagnosis?   (In  
the  q  they  mentioned  something  about  histoimmune  and  positive  CD?)  
A)   Mycosis  fungoides  
B)   Bowen  disease  
C)   Basel  cell  carcinoma  
If   the   plaques   are   hypopigmented,   with   CD   3   positive,   CD4   positive   and   CD8   negative   this   is   mycosis  
fungoides.  
 
 
87)  Pediatric   age   group,   mass   on   the   inner   side   of   lower   lip,   non-­‐tender   and   bluish   in   color.  
What  is  the  diagnosis?  
A)   Mucocele  
B)   Gingival  cyst            
C)   Ranula  
D)   Epiula  
 
Answer  is  A  
Source:  wikipedia  
The   mucocele   has   a   bluish   translucent   color,   and   is   more   commonly   found   in   children   and   young  
adults.  The  most  common  location  to  find  a  mucocele  is  the  inner  surface  of  the  lower  lip.  It  can  also  
be   found   on   the   inner   side   of   the   cheek   (known   as   the   buccal   mucosa),   on   the   anterior   ventral  
tongue,  and  the  floor  of  the  mouth.      
 
88)  Most  common  known  side  effect  of  steroid  use  in  children?  
A)   Excitable  behavior    
B)   Labile  mood  
C)   Growth  retardation    
D)   Intraocular  Hypertension  
 
Answer  is  C?  
 
89)  Girl   or   boy   (I   can’t   remember)   hit   puberty.   Till   which   age   the   bone   growth   will  
continue?  
A)   12  months  
B)   24  months          
C)   36  months  
 
Answer  is  B  
 
90)  A   young   girl   came   with   leg   pain   more   in   night.   Examination   is   normal.   Labs:   ESR   and  
ANA  high.  What’s  the  diagnosis?      
A)   Rheumatoid    
B)   Growing  pain  
C)   Osteoid  osteoma  
D)   Osteosarcoma  
 
 
91)  10  year  old  child  with  acute  rheumatic  fever  and  cardiac  involvement.  For  how  long  will  
you  continue  him  on  prophylactic  antibiotics?  
A)   1  year  
B)   5  years            
C)   10  years            
D)   15  years  
 
Answer  is  C  
Source:  

 
 
92)  A   baby   has   depigmentation   since   birth,   photophobia   and   refractory   error   nystagmus.  
What’s  the  most  likely  complication  associated  with  his  condition?  (Case  of  albinism)    
A)   Brain  tumor  
B)   Skin  cancer              
C)   Renal  dysfunction  
 
Answer  is  B  
Source:  https://emedicine.medscape.com/article/1200472-­‐followup  
 
Complications   of   albinism  
See   the   list   below:  
Skin   cancer,   sunburn  
Reduced   visual   acuity  
Social  stigma  
 
 
93)  A   child   with   cough,   coryza   and   conjunctivitis.   Presented   with   maculopapular   rash   in  
palm  and  buttocks.  On  examination,  there  are  pustules  in  tonsils.  What  other  physical  
signs  will  find  in  the  patient?  
A)   Spot  on  the  sole  of  the  foot                
B)   Swollen  tonsils    
C)   Scratch  markers    
D)   Diarrhea  
 
Answer:  It’s  measles,  diarrhea  is  a  common  complication.??  
Ref.  http://emedicine.medscape.com/article/966220-­‐clinical#b3  
 
94)  5  month  old  baby  found  to  have  developmental  delay.  Which  one  of  the  following  is  a  
reassuring  sign  for  the  doctor?  
A)   Pincer  grasp.  
B)   Reaching  for  objects.    
C)   Sits  independently.  
 
Answer  is  B  
 
95)  Child  with  autoantibodies..  Chances  of  getting  DM  type  1?          
A)   40  %    
B)   60  %    
C)   80  %    
D)   100  %    
 
Source:  http://diabetes.diabetesjournals.org/content/54/suppl_2/S52  
   
in   the   large   U.S.   study,   the   Diabetes   Prevention   Trial   (DPT-­‐1),   four   autoantibodies   (ICA,   IAA,  
GAD65Ab,  and  IA-­‐2Ab)  were  analyzed  to  assess  the  risk  for  developing  diabetes;  98%  of  first-­‐degree  
relatives  who  went  on  to  develop  type  1  diabetes  had  one  or  more  autoantibodies,  and  80%  had  two  
or   more   autoantibodies.   Individuals   with   two   or   more   positive   biochemical   autoantibodies   had   a  
68%  5-­‐year  risk  for  developing  type  1  diabetes,  and  those  with  all  three  biochemical  antibodies  had  
an  estimated  100%  5-­‐year  risk        
 
96)  Child  with  fever  and  productive  cough  and  diarrhea  5  times  a  day.  Now,  tolerating  oral  
rehydration?  
A)   Hospitalize  
B)   Augmentin  and  discharge  
C)   Stool  analysis  and  culture  
 
97)  Which   of   the   following   vaccines   is   contraindicated   in   a   child   with   progressive  
neurological  problems?    
A)   Pneumococcal  conjugate  vaccine  
B)   Hib  vaccine  
C)   Dtap  
D)   Varicella  
 
Answer  is  C  
Source:  https://www.cdc.gov/vaccines/hcp/acip-­‐recs/general-­‐recs/contraindications.html  
 
DTaP  
Contraindications:  
1)   Severe  allergic  reaction  (e.g.,  anaphylaxis)  after  a  previous  dose  or  to  a  vaccine  component  
2)   Encephalopathy   (e.g.,   coma,   decreased   level   of   consciousness,     prolonged   seizures),   not  

attributable   to   another   identifiable   cause,   within   7   days   of   administration   of   previous   dose   of  

DTP  or  DTaP  

Precautions:  
1)   Progressive   neurologic   disorder,   including   infantile   spasms,   uncontrolled   epilepsy,   progressive  
encephalopathy;  defer  DTaP  until  neurologic  status  clarified  and  stabilized  
2)   Temperature  of  ≥105°F  (≥40.5°C)  within  48  hours  after  vaccination  with  a  previous  dose  of  DTP  

or  DTaP  

3)   Collapse   or   shock-­‐like   state   (i.e.,   hypotonic   hyporesponsive   episode)   within   48   hours   after  

receiving  a  previous  dose  of  DTP/DTaP  

4)   Seizure  ≤3  days  after  receiving  a  previous  dose  of  DTP/DTaP  

5)   Persistent,  inconsolable  crying  lasting  ≥3  hours  within  48  hours  after  receiving  a  previous  dose  of  

DTP/DTaP  

6)   GBS  <6  weeks  after  previous  dose  of  tetanus-­‐toxoid–containing  vaccine  

7)   History   of   Arthus-­‐type   hypersensitivity   reactions   after   a   previous   dose   of   diphtheria-­‐toxoid–

containing   or   tetanus-­‐toxoid–containing   vaccine;   defer   vaccination   until   at   least   10   years   have  

elapsed  since  the  last  tetanus-­‐toxoid–containing  vaccine  

8)   Moderate  or  severe  acute  illness  with  or  without  fever  

             
98)  A   6   year   old   came   to   your   clinic   with   no   vaccine   history   except   bgg   (I   never   read   this  
vaccine  except  in  this  question  in  exam).  What  is  the  appropriate  vaccination  for  him  on  
his  first  clinic  visit?  (time  of  clinic  visit  not  mentioned.  Means  now  or  before  when  he  
was  infant???  To  catch  up??  Or  to  give  6  years  vaccines?)          
A)   MMR  OPV  Varicella  DTap  -­‐-­‐-­‐  6  yrs  vaccines    
B)   MMR  OPV  HiB  
C)   MMR  OPV  Hib  HBV  
 
99)  Child  with  multiple  infections:  viral,  fungal  and  bacterial.  He  has  a  defect  in  Cd4?  
A)   Ada  deficiency  
Adenosine   deaminase   deficiency   (also   called   ADA   deficiency   or   ADA-­‐SCID[1])   is   an   autosomal  
recessive[2]metabolic  disorder  that  causes  immunodeficiency.  It  occurs  in  fewer  than  one  in  100,000  
live  births  worldwide.  
It  accounts  for  about  15%  of  all  cases  of  severe  combined  immunodeficiency  (SCID).[3]  ADA  deficiency  
may  be  present  in  infancy,  childhood,  adolescence,  or  adulthood  
……..  
   
1)           Child  with  a  history  of  fall  without  losing  consciousness.  He  vomited  twice  and  is  
crying  and  complaining  of  a  headache.  What  should  you  do?  
A)          CT  
B)          Close  observe  
Answer:  A  
Indications   for   CT   scanning   in   a   patient   with   a   head   injury   include   anisocoria,   GCS   score   less   than   12  
(some  studies  suggest  CT  scanning  in  any  pediatric  patient  with  a  GCS  score  of  <  15),  posttraumatic  
seizures,   amnesia,   progressive   headache,   an   unreliable   history   or   examination   because   of   possible  
alcohol  or  drug  ingestion,  loss  of  consciousness  for  longer  than  5  minutes,  physical  signs  of  basilar  
skull   fracture,   repeated   vomiting   or   vomiting   for   more   than   8   hours   after   injury,   and   instability   after  
multiple  trauma.  
https://emedicine.medscape.com/article/907273-­‐workup#c8  
   
2)          An  18  month  old  female  baby  with  anemia.  What’s  the  diagnosis?  INCOMPLETE  
A)          Homozygous  b  thalassemia  
B)          Homozygous  a  thalassemia  
C)          Carrier  of  a  thalassemia  
Answer:  
-­‐                 Alpha   thalassemia   silent   carrier:   Patients   are   likely   to   be   asymptomatic   and   hematologically  
normal.  
-­‐                 Alpha-­‐thalassemia  major  or  homozygous  alpha(0)  thalassemia:  Hemoglobin  Bart  hydrops  fetalis  
syndrome  is  typically  caused  when  all  4  alpha-­‐globin  genes  are  deleted.  
-­‐                Beta-­‐thalassemia  minor  (commonly  referred  to  as  beta-­‐thalassemia  trait):  usually  asymptomatic;  
the  mild  microcytic  anemia  is  often  misdiagnosed  as  iron  deficiency  anemia.  
-­‐                 Beta-­‐thalassemia  intermedia:  usually  a  similar  presentation  to  beta-­‐thalassemia  major  but  as  a  
toddler   or   older   child;   symptoms   are   usually   less   pronounced   and   the   course   is   usually   more  
insidious.  
-­‐                Beta-­‐thalassemia   major   (also   called   Cooley's   anemia):   complete   absence   of   hemoglobin   A;   often  
presents  at  a  few  months  of  age  with  progressive  pallor  and  abdominal  distension;  perinatal  history  
is   most   often   uneventful,   and   the   infant   may   be   pale,   possibly   with   poor   feeding   and   decreased  
activity;  hepatosplenomegaly  and  bony  abnormalities  are  often  present  at  presentation,  most  often  
of  the  skull  (frontal  and  parietal  bossing,  and  chipmunk  facies).  
BUT   (A)   is   closer   because   approx.   after   18   months   HbF   production   reduces   and   HbA   cannot   be  
produced.  
Reference:  http://bestpractice.bmj.com/best-­‐practice/monograph/251.html  
   
3)           Before   giving   influenza   vaccine   you   have   to   ask   if   the   child   has   an   allergy   from  
which  of  the  following  substances?  
A)          Egg  
Answer:  A  
Reference:  https://www.cdc.gov/flu/professionals/vaccination/vax-­‐summary.htm  
   
4)           A   child   with   barking   cough   only.   No   fever   or   shortness   of   breath.   Vitals   are   normal.  
What’s  your  management?  
A)          Moist  oxygen  
B)          Nebulizer  racemic  acid                                                                                                                                              
C)          Antibiotics  
Answer:  A.  Cool  mist  from  a  humidifier  and/or  sitting  with  the  child  in  a  bathroom  filled  
with  steam  generated  by  running  hot  water  from  the  shower,  help  minimize  symptoms.  
Reference:https://emedicine.medscape.com/article/962972-­‐treatment  
         
5)           A   patient   with   hematuria   and   upon   examination,   he   has   aniridia   (absence   of   the  
iris).  What’s  the  diagnosis?  
A)          Wilms  tumor  
B)          Neuroblastoma  
Answer:  A  
Reference:  https://emedicine.medscape.com/article/989329-­‐overview                        
   
   
6)           A  child  came  with  abdominal  mass.  Imaging  showed  renal  tumor.  The  patient  has  
absent  iris.  What  is  the  most  likely  diagnosis?                                                                        
A)          Wilms's  tumor  
Answer:  A  
Reference:  https://emedicine.medscape.com/article/989329-­‐overview                        
   
7)           Pediatric  patient  known  case  of  acute  lymphoblastic  leukemia  (ALL),  presented  to  
the  ER  with  fever  and  pancytopenia  (lab  results  were  provided).  What  is  your  action?  
A)          Blood  transfusion  
B)          Refer  to  oncology                                                                                                                                          
C)          Start  Antibiotics  
Answer:  C  
   
8)           Most   common   presentation   of   congenital   heart   disease?                                                          
   
A)          JVD  
B)          Difficulty  feeding  
C)          Ascites                                                                                                                                                      
Answer:   the   most   common   presentation   of   congenital   heart   disease   is   with   a   heart  
murmur.  
Reference:  Illustrated  textbook  of  pediatrics  
   
9)          Which  of  the  following  congenital  heart  defects  is  associated  with  Down  syndrome?  
A)          Endocardial  cushion  defects  such  as  AVSD  
Answer:  A  
Reference:  https://emedicine.medscape.com/article/943216-­‐clinical#b2  
   
                                                                                                                                           
10)     A   child   with   foul   smelling   breath   and   seed   like   structures   coming   out   of   the   mouth.  
He  is  also  a  mouth  breather.  No  history  of  fever.  What’s  the  most  likely  diagnosis?  
A)          Pulmonary  disease  
B)          Focal  tonsillitis  
Answer:   B.   Individuals   with   acute   tonsillitis   present   with   fever,   sore   throat,   foul   breath,  
dysphagia,   odynophagia,   and   tender   cervical   lymph   nodes.   Airway   obstruction   may  
manifest   as   mouth   breathing,   snoring,   sleep-­‐disordered   breathing,   nocturnal   breathing  
pauses,  or  sleep  apnea.  
Reference:  https://emedicine.medscape.com/article/871977-­‐clinical  
   
11)     A  2-­‐year  child  came  to  the  clinic  with  his  mother.  He  scribbles  circles,  runs  around  
and  climbs  onto  the  chair,  plays  with  his  friends  but  does  not  share  his  toys.  He  speaks  
10  words.  He  names  the  picture  you  point  to.  What  is  the  best  thing  to  tell  his  mom?  
A)          He  is  normal  
B)          Delayed  social  development  
C)          Delayed  language  development  
Answer:   A.   He   is   normal   since   sharing   toys   and   cooperative   play   only   starts   at   4   years   of  
age.  
Reference:  Toronto  notes  
   
12)    6  month  old  baby  presented  with  jaundice,  seizure,  irritability  and  vomiting.  He  has  
been   breast-­‐fed   during   his   first   3   months   of   life,   then   he   has   been   started   on  
commercially  available  milk  formula.  More  recently,  his  mother  introduced  fruit  juices  
into  his  diet.  Investigations  revealed  positive  urine  reducing  substances.  What  element  
should  avoided  in  his  diet?                                                                                                                                    
A)          Galactose.  
B)          Fructose.  
C)          Phenylalanine.  
D)        Irrelevant  choice.  
Answer:  B  
   
13)     4   year   old   child   brought   by   his   parents   to   pediatric   outpatient   clinic   with   them  
complaining   of   his   massive   uncontrolled   appetite   during   the   last   18   months.   Weight  
was   above   95th   percentile,   while   height   was   below   5th   percentile.   Mother   reported  
that  her  son  was  failing  to  thrive  during  his  first  two  years,  beside  the  fact  that  he  was  
developmentally  delayed  compared  to  his  siblings  until  he  caught  up  late.  O/E  he  had  a  
high   forehead,   broad   nose,   small   peripheries   (Hands   and   Feet).   What   is   the   cause  
behind  his  symptoms?                                                
A)          Genetic.  
B)          Metabolic.  
C)          Nutritional.  
D)        Irrelevant  choice.  
Answer:  A  (Prader-­‐Willi  syndrome)  
Reference:  Medscape  
   
14)     A  5  year  old  boy  presents  with  tender,  swollen  and  painful  testicle.  O/E  there  was  
absent  cremasteric  reflex.  Which  of  the  following  is  the  correct  diagnosis?  
A)          Epididymo-­‐orchitis.  
B)          Testicular  torsion.  
Answer:   B.   The   most   sensitive   physical   finding   in   testicular   torsion   is   the   absence   of   the  
cremasteric  reflex.  
Reference:  aafp.org  
                                                                                                                                                                               
15)     Pediatric   case   of   Hemolytic   uremic   syndrome:   he   has   petechia,   hematuria   and  
proteinuria.   2   weeks   ago,   he   develop   bloody   diarrhea.   The   doctor   prescribed  
symptomatic  treatment  and  probiotics.  Platelets  95,  WBC  48.  What’s  your  next  step?  
A)          Antibiotic  
B)          Platelet  transfusion  
C)          Anticoagulant  
Answer:  Treatment  of  HUS:  
-­‐                Avoid  antibiotics  and  platelet  transfusion.  
-­‐                Good  hydration  with  IV  fluids.  
-­‐                Blood  transfusion  in  anemic  patients  when  indicated.  
-­‐                CCB  for  HUS  with  hypertension.  
-­‐                Dialysis  for  renal  failure.  
-­‐                Renal  transplant  for  irreversible  failure.  
Reference:http://bestpractice.bmj.com/bestpractice/monograph/470/treatment/details.html  
   
16)    Pediatric  patient  has  symptoms  of  epiglottitis  with  stridor  and  distress.  What’s  your  
next  step?  
A)          Antibiotic  
B)          Admit  to  ICU  and  refer  to  ENT  
(No  intubation  in  choices)  
Answer:  B  
Reference:  Medscape  
   
17)    Pediatric  patient  comes  with  congested  pharynx,  tonsils  and  plaques  on  the  tongue,  
lips,  and  gingivitis.  No  lesions  on  the  hands  and  feet.  What  is  the  diagnosis?  
A)          Herpes  simplex  virus  
B)          Coxsackie  virus  
Answer:  A.  
Reference:  https://emedicine.staging.medscape.com/article/218502-­‐differential  
   
18)     Scenario:  a  patient  developed  bloody  diarrhea,  abdominal  pain  and  vomiting  after  
eating   from   a   restaurant.   History   of   hematuria.   Lab   results   show   anemia.   What   is   the  
diagnosis?                      
A)          E.coli                                                                                                                                                                    
B)          Hemolytic  uremic  syndrome  
Answer:   B.   HUS   is   characterized   by   progressive   renal   failure,   microangiopathic   hemolytic  
anemia  (MAHA),  and  thrombocytopenia.  In  typical  HUS,  diarrhea  usually  occurs.  
Reference:  Medscape  
                               
19)     What   is   the   treatment   for   the   above   question?                                                          
   
A)          Steroids                                                                                                                                                        
B)          Antibiotics  
Answer:  Treatment  of  HUS:  
-­‐                Maintenance  of  good  hydration  is  important  to  minimize  the  likelihood  of  renal  damage.  Careful  
attention  needs  to  be  paid  to  avoid  cardiopulmonary  overload,  especially  because  these  patients  are  
at  risk  of  developing  oliguria.  
-­‐                 Avoidance   of   antibiotics,   antimotility   (antidiarrheal)   agents,   opioids,   or   non-­‐steroidal   anti-­‐
inflammatory  drugs  is  advised.  
-­‐                 Platelet  transfusions  have  been  associated  with  clinical  deterioration  and  should  be  avoided  if  
possible.  
-­‐                For  the  anemia:  blood  transfusion  if  needed.  
-­‐                For  the  HTN:  CCB  
-­‐                 If   renal   failure   is   present:   dialysis   is   performed   if   clinically   indicated:   signs   and   symptoms   of  
uremia,   hyperkalemia   (potassium   >6.5   with   ECG   changes),   persistent   severe   acidosis   (bicarbonate  
<10),   hypertension   secondary   to   volume   overload   that   cannot   be   controlled   with   medical   therapy,  
and  necessity  for  transfusion  in  patient  with  volume  overload  and/or  oliguria.  
-­‐                In  irreversible  renal  failure:  renal  transplant.  
Reference:http://bestpractice.bmj.com/bestpractice/monograph/470/treatment/details.html  
   
20)    What  age  in  months  can  a  baby  say  a  few  words?  
A)          6  months  
B)          9  months  
C)          12  months                                                                                                                                                                                
D)        24  months  
At  12  months:  the  baby  can  say  2  words  
At  24  months:  2-­‐3  word  phrases  and  uses  “I,  me,  you”.  
Reference:  Toronto  notes                      
                           
21)     During   delivery   of   a   baby,   there   was   stylomastoid   foramen   trauma.   Which   of   the  
following  features  will  be  evident  when  you  examine  this  baby?  
A)          Loss  of  eye  close                                              
B)          Loss  of  facial  sensation                
C)     Loss  of  mastication  function  
Answer:  A  
Facial  Palsy  (Bell’s  palsy):  
-­‐                 It   is   usually   due   to   pressure   by   the   forceps   blade   on   the   facial   nerve   at   its   exit   from   the  
stylomastoid  foramen  or  in  its  course  over  the  mandibular  ramus.  
-­‐                 It  appears  within  1-­‐2  days  after  delivery  due  to  resultant  edema  and  hemorrhage  around  the  
nerve.  
-­‐                 Manifestations:  There  is  paresis  of  the  facial  muscles  on  the  affected  side  with  partially  opened  
eye  and  flattening  of  the  nasolabial  fold.  The  mouth  angle  is  deviated  towards  the  healthy  side.  
                             Reference:  https://www.gfmer.ch/Obstetrics_simplified/foetal_birth_injuries.htm  
                               
22)     Clear  case  about  crohn’s  disease:  a  child  with  abdominal  cramping,  diarrhea...etc.  
Endoscopy   shows   skip   lesions   and   transmural   inflammation.   What’s   the   diagnosis?                                                          
   
A)          Crohn’s  disease                                                                                                                        
B)          Ulcerative  colitis  
C)          Celiac  disease  
Answer:  A  
   
23)    Similar  case:  child  with  abdominal  pain  and  tenderness,  bloody  diarrhea  and  weight  
loss.  (no  labs).  What's  the  diagnosis?                                                                                                  
A)          Crohn’s  disease                                                                                                                                                            
B)          Celiac  disease  
C)          Ulcerative  colitis  
Answer:  abdominal  pain  and  weight  loss  go  more  with  crohn’s,  while  the  bloody  diarrhea  is  
more   with   UC.   Most   likely,   crohn’s   since   two   of   the   features   in   the   scenario   support   this  
diagnosis.  

  
24)    15  month  old  boy  with  meningitis.  What’s  the  best  antibiotic  choice  for  him?  
A)          Vancomycin  +  ceftriaxone
  
B)          Ampicillin  (or  penicillin?)  +  gentamicin  
Answer:   A.   In   infants   and   children:   Initial   antibiotic   selection   should   provide   coverage   for   the   3   most  
common   pathogens:   S   pneumoniae,   N   meningitidis,   and   H   influenzae.   According   to   the   2004  
Infectious   Diseases   Society   of   America   (IDSA)   practice   guidelines   for   bacterial   meningitis,  
vancomycin   plus   either   ceftriaxone   or   cefotaxime   is   recommended   for   those   with   suspected  
bacterial  meningitis  
Reference:  https://emedicine.medscape.com/article/961497-­‐treatment#d10  
   
25)     14   year   old   boy   with   swollen   lips.   Deficiency   of   which   of   the   following   causes   his  
presentation?  
A)          Hereditary  angioedema  
B)          Factor  D  
C)          Anaphylactic  inhibitor  
D)        C1  esterase  inhibitor  
Answer:  D  
Reference:  https://emedicine.medscape.com/article/135604-­‐overview  
   
26)     A  three  year-­‐old  girl  presented  to  Emergency  Department  with  fever,  vomiting  and  
abdominal   pain   which   began   10   hours   ago.   Radiological   examination   confirmed   a  
dilated  intestinal  pouch  attached  to  the  anterior  abdominal  wall.  Her  diagnosis  was  the  
persistence  of  a  Meckel’s  diverticulum.  
Which  of  the  following  sites  will  the  surgeon  look  for  this  diverticulum?  
A)          Lower  Duodenum  
B)          Lower  Jejunum  
C)          Lower  Ileum  
D)        Cecum  
Answer:  C  
   
27)     Neonate   presents   with   lethargy,   irritability   and   fever.   Which   of   the   following   is   the  
most  likely  causative  organism?  
A)          Listeria  monocytogens  
B)          Staph  aureus  
C)          N.meningitidis  
Answer:  A  
Reference:  Medscape  
   
28)     A   patient   presents   with   a   cough,   fever,   rhinorrhea,   malaise,   with   conjunctival  
suffusion.   There   are   small,   grayish,   irregular   lesions   surrounded   by   an   erythematous  
base,  on  the  buccal  mucus  membrane  near  the  second  molar  teeth.  
What  is  the  most  likely  diagnosis?  
A)          Measles  
B)          Rubella  
C)          Parainfluenza  
D)        Respiratory  syncytial  infection  
Answer:  A  
29)     A  baby  with  greasy  scaly  rash  at  the  edge  of  the  forehead  and  over  the  cheeks  not  
sparing   the   folds.   Which   of   the   following   is   the   appropriate   treatment?   (Seborrheic  
dermatitis)  
A)    Muropicin  topical  (antibiotic)  
Answer:   Low-­‐potency   topical   corticosteroids,   such   as   hydrocortisone,   desonide,   and  
mometasone   furoate,   have   shown   to   be   efficacious   on   the   face.   Antifungal   therapies   are  
first-­‐line   therapies   (Ketoconazole,   naftifine,   or   ciclopirox   creams   and   gels)   are   effective  
therapies.  Systemic  fluconazole  may  help  if  seborrheic  dermatitis  is  severe  or  unresponsive.  
Combination  therapy  has  been  recommended.    
Reference:  https://emedicine.medscape.com/article/1108312-­‐treatment  
   
30)    How  to  manage  croup?  
A)          Nebulized  epinephrine  and  steroid  
B)          Inhaled  salbutamol  and  betamethasone  
Answer:  A  
Reference:  Medscape  
   
31)    Minimum  age  to  give  influenza  vaccine?  
A)          3  months  
B)          6  months  
C)          9  months  
D)        12  months  
Answer:  B.  Children  younger  than  6  months  of  age  should  not  be  vaccinated.  
Reference:  https://www.cdc.gov/flu/professionals/vaccination/vax-­‐summary.htm  
   
32)    Which  of  these  patients  will  most  likely  be  diagnosed  with  rheumatic  fever  from  his  
symptoms?  
A)     Child  with  knee  swelling  and  joint  pain  and  sore  throat.  
Answer:  A  
 …  
Part  3  
   
1)           A  3  year  old  with  symptoms  of  DM  since  2  weeks  (weight  loss  of  3  kg,  dysphagia,  
and  polyuria.  What  is  the  appropriate  step  to  diagnose  him?                                                                                                      
   
A)          HLA  DR  3  
B)          Urine  dipstick  
Answer:  B  
Reference:  https://emedicine.medscape.com/article/919999-­‐workup  
   
2)          Long  scenario  of  a  boy  k/c  of  G6PD.  Came  with  Hb  of  7.  What  will  you  do?  
A)          Blood  transfusion  
B)          Give  iron                                                                                                                                                                                
C)          Reassurance  
D)        Give  folic  acid  
Answer:  
-­‐                 Affected  patients  should  be  encouraged  to  maintain  a  good  fluid  intake  and  to  eat  a  
light  diet,  as  nausea  is  common.  
-­‐                A  haematology  consultation  is  warranted  once  haemolytic  anaemia  is  diagnosed.  
-­‐                Folic  acid  is  required  to  supply  increased  RBC  production.  
-­‐                 If  severe  anaemia  (haemoglobin  <70  g/L  [<7  g/dL])  with  no  renal  impairment:  blood  
transfusion  
-­‐                 If   severe   anaemia   (haemoglobin   <70   g/L   [<7   g/dL])   with   renal   impairment:   blood  
transfusion  and  renal  dialysis.  
Reference:  http://bestpractice.bmj.com/best-­‐practice/monograph/704/treatment.html  
   
3)          A  mother  comes  with  her  9  year  old  daughter.  She’s  looking  short  and  her  mother  is  
also  short.  The  bone  age  is  7  years  and  her  labs:  all  normal  except  insulin  like  growth  
hormone  (below  the  normal  range).  What  will  you  do?  
A)          Reevaluation  after  1  year  
B)          Give  growth  hormone  
Answer:  B  
Reference:  https://emedicine.medscape.com/article/920446-­‐treatment  
   
4)           Scenario   of   a   child   with   recurrent   chest   infections   and   sinusitis.   What’s   the  
diagnosis?  
A)          Cystic  fibrosis  
B)          Primary  ciliary  dyskinesia  
Answer:  According  to  BMJ  best  practice  Primary  ciliary  dyskinesia  Usually  not  associated  
with   pancreatic   insufficiency;   chronic   purulent   middle   ear   infections,   which   are   less  
common  in  children  with  CF.  
Reference:http://bestpractice.bmj.com/bestpractice/monograph/403/diagnosis/differen
tial.html  
   
5)           Month-­‐old  girl  (I  don’t  remember  how  old  exactly),  found  to  have  meningitis  with  
negative  diplococci,  parents  are  also  concerned  about  her  6  year  old  brother.  What  to  
give  her  brother?  
A)          Oral  rifampicin
  
B)          Active  immunization
  
C)          Ampicillin  &  other  antibiotic
  
Answer:  A  
Reference:  https://www.medscape.com/viewarticle/451596_4  
   
6)          A  child  with  history  of  trauma  then  developed  knee  pain  and  swelling  with  
tenderness  on  passive  movement.  What’s  the  next  step?  
A)          Blood  culture  
B)          Empirical  Antibiotics  
C)          Knee  x-­‐ray  
D)        Examination  of  synovial  fluid  
Answer:  C  
Reference:  http://bestpractice.bmj.com/best-­‐practice/monograph/575/diagnosis/step-­‐
by-­‐step.html  
   
7)          Which  of  the  following  deficiencies  is  a  contraindication  for  BCG  vaccine?  
A)          IL  
B)          TNF  
C)          INF  gamma  
Answer:    C.  IL-­‐12  deficiency  causes:  low  INF  gamma,  disseminated  mycobacterial  and  viral  
infections.  BCG  is  contraindicated.  
Reference:  
-­‐  https://www.immunodeficiencysearch.com/interferon-­‐gamma-­‐pathway-­‐defects  
-­‐  https://www.ncbi.nlm.nih.gov/pmc/articles/PMC1905729/  
 
8)          Which  of  the  following  is  a  complication  of  mumps?  
A)          Encephalitis  
B)          Hearing  loss  
C)          Facial  palsy  
Answer:  both  A  and  B  are  complications  of  mumps.  
Reference:  http://bestpractice.bmj.com/best-­‐practice/monograph/1037/follow-­‐
up/complications.html  
   
9)          Scenario  of  a  patient  with  sickle  cell  with  a  picture  showing  swollen  foreskin  of  the  
head  of  the  penis?  
A)          Phimosis  
B)          Paraphimosis  
C)          Balanitis  
D)        Priapism  
Answer:  B  
Reference:  https://emedicine.medscape.com/article/777539-­‐clinical#showall  
   
10)    Tanner  stage  description  in  a  female  (stage  3).  
Answer:  breast  begins  to  become  more  elevated,  and  extends  beyond  the  borders  of  the  
areola,  which  continues  to  widen  but  remains  in  contour  with  surrounding  breast  (11.5–
13)  Pubic  hair  becomes  more  coarse  and  curly,  and  begins  to  extend  laterally  (11.5–13)  
Reference:  https://www.ncbi.nlm.nih.gov/pmc/articles/PMC2020314/  
   
11)    Child  chewing  his  toys,  associated  with  abdominal  pain,  looks  pale  with  high  
transferrin.  Peripheral  smear:  anisocytosis,  basophilic  stippling  with  target  cells,  
erythroid  hyperplasia  with  maturation  arrest:  
A)Hemolytic  anemia  
B)Sideroblastic  anemia,  
C)IDA  
D)            Thalassemia  trait  
Answer:  B.  
Reference:  https://emedicine.medscape.com/article/1389794-­‐workup#c7  
   
12)    Stills  disease  
Answer:  Still's  disease,  a  disorder  featuring  inflammation,  is  characterized  by  high  spiking  
fevers,  salmon-­‐colored  rash  that  comes  and  goes,  and  arthritis.  Still's  disease  is  also  
referred  to  as  systemic-­‐onset  juvenile  idiopathic  arthritis.  
Reference:  https://www.medicinenet.com/stills_disease/article.htm  
   
13)    Patient  with  hypotonia,  broad  nose  and  large  forehead,  his  growth  is  ...(smaller  
than  peers?).  What’s  the  most  likely  cause?  
A)          Metabolic  
B)          Genetic  
Answer:  B.  
-­‐                The  scenario  is  incomplete  however;  the  description  of  dysmorphic  features  goes  
more  with  chromosomal  and  genetic  abnormalities  like  (Pallister-­‐Killian  mosaic  
syndrome:  characterized  by  severe  hypotonia  in  infancy  and  early  childhood,  intellectual  
disability,  distinctive  facial  features,  sparse  hair,  areas  of  unusual  skin  pigmentation,  and  
other  birth  defects.  
-­‐                Pallister-­‐Killian  mosaic  syndrome  is  associated  with  a  distinctive  facial  appearance  
that  is  often  described  as  "coarse."  Characteristic  facial  features  include  a  high,  rounded  
forehead;  a  broad  nasal  bridge;  a  short  nose;  widely  spaced  eyes;  low-­‐set  ears;  rounded  
cheeks;  and  a  wide  mouth  with  a  thin  upper  lip  and  a  large  tongue.  
Reference:  https://ghr.nlm.nih.gov/condition/pallister-­‐killian-­‐mosaic-­‐syndrome  
   
14)    A  patient  with  seizures  on  anti-­‐epileptic  treatment.  What  will  u  do  regarding  his  
vaccinations?  
A)          Give  all  vaccines  
B)          Don't  give  live  attenuated  vaccine  
C)          Give  all  vaccine  except  DTap  
Answer:  A  
Reference:  https://www.medscape.com/viewarticle/413688  
   
15)    A  32  week  preterm  baby  stayed  in  incubation  for  2  weeks  then  discharged  with  
good  health.  When  to  give  vaccine?  
A)          Correct  for  chronological  age.  
B)          Give  vaccine  as  scheduled.  
C)          Give  vaccine  half  the  dose.  
Answer:  B.  Whether  full-­‐term  or  preterm,  the  vaccination  schedule  doesn’t  change.  
Reference:  https://www.medscape.com/viewarticle/775544_1  
 
16)     Absence   of   moro   reflex   on   the   right   side   of   the   infant   is   due   to?                                                          
   
A)          Intracranial  hemorrhage.  
B)          Neonatal  hypoglycemia.  
C)          Neonatal  sepsis.  
D)        Erb’s  palsy.  
Answer:  D  
Reference:  
-­‐               http://bestpractice.bmj.com/best-­‐practice/monograph/746/diagnosis/history-­‐and-­‐
examination.html  
-­‐                https://medlineplus.gov/ency/article/003293.htm  
   
17)    Child  with  DM  type  1.  What's  the  mechanism  of  action  of  the  disease?  
A)          Triglyceride  uptake  
B)          Liver  increase  of  fatty  acid  
 Answer:    
 
18)     A  long  case  with  barking  cough  that  comes  only  at  night  and  resolves.  What’s  the  
diagnosis?  
   
19)    A  baby  has  a  hemangioma.  When’s  the  best  time  to  resect  it?  
A)          1  day  
B)          1  week  
C)          3  months  
D)        6  months  
   
20)     A   baby   fell   from   the   stairs.   He   presented   with   multiple   lacerations   of   different  
degrees  and  a  fracture.  What’s  your  next  step?  
A)          Refer  to  orthopedics  
B)          Call  a  social  worker  
   
21)     Child  woke  up  from  a  nightmare  screaming  and  crying.  Parents  asked  the  child  and  
he  doesn’t  remember  anything  what  sleep  stage?  
A)          1  
B)          2  
C)          3                                                                                                                                                                              
D)        4  
Answer:  C  or  D.  These  slow  wave  sleep  parasomnias  include  confusional  arousals,   night  
terrors  (pavornocturnus),  and  sleepwalking  (somnambulism).  
Reference:  https://emedicine.medscape.com/article/1140322-­‐overview#a4  
   
22)     3-­‐year-­‐old  child  presenting  with  high  fever  for  the  last  2  days,  vomiting,  refusal  to  
eat  and  red  ears.  Which  of  the  following  will  help  you  find  the  diagnosis?  
A)          CSF  analysis                                                                                                                                                                    
B)          Blood  culture                                                                                                                    
C)          Urine  culture  
   
23)    A  mother  brought  her  child  saying  that  he’s  always  sucking  his  thumb  and  when  she  
asks  him  to  stop,  he  doesn’t.  What  are  you  going  to  do?  
A)          Ignore  him  
 
24)     Long  scenario:  18  month  old  baby  presents  with  irritation  and....,  he  was  drinking  
breast  milk  till  the  age  of  9  months.  Then,  he  was  switched  to  cow’s  milk  around  one  
liter  daily.  The  mother  noticed  bad  smelling  stool  for  3  days  with  change  in  consistency.  
The   baby   looks   pale.   Labs   were   provided   and   show   low   Hb.   What   is   the   cause   of   the  
child’s  symptoms?  
A)          Cow  milk  allergy  
B)     Gastrointestinal  infection?                      
Answer:    
   
25)    When  to  screen  for  congenital  hypothyroidism?                                                                                                                                                              
A)          At  birth  by  cord  blood                                                                                                                                                        
B)          After  1  week                                                                                                                              
C)          After  1  month  
D)        After  6  months  
Answer:   updated   AAP   Guidelines   on   Newborn   Screening   and   Therapy   for   Congenital  
Hypothyroidism   recommend   the   screening   of   all   infants   should   be   performed   between  
two   and   four   days   of   birth.   If   this   is   not   possible,   testing   should   be   performed   before  
discharge  or  within  seven  days  of  birth.  
   
26)     24   Year   old   mother   had   a   child   with   suspected   Down   syndrome;   she's   asking   if  
there  is  a  chance  of  another  child  to  have  this  disease.  What  test  will  you  order  to  help?  
A)          Karyotyping  of  child.                                                                                                                          
B)          Karyotype  both  the  mother  and  child  
Answer:   Most   probably   A,   because   you   have   to   karyotype   the   child   first,   then   you  
karyotype  both  parents  depending  on  the  results.  
The   extra   chromosome   21   may   result   from   meiotic   disjunction,   translocation   or  
mosaicism.  So,  if  the  karyotype  of  the  child  shows:  
1)           Meiotic  disjunction:  no  need  to  karyotype  parents  and  the  incidence  of  trisomy  
21  is  related  to  maternal  age.  
2)           Robertsonian  translocation:  Both  parents  should  be  tested.  If  the  mother  is  the  
carrier,  the  risk  is  10-­‐15%,  and  if  the  father  is  the  carrier  the  risk  is  2.5%.  If  one  of  the  
parents   carries   the   rare   21:21   translocation,   then   all   children   will   have   Down  
syndrome.  If  none  of  the  parents  are  carry  the  translocation,  the  risk  is  <1%.  
Reference:  illustrated  textbook  of  pediatrics  
   
27)     A   child   swallowed   safety   pins.   Abdominal   x-­‐ray   revealed   multiple   pins   in   the  
intestines.  What  are  you  going  to  do?  
A)          Admit  for  observation  &  repeat  x-­‐ray  
B)          Discharge  
C)          Exploratory  laparotomy  
Answer:  A.  Objects  with  sharp  edges  or  points  present  a  special  problem  because  of  the  
possibility   for   erosion   or   perforation.   These   include   pins,   needles,   tacks,   razor   blades,  
pieces   of   glass,   or   open   safety   pins.   Children   who   have   swallowed   such   objects   should   be  
vigilantly  observed.  Esophageal  impaction  demands  surgical  removal;  however,  many  of  
these   objects   also   pass   through   the   GI   tract   without   incident   once   they   are   past   the  
gastroesophageal   junction.   Obtain   a   daily   radiograph   (for   radiopaque   objects)   and  
monitor   closely   for   signs   of   peritonitis   or   GI   bleeding.   In   these   cases,   stools   are   examined  
for  the  foreign  body  in  question.  GI  hemorrhage  or  signs  of  peritonitis  mandate  surgical  
exploration  and  removal  of  the  object.  
Reference:  https://emedicine.medscape.com/article/933015-­‐treatment  
   
28)     A   child   with   drug   overdose   (they   didn't   mention   the   drug).   What's   the  
management?  
A)          Activate  charcoal  
   
29)     A   baby   (I   am   not   sure   about   the   age)   did   not   take   vaccines.   Presented   with   seizure.  
Lab:  high  Ca.  What’s  the  diagnosis?  
A)     Neonate  tetanus  
   
30)     A   mother   came   with   her   boy   for   6   month   vaccine,   she   mentioned   that   he  
developed  anaphylaxis  after  the  4  month  vaccines  &  needed  hospital  admission.  Which  
of  the  following  is  appropriate?  
A)          Defer  all  vaccines  
B)          Defer  DTP  vaccine  only  
C)          Do  allergy  test  
D)        Give  all  vaccines  &  observe  for  1  hour  
Answer:  C.  
Reference:https://www.uptodate.com/contents/image?imageKey=ALLRG%2F81252&topicKey=ALLRG
%2F2074&source=see_link  
   
31)     Child   presented   with   heaves   on   examination.   ECG   shows   RBBB.   On   ECHO:   right  
ventricle   motion   abnormality   and   right   ventricle   hypertrophy.   What   is   the   most   likely  
cause?  
A)          Mitral  prolapse  
B)          ASD  
C)          VSD  
D)        Coarctation  of  aorta  
Answer:  B.  
-­‐                Patients  may  have  a  hyperdynamic  right  ventricular  impulse  by  palpation,  especially  
in  older  children  and  adults  with  a  large  left-­‐to-­‐right  shunt.  
-­‐                 ECG   shows   normal   or   tall   p   waves   indicating   RA   enlargement   or   R-­‐wave   voltages   in  
lead   V1   greater   than   the   upper   limit   of   normal   for   age,   suggesting   right   ventricular  
hypertrophy,  may  be  present  in  larger  defects.  
-­‐                 Two-­‐dimensional   echocardiography   demonstrates   right   atrial   and   ventricular  
enlargement,  as  well  as  the  defect  itself,  especially  for  secundum-­‐type  defects.    
Reference:http://bestpractice.bmj.com/best-­‐practice/monograph/1099/diagnosis/step-­‐by-­‐
step.html  
   
32)     Child   with   a   history   of   fall   without   losing   consciousness.   He   vomited   twice   and   is  
crying  and  complaining  of  a  headache.  What  should  you  do?  
A)          CT  
B)          Close  observe  
Answer:  A  
Indications  for  CT  scanning  in  a  patient  with  a  head  injury  include  anisocoria,  GCS  score  
less  than  12  (some  studies  suggest  CT  scanning  in  any  pediatric  patient  with  a  GCS  score  
of  <  15),  posttraumatic  seizures,  amnesia,  progressive  headache,  an  unreliable  history  or  
examination   because   of   possible   alcohol   or   drug   ingestion,   loss   of   consciousness   for  
longer   than   5   minutes,   physical   signs   of   basilar   skull   fracture,   repeated   vomiting   or  
vomiting  for  more  than  8  hours  after  injury,  and  instability  after  multiple  trauma.  
Reference:https://emedicine.medscape.com/article/907273-­‐workup#c8  
   
33)    An  18  month  old  female  baby  with  anemia.  What’s  the  diagnosis?  INCOMPLETE  
A)          Homozygous  b  thalassemia  
B)          Homozygous  a  thalassemia  
C)          Carrier  of  a  thalassemia  
Answer:  
-­‐                 Alpha   thalassemia   silent   carrier:   Patients   are   likely   to   be   asymptomatic   and  
hematologically  normal.  
-­‐                 Alpha-­‐thalassemia   major   or   homozygous   alpha(0)   thalassemia:   Hemoglobin   Bart  
hydrops  fetalis  syndrome  is  typically  caused  when  all  4  alpha-­‐globin  genes  are  deleted.  
-­‐                Beta-­‐thalassemia  minor  (commonly  referred  to  as  beta-­‐thalassemia  trait):  usually  
asymptomatic;  the  mild  microcytic  anemia  is  often  misdiagnosed  as  iron  deficiency  
anemia.  
-­‐                Beta-­‐thalassemia  intermedia:  usually  a  similar  presentation  to  beta-­‐thalassemia  major  
but  as  a  toddler  or  older  child;  symptoms  are  usually  less  pronounced  and  the  course  is  
usually  more  insidious.  
-­‐                Beta-­‐thalassemia  major  (also  called  Cooley's  anemia):  complete  absence  of  
hemoglobin  A;  often  presents  at  a  few  months  of  age  with  progressive  pallor  and  
abdominal  distension;  perinatal  history  is  most  often  uneventful,  and  the  infant  may  be  
pale,  possibly  with  poor  feeding  and  decreased  activity;  hepatosplenomegaly  and  bony  
abnormalities  are  often  present  at  presentation,  most  often  of  the  skull  (frontal  and  
parietal  bossing,  and  chipmunk  facies).  
BUT   (A)   is   closer   because   approx.   after   18   months   HbF   production   reduces   and   HbA  
cannot  be  produced.  
Reference:  http://bestpractice.bmj.com/best-­‐practice/monograph/251.html  
   
34)     Before   giving   influenza   vaccine   you   have   to   ask   if   the   child   has   an   allergy   from  
which  of  the  following  substances?  
A)          Egg  
Answer:  A  
Reference:  https://www.cdc.gov/flu/professionals/vaccination/vax-­‐summary.htm  
   
35)     A   child   with   barking   cough   only.   No   fever   or   shortness   of   breath.   Vitals   are   normal.  
What’s  your  management?  
A)          Moist  oxygen  
B)          Nebulizer  racemic  acid                                                                                                                                                      
C)          Antibiotics  
Answer:  A.  Cool  mist  from  a  humidifier  and/or  sitting  with  the  child  in  a  bathroom  filled  
with  steam  generated  by  running  hot  water  from  the  shower,  help  minimize  symptoms.  
Reference:https://emedicine.medscape.com/article/962972-­‐treatment          
36)     A   patient   with   hematuria   and   upon   examination,   he   has   aniridia   (absence   of   the  
iris).  What’s  the  diagnosis?  
A)          Wilms  tumor  
B)          Neuroblastoma  
Answer:  A  
Reference:  https://emedicine.medscape.com/article/989329-­‐overview                        
   
37)     A  child  came  with  abdominal  mass.  Imaging  showed  renal  tumor.  The  patient  has  
absent  iris.  What  is  the  most  likely  diagnosis?                                                                        
A)          Wilms's  tumor  
Answer:  A  
Reference:  https://emedicine.medscape.com/article/989329-­‐overview                        
   
38)     Pediatric  patient  known  case  of  acute  lymphoblastic  leukemia  (ALL),  presented  to  
the  ER  with  fever  and  pancytopenia  (lab  results  were  provided).  What  is  your  action?  
A)          Blood  transfusion  
B)          Refer  to  oncology                                                                                                                                          
C)          Start  Antibiotics  
Answer:  C  
Reference:  BMJ  
   
39)    Most  common  presentation  of  congenital  heart  disease?                                                                                                                    
A)          JVD  
B)          Difficulty  feeding  
C)          Ascites                                                                                                                                                      
Answer:   the   most   common   presentation   of   congenital   heart   disease   is   with   a   heart  
murmur.  
Reference:  Illustrated  textbook  of  pediatrics  
   
40)    Which  of  the  following  congenital  heart  defects  is  associated  with  Down  syndrome?  
A)          Endocardial  cushion  defects  such  as  AVSD  
Answer:  A  
Reference:  https://emedicine.medscape.com/article/943216-­‐clinical#b2  
                                                                                                       
41)     A   child   with   foul   smelling   breath   and   seed   like   structures   coming   out   of   the   mouth.  
He  is  also  a  mouth  breather.  No  history  of  fever.  What’s  the  most  likely  diagnosis?  
A)          Pulmonary  disease  
B)          Focal  tonsillitis  
Answer:   B.   Individuals   with   acute   tonsillitis   present   with   fever,   sore   throat,   foul   breath,  
dysphagia,   odynophagia,   and   tender   cervical   lymph   nodes.   Airway   obstruction   may  
manifest   as   mouth   breathing,   snoring,   sleep-­‐disordered   breathing,   nocturnal   breathing  
pauses,  or  sleep  apnea.  
Reference:  https://emedicine.medscape.com/article/871977-­‐clinical  
   
42)     A  2-­‐year  child  came  to  the  clinic  with  his  mother.  He  scribbles  circles,  runs  around  
and  climbs  onto  the  chair,  plays  with  his  friends  but  does  not  share  his  toys.  He  speaks  
10  words.  He  names  the  picture  you  point  to.  What  is  the  best  thing  to  tell  his  mom?  
A)          He  is  normal  
B)          Delayed  social  development  
C)          Delayed  language  development  
Answer:  A.  He  is  normal  since  sharing  toys  and  cooperative  play  only  starts  at  4  years  of  
age.  
Reference:  Toronto  notes  
   
43)    6  month  old  baby  presented  with  jaundice,  seizure,  irritability  and  vomiting.  He  has  
been   breast-­‐fed   during   his   first   3   months   of   life,   then   he   has   been   started   on  
commercially  available  milk  formula.  More  recently,  his  mother  introduced  fruit  juices  
into  his  diet.  Investigations  revealed  positive  urine  reducing  substances.  What  element  
should  avoided  in  his  diet?                                                                                                                                    
A)          Galactose.  
B)          Fructose.  
C)          Phenylalanine.  
D)        Irrelevant  choice.  
Answer:  B  
   
44)     4   year   old   child   brought   by   his   parents   to   pediatric   outpatient   clinic   with   them  
complaining   of   his   massive   uncontrolled   appetite   during   the   last   18   months.   Weight  
was   above   95th   percentile,   while   height   was   below   5th   percentile.   Mother   reported  
that  her  son  was  failing  to  thrive  during  his  first  two  years,  beside  the  fact  that  he  was  
developmentally  delayed  compared  to  his  siblings  until  he  caught  up  late.  O/E  he  had  a  
high   forehead,   broad   nose,   small   peripheries   (Hands   and   Feet).   What   is   the   cause  
behind  his  symptoms?                                                
A)          Genetic.  
B)          Metabolic.  
C)          Nutritional.  
D)        Irrelevant  choice.  
Answer:  A  (Prader-­‐Willi  syndrome)  
Reference:  Medscape  
   
45)     A  5  year  old  boy  presents  with  tender,  swollen  and  painful  testicle.  O/E  there  was  
absent  cremasteric  reflex.  Which  of  the  following  is  the  correct  diagnosis?  
A)          Epididymo-­‐orchitis.  
B)          Testicular  torsion.  
Answer:  B.  The  most  sensitive  physical  finding  in  testicular  torsion  is  the  absence  of  the        
  cremasteric  reflex.  
Reference:  aafp.org  
                                                                                                                                                                               
46)     Pediatric   case   of   Hemolytic   uremic   syndrome:   he   has   petechia,   hematuria   and  
proteinuria.   2   weeks   ago,   he   develop   bloody   diarrhea.   The   doctor   prescribed  
symptomatic  treatment  and  probiotics.  Platelets  95,  WBC  48.  What’s  your  next  step?  
A)          Antibiotic  
B)          Platelet  transfusion  
C)          Anticoagulant  
Answer:  Treatment  of  HUS:  
-­‐                Avoid  antibiotics  and  platelet  transfusion.  
-­‐                Good  hydration  with  IV  fluids.  
-­‐                Blood  transfusion  in  anemic  patients  when  indicated.  
-­‐                CCB  for  HUS  with  hypertension.  
-­‐                Dialysis  for  renal  failure.  
-­‐                Renal  transplant  for  irreversible  failure.  
Reference:http://bestpractice.bmj.com/bestpractice/monograph/470/treatment/details.
html  
   
47)    Pediatric  patient  has  symptoms  of  epiglottitis  with  stridor  and  distress.  What’s  your  
next  step?  
A)          Antibiotic  
B)          Admit  to  ICU  and  refer  to  ENT  
(No  intubation  in  choices)  
Answer:  B  
Reference:  Medscape  
   
48)    Pediatric  patient  comes  with  congested  pharynx,  tonsils  and  plaques  on  the  tongue,  
lips,  and  gingivitis.  No  lesions  on  the  hands  and  feet.  What  is  the  diagnosis?  
A)          Herpes  simplex  virus  
B)          Coxsackie  virus  
Answer:  A.  
Reference:  https://emedicine.staging.medscape.com/article/218502-­‐differential  
   
49)     Scenario:  a  patient  developed  bloody  diarrhea,  abdominal  pain  and  vomiting  after  
eating   from   a   restaurant.   History   of   hematuria.   Lab   results   show   anemia.   What   is   the  
diagnosis?                      
A)          E.coli                                                                                                                                                      
B)          Hemolytic  uremic  syndrome  
Answer:  B.  HUS  is  characterized  by  progressive  renal  failure,  microangiopathic  hemolytic  
anemia  (MAHA),  and  thrombocytopenia.  In  typical  HUS,  diarrhea  usually  occurs.  
Reference:  Medscape  
                               
50)     What   is   the   treatment   for   the   above   question?                                                          
   
A)          Steroids                                                                                                                                                
B)          Antibiotics  
Answer:  Treatment  of  HUS:  
-­‐                 Maintenance   of   good   hydration   is   important   to   minimize   the   likelihood   of   renal  
damage.   Careful   attention   needs   to   be   paid   to   avoid   cardiopulmonary   overload,  
especially  because  these  patients  are  at  risk  of  developing  oliguria.  
-­‐                 Avoidance  of  antibiotics,  antimotility  (antidiarrheal)  agents,  opioids,  or  non-­‐steroidal  
anti-­‐inflammatory  drugs  is  advised.  
-­‐                 Platelet  transfusions  have  been  associated  with  clinical  deterioration  and  should  be  
avoided  if  possible.  
-­‐                For  the  anemia:  blood  transfusion  if  needed.  
-­‐                For  the  HTN:  CCB  
-­‐                 If   renal   failure   is   present:   dialysis   is   performed   if   clinically   indicated:   signs   and  
symptoms  of  uremia,  hyperkalemia  (potassium  >6.5  with  ECG  changes),  persistent  severe  
acidosis   (bicarbonate   <10),   hypertension   secondary   to   volume   overload   that   cannot   be  
controlled   with   medical   therapy,   and   necessity   for   transfusion   in   patient   with   volume  
overload  and/or  oliguria.  
-­‐                In  irreversible  renal  failure:  renal  transplant.  
Reference:http://bestpractice.bmj.com/bestpractice/monograph/470/treatment/details.
html  
   
51)    What  age  in  months  can  a  baby  say  a  few  words?  
A)          6  months  
B)          9  months  
C)          12  months                                                                                                                                                                        
D)        24  months  
At  12  months:  the  baby  can  say  2  words  
At  24  months:  2-­‐3  word  phrases  and  uses  “I,  me,  you”.  
Reference:  Toronto  notes              
                           
52)     During   delivery   of   a   baby,   there   was   stylomastoid   foramen   trauma.   Which   of   the  
following  features  will  be  evident  when  you  examine  this  baby?  
A)          Loss  of  eye  close                                    
B)          Loss  of  facial  sensation        
C)     Loss  of  mastication  function  
Answer:  A  
Facial  Palsy  (Bell’s  palsy):  
-­‐                 It  is  usually  due  to  pressure  by  the  forceps  blade  on  the  facial  nerve  at  its  exit  from  
the  stylomastoid  foramen  or  in  its  course  over  the  mandibular  ramus.  
-­‐                 It   appears   within   1-­‐2   days   after   delivery   due   to   resultant   edema   and   hemorrhage  
around  the  nerve.  
-­‐                 Manifestations:   There   is   paresis   of   the   facial   muscles   on   the   affected   side   with  
partially   opened   eye   and   flattening   of   the   nasolabial   fold.   The   mouth   angle   is   deviated  
towards  the  healthy  side.  
Reference:  https://www.gfmer.ch/Obstetrics_simplified/foetal_birth_injuries.htm  
                               
53)     Clear  case  about  crohn’s  disease:  a  child  with  abdominal  cramping,  diarrhea...etc.  
Endoscopy   shows   skip   lesions   and   transmural   inflammation.   What’s   the   diagnosis?                                                          
   
A)          Crohn’s  disease                                                                                                                
B)          Ulcerative  colitis  
C)          Celiac  disease  
Answer:  A  
   
54)    Similar  case:  child  with  abdominal  pain  and  tenderness,  bloody  diarrhea  and  weight  
loss.  (no  labs).  What's  the  diagnosis?                                                                                                  
A)          Crohn’s  disease                                                                                                                                                    
B)          Celiac  disease  
C)          Ulcerative  colitis  
Answer:  abdominal  pain  and  weight  loss  go  more  with  crohn’s,  while  the  bloody  diarrhea  
is  more  with  UC.  Most  likely,  crohn’s  since  two  of  the  features  in  the  scenario  support  this  
diagnosis.  

  
55)    15  month  old  boy  with  meningitis.  What’s  the  best  antibiotic  choice  for  him?  
A)          Vancomycin  +  ceftriaxone
  
B)          Ampicillin  (or  penicillin?)  +  gentamicin  
Answer:  A.  In  infants  and  children:   Initial  antibiotic  selection  should  provide  coverage  for  
the   3   most   common   pathogens:   S   pneumoniae,   N   meningitidis,   and   H   influenzae.  
According   to   the   2004   Infectious   Diseases   Society   of   America   (IDSA)   practice   guidelines  
for   bacterial   meningitis,   vancomycin   plus   either   ceftriaxone   or   cefotaxime   is  
recommended  for  those  with  suspected  bacterial  meningitis  
Reference:  https://emedicine.medscape.com/article/961497-­‐treatment#d10  
   
56)    14  year  old  boy  with  swollen  lips.  Deficiency  of  which  of  the  following  causes  his  
presentation?  
A)          Hereditary  angioedema  
B)          Factor  D  
C)          Anaphylactic  inhibitor  
D)        C1  esterase  inhibitor  
Answer:  D  
Reference:  https://emedicine.medscape.com/article/135604-­‐overview  
   
57)    A  three  year-­‐old  girl  presented  to  Emergency  Department  with  fever,  vomiting  and  
abdominal  pain  which  began  10  hours  ago.  Radiological  examination  confirmed  a  
dilated  intestinal  pouch  attached  to  the  anterior  abdominal  wall.  Her  diagnosis  was  the  
persistence  of  a  Meckel’s  diverticulum.  
Which  of  the  following  sites  will  the  surgeon  look  for  this  diverticulum?  
A)          Lower  Duodenum  
B)          Lower  Jejunum  
C)          Lower  Ileum  
D)        Cecum  
Answer:  C  
   
58)    Neonate  presents  with  lethargy,  irritability  and  fever.  Which  of  the  following  is  the  
most  likely  causative  organism?  
A)          Listeria  monocytogens  
B)          Staph  aureus  
C)          N.meningitidis  
Answer:  A  
Reference:  Medscape  
   
59)    A  patient  presents  with  a  cough,  fever,  rhinorrhea,  malaise,  with  conjunctival  
suffusion.  There  are  small,  grayish,  irregular  lesions  surrounded  by  an  erythematous  
base,  on  the  buccal  mucus  membrane  near  the  second  molar  teeth.  
What  is  the  most  likely  diagnosis?  
A)          Measles  
B)          Rubella  
C)          Parainfluenza  
D)        Respiratory  syncytial  infection  
Answer:  A  
   
60)    A  baby  with  greasy  scaly  rash  at  the  edge  of  the  forehead  and  over  the  cheeks  not  
sparing  the  folds.  Which  of  the  following  is  the  appropriate  treatment?  (Seborrheic  
dermatitis)  
A)     Muropicin  topical  (antibiotic)  
Answer:  Low-­‐potency  topical  corticosteroids,  such  as  hydrocortisone,  desonide,  and  
mometasone  furoate,  have  shown  to  be  efficacious  on  the  face.  Antifungal  therapies  are  
first-­‐line  therapies  (Ketoconazole,  naftifine,  or  ciclopirox  creams  and  gels)  are  effective  
therapies.  Systemic  fluconazole  may  help  if  seborrheic  dermatitis  is  severe  or  
unresponsive.  Combination  therapy  has  been  recommended.    
Reference:  https://emedicine.medscape.com/article/1108312-­‐treatment  
   
61)    How  to  manage  croup?  
A)          Nebulized  epinephrine  and  steroid  
B)          Inhaled  salbutamol  and  betamethasone  
Answer:  A  
Reference:  Medscape  
   
62)    Minimum  age  to  give  influenza  vaccine?  
A)          3  months  
B)          6  months  
C)          9  months  
D)        12  months  
Answer:  B.  Children  younger  than  6  months  of  age  should  not  be  vaccinated.  
Reference:  https://www.cdc.gov/flu/professionals/vaccination/vax-­‐summary.htm  
   
63)    Which  of  these  patients  will  most  likely  be  diagnosed  with  rheumatic  fever  from  his  
symptoms?  
A)     Child  with  knee  swelling  and  joint  pain  and  sore  throat.  
Answer:  depends  on  the  other  choices,  but  most  likely  A.  
Reference:  http://bestpractice.bmj.com/best-­‐
practice/monograph/404/diagnosis/history-­‐and-­‐examination.html  
   
   
64)    Child  with  hip  pain,  x-­‐ray  was  normal  but  US  showed  fluid.  Labs  revealed:  high  ESR  
and  CRP,  otherwise  normal.  What  to  do?  
A)          MRI  
B)          CT  hip  and  pelvis.  
C)          Aspiration  
Answer:  most  likely  C  
 
65)    18  month  old  child  took  Hib,  MMRV,  and  ...  vaccine  one  week  ago.  He  came  for  HAV  
but  was  not  available.  When  to  give  HAV?  
A)          Immediately                
B)          After  1  week  
C)          After  3  weeks  
D)        After  7  weeks  
Answer:  most  likely  A  
   
66)    4  month  old  child  came  for  vaccination  he  has  a  2  day  history  of  watery  diarrhea,  
abdominal  pain  and  vomiting.  What  are  you  going  to  do  regarding  his  vaccination?  
A)          Give  all  vaccines  except  DTap  
B)          Only  give  hep  B  
C)          Give  all  vaccines  except  for  OPV  
D)        Defer  all  vaccines  
Answer:  most  likely  C  
 
 

Family  Medicine  
   
   
Notes:  
-­‐  Drug  for  HTN  reduces  heart  rate  and  peripheral  resistance:  Carvedilol  
-­‐  Drug  for  HTN  reduced  preload  and  cause  vasodilatation:  ACEI  
-­‐  Drug  induced  Hyperpigmentation:  amidarone  
-­‐  Aspirin  SE:  diarrhea  
-­‐  Bisphosphonate  inhibits  osteoclast  activity:  alendronate  
-­‐  Bisphosphonate  works  by  ADP  bla  bla:  coldronate,  etodronate  and  toldronate  
-­‐  Med  for  osteoprosis  causes  heartburn:  risedronate  
-­‐  analgesia  for  cholecystitis:  Meperidine  
-­‐  HgF  inducer:  hydroxyurea,  Na  butyrate,  decitibine  and  5-­‐azacytidine  
-­‐  Monitor  high  cholesterol:  every  6  months  
-­‐  Borderline  HgA1c;  repeat  after  1  year  
-­‐  Angular  cheilosis:  Vit  b6  
-­‐  Measles:  10-­‐14  D  occupation  
-­‐  Varicella:  10-­‐21  D  
-­‐  Smoking  withdrawal  peak:  3-­‐5  days  
-­‐  Optha  screening  for  DM:  
-­‐  type  1  DM:  annually  beginning  5  years  after  onset  of  DM  
-­‐  Type  2  DM:  annually  beginning  at  time  of  diagnosis  
-­‐  Breast  self  exam:  monthly  
-­‐  Mammogram  normal:  repeat  every  2  years  
-­‐  Most  common  infection  in  saudi  arabia  and  disable  patient  from  work:  HBV  
-­‐  High  cholesterol  pt,  next  follow:  6  months  
-­‐  DM  screening:  if  normal  repeat  after  3  years,  if  pre-­‐diabetic:  yearly.  
-­‐  Safe  in  prgenancy:  DTAP,  HBV  and  influenza  
-­‐  HBV:  HBsAg,  then  HBc  Igm,  then  HBs  antibody  
-­‐  Treat  water  from  entameba:  boiling  
-­‐  Cow  milk:  more  in  protein  
-­‐  Parasite  with  meat:  trichinosis  
-­‐  CRC  screening  in  UC  patients  starts  8  years  after  the  Diagnosis.  
-­‐  Burkitt  lymphoma:  BCL2  overexpression,  ki67  positive  in  hitsopathology  
-­‐  HIV  CD  &gt;200:  can  be  given  MMR  and  varicell  
1.     Researcher  want  to  measure  obesity  in  children  he  included  BMI  and  gender.  what  to  add  
to  asses  risk  of  obesity?  
A.     Girth  measurement    
B.     HDL/LDL  ratio  
C.     Dietary  habits  
D.        Skin  fold  Thickness  
Answer:  D  
BMI  correlates  with  fat  content  measured  by  skin  fold  thickness.  
   
2.     Pt  every  menstrual  period  has  depression  even  post  menstrual,  these  problem  are  
continuous  for  6  month,  on  examination  pt  depressed  >>  
Answer:  major  depression  disorders”  and  need  referral  to  psychiatry.  
   
3.     About  secondry  prevention  
a)          -­‐personal  education  
b)     -­‐secreening  
Answer:  B.  
   
5.     nulliparous  came  to  you  age  53  he  mother  got  breast  cancer  when  she  was  38  what  is  the  
most  important  thing  to  check  for  her  abnormal  vaginal  bleeding  ?  
Answer:  AUB  is  considered  a  very  alarming  in  postmenopausal  and  highly  suggests  endometrial  
cancer  which  mandates  full  work  up.  [  Endometrial  cancer  wkx:    sampling  by  biopsy  or  D&C  +  
hysteroscopy,  pelvic  ultrasound.  ]  All  DDx  in  the  figure  are  important  causes  that  we  need  to  rule  
out.  
   
 
 
   
   
 
   
6.     A  man  tested  HIV  +ve.  You  told  him  that  he  should  inform  his  wife  but  he  refused.  What  
should  you  do?  
A-­‐Inform  the  ministry  of  health  
B-­‐Inform  his  wife  regardless  
C-­‐Try  to  convince  him  to  tell  her    
Answer:  B.  
Reference:  https://www.webmd.com/hiv-­‐aids/features/talk-­‐about-­‐hiv-­‐positive#1  
   
7.     Best  diagram  used  for  screening  test  ?  
Answer:  Nomogram.  A  Thorough  search  was  done  without  knowing  the  answer.  
   
8.     Best  way  to  treat  water  against  entamebeoa  histolytica  ?  
a)     Cholorization  
b)     Boiling  
 Answer:  B.  
   
9.     1st  responsible  for  Bp  regulation  :                                                                                                                                                                                        
A.                      heart  
B.                      aorta  
C.                      arterioles                                                                                            
D.                     capillaries  
Answer  :       B      
   
High  BP  in  aorta  and  carotid  sinuses  increase  =>    +  Baroreceptor  =>  CVS  centre  in  the  medulla.  
“sympathetic  to  parasympathetic  ratio]                                                                                                                                                                                    
Baroreceptor  reflex:  Baroreceptors  in  the  high  pressure  receptor  zones  detect  changes  in  arterial  

pressure.  These  baroreceptors  send  signals  ultimately  to  the  medulla  of  the  brain  stem,  specifically  

to  the  rostral  ventrolateral  medulla  (RVLM).  The  medulla,  by  way  of  the  autonomic  nervous  system,  

adjusts  the  mean  arterial  pressure  by  altering  both  the  force  and  speed  of  the  heart's  contractions,  
as  well  as  the  systemic  vascular  resistance.  The  most  important  arterial  baroreceptors  are  located  in  

the  left  and  right  carotid  sinuses  and  in  the  aortic  arch  

10.    Pic  of  diagram  showing  progress  of  diabetic  nephropathy  (GFR  and  albuminuria)  cases  in  
years,  then  ask  about  the  specific  point  of  change  
a)          10  year                                                                                                                                                                                    
b)          15  year  
c)          20  year  
d)          25  year                                                                                                      
Answer:  C.  
   
   
   
   
   
   
 

 
   
   
   
 
11.    Which  of  these  vitamins  r  involved  in  degeneration  of  spinal  cord  lead  to  sensory  and  motor  
manifestation  ?  
Answer:  Vit  b6,  Vit  b12,  Vit  b1  Vit  E                                                                                                                                                                      
http://emedicine.medscape.com/article/1152670-­‐overview#a5  
   
12.    which  one  of  the  following  cancers  directly  related  to  smoking:  
A-­‐  Colon  
B-­‐  Bladder  
C-­‐  Testicular  
D-­‐  Small  cancer.  
Answer:  B.      
Compared  to  never  smokers,  TCC  risk  was  threefold  higher  in  former  smokers  (95%  CI  2.07-­‐4.18)  and  more  
than  sixfold  higher  in  current  smokers  (95%  CI  4.54-­‐9.85).  TCC  risk  steadily  increased  with  increasing  intensity  
(OR  for  ≥25  cigarettes/day  8.75;  95%  CI  3.40-­‐22.55)  and  duration  of  smoking  (OR  for  ≥50  years  5.46;  95%  CI  
2.60-­‐11.49).        
References:  https://www.ncbi.nlm.nih.gov/pubmed/24964779                                                                                                                                                                                                    
                                                                                                                                                                                                   
13.    Treatment  of  HBV  (started  since  one  month)  
a)          lamivudine  
b)          Interferon  
Answer:  B.  
   
   
14.    Dose  of  glucagon  in  IV?  
a)          0.1  
b)          0.2  
c)          1  
d)          2  
Answer:  C.    
In  severe  hypoglycaemia  give  1mg  ‘1  unit’  and  repeat  q15min  once  or  twice  and  give  dextrose  as  
soon  as  its  available  if  nor  response.  
References:  https://reference.medscape.com/drug/glucagen-­‐glucagon-­‐342712  
   
15.    Reason  to  start  screening  for  cancer?  
a)     If  it  can  change  the  natural  history  of  the  disease.  
Answer:  A.  no  other  options  are  available.  
   
16.    Evidence  of  screening  for  hyperlipidemia  in  adults  with  no  cardiac  risk  factors  
Answer:    not  recommended  if  CHD  do  not  exist.  
   
 

   
 
 
   
17.    how  to  monitor  unfractionated  heparin?  
a)          aptt  
b)          pt  INR  
Answer:  A.      
The  most  common  methods  for  monitoring  UFH  are  the  activated  partial  thromboplastin  time  (aPTT)  
and  antifactor  Xa  heparin  assay  (anti-­‐Xa  HA).  
References:  https://www.medscape.com/viewarticle/746710  
   
18.    Patient  with  small  cell  cancer  grade  III,  developed  back  pain  in  the  last  24  hours,  something  
like  that.  What  to  do?  
a)          MRI  
b)          steroids  and  MRI  
c)          do  nothing  
d)     radiation  
   
Answer:  B.  
In  SCLC  the  most  common  initial  presentation  is  backpain  and  in  patient  with  stage  III  the  
management  is  palliative  or  by  chemotherapy  and  even  if  it’s  an  indication  for  Mets  and  no  
recommendation  for  Steroids  or  MRI  unless  there’s  focality,  and  radiation  is  not  used  in  SCLC  III.  
   
 
 
 
   
19.    Colonoscopy  shows  ~2  hyperplastic  polyps.  When  to  repeat?  
A.                      1  
B.                      3  
C.                      5  
Answer:  10  years.    
hyperplastic  polyps  are  benign      Ref:  http://www.aafp.org/afp/2015/0115/p93.html  
   
20.    23  year  old  man  is  complaining  of  polydpisa  and  polyurea  and  he  is  worry  and  afraid  about  
getting  dm  what  is  the  best  value  for  diagnosis?  Typical  senario  and  answer  
1-­‐  7.7  
2-­‐8  
3-­‐  9  
4-­‐  12                                                                                                                      
(UpTodate)                                                                                                                                                            
The  diagnosis  of  diabetes  in  an  asymptomatic  individual  can  be  established  with  any  of  the  following  
criteria:  fasting  plasma  glucose  (FPG)  values                                                                                                                                                                                              
   
≥126  mg/dL  (7.0  mmol/L),  two-­‐hour  plasma  glucose  values  of                                        
≥200  mg/dL  (11.1  mmol/L)  during  an  oral  glucose  tolerance  test  (OGTT),  and  A1C  values  ≥6.5  percent  
(48  mmol/mol)          
   
21.  young  female  came  for  checkup  her  labs  show  
Low  Hbg  
High  MCV  
High  AST  
What  is  the  cause?  
A.     folate  deficiency  
B.     vitamin  B12  deficiency  
C.     alcohol  abuse  
Answer:  C  
not  sure  
   
22.  Colon  cancer  screen  recommended  grad  A  which  age  group:  
A-­‐45-­‐65  
B-­‐50-­‐65  
C-­‐50-­‐75  
Answer:  C  
Not  sure!  
   
23.    70  years  old  professor  presented  with  episodes  of  headache,  all  his  labs  is  normal,  BP  >  140/80  
What  is  the  diagnosis:  
A.          Essential  
B.          Secondary  
 Answer:A    
24.    Community  medicine  employee  want  to  implement  a  campaign  for  a  leading  cause  of  death  in  
Saudi  Arabia.  What  is  the  appropriate  answer:  
A.          Breast  cancer  
B.          DM  
C.          Coronary  artery  disease  
Answer:  DM  
Ref:  https://www.cdc.gov/globalhealth/countries/saudi_arabia/default.htm  
   
25.    17  yo  athlete  male  gain  7kg  lately  and  he  has  all  characteristic  of  normal  puberty,  he  also  have  
a  foul  smell  breathing?  
A-­‐  anabolic  steroid  
B-­‐  puberty  
C-­‐..  
 Answer:  most  likely  A    
   
26.    Scenario  of  a  couple  pre-­‐marital  screening  and  they  poor  blood  labs  for  both,  
Male:  
Hb  low  
Mcv  low  
Hb  A2  more  than  3.5  
Similar  lab  results  for  the  female  
What  is  the  risk  their  child  will  develop  thalassemia?  
1-­‐  25%  
2-­‐  50%  
3-­‐  75  
4-­‐  100  
Answer:A  
   
27.    Middle  aged  man  came  to  PCC  for  regular  check  up.  
He  has  been  smoking  1  pack  a  day  for  40  years.  He  also  consumes  chewed  tobacco.  
Vital  signs:  
BP  =  120/80  
Labs:  
Fasting  blood  glucose  normal  
Lipid  profile  normal  
Renal  function  normal  
What  should  you  screen  for  next  year?  
A-­‐  lung  cancer  
B-­‐Hypertension  (correct)  
C-­‐DM  
D-­‐  dyslipidemia  
Answer:  A  
Ref:  https://www.cdc.gov/cancer/lung/basic_info/screening.htm  
   
28.    Best  way  for  pain  measurement  in  pt  with  different  language:  
Answer:  Faces  (Drawings)  
   
29.    which  of  the  following  supplements  can  decrease  risk  of  some  cancer?  
A.                      -­‐fiber  
B.                      -­‐  vitamin  D  
C.                      ***folic  acid  
 Answer:  
   
30.    The  maximum  accepted  level  of  LDL  in  mmol/l??  
8.8  
   
http://www.cholesterolmenu.com/cholesterol-­‐levels-­‐chart/  
   
31.    Pre-­‐diabetic  what  will  be  the  next  visit?  
A.     3  months  
B.     6  months  
C.     12  months  
Answer:  C  
Ref:  http://www.dartmouth-­‐hitchcock.org/endo/pre-­‐diabetes.html  
   
32.    Patient  came  to  the  clinic  and  everything  was  normal  except  HBA1C  which  was  5.9.  When  are  
you  going  to  repeat  and  see  her  again?  
1.          3  months  
2.          6  months  
3.          12  months  
4.          36  months  
   
Answer:  6  months  
Ref:  https://www.diabetes.org.uk/professionals/position-­‐statements-­‐reports/diagnosis-­‐ongoing-­‐
management-­‐monitoring/new_diagnostic_criteria_for_diabetes  
   
33.    Patient  with  high  BP,  otherwise  normal,  when  to  check  BP  again?  
A.          6  month,  (3  MONTHS  if  has  a  disease  with  it,  like  HF)  
B.          1  year  
C.          3  years  
 Answer:B  
   
34.    Osteoporosis  most  commonly  due  to?  
Answer:  Aging-­‐  menopause  
 Ref:  https://www.emedicinehealth.com/osteoporosis/page2_em.htm  
   
35.    patient  with  hypertrigelicridemia,  came  with  abdominal  and  back  pain  ,his  symptomes  due  to  
?  
Answer:  Hypertrigylesride-­‐  pancreatitis  
   
36.    Pt  on  statin  for  high  cholesterol  ,now  hight  trigylesride  ,what  to  add  ?  
Answer:  Fibrate  
   
37.    patient  diagnosed  with  scoliosis  according  to  the  cobb  angle,  when  the  orthopedic  refer  should  
be  done?  
A.  5  
B.  10  
C.  15  
D.  20  
 Answer:D  
   
38.    -­‐Man  smoker  ,  55  y  ,  no  family  history  of  DM  ,  his  hemoglobin  A1c  and  blood  sugar  in  the  
border  line  of  being  diabetics  according  to  lap  reading  ,  when  to  follow  up  again  ?  
3  m  
6  m  
12  m  ✅  borderline  
24  m  
36  m  
Answer:  every  12m    
Ref:  http://www.dartmouth-­‐hitchcock.org/endo/pre-­‐diabetes.html  
   
40.    Exercise  to  prevent  osteoporosis  
A.     Weight  bearing  exercises  
B.          Low  resistance  high  repetitive  
Answer:      
   
41.    The  daughter  of  an  old  age  patient  complains  of  progressively  decreasing  in  memory  and  
change  in  personality  of  her  father,    ttt:  
Answer:  Refer  to  geriatric  clinic  
   
42.    -­‐Treatment  of  refractory  hiccup?  
A.     Gabapentin  
B.     Chlorapromazine  
Answer:  B  
https://emedicine.medscape.com/article/775746-­‐medication  
   
43.    -­‐When  to  screen  for  congenital  thyroid  disease  ?  
A.     Umbalical  blood  sample  
B.     After  3  month  
C.     After  year  
 Answer:  A  
   
44.    Want  to  stop  smoking,  but  now  he  wants  to  stop,  which  phase  he  is  in  ?  
A.                      Precontplation  
B.                      Contplation  
C.                      action  
D.                     Preparation  
Answer:  D  
1.  Precontemplation—not  yet  acknowledging  that  there  is  a  problem  
2.  Contemplation—acknowledging  that  there  is  a  problem,  but  not  yet  ready  or  willing  to  make  a  
change  
3.  Preparation/determination—getting  ready  to  change  behaviors  
4.  Action/willpower—changing  behaviors  
5.  Maintenance—maintaining  the  behavior  changes  
6.  Relapse—returning  to  old  behaviors  and  abandoning  new  changes  
   
45.    Patient  can't  sustain  erection,  he  had  similar  problem  2  years  ago  (or  2-­‐3  months  ago).  he  has  
daily  morning  erection.  
To  whom  you  should  refer  him?  
A.  Urology  
B.  Psychiatry  
C.  Endocrinology  
D.  Neurology  
Answer:  B  
   
46.    Which  one  of  the  following  is  the  best  for  screening  male  above  59  years  old?  
1.          Highly  sensitive  occult  blood  test  annualy  
2.          Sigmoidoscopy  every  3  years  with  Highly  sensitive  occult  blood  
3.          Sigmoidoscopy  every  5  years  
4.          Colonscopy  every  5  years  
Answer:  
   
47.    Saudi  man  came  for  routine  checkup  what  you  will  screen  him  
A.          Hypertension,obesity,DM  
B.          Hyperlipidemia,hypertension,obesity  
C.          Hypertension,DM,hyperlipidemia  
Answer:  D  ?  
   
48.    adolescent  female,  found  to  have  BMI  greate  than  95th  percentile  on  her  routince  visit.  What  
is  the  most  appropriate  action?  
A.          avoid  discussion  about  weight.  
B.          decrease  calories  intake  
C.          Interdisciplinary  plan  something  
D.        Tell  her  she  is  larger  than  most  people  her  age.  
 Answer:C  
   
49.    If  you  treat  diabetic  with  Vit  D.  What  type  of  prevention  is  this?  
A.     Primary  
B.     Secondary  
C.     Tertiary  
Answer:  B  
   
Q.  Newly  diagnosed  osteoprosis,  you  gave  vitamin  D,  what  is  the  type  of  prevention  ?  
 
50.    Baby  with  yellow  teeth  and  dental  carries  in  both  side  
A.          nursing  bottle  caries  
B.          other  options  
 Answer:    A  
   
51.    Chronic  pain  syndrome  management  ?  
NSAID  
   
52.    Term  used  to  call  ppl  who  eat  ice  
-­‐  amylophagia  (starch)  
-­‐  Geophagia(dirt/soil)  
-­‐  Pagophagia(ice)  
Answer:  C  
   
53.    What’s  minimum  volume  of  blood  to  do  a  culture:  
10_20  ml  
   
54.    Patients  have  D.M.  On  metformin  1  g  and  another  anti  dm  present  with  increasing  
blood  glucose  at  morning,  what  you  will  give  :  
A.  NPH  
B.  Lispro  
C.  Regular  
Answer:  A  
 It  depends,    before  breakfast  it  means  fasting,  so  the  answer  would  be  long  acting  insulin,  most  
likely  NPH  or  LANTUS  
   
55.    Female  with  CAD  ,  prescribe  for  her  drug  lower  lipid  .  Present  with  facial  flush  ,  what  is  the  
drug  ?  
-­‐  statin  
-­‐  Nicain  my  answer  
-­‐  Fibrate  
Answer:  statin  
   
56.    what  is  the  MOA  of  glipizide  ?  
A.  Increase  insulin  secretions  from  pancreas  
Answer:  A  
   
57.    Degree  for  screening  of  abdominal  aortic  aneurysm  for  pt  age  70  y/o  who  never  smoked  ?  
   
   
58.    A  16  YO  boy  with  unilateral  gynecomastia,  what  to  advice  him?  
A.  reassure  that  most  males  have  this  problem  and  will  disappear  in  a  few  years.  
B.  Compress  with  bandage  overnight.  
 Answer:  A  
Ref:  https://emedicine.medscape.com/article/120858-­‐treatment  
   
59.    A  Pt  with  CHF  and  HTN  on  meds,  he  developed  dizziness  every  morning  due  to  a  certain  
medication  and  we  stopped  it,  what  Type  of  prevention  is  this:  
A.  primary  
B.  secondary  
C.  tertiary  
D.  quaternary  
My  answer:  B  not  sure  
   
60.    Pt  with  untreated  lower  UTI  for  14  day,  what  is  the  percentage  that  this  infection  will  ascend  
to  the  kidney?  
A.     .05  
B.     .5  
C.     5  
D.   50  
 Answer:    
   
61.    Elderly  patient  with  dm,  HTN  ,  dyslipdemia  ,  what  is  the  target  LDL  level  for  him?  
-­‐  2.3-­‐4.6  
‫ ﺍاﺭرﻗﺎﻡم ﻏﺮﻳﯾﺒﻪﮫ ﺑﺎﻟﻔﻮﺍاﺻﻞ ﺟﺖ‬ -­‐  .  
   
A  target  LDL  cholesterol  level  below  70  to  80  mg/dL  (1.81  to  2.07  mmol/L)  is  recommended  for  
people  who  have  CVD  and  have  multiple  major  risk  factors  (eg,  people  with  diabetes  or  who  smoke).  
A  target  LDL  cholesterol  level  less  than  100  mg/dL  (2.59  mmol/L)  is  recommended  for  people  who  
have  CVD  but  do  not  have  many  additional  risk  factors.  Lifestyle  changes  as  well  as  nonstatin  
medications  may  be  recommended  when  LDL  cholesterol  levels  are  higher  than  100  mg/dL  (2.59  
mmol/L).  
refL  https://www.uptodate.com/contents/high-­‐cholesterol-­‐treatment-­‐options-­‐beyond-­‐the-­‐basics  
   
62.    -­‐old  patient  known  to  have  acquired  immunodeficiency  presented  with  cough  and  night  sweat  
,  he  did  mentoux  test  which  was  negative  but  culture  was  positive  for  tuberculosis  what  is  most  
likely:  
A.          culture  is  false  positive  
B.          mentoux  is  false  negative  
C.          mentoux  is  not  a  screening  test  for  TB  
D.        the  patient  should  be  screened  with  heaf  test.  
Answer:  B  
https://en.m.wikipedia.org/wiki/Mantoux_test  
 
 
Ethics  and  community  Medicine  
   
1.          Patient  trying  to  quit  smoking,  but  he  couldn’t,  he  came  to  you  for  help:  what  behavior  change  
at  this  stage?  
A-­‐precontemplation  
b-­‐contemplation  
C-­‐Action  
D-­‐preparation  
   
Answer:  Preparation.  
         

         
     

     

   
   
   
   
   
   
   
   
   
   
   
   
   
   
   
 
2.          A  doctor  is  giving  a  lecture  about  hypertension  and  its  medication,  A  drug  company  
representative  came  to  you:  what  will  you  do?  
   
ANSWER:  Physicians  ought  to  refuse  to  visit  with  representatives  as  a  matter  of  both  professional  
integrity  and  sensible  time  management.  
REFERENCES:  https://www.ncbi.nlm.nih.gov/pmc/articles/PMC1466797/  
   
3.          What’s  minimum  volume  of  blood  to  do  a  culture:  
   

 
REFERENCES:  
https://www.amc.edu/pathology_labservices/specimen_collection/specimen_col
lection_docs/Blood_Cultures022713.pdf  
   
4.          Female  150k.g.  Weight  and  height  160  according  to  BMI?  
A-­‐I  Obesity  
B-­‐II  obesity  
C-­‐III  obesity  
   
ANSWER:  Class  III.    Formula  :  kg/  height  squared  meters.  [    150/  [  1.6*1.6]  =  58]  
   
 

 
 
 
   
 
5.          Example  of  opening  question?  
A-­‐Tell  me  about  the  pain  
   
6.          A  type  2  DM  pt  has  an  appointment  in  the  clinic  at  11  o'clock,  the  doctor  came  at  12:30  
because  there  was  a  serious  case  in  the  hospital,  the  pt  was  angry,  what  will  you  do  as  a  doctor  ?  
A-­‐Apologize  and  expect  that.  
   
ANSWER:    the  answer  depends  on  the  options  provided  in  the  Q  but  the  following  points  should  be  
considered;    Do’s  [  keep  a  safe  distance,  stay  calm  and  cool,    acknowledge  the  emotional  status  “  
angry,anxious  etc”,  acknowledge  legitimacy  “  without  blaming  anyone”,  listen  actively  with  eye-­‐to-­‐
eye  contact,  ask  open  Q  “  what  makes  you  feel  so”,  explore  the  reasons  and  possible  ways  of  
treatments  when  suitable,  apologies    when  appropriate]  Don’t  [  interrupt,  rude  language,  deny  
reality,  get  angry,  challenge  the  pt.,  defensive  responses  “We  have  been  doing  our  best,  don't  you  
realize  how  hard  we  have  been  trying?.]  
REFERENCES:  http://careers.bmj.com/careers/advice/view-­‐article.html?id=1854  
   
7.          best  one  considered  as  open-­‐ended  question  in  pt  with  chest  pain?  
A-­‐Tell  me  about  the  pain  
B-­‐when  did  the  pain  start  
C-­‐where  is  the  pain.  
   
ANSWER:  Tell  me  about  your  pain.  
   
   
   
   
   
   
   
   
8.          definition  of  epidemiology?  
A-­‐Distribution  and  determinant  of  the  disease.  
   
ANSWER:    Epidemiology  is  the  study  of  the  distribution  and  determinants  of  health-­‐related  states  or  
events  (including  disease),  and  the  application  of  this  study  to  the  control  of  diseases  and  other  
health  problems.  Various  methods  can  be  used  to  carry  out  epidemiological  investigations:  
surveillance  and  descriptive  studies  can  be  used  to  study  distribution;  analytical  studies  are  used  to  
study  determinants.  
REFERENCES:    http://www.who.int/topics/epidemiology/en/  
   
   
9.          You  got  invited  in  diabetes  campaign,  and  they  offered  you  to  advertise  about  a  drug,  what  to  
do:  
A-­‐Avoid  any  inform  about  drug...  
ANSWER:  Physicians  ought  to  refuse  to  visit  with  representatives  as  a  matter  of  both  professional  
integrity  and  sensible  time  management.  
   
   
10.    Type  of  hepatitis  B  vaccine?  
   
ANSWER:    two  types  exist;  plasma  derived  [purified  HBsAg  obtained  from  the  plasma  of  persons  
with  chronic  HBV  infection.]  ,  and  recombinant  vaccines  [use  HBsAg  synthesized  in  yeast  or  
mammalian  cells  into  which  the  HBsAg  gene  (or  HBsAg/pre-­‐HBsAg  genes)  has  been  inserted  by  
plasmids].  
REFERENCES:  http://www.who.int/immunization/topics/WHO_position_paper_HepB.pdf  
   
11.    Which  of  the  following  reduce  cancers?  
A-­‐Vit  D  
B-­‐Fibers              
C-­‐Salt  
ANSWER:  Vitamin  D.  
·∙            Molecular,  genetic  and  clinical  data  in  humans  are  scarce  but  they  suggest  that  vitamin  D  is  
protective  against  colon  cancer.  
·∙            Intake  of  dietary  fiber  is  inversely  associated  with  colorectal  cancer  risk.    
REFERENCES:  
·∙                https://www.ncbi.nlm.nih.gov/pubmed/22383428  
·∙              https://academic.oup.com/jnci/article/102/9/614/893779/Dietary-­‐Fiber-­‐and-­‐Colorectal-­‐Cancer-­‐Risk-­‐A-­‐Nested.  
   
12.    If  patient  that  you  treated  gave  an  expensive  watch  as  a  gift,  what  will  you  do?  
A-­‐Accept  it  and  tell  to  not  do  it  again  
B-­‐Refuse  it  
C-­‐Accept  it  
ANSWER:  Refuse  it.      It’s  quite  controversial  and  the  actions  depends  on  the  timing  of  the  gift,  
relationship  with  the  patient,  type  of  gifts  but  usually  for  expensive  gifts  its  best  to  refuse.    
REFERENCES:    https://www.ncbi.nlm.nih.gov/pmc/articles/PMC3860914/  
   
13.    Most  common  infection  in  saudi  arabia  and  disable  patient  from  work:  
   
ANSWER:    Hepatitis  B  infection  was  the  most  common  cause  (57.5%),  followed  by  noncommunicable  
diseases  (21.2%)  and  hepatitis  C  infection  (17.4%)  
REFERENCES:  https://www.ncbi.nlm.nih.gov/pubmed/24975313  
   
14.    Treat  water  from  entameba:  
   
ANSWER:  Amebiasis  can  be  prevented  by  increased  sanitation  and  effective  and  safe  disposal  of  
human  excreta.  Travelers  should  avoid  unpeeled  fresh  vegetables  and  fruits  and  drink  only  boiled  or  
bottled  water.  Avoiding  sexual  practices  that  involve  fecal-­‐oral  contact  can  reduce  infection  in  
homosexuals.  In  mental  institutions  recurrent  outbreaks  of  amebiasis  can  be  prevented  by  routine  
screening  of  stool  and  treating  infected  patients.  
REFERENCES:  http://www.antimicrobe.org/new/b137.asp  
   
15.    Researcher  want  to  measure  obesity  in  children  he  took  BMI  what  else  he  should  take?  
1-­‐girth  measurement    
2-­‐HDL/LDL  level  
3-­‐dietary  habits  
4-­‐forgot  
ANSWER:    girth/  waist  measuruments.    
References:    https://www.hsph.harvard.edu/obesity-­‐prevention-­‐source/obesity-­‐definition/how-­‐to-­‐
measure-­‐body-­‐fatness/  
   
   
16.    How  to  prevent  plague  
A-­‐Killed  rodent  
ANSWER:  kill  rodents  and  their  habitnant,  wear  gloves  while  touching  infected  animals,    DEET  
repellent  sprays,  flea  control  products  on  animals.  
REFERENCES:  https://www.cdc.gov/plague/prevention/index.html  
   
17.    Ministry  of  health,  prevent  some  group  of  ppl  to  do  Hajj  &  Umrah  in  2015,  due  to  MERSA-­‐  Co;  
whose  those  population?  
A-­‐  Lactating  women  
B-­‐  Young    kids    >  12  years  
C-­‐  elderly  with  DM  
D-­‐  HBV  
   
ANSWER:  Elderly  +  DM.  Using  a  case–control  design,  we  assessed  differences  in  underlying  medical  
conditions  and  environmental  exposures  among  primary  case-­‐patients  and  2–4  controls  matched  by  
age,  sex,  and  neighborhood.  Using  multivariable  analysis,  we  found  that  direct  exposure  to  
dromedary  camels  during  the  2  weeks  before  illness  onset,  as  well  as  diabetes  mellitus,  heart  
disease,  and  smoking,  were  each  independently  associated  with  MERS-­‐CoV  illness.    
REFERENCES:  https://wwwnc.cdc.gov/eid/article/22/1/15-­‐1340_article  
   
18.    A  doctor  is  going  to  present  in  hypertension  conference.  Company  that  produced  new  drug  
suggested  to  sponsor  his  trip.  What  he  is  supposed  to  do:  
A-­‐Disclose  that  he’s  sponsored  
B-­‐Avoid  mentioning  the  new  drug  at  all  
C-­‐Reject  the  offer.  
   
ANSWER:  Reject  the  offer.  
REFERENCES:  
 
19.    Inactive  person,  overweight,  smoker,  44  years  old  man  with  heart  problems,  What  is  the  most  
common  associated  with  heart  disease:  
A-­‐Smoking  
B-­‐Obesity  
C-­‐Inactivity  
ANSWER:  Smoking.  Smoking  is  a  major  cause  of  heart  disease.  It  is  estimated  that  smoking  increases  
the  risk  of  stroke,  coronary  heart  disease  and  impotence  by  100%.  Smoking  increases  the  risk  of  
death  from  undiagnosed  coronary  heart  disease  by  300%.  
REFERENCES:  https://www.world-­‐heart-­‐federation.org/resources/risk-­‐factors/  
   
20.    how  to  prevent  MERSA  ?  
A-­‐by  hand  washing  
B-­‐vaccine.  
ANSWER:  hand  washing.  As  a  general  precaution,  anyone  visiting  farms,  markets,  barns,  or  other  places  
where  dromedary  camels  and  other  animals  are  present  should  practice  general  hygiene  measures,  including  
regular  hand  washing  before  and  after  touching  animals,  and  should  avoid  contact  with  sick  animals  
REFERENCES:  http://www.who.int/mediacentre/factsheets/mers-­‐cov/en/  
   
21.    The  daughter  of  an  old  age  patient  complains  of  progressively  decreasing  in  memory  and  
change  in  personality  of  her  father,  ttt:  
A-­‐Refer  to  geriatric  clinic.  
ANSWER:  refer  to  geriatric.  
References:    previous  MCQ’s  
   
22.    Important  in  Quit  Smoking?  
A-­‐Patient  desiree.  
   
23.    what  is  more  risk  for  CAD  
A-­‐55    male  Dm  
B-­‐50  male  hyperlipidemia  
C-­‐HTN+  obese  
AMSWER:    C.  
 REFERENCES:  Toronto  Notes  2016.  
 

 
24.    What  is  have  more  risk  for  CAD  
A-­‐LDL.  
B-­‐HDL  
C-­‐Triglycerides  
D-­‐Total  cholesterol  
 ANSWER:  LDL.  
 
25.    Calculate  disabilities  days  of  URTI  ?  
A-­‐292.5  (  correct  ).  
   
ANSWER:    usually  the  number  of  disability  days  is  calculated  by  subtracting  the  number  of  absent  
days  from  the  total  number  of  days  per  year.  The  reported  Duration  of  an  URI  episode  is  7.4  days  
with  25%  of  cases  lasting  up  to  two  weeks.    
   
26.    mammogram  can  detect  breast  cancer  before  clinical  examination  by  how  many  years  ?  
A-­‐1  
B-­‐2  
C-­‐3  
D-­‐4  
ANSWER:    2  years.  
   
28.    Which  of  one  of  the  following  is  considered  as  a    secondary  prevention  method?  
A-­‐personal  education  
B-­‐screening  
ANSWER:  screening.  
   
 
 
   
   
29.    commonest  virus  which  make  rejection  of  the  expatriates  or  labors  from  working  in  Saudi?(  
community  )  
a)  hbv  
b)  hcv  
c)  hiv  
Answer:  A  
same  Q  from  smle  13  
References:    http://applications.emro.who.int/emhj/v19/07/EMHJ_2013_19_7_664_670.pdf?ua=1  
   
30.    In  a  village  where  the  incidence  of  cretinism  and  iodine  was  less  than  1  microgram  the  health  
promoters  want  to  issue  a  director  for  the  for  the  management  of  those  with  cretinism..  what  is  
the  best  initial  management  
a)     TSH  and  t4  measurements  
b)     Start  thyroxine  medication.  
c)     Iodine  supplementation.  
   
ANSWER:  B.  
   
   
   
31.    varicella  vaccine:  
A-­‐2  doses  6  weeks  apart      
B-­‐2  doses  4  weeks  apart  
   
ANSWER:      2  doses  4  weeks  apart.  For  children  aged  7  through  12  years,  the  recommended  minimum  
interval  between  doses  is  3  months  (if  the  second  dose  was  administered  at  least  4  weeks  after  the  first  
dose,  it  can  be  accepted  as  valid);  for  persons  aged  13  years  and  older,  the  minimum  interval  between  
doses  is  4  weeks.  
REFRENCES:  https://www.cdc.gov/vaccines/schedules/hcp/imz/catchup-­‐shell.html  
   
32.    What’s  the  most  effective  way  to  disseminate  health  education?  
A-­‐mass  media  
   
ANSWER:    Mass  media.  
   
33.    Best  diagram  used  for  screening  test  ?  
Nomogram,  and  other  s  ??  
   
ANSWER:  I  couldn’t  find  a  reference  of  any  information  relevant  to  this.  
   
   
34.    Want  to  stop  smoking,  but  now  he  wants  to  stop,  which  phase  he  is  in?  
A-­‐  Precontemplation  
B-­‐  Contemplation  
C-­‐action  
D-­‐Preparation  
   
1.  Precontemplation—not  yet  acknowledging  that  there  is  a  problem  
2.  Contemplation—acknowledging  that  there  is  a  problem,  but  not  yet  ready  or  willing  to  make  a  
change  
3.  Preparation/determination—getting  ready  to  change  behaviors  
4.  Action/willpower—changing  behaviors  
5.  Maintenance—maintaining  the  behavior  changes  
6.  Relapse—returning  to  old  behaviors  and  abandoning  new  changes  
   
35.    best  parameter  for  screening  ?  
A-­‐specify,  
B-­‐sensitivity  
   
ANSWER:    Sensitivity.  
REFERENCES:  http://sphweb.bumc.bu.edu/otlt/mph-­‐
modules/bs/bs704_probability/bs704_probability4.html  
   
36.    Community  medicine  employee  want  to  implement  a  campaign  for  a  leading  cause  of  death  in  
Saudi  Arabia.  What  is  the  appropriate  answer:  
A-­‐Breast  cancer  
B-­‐DM  
C-­‐Coronary  artery  disease  
ANSWER:    DM.  top  ten  leading  causes  of  death  in  KSA  in  order  [  RTA  >  DM  >  back/neck  injuries  >  IHD  

>  depressive  disorders>congenital  anomalies  of  the  heart  >    Drug  use  >  Skin  disorders  >  CKD  >  

cancer.  

REFERENCES:  https://www.cdc.gov/globalhealth/countries/saudi_arabia/default.htm  

37.    Want  to  stop  smoking,  went  to  hospital  and  asked  for  treatment:  
ANSWER:  action  
   
38.    Colon  cancer  screen  recommended  grad  A  which  age  group:  
A-­‐45-­‐65.  
B-­‐50-­‐65.  
C-­‐50-­‐75.  
ANSWER:  C  
REFERENCES:  https://www.cancer.org/cancer/colon-­‐rectal-­‐cancer/detection-­‐diagnosis-­‐staging/acs-­‐
recommendations.html  
   
39.    A  man  newly  diagnosed  with  type  2  diabetes  2  weeks  ago,  he  came  for  follow  up  with  you  at  
the  clinic.  You  were  busy  in  the  ward.  His  appointment  is  supposed  to  be  at  10:00  and  now  it  is  
11:15,  the  patient  is  very  angry.  What  should  you  do?  
A)  explore  the  reason  of  his  anger  
B)  explain  why  you  are  late  
C)  show  empathy  to  his  newly  diagnosed  condition.  
ANSWER:  A  
   
40.    Smoking  withdrawal  peak:  
ANSWER:  3-­‐5  days.  
REFERENCES:  https://vapingdaily.com/quitting-­‐effects/nicotine-­‐withdrawal-­‐timeline/  
   
41.    Breast  self  exam:  
A.     monthly  
ANSWER:  Breast  exams,  either  from  a  medical  provider  or  self-­‐exams,  are  no  longer  recommended.  

REFERENCES:  https://www.cancer.org/latest-­‐news/american-­‐cancer-­‐society-­‐releases-­‐new-­‐breast-­‐
cancer-­‐guidelines.html  
   
42.    Blood  preserved  at  22  C  degree?  
ANSWER:    PRBC;s  is  stored  between  1-­‐6  degrees,  FFP  stored  -­‐18  to  -­‐30,    platelet  stored  at  22  
degrees.  
REFERENCES:  https://library.med.utah.edu/WebPath/EXAM/LabMedCurric/LabMed06_02.html  
   
43.    Newly  diagnosed  osteo,  you  gave  vitamin  D,  what  is  the  type  of  prevention?  
ANSWER:  tertiary  prevention.  
   
44.    prevention  of  brucellosis?  
A-­‐Pasteurization  of  milk.  
ANSWER:  The  most  rational  approach  for  preventing  human  brucellosis  is  the  control  and  elimination  of  the  
infection  in  animals.  Pasteurization  of  milk  is  another  protective  mechanism.  Vaccination  of  cattle  is  
recommended  for  control  of  bovine  brucellosis  in  enzootic  areas  with  high  prevalence  rates.  
REFERENCES:  http://www.who.int/zoonoses/diseases/brucellosis/en/  
 

Research    
   
   
1.     What  is  the  type  of  bias  in  meta-­‐analysis?  
   
A.     Tendancy  to  answer  questions  untruthfully  or  misleadingly  on  a  survey  
B.   Recall  retrospective  studies.  
C.      Loss  of  follow  up  
D.        Publication      
   
Answer:  D  
Reference:  https://www.ncbi.nlm.nih.gov/pmc/articles/PMC3868184/  
   
2.     What  is  the  most  common  type  of  study  that  risks  a  recall  bias?  
   
Answer:  Case  control  
Reference:  http://jech.bmj.com/content/58/8/635  
   
3.  What  is  the  most  common  type  of  study  that  risks  non-­‐
response  bias?  
   
Answer:  Cross  sectional  
Reference:  http://www.bmj.com/content/348/bmj.g2573.full  
 
4.  What  does  a  confidence  interval  of  95%  mean?  
 
Answer:  A  95%  confidence  interval  reflects  a  significance  level  of  
0.05.  If  it  is  hypothesized  that  a  true  parameter  value  is  0  but  the  
95%  confidence  interval  does  not  contain  0,  then  the  estimate  is  
significantly  different  from  zero  at  the  5%  significance  level.  
Reference:  https://en.wikipedia.org/wiki/Confidence_interval  
   
 
5.  If  a  raesearcher  wants  to  start  a  study  and  wants  only  the  subjects  that  do  not  have  diabetes  to  
participate.  What  will  be  high  in  the  test  ?  
Answer:  Specifity  
Reference:  https://www.ncbi.nlm.nih.gov/pmc/articles/PMC2636062/  
   
6.  What  is  the  definition  of  epidemiology?  
 Answer:  Epidemiology  is  the  study  of  the  distribution  and  determinants  of  health-­‐related  states  or  
events  in  specified  populations,  and  the  application  of  this  study  to  the  control  of  health  problems  
Reference:  https://www.cdc.gov/ophss/csels/dsepd/ss1978/lesson1/section1.html    
7.                       What  does  “p-­‐value”  mean?  
 Answer:  P-­‐value  or  probability  value  is  the  probability  for  a  given  statistical  model  that,  when  the  
null  hypothesis  is  true,  the  statistical  summary  would  be  the  same  as  or  of  greater  magnitude  than  
the  actual  observed  results.  
   
Reference:  https://en.wikipedia.org/wiki/P-­‐value  
   
8.                       What  is  a  Standard  deviation?  
 Answer:  The  standard  deviation  is  a  measure  that  is  used  to  quantify  the  amount  of  variation  or  
dispersion  of  a  set  of  data  values.  A  low  standard  deviation  indicates  that  the  data  points  tend  to  be  
close  to  the  mean  (also  called  the  expected  value)  of  the  set,  while  a  high  standard  deviation  
indicates  that  the  data  points  are  spread  out  over  a  wider  range  of  values.  
Reference:  https://en.wikipedia.org/wiki/Standard_deviation  
   
   
9.  Best  test  for  screening?  
 Answer:  sensitivity.  
   
10.    A  study  aims  at  exploring  the  association  of  cigarette  smoking  and  the  risk  of  IHD.  Results  were  
as  follows  (Numbers  are  not  exact,  but  are  used  for  clarification  purposes):  
-­‐  Non-­‐smokers  OR:  0.1.                                                                                
-­‐  Mild  smokers    OR:  1.  
-­‐  Heavy  smokers  OR:  2.  
-­‐  Extensive  heavy  chain  smokers  OR:  5.  
Which  of  these  is  true  about  this  study  findings?  
A)  Risk-­‐Association  relationship.  
B)  Dose-­‐dependant  relationship.  
Answer:  B  
Explanation:  The  odd  ratio  is  increasing  as  the  patient  is  smoking  heavier.  
                                     
11.    Post  test  probability  of  a  diagnostic  test:  
A.  likelihood  ratio  
B.  predictive  value  
Answer:  A  
Reference:  https://www.ncbi.nlm.nih.gov/pmc/articles/PMC4025141/  
Explanation:  The  liklihood  ratio  is  a  probability  of  an  individual  without  the  condition  having  the  test  
result.  
   
   
   
   
12.    At  daycare  center  10  out  of  50  had  red  eye  in  the  1st  week,  another  30  developed  same  
condition  in  the  next  week  .What  is  the  attack  rate?  
20%  
40%  
60%  
80%  
Answer:  80%  
Referance:  https://www.cdc.gov/ophss/csels/dsepd/ss1978/lesson3/section2.html  
Explanation:  Attack  rate  =  Number  of  new  cases  in  the  population  at  risk  /  Number  of  persons  at  risk  

in  the  population  

13.    long  scenario,  he  wants  the  equation  of  relative  risk:  
Answer:  RR  =  
Reference:  https://en.wikipedia.org/wiki/Relative_risk  
Explanation:  
▪        A  =  The  number  of  people  who  both  had  the  exposure  and  developed  the  disease  
▪        B  =  The  number  of  people  who  had  the  exposure  but  did  not  develop  the  disease  
▪        C  =  The  number  of  people  who  did  not  have  the  exposure  but  did  develop  the  disease  
▪        D  =  The  number  of  people  who  neither  had  the  exposure  nor  developed  the  disease  
   
14.    A  study  aims  at  exploring  the  association  of  high  fat  intake  and  prostatic  cancer.,  Group  1  has  
prostatic  cancer  (1000)  patients  with  50  high  fat  intake,  group  2  doesn't  have  cancer  (1000)  pt  with  
10  high  fat  intake  
a)OR  0.52  
b)OR  5.2  
c)RR  0.52  
d)RR  5.2  
Answer:  b  
Reference:  First  aid  
Explanation:  OR  is  typically  used  in  case  control  studies  

 
   
15.    In  a  study  they  are  selecting  every  10th  family  in  the  city,  what  is  the  type  of  study?  
A.     Systematic  random  study  
B.     Stratified  random  study  
C.     Non  randomized  study  
Answer:  A  
Reference:  https://en.wikipedia.org/wiki/Systematic_sampling  
   
   
16.    In  a  cohort  study  on  lubricant  oil  use  and  urinary  bladder  CA  done  over  20  years  10,000  
exposed  10,000  non  exposed  750  exposed  got  CA  150  non  exposed  got  CA  Then  they  asked  about  
the  incidence  in  1000  in  one  year?  
A.     2.25  
B.     45  
C.     .45  
D.   .225  
Answer:  A  
Explanation:  In  20  years:  20,000  patients  were  studied  ,  900  cases  of  CA  in  total.  

So  in  one  year  900/20=45  case  for  20,000  patients  

For  1000:  45  x1000/20,000=2.25  


17.    Lung  cancer  affected  80  of  100  smokers,6  of  500  nonsmokers.  What  is  Relative  risk  reduction?  
A.     33:1  
B.     55:1  
C.     66:1  
Answer:  C  
Reference:  :  https://en.wikipedia.org/wiki/Relative_risk  
   

Explanation:    
   
18.  Repeated  question  (Question  5)  
   
19.    In  a  systematic  review,  which  one  of  the  following  can  be  done  to  decrease  selection  bias?  
A.     English  literature  
B.     Setting  inclusion/exclusion  criteria  
C.     Including  papers  with  positive  outcomes  only  
Answer:  B  
Reference:  https://www.ncbi.nlm.nih.gov/books/NBK126701/  
   
20.    Research  question  about  cumulative  incidence:  
Answer:  Number  of  new  cases  of  disease  or  injury  during  a  specified  period  divided  by  Size  of  
population  at  start  of  period  
Reference:  https://www.cdc.gov/ophss/csels/dsepd/ss1978/lesson3/section2.html  
Explanation:  Cumulative  incidence  is  defined  as  the  probability  that  a  particular  event,  such  as  
occurrence  of  a  particular  disease,  has  occurred  before  a  given  time.  It  is  equivalent  to  the  incidence,  
calculated  using  a  period  of  time  during  which  all  of  the  individuals  in  the  population  are  considered  
to  be  at  risk  for  the  outcome.  It  is  sometimes  also  referred  to  as  the  incidence  proportion.  
   
   
   
21.    Wha  is  the  definition  of  a  null  hypothesis?  
Answer:  A  null  hypothesis"  is  a  general  statement  or  default  position  that  there  is  no  relationship  
between  two  measured  phenomena,  or  no  association  among  groups.  
Reference:  https://en.wikipedia.org/wiki/Null_hypothesis  
   
   
22.    Definition  of  case  control  
Answer:  A  case-­‐control  study  is  a  type  of  observational  study  in  which  two  existing  groups  differing  
in  outcome  are  identified  and  compared  on  the  basis  of  some  supposed  causal  attribute.  
Reference:  https://en.wikipedia.org/wiki/Case-­‐control_study  
   
23.    What  determines  how  precise  the  study  is?  
A.                     P  value  
B.                     Confidence  interval  
C.                     Relative  risk  
D.                     Odds  ratio  
Answer:  B  
Reference:  https://www.ncbi.nlm.nih.gov/pmc/articles/PMC2920077/  
   
   
   
24.    A  study  of  something  shows  reduced  risk  of  dm  from  10%  to  5%  what  is  called?  
A.     Efficacy  
B.     Effectiveness  
 Answer:  B  
Reference:  https://www.ncbi.nlm.nih.gov/books/NBK44024/  
   
25.    A  scenario  and  asked  about  sensitivity  &  specificity  (in  statistics)  you  should  understand  the  
definition  to  answer  according  to  scenario  
Reference:  First  Aid  
   
 

 
   
   
26.    Exposed  group  3  non  exposed  group  2  which  is  correct:  that  the  question  
 
27.    Exposed  group  0.02  non  exposed  group  0.09  
 

 
 
 Reference:  First  Aid  
   
28.    Most  normograph  used  for  positive  predictive  value:  
Answer:  Roc  curve  
Reference:  https://www.omicsonline.org/evaluating-­‐measures-­‐of-­‐
indicators-­‐of-­‐diagnostic-­‐test-­‐performance-­‐fundamental-­‐meanings-­‐
and-­‐formulars-­‐2155-­‐6180.1000132.php?aid=4054  
Explanation:  In  a  ROC  curve  the  true  positive  rate  (Sensitivity)  is  
plotted  in  function  of  the  false  positive  rate  (100-­‐Specificity)  for  
different  cut-­‐off  points  of  a  parameter  
   
29.    What  is  a  funnel  plot?  
Answer:  A  funnel  plot  is  a  scatter  plot  of  the  effect  estimates  from  
individual  studies  against  some  measure  of  each  study’s  size  or  
precision.  
Reference:  http://www.bmj.com/content/343/bmj.d4002  
Explanation:  The  standard  error  of  the  effect  estimate  is  often  chosen  as  the  measure  of  study  size  
and  plotted  on  the  vertical  axis8  with  a  reversed  scale  that  places  the  larger,  most  powerful  studies  
towards  the  top.  The  effect  estimates  from  smaller  studies  should  scatter  more  widely  at  the  
bottom,  with  the  spread  narrowing  among  larger  studies  
   
 
30.    What  is  the  graph  that  is  usually  used  to  illustrate  a  relation  between  two  variables?  
 Answer:  Scatter  plot  
Reference:  
http://www.stat.ucla.edu/~rgould/m12s01/relations.pdf  
   
E  
Answer:  c.  
References:  
https://www.ncbi.nlm.nih.gov/pmc/articles/PMC3737004/#!p
o=20.5882  
 
   
●            ·∙         Different  Qs  about  type  of  study  needed  
●            ·∙         About  20Q  about  incedince  and  prevalence  and  relative  risk  
●            ·∙         Multiple  qs  about  RR  OR  
●            ·∙         Memorize  the  equation  of  Specificity.  Few  questions  came  asking  about  it.  
   
Notes:  
-­‐Study  selective  every10th  family  in  city  =systematic  study.  
-­‐  non  response  bias  =cs  
-­‐Lung  ca  90%  30%  smokers  =70%.  
-­‐Dadycare  another  developed  same  =80%.  
Typhoid  fever      
-­‐Sensitive  =probability  that  a  disease  pt  have  +ve  test  results  
-­‐  specificty  =  probability  thr  non  disease  have  -­‐ve  test  results.  
-­‐Balder  ca  =  2.25.  
-­‐    IR  1000/8000*100=12.5.  
-­‐Cohort  study  +ve  mersa  ration  of  exp  on  non  exp  =66:1  
Risk  exp  to  mersa  -­‐vir=66  
-­‐Lung  ca  =  (80/100)/(6/500)=66:1  
-­‐  RR    =a/(a+b)/  c/(c+d)=  20/(20+80)  /494/(494+80)=0.  2  
 -­‐Insulin  =near  to  treatment  14  
-­‐Congenital  hearts  -­‐GD  
OR=  (20/980)/(80/4930)=1.255  
RR  =(20/1000)/(80+5000)=1.25  
-­‐GDM    -­‐>  OR  =2  
-­‐    new  case  200  old  case  80  total  280  so  (280/8000000)*100000=3.5  
-­‐odd  ration  75  control  rate  0.5  
Relative  risk  is  .12  
-­‐  group  of  breast  ca    mastectomy  with  radiation  and  mastectomy  with  chemo  =  randomized  
controlled  trial.    
-­‐Epidemic  investigations  wt  first  =  identiving  population  at  risk.    
   Risk  of  un  exp  2  and  risk  of  exp  3    wt  true  =1  
Bladder  ca  cohort  study  =5  
2  groups  disease  and  non  disease  =  RR=.5  
Incident  of  RS  infection  117  in  1000.  =  1.2  
-­‐Study  compared  and  followed  till  birth  type  =cohort  study  
-­‐Research  about  creatin  disease  in  the  beginning  were  2000  but  after  3y  were  1000  wt  incidence  in  
one  year  =10%  
-­‐Most  determine  study  =positive  predictive  value.  
-­‐Study  used  to  comparing  2  groups  =  cohort  study  
-­‐Study  used  to  comparing  2  groups  pregnet  spesfic  diet    =  cohort  study  l2nha  follow  up  pregnancy  
-­‐  Epileptic  pt  on  carbamazepine  =prospective  cohort  study.  
-­‐  Red  eye  case  à  80  
-­‐  cumulative  incidence  à  2.5  
-­‐  OR  5.2  (  high  fat  diet  pregnant  ladies  comparison  )  
-­‐  Lung  ca  à  70%  
-­‐  MERS-­‐CO  study  Relative  risk  à  66:1  
-­‐  Calculate  disabilities  days  of  URTI  à  292.5  (correct)  
*  GDM  and  congenital  malformation  OR  or  RR?  
*  Research  question  with  numbers  0.3  and  0.2.  
   
   
   
   
 
 
 
 
 
 
 
 
 
 
 
 
 

Emergency  Medicine  
 
 
1-­‐  Aspirin  toxicity?  
Respiratory  alkalosis  and  metabolic  acidosis  
 
2-­‐  Acute  asthma  in  ER,  took  SABA  what  to  do  next?  
A.   Thyophilin  
B.   Inhaled  ipratropium  
Answer:B  
 
3-­‐  Elderly  patient  came  to  the  ER  looking  toxic,  comatose,  and  hypotensive?  
A.   septic  shock  
B.   insulin  overdose  
 
Correct  answer  is  A  
 
4-­‐  Case  of  MG  take  neostagmine  then  present  with  worse  symptoms    to  ER  what  is  the  Rx?  No  IVIG  
in  answers  
A.   Plasmapheresis    
 
Correct  answer  is    
Source:  https://emedicine.medscape.com/article/793136-­‐overview#a9  
 
Emergency  Department  Care  
Patients  with  myasthenia  gravis  who  are  in  respiratory  distress  may  be  experiencing  a  myasthenic  
crisis  or  a  cholinergic  crisis.  Before  these  possibilities  can  be  differentiated,  ensuring  adequate  
ventilation  and  oxygenation  is  important.    

Inpatient  Care:  
Plasmapheresis  has  been  found  to  be  an  effective  short-­‐term  treatment  of  acute  exacerbations  of  
myasthenia  gravis.  Clinical  improvement  takes  several  days  to  occur  and  lasts  up  to  3  weeks.  
Because  of  the  delayed  onset  of  beneficial  effects,  plasmapheresis  has  limited  utility  in  the  ED  
setting,  but  often  is  used  in  the  ICU  setting.  Immunotherapy  with  intravenous  gamma  globulin  
appears  to  diminish  the  activity  of  the  disease  for  unknown  reasons.  The  benefit  begins  within  2  
weeks  and  may  last  for  several  months.  The  AAN  considers  IVIG  an  effective  therapy  for  moderate-­‐
to-­‐severe  cases  of  myasthenia  gravis,  as  per  their  2012  guidelines.    
 
5-­‐21  years  old  patient  with  known  case  of  depression,  has  been  found  on  the  floor  unconscious  
with  empty  pill  bottle  ,  patient  was  obtunded  ,  dilated  pupil  and  unreactive  bilaterally  and  other  
symptom(  I  can  not  remember)    
What  is  the  medication  the  patient  most  likely  on?  
A.   Sertraline  
B.    Fluoxetine    
C.   Other  SSRI  
D.   Amitriptyline  
 
 
Correct  answer:  D  
 
6-­‐  Abdominal  trauma  with  pancreatic  body  injury,  pancreatic  juice  split  anteriorly  ,  where  this  
juice  will  collected  ?  
A.   Omentum  bursa  
B.   Left  para  colic    
C.   Sub  hepatic    
 
Correct  answer:  A  
 
7-­‐  What  is  most  common  cause  of  death  in  flame  burn  ?  
A.   Hypovolemic  shock    
B.   Inhalation  Smoke    
 
Correct  answer:  B  
Source:  
http://msue.anr.msu.edu/news/smoke_inhalation_is_the_most_common_cause_of_death_in_hous
e_fires  
Smoke  inhalation  is  the  most  common  cause  of  death  in  house  fires  

8-­‐  Patient  came  with  snake  bite  what  is  your  first  action?  
A.   Incision  on  the  site  of  bite  
B.   Mechanical  suction  of  venom  
C.   Immobilize  limb    
D.   tourniquet  application  proximal  to  the  site  of  bite  
 
Correct  answer:  C  
Source:  WHO,  Guidelines  for  the  management  of  snake-­‐bites  
http://apps.searo.who.int/PDS_DOCS/B4508.pdf  
 
Recommended  first-­‐aid  methods    
1)  Reassure  the  victim  who  may  be  very  anxious      
2)  Immobilize  the  whole  of  the  patient’s  body  by  laying  him/her  down  in  a  comfortable  and  safe  
position  and,  especially,  immobilize  the  bitten  limb  with  a  splint  or  sling.  Any  movement  or  muscular  
contraction  increases  absorption  of  venom  into  the  bloodstream  and  lymphatics    
3)  Avoid  any  interference  with  the  bite  wound  (incisions,  rubbing,  vigorous  cleaning,  massage,  
application  of  herbs  or  chemicals)  as  this  may  introduce  infection,  increase  absorption  of  the  venom  
and  increase  local  bleeding      
4)  Tight  (arterial)  tourniquets  are  not  recommended:  Traditional  tight  (arterial)  tourniquets  are  not  
recommended.  To  be  effective,  these  had  to  be  applied  around  the  upper  part  of  the  limb  so  tightly  
that  the  peripheral  pulse  gets  occluded.  This  method  can  be  extremely  painful  and  very  dangerous  if  
the  tourniquet  was  left  on  for  too  long  (more  than  about  40  minutes),  as  the  limb  might  be  damaged  
by  ischaemia.  Tourniquets  have  caused  many  gangrenous  limbs  
 
 
9-­‐  Massive  pleural  effusion?  
A.   Thoracocentesis  
B.   Needle  decompression  
C.   Inhalator  in  high  dose    
 
Correct  answer:  A  
Source:  https://emedicine.medscape.com/article/807375-­‐overview#a10  
 
Thoracentesis,  which  involves  the  removal  of  50 -­‐100  mL  for  laboratory  analysis,  is  the  first -­‐line  
invasive  diagnostic  procedure  and  can  be  safely  performed  in  most  patients,  including  those  
undergoing  mechanical  ventilation.    
 
11-­‐  patient  came  to  ER  he  was  semiconscious  you  give  naloxone  he  became  conscious  what  is  the  
drug  toxicity?  
A.    Opioids  
 
Correct  answer:  A  
 
12-­‐  Patient  with  drug  overdose  O/E  there  was  pinpoint  pupils  what  is  the  cause?  
A.   Morphine  
Correct  answer  is  A  
 
13-­‐  Antidote  of  digoxin?    
A.   FAB  immune  globulin.  
 
Correct  answer:  A  
Source:  emedicine  
https://emedicine.medscape.com/article/154336-­‐treatment  
 
Digoxin  immune  Fab  (Digibind)  is  an  immunoglobulin  fragment  that  binds  with  digoxin.  It  is  currently  
considered  first-­‐line  treatment  for  significant  dysrhythmias  (eg,  severe  bradyarrhythmia,  second-­‐  or  
third-­‐degree  heart  block,  ventricular  tachycardia  or  fibrillation)  from  digitalis  toxicity.  
 
14-­‐  Patient  with  paracetamol  overdose  you  after  doing  ABC  you  measure  the  drug  level  you  found  
it  toxic  what  is  the  proper  management  ?  
A.    Give  N-­‐acetylcystiene  
 
Correct  answer:  A  
source  :  
http://bestpractice.bmj.com/best-­‐practice/monograph/337/treatment/step-­‐by-­‐step.html  
 
 

 
 
 
 
15-­‐  Patient  with  cherry  red  skin  what  is  he  toxic  of?    
A.   Carbon  monoxide  
 
Correct  answer:  A  
 
16-­‐  Patient  collapsed  at  clinic,  there  is  no  sign  of  breathing,  u  call  cardiac  arrest  team,  pt  shows  no  
signs  of  breathing  what  is  next  step?        
A.   Start  chest  compression  30:2  
B.   Give  2  rescue  breathing  
C.   Asses  carotid  pulse  
 
Correct  answer:  C  
 
17-­‐  Patient  came  with  bee  sting,  what  to  do  for  him?  
A.    Remove  stinger,  and  monitor  the  patient.  
B.   Antihistamines.  
C.   Epi,  antihistamines,  and  IV  fluid.  
D.   Forgot  it.  
 
Correct  answer:  A  
Source:  uptodate  
https://www.uptodate.com/contents/bee-­‐and-­‐insect-­‐stings-­‐beyond-­‐the-­‐basics  
 
INSECT  STING  REACTION:  
After  being  stung,  you  should  remove  the  stinger  from  your  skin  as  soon  as  possible  to  prevent  any  
more  venom  from  being  released  into  the  skin.  However,  all  the  venom  is  released  from  the  stinger  
within  the  first  few  seconds,  so  this  is  only  helpful  if  done  quickly.  You  do  not  need  to  use  any  special  
technique  (eg,  flicking  or  scraping)  to  get  the  stinger  out.  
Most  people  who  are  stung  by  an  insect  will  develop  a  local  reaction  (an  area  of  swelling  and  
redness).  Some  people  will  also  develop  a  severe  allergic  reaction,  called  anaphylaxis.  

Local  reaction  —  Immediately  after  being  stung,  most  people  have:  

●Sharp  or  burning  pain  


●Skin  swelling  and  redness    

The  swelling  and  pain  usually  improve  within  a  few  hours.  

Local  reaction  treatment  —  To  reduce  pain  and  swelling  after  an  insect  sting,  you  can  try  the  
following:  

●Apply  a  cold  compress  (a  cold,  damp  washcloth  or  damp  cloth  wrapped  around  an  ice  pack)  to  
the  area.  
●If  you  develop  itching,  you  can  take  a  nonprescription  antihistamine,  such  as  cetirizine  
(Zyrtec).  
●A  pain  reliever,  such  as  ibuprofen  (sold  as  Advil,  Motrin,  and  store  brands),  may  help  reduce  
pain.  

Allergic  reaction  —  Insect  stings  cause  allergic  reactions  in  some  people.  Symptoms  of  an  allergic  
reaction  usually  develop  quickly,  and  include:  

●Skin  symptoms,  such  as  hives,  redness,  or  swelling  of  skin  away  from  the  area  that  was  stung  
(for  example,  the  face  or  lips  may  swell  after  being  stung  on  the  hand)  
●Belly  cramps,  nausea,  vomiting,  or  diarrhea  
●Hoarse  voice,  shortness  of  breath,  and  difficulty  breathing  
●Lightheadedness,  dizziness,  passing  out  
Severe  allergic  reactions  are  called  anaphylaxis.  You  can  have  an  anaphylactic  reaction  the  first  time  
you  are  stung.  The  first  and  most  important  treatment  for  a  severe  allergic  reaction  is  a  shot  of  
epinephrine.  

 
18-­‐Patient  has  hemorrhage,  baroreceptor  activated  which  lead  to  increase  heart  rate,  what  of  
those  will  be  decrease?  
A.   Ventricular  contractility  
B.   Venous  capacitance    
C.   Coronary  blood  flow  
 
Correct  answer:  B  
Source:  SMLE  malzama  12    
http://www.cvphysiology.com/Blood%20Pressure/BP031  
 
The  body  can  quickly  sense  a  fall  in  blood  pressure  through  its  arterial  and  cardiopulmonary  
baroreceptors,  and  then  activate  the  sympathetic  adrenergic  system  to  stimulate  the  heart  (increase  
heart  rate  and  contractility)  and  constrict  blood  vessels  (increase  systemic  vascular  resistance).  
Sympathetic  activation  has  little  direct  influence  on  brain  and  coronary  blood  vessels,  so  these  
circulations  can  benefit  from  the  vasoconstriction  that  occurs  in  other  organs  (particularly  in  the  
gastrointestinal,  skeletal  muscle  and  renal  circulations)  that  serve  to  increase  systemic  vascular  
resistance  and  arterial  pressure.  In  other  words,  cardiac  output  is  redistributed  from  less  important  
organs  to  the  brain  and  myocardium,  both  of  which  are  critical  for  survival.  Reduced  organ  blood  
flow  caused  by  vasoconstriction  and  reduced  arterial  pressure,  leads  to  systemic  acidosis  that  is  
sensed  by  chemoreceptors.  The  chemoreceptor  reflex  further  activates  the  sympathetic  adrenergic  
system  thereby  reinforcing  the  baroreceptor  reflex.  When  the  hypotension  is  very  severe  (e.g.,  mean  
arterial  pressures  <50  mmHg)  and  the  brain  becomes  ischemic,  this  can  produce  a  very  intense  
sympathetic  discharge  that  further  reinforces  the  other  autonomic  reflexes.  
 
19-­‐Patient  swallows  pins  what  to  do?  
 
20-­‐  Patient  came  to  ER  with  hemorrhage,  hypotensive.  What  will  you  do  next?  
A.   Ringer  lactate  
B.   Packed  RBC  
C.   Blood  
D.   Fresh  frozen  plasma      
   
Correct  answer:  A  
source  :  emedicine    
https://emedicine.medscape.com/article/760145-­‐treatment#d11  
 
Once  IV  access  is  obtained,  initial  fluid  resuscitation  is  performed  with  an  isotonic  crystalloid,  such  as  
lactated  Ringer  solution  or  normal  saline.  An  initial  bolus  of  1-­‐2  L  is  given  in  an  adult  (20  mL/kg  in  a  
pediatric  patient),  and  the  patient's  response  is  assessed.  
 
21-­‐  Patient  18  years  old  was  playing  tennis,  he  presented  to  ER  with  abdominal  pain  (left  para  
umbilical  pain),  without  history  of  trauma.  What  is  your  next  action?  
A.   Erect  chest  xray  
B.   Abdominal  ct  
C.   Kidney  us  
D.    
Correct  answer:  B  
 
Source:  https://reference.medscape.com/article/776871-­‐workup#c5  
CT  is  the  diagnostic  test  of  choice  for  rectus  sheath  hematoma  and  is  superior  to  ultrasonography  in  
sensitivity  and  specificity.  Patients  who  are  pediatric,  pregnant,  or  have  renal  insufficiency  may  
benefit  from  ultrasonography  as  a  first-­‐line  test  to  avoid  radiation  and  intravenous  contrast  material  
 
Ultrasonography  should  be  used  as  a  first-­‐line  diagnostic  test  in  pediatric  patients,  pregnant  
patients,  or  perhaps  in  patients  with  renal  insufficiency.  However,  in  other  patients,  its  primary  role  
may  be  to  follow  hematomas  to  maturation  and  resolution  after  definitive  diagnosis  by  CT  scanning  
 
22-­‐  Patient  presented  to  ER  with  congested  neck  veins,  no  chest  sounds,  difficulty  breathing.  What  
is  your  first  action?  
A.   Needle  thoracostomy        
 
Correct  answer:  A  
Source:  http://www.trauma.org/archive/thoracic/CHESTtension.html  
 
Classical  management  of  tension  pneumothorax  is  emergent  chest  decompression  with  needle  
thoracostomy.  A  14-­‐16G  intravenous  cannula  is  inserted  into  the  second  rib  space  in  the  mid-­‐
clavicular  line.  The  needle  is  advanced  until  air  can  be  aspirated  into  a  syringe  connected  to  the  
needle.  
 
23-­‐  Patient  presented  to  ER  with  pneumothorax.  What  is  your  next  action?  
A.   Tube  thoracostomy    
       
Correct  answer:  A  
 
 
 24-­‐  Contraindication  for  gastric  lavage?  
A.   Ingestion  of  detergents  
B.   Ingestion  of  aspirin    
 
Correct  answer:  A  
Source:  https://wikem.org/wiki/Gastric_lavage  
 
Gastric  Lavage  
Contraindications:  
●   Corrosive  ingestions  or  esophageal  disease  
●   The  poison  ingestion  is  not  toxic  at  any  dose  
●   The  poison  ingestion  is  adsorbed  by  charcoal  and  adsorption  is  not  exceed  by  quantity  ingestion  
●   Presentation  many  hours  after  poisoning  
●   A  highly  efficient  antidote  such  as  NAC  is  available  
 
Indications:  
●   Life-­‐threatening  poisoning  (or  history  is  not  available)  and  unconscious  presentation  (eg  
Colchicine)  (Protect  airway  (endotracheal  intubation)  if  patient  is  stuporous  or  comatose)  
●   Life-­‐threatening  poisoning  and  presentation  within  1  hour  
●   Life-­‐threatening  poisoning  with  drug  with  anticholinergic  effects  and  presentation  within  4  
hours  
●   Ingestion  of  sustained  release  preparation  of  significantly  toxic  drug  
●   Large  salicylate  poisonings  presenting  within  12  hours  
●   Iron  or  lithium  poisoning  
 
25-­‐  Infant  with  abdominal  colics,  vomiting  and  bloody  stool,  other  data  provided,  diagnosis  is  
intussusception,  the  question  asked  about  the  1st  step  in  management?    
A.    IV  fluids  and  bowel  rest  
B.   CT  abdomen  
C.   Surgery  
D.   Barium  enema  
 
Correct  answer:  A  
Source:  https://emedicine.medscape.com/article/930708-­‐treatment#d10  
 
Tailor  treatment  of  the  child  with  intussusception  to  the  stage  at  presentation.  For  all  children,  start  
intravenous  fluid  resuscitation  and  nasogastric  decompression  as  soon  as  possible.  
 
26-­‐  A  patient  with  depression  on  medications  found  comatose  with  empty  pill  bottle  beside  her  
On  invx;  ABG  showed  Metabolic  acidosis,  what  drug  overdosed?    
A.   Aspirin    
B.   SSRI    
 
Correct  answer:  A  
 
27-­‐  A  teenager  came  with    abdominal  pain  after  a  basket  ball  game,  onExamination  ,  there's  
periumbilical  tenderness.    What  is  the  best  action.  ?    
A.   CT  abdomen  
B.   Abdominal  US    
C.   Erect  chest  x  ray    
D.   Re  examine  after  24  hours.  
 
Correct  answer:  C?  
 
28-­‐  Clear  case  of  pericardial  tamponade(  Hypotension,  distended  neck  veins  and  muffled  heart  
sounds)  
 
29-­‐  RTA  victim  e  closed  head  injury  and  LOC    
What  is  the  best  thing  to  do  ?    
A.   Intubation  and  ventilation    
B.   Check  the  pulses    
C.   Check  the  pupils  
D.   Check  the  airway  
 
Correct  answer:  D  
 
 30-­‐  RTA  victim  was  brought  to  hospital  with  multiple  injuries,  transferred  to  ICU.  There,  he  
developed  multi  organ  failure,  the  phosphate  level  in  his  blood    is  raised  ,  what  is  the  cause  of  his  
multi  organ  failure?  
A.   Heart    
B.   Lungs  
C.   Kidneys    
D.   Liver    
 
Correct  answer:  A  
 
 31-­‐  2  cases  about  iron  toxicity.  Best  way  to  decontaminate  the  stomach  of  a  patient  with    iron  
poisoning?  
A.   Gastric  lavage,    
B.   Syrup  epicac  ,    
C.   Something  called  tacren  I'm  not  sure  about  it  and  last  option  was    
D.   induced  vomiting,    
 there  was  no  activated  charcoal  in  the  choices.    
 
Correct  answer:  A  
Source:  https://emedicine.medscape.com/article/815213-­‐treatment#d10  
 
Gastric  lavage  with  a  large-­‐bore  orogastric  tube  may  remove  iron  from  the  stomach.  Ideally,  lavage  
should  be  performed  1-­‐2  hours  postingestion,  although  later  use  may  be  appropriate  if  evidence  of  
iron  products  in  the  stomach  is  observed  on  a  radiograph.  However,  iron  has  a  gelatinous  texture  
and  may  be  difficult  to  remove  by  lavage.    
The  American  Academy  of  Clinical  Toxicology  advises  that  the  routine  administration  of  ipecac  in  the  
emergency  department  should  definitely  be  avoided.    
Activated  charcoal  does  not  bind  iron.  However,  it  should  be  utilized  if  co-­‐ingestants  are  suspected.  
In  acute  or  chronic  iron  toxicity,  chelation  therapy  with  deferoxamine  is  indicated  for  patients  with  
serum  iron  levels  >350  mcg/dL  who  have  evidence  of  toxicity,  or  levels  of  >500  mcg/dL  regardless  of  
signs  or  symptoms.  In  patients  with  significant  clinical  manifestations  of  toxicity,  chelation  therapy  
should  not  be  delayed  while  one  awaits  serum  iron  levels.  
32  -­‐Iron  poisoning,  a  child  swallowed  a  bottle  filled  iron  pills,  presented  to  ER  e  abdominal  pain,  
nausea  and  confusion.  
Best  way  to  manage  is  ?  
A.   IV  deferoxamine  
 
Correct  answer:  A  
Source:  https://emedicine.medscape.com/article/815213-­‐treatment#d10  
 
In  acute  or  chronic  iron  toxicity,  chelation  therapy  with  deferoxamine  is  indicated  for  patients  with  
serum  iron  levels  >350  mcg/dL  who  have  evidence  of  toxicity,  or  levels  of  >500  mcg/dL  regardless  of  
signs  or  symptoms.  In  patients  with  significant  clinical  manifestations  of  toxicity,  chelation  therapy  
should  not  be  delayed  while  one  awaits  serum  iron  levels.  

33  -­‐Patient  drank  an  ethylene  glycol  containing  product  ,what  renal  damage  is  suspected?    
A.   Acute  tubular  necrosis  (ATN)    
B.    
Correct  answer:  A  
Source:  https://en.wikipedia.org/wiki/Acute_tubular_necrosis#Toxic_ATN  
 
Toxic  ATN  can  be  caused  by  free  hemoglobin  or  myoglobin,  by  medication  including  antibiotics  such  
as  aminoglycoside,  statins  such  as  atorvastatin,  and  cytotoxic  drugs  such  as  cisplatin,  or  by  
intoxication  (ethylene  glycol,  "anti-­‐freeze").  
 
34-­‐  Trauma  to  abdomen  Omentum  out.  What  is  the  management?  
A.   immediate  laporatomy    
 
Correct  answer:    A  
Source:  uptodate  
http://cursoenarm.net/UPTODATE/contents/mobipreview.htm?23/36/24143  
 
Indications  for  emergent  laparotomy  include:  hemodynamic  instability,  unequivocal  peritoneal  signs  
on  physical  examination,  signs  of  gastrointestinal  hemorrhage,  and  implement  in  situ.  Evisceration  of  
bowel  or  mesentery  is  an  indication  for  immediate  laparotomy  at  most  trauma  centers  
 
35-­‐  What  is  the  maximum  duration  that  N  acetylcystein  can  be  administered?  
A.   Given  after  8  hrs    
B.      
NAC  is  most  effective  in  managing  paracetamol  toxicity  when  administered  in  the  first  8  hrs.    
Source:bmj  
   
   
36-­‐  MVA  injury  to  jugular  Foramen  which  muscle  will  be  still  functioning  ?  
A.   sternocleidomastoid  muscle  
B.   hypoglossus  muscle  
C.   stylopharyngus  muscle  
D.   Trapazius    
 
 Correct  answer:  B  
Hypoglossus  muscle  is  supplied  by  XII  nerve,  which  does  not  pass  through  the  jugular  foramen.  
   
   
37-­‐  Patient  with  organophosphate  poisoning  what  to  give  ?  
A.   Atropine    
B.   Nalxoprine  
C.   N-­‐Actylecystein    
D.    
Correct  answer:  A  
The  antidote  for  organophosphate  toxicity  is  Atropine  &  pralidoxime    
   
38-­‐  Child  swallowed  battery,  presenting  with  increase  salivation.  What  is  the  first  step  in  
management?  
A.   X-­‐ray  abdomen,  chest,  and  neck  
B.   Upper  GI  endoscopy  
C.   Watchful  waiting  for  24  hours  
 
Correct  answer:  A  
Source:  http://adc.bmj.com/content/archdischild/61/4/321.full.pdf  
   
39-­‐  What  to  give  after  CPR  in  a  child  with  asystole?  
A.   Atropine  
B.   Epinephrine  
 
Correct  answer:  B    
Source:  AHA-­‐  pediatric  cardiac  arrest  algorithm    
 

40-­‐  Dehydrated  child  (I  think  moderate)  with  severe  vomiting.  Management  ?  
A.   Pedialyte  10  ml  over  5  min  
B.   Normal  saline  50ml/kg  
C.   ORS  with  ratio  of  1:1  glocose  to  sodium  
D.    
Correct  answer:  B  
Source:  clinical  emergency  medicine      
 

41-­‐  Patient  with  tachycardia  and  SOB  after  bee  sting  what  is  the  "OPTIMAL"  treatment:  
A.   Remove  stinger  and  watch  the  pt  
B.   SC  epinephrine  and  IV  antihistamine.  
C.   IM  epinephrine    
 
Correct  answer:  C    
The  initial  management  include  IM  or  SC  epinephrine  ,  while  the  optimal  treatment  include  
monitoring  the  patient,    H1  antagonist,  H2  antagonist  ,  corticosteroids.  
Source  :  Toronto  notes    
 
42-­‐  Man  with  aortic  stenosis  develop  syncope  what  is  the  cause  :  
A.   Systemic  hypotension  
 
43-­‐  women  found  unconsious  and  empty  bottle  of  medication  near  to  her  and  she  was  diagnosed  
with  depression  previously  on  exam  dilated  unresponsive  pupils  what  is  the  drug:  
A.   acetylcholine    
B.   Dopamine  
C.   serotonin    
 
Correct  answer:  C  or  TCA    
Dilated  pupils  are  more  diagnostic  of  anticholinergic  syndrome  than  serotonin  syndrome.    
Source:  bmj          
 
44-­‐  Patient  presented  to  ER  with  hx  of  drug  overdose  for  last  8  hours    on  the  examination  the  gag  
reflex  was  absent  .  the  best  management  is?  
A.   Iv  naloxone  
B.   Immediate  endortracheal  intubation*  
C.   Gastric  lavage  

Correct  answer:  B    

Source:  https://www.ncbi.nlm.nih.gov/pmc/articles/PMC1671555/pdf/bmj00153-­‐0032.pdf  

45-­‐  70  Y/O  came  with  urinary  retention  ,1st  step  ?  


A.   Urinary  catheter  
 
46-­‐    29  year  old  lady,  presented  to  the  ER  with  nausea  vomiting,  generalized  rash  with  
desquamation  and  bilateral  conjunctivitis  and  vaginitis.  No  history  of  travel.  Past  medical  
unremarkable  except  for  vaginitis  a  few  days  back.  Which  of  the  following  is  the  most  likely  
diagnosis?  
A.   Toxic  shock  syndrome  
B.   Meningiococcal  septicaemia  
 
Correct  answer:  A  
 
47-­‐  Elderly  with  fever,night  sweats,    and  loss  of  weight  what  you  will  do  in  the  ER?  
A.   Isolation  in  negative  pressure    
B.   CXR  
C.   Tb  medication  
D.    
Correct  answer:  A    
 
 
48-­‐  Methotrexate  antidote?  
A.   Folinic  acid  
 
Correct  answer:A    
 
49-­‐  Acetaminophen  antidote?  
A.   N-­‐  actylecyctin    
 
Correct  answer:  A  
 
50-­‐  Alcohol  abuse  antidote?  
naltrexone  or  disulfiram    
 
51-­‐  What  drug  reverses  the  effect  of  Benzodiazepines?  
A.   Flumazanil    
 
Correct  answer:A  
 
52-­‐  30  yrs  old  male  presented  to  you  after    45  minutes  from  ingesting  multiple  paracetamol  ,  he  is  
awake  alert  and  not  complaining  of  anything,  what  is  your  next  step?  
A.   N  acetylcysteine  
B.   Activated  charcoal  
C.   Gastric  lavage  
D.   Serum  acetaminophen  concentration  
 
Correct  answer:  D  
When  suspecting  paracetamol  toxicity,  serum  concentration  should  be  checked,  especially  at  the  
earliest  4  hrs  after  ingestion.    
Source:bmj  
 
53-­‐  Comatose  patient  came  to  the  ER  the  1st  thing  to  do?  
A.   Intubation  
 
54-­‐  Boy  coming  to  the  ER  complaining  on  the  right  side  ,examination  all  normal  except  
hypertensive  ,  one  day  before  he  is  fallen  on  right  side    what  is  the  most  organ  involved  ?  (not  
clear)  
A.   Right  kidney  
B.   Liver  
 
Correct  answer:  A  (  NOT  SURE)    
 

55-­‐    Patient  comes  to  you  with  drooling  saliva  constricted  pupil  same  scenario  almost  as  in  notes  
what  to  do
?  

A.   IV  atropine  
B.   
Lavage    
Correct  answer:  A    

Constricted  pupils  and  drooling  are  parts  of  organophosphate  toxicity  syndrome,  which  is  managed  
with  atropine.    

 
56-­‐  20  years  old  student  came  to  you  lethargic  change  of  sense  of  time  blood  test  normal  and  
waiting  for  urine  test  .  What  is  the  cause?  

A.   Caffeine
    
B.   Cannabis    
C.   Cocaine    
D.   Opioid  
Correct  answer:  B  
 
57-­‐  Overdosed  drug  caused  resp  depression?  

opioids  

58-­‐  Overdosed  lead  to  comatose?  

59-­‐  Overdosed  lead  to  dilated  pupils?  

Anticholinergic  drugs    

60-­‐    Food  allergy  with  symptoms  of  anaphlyctic  shock..  

61-­‐  Pscych  pt  swallowed  safety  pin  reached  dudenum  what  to  do  

62-­‐    Organophosphate  poisoning  case  .  Tx?  


A.   Praldoxime  
 
63-­‐    Dilated  pupil,  no  neuro    deficit?  
A.   TCA  
 

64-­‐  Which  of  the  following  is  the  diagnostic  test  for  tension  pneumothorax?  
A.   CXR  
B.   CT  
Correct  answer:  A  
 
65-­‐  Patient  came  with  the  presentation  of  excess  salivation  and  difficult    breathing  due  to  
respiratory  secretions.  The  patient  has  a  history  of  pesticide  exposure  and  now  he  also  have  a  (  
garlic  odor)  What  you  will  give:  (ORAGANOPHOSPHATE  POISINING)  
A.   Nesostagmine    
B.   Pralidoxime  
C.    
Correct  answer:  B    
Atropine  and  pralidoxime  are  antidotes  of  organophosphate  toxicity    
 
67-­‐  How  to  treat  alcohol  withdrawal  (  no  diazepam  in  the  answers)?  
A.   Disulfiram  
B.   Propranolol  
C.   Chlorodizepoxide  
 
Correct  answer  :  C      
Chlorodizepoxide  (it  is  benzo  group)  
Disulfiram  is  used  is  alcohol  dependence.      
 
68-­‐Sign  of  uncomplicated  pneumothorax?  
(  decreased  breath  sounds  and  hyperresonance  over  the  affected  side,  mediastinal  shift  toward  the  
side  of  pneumothorax)    
 

69-­‐  17  years  old  from  MVA,  she  was  intubated  but  then  she  was  severely  hemodynamically  
unstable.  What  sign  will  warrant  you?  
A.   Engorged  IJV  
B.   Shifted  trachea  
           
Hypotension  Resistant  ventilation  
           
REF:  Deterioration  after  intubation  -­‐  DOPE:  
●   Displaced  ETT                  
●   Obstruction  (anywhere  along  circuit)  
●   Pneumothorax  
●   Equipment  failure  (ventilator  malfunction  or  disconnect)  
 
70-­‐  Patient  present  to  ER  intensive  knee  swelling  with  ballottement  pattern  next  step?  
A.   ESR  
B.   Arthrocentesis  
C.   CBC  
Correct  answer:B  
 
70-­‐  23  yo  with  agitation,  dilated  pupil,  diaphorisis  ,  tachycardia  ,  what  is  the  toxicity?  
A.   TCA  
B.   Digoxin  
C.   Sympathomymetic    
D.   Anticholenergic  
 
Correct  answer:  C  
                     
72-­‐  Female  taking  paracetamol  500mg  bid  daily  presented  with  liver  enzymes  elevated  and  
hepatomegaly?  
A.   Alpha  1  antitrypsin  deficiency  
B.   Liver  damage  due  to  paracetamol            
C.   Hepatitis  i  guess  
 
Correct  answer:  B  (not  sure)    
Paracetamol  ingestion  may  cause  hepatocyte  death  which  causes  acute  liver  failure.    
Source:  BMJ  
 
 
 
73-­‐    Treatment  of  MS  attack  in  ER:  
A.   Oral  antibiotic  
B.   IV  antibiotic  
C.   Oral  steroid  
D.   IV  steroid  
 
Correct  answer:  D    
Source:  step  up  to  medicine    
 
74-­‐  Patient  take    a  lot  of  sleeping  drugs    unresponsive  pulse  rapid  weak,  show  reflex  gasping  
breathing,  give  2  breathing  by  mask,  next  step?  
A.   Wait  for  blue  team    
B.   Perform  CPR    
C.   Intubate    
 
Correct  answer:  C    
CPR  is  not  indicated  if  there  is  pulse.    
Source:  AHA  
 
75-­‐  Case  scenario  about  patient  with  basal  skull  fracture  what  expected  nerve  injury?  
A.   Olfactory  
B.   Optic  
C.   Oculomotor  
 
Correct  asswer:A  
 
76-­‐  Unconscious  patient  with  agonal  gasps?  
A.   check  carotid  pulse  
B.   Start  CPR  
C.   (Rescue  breaths?)  
 
Correct  answer:  B    
Agonal  gasping  (  breathing)  is  associated  with  cardiac  arrest.    
Source:  AHA  
 
77-­‐  Epileptic  pt  with  trauma?  
A.   Posterior  dislocation  of  shoulder  
 
78-­‐  RTA  victim  come  with  closed  head  injury  and  LOC.  What  is  the  best  thing  to  do?  
A.   Intubation  and  ventilation  
B.   Check  pulse  
C.   Check  pupils  
D.   Check  airway  
E.    
Correct  answer:  D  
Head  injury  with  LOC  requires  immediate  intubation    
Source:  Toronto  notes-­‐  traumatology    
 
 
 
79-­‐  A  child  brought  to  ER  with  barking  cough,  red  epiglottis,  thumb  sign  on  x  ray,  
Best  initial  management?  
A.   examination  of  epiglottis    
B.   Endotracheal  intubation.    
C.   Emergent  tracheostomy  
D.   Nasopharyngeal  Tube  
 
Correct  answer:  B    
 
 
80.  What  is  your  concern  with  the  following  when  giving  a  patient  opioids  (patient  on  opioid,  
contraindicated  drug  is)?  
A.   Aspirin  
B.   Antidepressants  
C.   NSAID  
D.   Laxative  
 
Correct  answer:  B  
Source:  https://www.healthline.com/health-­‐news/fda-­‐warning-­‐involving-­‐opioid-­‐prescriptions-­‐
antidepressants#2  
 
81.  paracetamol  antidote?  
A.   Acetylcysteine  
 
82.  When  to  give  paracetamol  antidote  (  maximum  or  something)?  
A.   2h  
B.   4h  
C.   6h  
D.   8h  
 
Correct  answer:D    
NAC  is  most  effective  in  managing  paracetamol  toxicity  when  administered  in  the  first  8  hrs.    
Source:bmj  
 
83.  20  yrs  girl  come  to  hospital  after  20  mins  of  complain  of  light  headiness  and  headache,....When  
she  went  to  the  doctor  she  start  complain  of  peri  oral  numbness  and  fingers  tingling.  What  to  do?  
A.   alcohol  leveling                                                                                                                                                                      
                                                                                                     
84.  Side  affects  of  atropine:  
A.   Seizure  
B.   Vomiting  
C.   Dry  mouth  
 
Correct  answer:C  
 
85.  SE  of  morphine:  
A.   N/V  
 
86.  1st  symptom  in  hypomagnesemia:  
A.   Decreased  deep  tendon  reflexes.  (hypermagnesemia)  
B.   Respiratory  depression.  (  hypermagnesemia)  
C.    
Hypomagnesemia  :  (muscle  twitch,  weakness,  tremor,  hyperreflexia,  seizure  )    
 
87.  What  drug  is  likely  to  cause  heat-­‐stroke  as  it  inhibits  sweating:  
A.   Hyoscamine  Sulfate  
B.    
Correct  answer:  A  
https://www.drugs.com/pro/hyoscyamine-­‐sulfate-­‐tablet.html  
 
88.  Usually  if  there  is  fentanyl  overdose  what  will  give?  
A.   naloxone    
 
89.  Patient  admitted  to  ER,  she  denies  eating  and  says  she's  not  hungry,  BMI  11.3  What  most  likely  
to  find  in  her  labs:                                                                                                                            
A.   inc  K                                                                                                            
B.   Dec  Creatinine                                                                                                                                  
C.   Other  labs  
D.    
Correct  answer:B    
 
GCS  
 

 
 
 
 
 
  APGAR  SCORE
 

 
 
 
   
 
 
 

OB/GYN  
 
 
1.What  indicates  the  labor  progress?  
a.  Descent  of  the  presenting  part  
b.  Head  manipulation  
c.  Strength  of  uterine  contraction  
d.  Frequency  of  uterine  contractions  
Answer:  A  
 
2.  Which  OCP  causes  hyperkalemia  
a.  Estradiol-­‐levonorgestrel  
b.  Estradiol-­‐drosperirenone  (  yasmin  )  
Answer:  B  (https://reference.medscape.com/drug/yasmin-­‐yaz-­‐drospirenone-­‐ethinyl-­‐estradiol-­‐
342768#4)  
 
3.  When  to  do  screening  for  bacterial  vaginosis?  
a.  All  trimester  
b.  First  trimester  
c.  Second  trimester  
Answer:  GBS  screening  at  35-­‐37  weeks  gestation    
 
4.  40  y/o  Female  preconception  counseling,  hx  of  fetal  death  after  delivery  with  neural  tube  
defect,  what’s  your  best  advice?  
a.  It  could  occur  because  of  your  age  
b.  Cvs  at  week  16  
C.  Folic  acid  supplementation    
Answer:  C?  
 
5.  Case  about  threatened  abortion  (hint  is:  POC  is  intact,  and  cervix  is  closed).  
 
6.  MgSO4  is  given  in  pregnancy  in  case  of  preeclampsia  for  what?  
Answer:  To  reduce  the  risk  of  seizures.  
 
7.  Greenish  foul  smelling  vaginal  discharge,  oE/  cervix  e  dots  and  erythema  (the  q  provided  the  
description  of  strawberry  cx)  
Answer:  trichomoniasis    
 
7-­‐  Same  scenario  (trichomoniasis),  what  is  the  treatment?  
A-­‐  Cerftriaxone    
B-­‐  Metronidazole  
Answer:  B  
 
8.  One  of  the  following  is  seen  in  bacterial  vaginosis  
a.  No  itching  or  burning  
b.  Yellow  discharge  on  Vaginal  PH  paper  
c.    Greenish  discharge  
Answer:  A    
itching  could  be  present,    
They  will  have  Alkaline  pH,  in  pH  paper  the  color  of  the  paper  changes  from  bright  yellow  at  pH  4.5  
and  lower  to  dark  blue  at  pH  7.0  and  higher.    
Characteristic  discharge  appearance  is  thin,  gray,  and  homogeneous.  Positive  whiff  test  and  clue  
cells  
 
9.  Young  female  with  fishy  odor  vaginal  discharge  and  positive  whiff  test  
a.  B  vaginosis  
b.  Trichomoniasis  
c.  Candidiasis  
d.  Chlamydia  infection  
Answer:  A    
 
10.A  female  came  to  GYN  clinic  with  post  coital  bleeding,  Source  of  bleeding  is  
a.  cervix  
b.  uterus  
c.  vulva  
d.  vagina  
Answer:  A  
 
11.  Couples  are  said  to  be  infertile  after  marriage  and  should  seek  for  fertility  after  
a.  3  M  
b.  6  M  
c.    9  M  
d.  12  M    
Answer:  D  
 
12.  Young  couples  trying  to  conceive  for  6  months,  both  are  healthy  
Answer:  Try  some  more.  
 
13.  38  wks,  multiparous,  with  a  trial  of  ECV  on  week  36,  her  amniotic  fluid  index  is  12,  now  her  
fetus  is  in  lateral  position,  her  membranes  ruptured  with  clear  fluids,  What  would  be  the  
indication  for  CS?  
a.  Repeated  ECV  
b.  amniotic  Fluid  index  
c.  ROM  
d.  Fetal  position  
e.  Failed  first  attempt  
Answer:  D  
 
14.  Patient  with  DUB,  what  to  give?  
Answer:  OCP  (progesterone)  
 
15.  Young  age  married  woman  trying  to  conceive  for  1  year  but  failed,  she  has  some  facial  hair  and  
hoarseness  of  voice,  her  husband  semen  analysis  is  normal,  what  is  the  ttt  
a.  Laparoscopy  
b.  Laparotomy  
c.    Clomiphene  citrate  
Answer  :  C  
 
16.  Post-­‐menopausal  woman  presented  with  vaginal  bleeding  cervical  and  vaginal  exam  normal  
except  for  blood  at  the  cervical  os  what  is  ur  next  step:  
a.  Endometrial  sampling  
b.  Pelvic  ultrasound  
c.  Bhcg  
d.  Ca  125  
Answer:  A  (https://emedicine.medscape.com/article/258148-­‐workup#c7)  
 
17.  Which  can  cross  placenta  and  cause  deafness  and  heart  problems?  
a.  Rubella  
b.  Measles  
c.  Mumps  
d.  HBV  
Answer:  A  
 
18.  30  year  old,  did  HPV  testing  and  pap  smear.  When  is  it  recommended  for  her  to  be  retested?  
a.  1  year  
b.  2  years  
c.  3  years  
d.  every  5  yrs.    
Answer:  D  (  correct)  (http://www.aafp.org/patient-­‐care/clinical-­‐recommendations/all/cw-­‐pap-­‐
smears.html)    
 
Notes  about  pap  smear  and  HPV  :  
Age  to  start  :  Screening  should  start  at  age  21  
Age  to  stop  :  Women  age  65  and  older  should  stop  getting  screened  if  they  meet  all  3  of  these  
requirements:  
They  have  never  smoked  
They  do  not  have  a  new  sex  partner  since  their  last  Pap  test  
They  had  Pap  tests  done  regularly  until  they  turned  65  
They  had  3  normal  Pap  tests  in  a  row  
They  had  no  abnormal  Pap  
From  21-­‐29  :  pap  test  every  3  years  
30  and  older  :  Pap  test  plus  HPV  test  every  5  years  
Or  Pap  test  every  3  years  
 
 
19.  which  medication  decrease  effect  of  OCP  ?  
A.Anticoagulant  
B.Anticonvulsant  
C.Steroids  
Answer:  B    
 
20.  Pregnant  in  labor,  fetal  heart  rate  deceleration  from  140  to  80.  What  is  the  most  appropriate  
anesthesia?  
Answer:  General  anesthesia  
 
21.  Turner  patient  diagnosed  with  premature  ovarian  failure,  which  of  the  following  you  will  
expect  to  find?  
Answer:  High  FSH,  LH,  low  estrogen  
 
37.  41  week  pregnant  women    came  to  u  with  some  fetus  position  and  u  did  External  cephalic  
version,    but  after  some  time  u  found  the  the  fetus    return  to  the  same  previous  position  ?  What  
prevent  u  from  doing  another  External  cephalic  version,  ?  
a.  failed  first  attempt  
b.  Polyhedron  ibis  
c.  Oligohydramnios    
d.  Gestational  Age  40  weeks  
Answer:  A  or  C,  Contraindication  of  ECV:  Fetal  abnormality,  placenta  previa,  oligohydromnios  or  
polyhydromnios,  Hx  of  antepartum  hemorrhage,  previous  c/s,  multiple  gestation,  pre-­‐eclampsia  or  
hypertension,  plan  to  deliver  c/s  anyway  
 
23.  Best  way  to  prevent  fracture  in  osteopenic  postmenopausal  lady!?  
a.  Daily  Vit  D    
b.  Exercise  
Answer  A  (but  if  there  bisphosphonates  chose  it)  
We  suggest  adequate  calcium  and  vitamin  D  for  all  postmenopausal  women  with  osteoporosis  and  
bisphosphonates  should  be  considered  as  first-­‐line  agents  for  the  prevention  of  osteoporosis.  
UpToDate  
 
24.  Lady  has  vaginal  discharge,  postcoital  bleed,  and  painful  urination.  Exam  show  red  cervix,  gram  
stain:  gram  negative  diploococci  .What  is  organism?  
a.  Gonorrhea    
b.  Chlamydia  
Answer:  A  
 
25.  Pregnant  lady,  1st  trimester,  UTI  which  Abx  contraindicated:  
a.  Tetracycline    
b.  Nitrufornitoin  
Answer:  A  
 
26.  which  of  the  following  exacerbate  symptoms  of  primary  dysmenorrhea!?  
a.  copper  IUD    
b.  Levinorgasterel  IUD  
c.  Mg  
d.  NIfidipine  
Answer:  A  
Dysmenorrhea  is  often  worse  in  the  first  few  cycles  after  insertion  of  a  copper  IUD,  and  along  with  
unscheduled  bleeding,  is  one  of  the  primary  reasons  for  copper  IUD  discontinuation.  However,  
discontinuation  rates  for  pain  are  low  (0.1  to  2.4  percent)  in  both  copper  and  LNg20  IUD  users.  
Moreover,  the  LNg20  and  LNg14  have  both  been  found  to  reduce  rates  of  dysmenorrhea  .  Mild  to  
moderate  dysmenorrhea  can  be  treated  with  nonsteroidal  antiinflammatory  drugs  (NSAIDs)  begun  
at  the  onset  of  menses  and  maintained  for  the  first  three  days  of  each  menstrual  cycle.  Women  with  
severe  dysmenorrhea  and  a  copper  IUD  should  consider  the  LNg20  or  LNg14  IUD  or  choose  another  
method  of  contraception.  
reference:  uptodate  
 
27.  Screening  for  Group  b  Streptococcus  in  pregnancy:*  
Answer:  35  wks  
 
28.  Female  patient  presented  with  symptoms  of  urinary  incontinence,,  it's  associated  with  cough  
and  stress,,  patient  tried  to  avoid  it  by  strengthening  pelvic  muscles  while  accidentally  urinating  
but  it  was  of  no  use...  what  to  do  for  her  ?  
a.  strengthening  pelvic  
b.  slings  
Answer  :  B  
conservative  approaches  to  treatment  of    Stress  urinary  incontinence  include  pelvic  floor  muscle  
training  and  incontinence  pessaries  and  Bladder  training.  However,  for  women  who  decline  or  have  
insufficient  improvement  following  conservative  therapy,  there  are  a  variety  of  surgical  treatments.  
The  introduction  of  midurethral  slings  has  changed  the  decision  process  for  surgical  treatment  and  is  
likely  a  factor  in  the  increase  in  the  rate  of  anti-­‐incontinence  surgery  in  the  United  States  
https://www.uptodate.com.ezp.uod.edu.sa/contents/surgical-­‐management-­‐of-­‐stress-­‐urinary-­‐
incontinence-­‐in-­‐women-­‐choosing-­‐a-­‐primary-­‐surgical-­‐
procedure?source=see_link&sectionName=Women%20who%20decline%20or%20have%20persisten
t%20symptoms%20following%20conservative%20therapy&anchor=H4#H4  
 
29.  Which  of  the  following  is  contraindicated  during  pregnancy?  
a.  nitrofuranroin  
b.  ampicillin  
c.  folorquinolones  
Answer  is  C  
nitrofurantoin  Pregnancy  Risk  Factor  B  (contraindicated  at  term)  
fluoroquinolones  are  considered  Pregnancy  Risk  Factor  C  
ampicilin  Pregnancy  Risk  Factor  B  
uptodate  
 
30.  Patient  presented  with  whitish  vaginal  discharge,  no  foul  smell,  wet  mount  showed  
psaeudohyphea  what's  your  treatment  
a.  Topical  miconzaole  
b.  Metronidazole  
c.  Ampicillin  
d.  Ceftriaxone  
Ans:  A  
Ref:  https://www.cdc.gov/std/tg2015/candidiasis.htm  
 
31.  Pt  presented  with  whitish  cheese  like  vaginal  discharge  no  foul  smell  what's  the  organism  
a.  Candida  
b.  Trichomonas  
c.  Syphilis  
d.  Chlamidiya  
Ans:  A  
Ref:  http://emedicine.medscape.com/article/2012015-­‐overview#a1  
 
32.  16  wk  pregnant  presented  with  severe  vomiting,  abdominal  distention  and  small  for  
gestational  age.  U/s  showed  snow  storm.  Bhcg  6.  What  is  the  dx?  
a.  Choriocarcinoma  
b.  Complete  mole  
c.  Partial  mole  
Ans:  B  
Ref:  https://radiopaedia.org/articles/hydatidiform-­‐mole  
 
33.  Pt  presented  with  mobile  vulvar  cyst  what  is  the  dx?  
A-­‐  Bartholin  cyst  
Ans:  incomplete  case,  read  more  from  the  following  link.  
Ref:  https://www.uptodate.com/contents/vulvar-­‐lesions-­‐differential-­‐diagnosis-­‐based-­‐on-­‐
morphology#H27977424  
 
34.  What  is  the  most  common  cause  of  bleeding  in  postmenopausal  woman?  
a.  cervical  polyp  
b.  cervical  erosion  
c.  atrophic  vaginitis    
d.  endometrial  hyperplasia  
Answer:    C  
 
35.  Case  scenario  about  postmenouposal  atrophic  vaginitis  and  ask  about  treatment:  
A.   topical  steroid  
B.   topical  estrogen  
 Answer:  B  
 
36.  39  weeks  pregnant  lady,  without  any  complications  in  her  pregnancy,  came  with  vaginal  
bleeding  (Fresh  blood  per  vaginal  with  minimal  clots)  what  is  the  diagnosis  ?  
a.  abruptio  placenta.  (it  is  painful)  
b.  placenta  previa.  
c.  attempted  to  abort  herself.  
d.  rupture  of  membrane.  (  clear  fluid  not  blood)  
Answer:    B  
 
38.  mechanism  of  Action  of  drugs  that  inhibit  Conversion  of  estriol  to  estrogen.  (I  forgot  the  exact  
question  but  it  mentioned  about  ovulation  and  who  inhibits  conversion  of  esterone  to  estrogen?)  
Answer:  Aromatase  inhibitors  
 
39.  A  women  wants  babies  but  she  doesn't  wants  to  get  pregnant  for  several  years  based  on  these  
statistics  of  her  condition  what  contraceptive  you  would  give  her  
a.    Diaphragm  
b.  Progestin  only  pills  
c.  Combined  OCP  
Answer:  Question  may  not  including  the  right  answer,  they  want  a  long  acting  contraception!  and  all  
of  them  are  used  regularly  
If  you  find  implant  (Implanon)  or  intrauterine  device  (IUD)  choose  one  of  them  
Reference:  http://m.acog.org/Resources-­‐And-­‐Publications/Committee-­‐Opinions/Committee-­‐on-­‐
Gynecologic-­‐Practice/Increasing-­‐Access-­‐to-­‐Contraceptive-­‐Implants-­‐and-­‐Intrauterine-­‐Devices-­‐to-­‐
Reduce-­‐Unintended-­‐Pregnancy?IsMobileSet=true  
 
40.  Female  17  year  old,  menarche  at  15,  regular.  Complaining  of  painful  menses.  Examination  
unremarkable.  What  to  give  her?  
Answer:  NSAID    
   
 
41.  lady  with  secondary  dysmenorrhea,  has  bilateral  ovarian  cyst,  US  showed  hypoehoic,  doppler  
showed  poor  perfusion,  what  next  to  confirm:  
a.  CT  
b.  MRI  
c.  Biopsy  
d.  CA125  
 
42.  Pregnant  4cm  dilation  cervix  90%  effacement  with  platelets  60  what's  ur  option  for  
anesthesia?  
a.  General  
b.  Pudendal  
c.  Paracervical  
d.  Narcotics  
Answer  :  A  
Pudendal  use  in  2nd  stage  of  labor  
Pudendal  and  epidural  are  contraindication  in  low  platelet  pt  
 
43.  38  week  pregnant  LL  swelling  fatigue  (the  hx  suggests  DVT)  
A.bed  rest  
B.LMWH  
C.venography  
Answer:  B  http://www.aafp.org/afp/2008/0615/p1709.html  
 
44.  what  can  u  palpate  in  the  lateral  fornix  
Answer:  Ovary    
 
45.  Hormonal  replacement  therapy  will  prevent  wt  from  the  following  
A.Menopausal  symptoms    
B.Osteoporosis  
Answer:  A  
 
46.  Pregnant  women  30  weeks  came  to  the  clinic  for  the  first  time,  with  symptoms  of  anemia  (  
tachypnea  ,  pale  ..  Etc  )  and  lower  limb  pitting  edema  +  heart  murmur  ,  her  labs.  Showed  Hg  of  6  
and  low  MCV  wt  is  the  diagnosis  ;  
A.Heart  disease  
B.Anemia    
Answer:  B  
 
47.  Case  scenario  about  female  patient  with  swelling  in  labia  majora  drainage  was  done  2  time  and  
she  ask  the  doctor  about  treatment  to  prevent  the  recurrence  ?  what  will  he  do  ?  
A.aspiration  
B.Marsupialization  
C.drainage.  
Answer:  B  
There  is  many  options  for  treatment  but  the  best  is  a  surgical  procedure  known  as  marsupialisation  
may  be  used.  
*Reference  :*  Gynecologist.  
 
48.  Multiparous  lady  with  cervical  dysplasia.  Which  of  the  following  is  the  most  likely  cause  of  her  
dysplasia?  
A.HPV  
B.multiparity  
Answer:  A  
 
49.  Lady  24  weeks  pregnant  with  DM2  with  nephritis  grade  F,  and  chronic  HTN  controlled  on  
meds,  which  one  of  the  following  is  the  most  likely  complication?  
A.Still  birth  
B.Preeclampsia  
C.Shoulder  dystocia  
D.Increased  birth  weight  
Answer:  B  
 
50.Case  of  amenorrhea  ,  there  is  breast  ,  absent  uterus  and  vagina  ?  
A.Mullerian  agenesis    
B.Androgen  insensitivity  
Answer:  A  
 
51.Case  of  pregnant  women  ,  sleep  a  lot,  want  to  eat  specific  food..  etc,  what  is  the  responsible  
hormone?  
A.Progesteron  
B.Estrogen  
C.bHCG    
D.Prolactin  
Answer:  C  
 
52.Pregnant  41  week  ,  cervix  5  cm  dilated  ,  60  %  effacement,  in  the  hospital  since  8  hours,  you  
checked  again  the  same  date  above,  what  to  do?  
(same  q  )Multigravida  came  with  regular  contractions.  Cervix  is  4  cm  dilated  and  60%  effaced.  2-­‐3  
hours  later  her  cervical  dilation  is  5  cm  but  still  60%  effacement.  What  is  the  management:  
A.C/S  
B.IV  oxytocin  and  rupture  of  amniotic  membrane    
Answer:  A  
Ref:  http://www.medscape.com/viewarticle/706359  
 
54.  26  year  old  came  with  foul  smelling  vaginal  discharge  and  irritated  uvula.  Which  one  of  the  
following  is  the  most  likely  diagnosis:  
A.  Trichomoniasis  
B.  Bacterial  vaginosis  
C.  Candida  
Answer:  A  
55.  a  woman  used  IUD  for  contraception,  now  has  vaginal  pain  and  discharge,  What  organism  
responsible?  
Answer:  IUD  causing  PID  by  actinomyces  israelii  
 
56.  Mother  with  Rh  -­‐ve  and  a  father  with  Rh  +ve,  what  the  probability  of  having  Rh  +ve  child?  
A.  25%  
B.  50%  
C.  75%  
D.  100%  
Answer:  B    
 
57.What  is  the  most  commonly  injured  organ  in  hysterectomy  
A.  Ureter  
B.  Colon  
C.  Bladder  
Answer:  C  
Ref:  http://laparoscopy.blogs.com/prevention_management_3/2010/07/complications-­‐of-­‐
laparoscopic-­‐gynecologic-­‐surgery.html  
 
58.  vaginal  discharge  watery,  yellow  ,  foul  smelling,  and  no  itching  ?  
Answer:  Bacterial  vaginosis.    (  no  itching  ,  watery  ,  odour)  
Take  care  If  they  mentioned  erythema  of  the  vulva  it  will  be  trichomonus  
 
59.  Case  of  miscarriage  ,  the  fetus  has  bilateral  renal    agenesis  ,  what  you  expect  also  to  find  !?  
A.polyhydramnios  
B.Oligohydramnios  
C.Renal  hyperplasia  
Answer:B  
 
60.  Dizygotic  twins  which  is  true?  
2  placenta  2  amniotic  fluid  regardless  sex  
2  placenta  1  amontic  same  sex  
1  placenta  1  amontic  same  sex  
1  placenta  1  amontic  regardless  sex  
Answer:A  
 
61.PCOS  have  a  risk  to  cause  which  type  of  tumor?  
A-­‐  endometrial  cancer  
B-­‐  ovarian  cancer  
C-­‐  cervical  cancer  
Answer:  A  
 
62.pregnant  women  already  have  a  history  of  two  miscarriage,  after  that  the  doctors  diagnosed  
her  as  cervical  incompetence,  she's  now  pregnant  and  have  a  bleeding,  what  is  the  cause  of  her  
condition?  "    
A-­‐  cervical  incompetence.  
B-­‐  chromosomal  disorders  
C-­‐  ectopic  pregnancy  
Answer:A  
 
63.  premature  menopause  at  which  age  :  
A.  30  
B.  35  
C.  40    
D.  45  
Answer:  C  
 
64.  Most  common  risk  factor  for  fibroid  or  (most  important):  
A-­‐age    
B-­‐  African  race    
C  -­‐multipara  
D-­‐  Smoking  
Answer:    B  (African  women  tend  to  have  larger  fibroid  and  they  are  3  times  more  likely  to  develop  
fibroid  than  white  women)  
Note:  smoking  and  multipara  is  protective  against  fibroid.      
 
65.  28  years  old  pregnant  lady  with  tubal  pregnancy,  Which  one  of  the  following  most  common  
risk  factor  of  ectopic  pregnancy  
A-­‐  Induction  by  ovulation  
B-­‐  Previous  pregnancy  (ectopic)  
C-­‐  IUCD  
D-­‐  Pelvic  inflammatory  disease.  
Answer:  B,D  
 
66.pregnant  lady,  we  want  to  give  her  prostaglandin  F2  alpha,  Which  one  of  the  following  diseases  
should  be  taken  into  consideration?  
A-­‐  Asthma    
B-­‐DM  
C-­‐HTN  
D-­‐  cholelithsis      
Answer:  A  
 
67.  Best  investigation  to  diagnose  ECTOPIC  pregnancy:  
A-­‐Ultrasound  
B-­‐Laparoscopy    
C-­‐B  HCG  
Answer:  B  
 
68.pregnant  woman  when  to  take  vaginal  swab  for  meningiococcus  or  something  about  infection  
to  protect  the  fetus  from  this  infection  ?  
A.38  weeks    
B.40  weeks    
C.26  weeks    
Answer:  35-­‐37  weeks  
I  think  they  mean  GBS  infection    
 
69.  women  diagnosed  with  trichomonas  and  she  is  asymptotic,  when  to  start  treatment?  
A.   immediate    
B.   when  symptomatic  
Answer:  A  
 
70.  when  birth  consider  premature?  
A.40    
B.38  
C.26  
Answer  :  Less  than  37  weeks    
 
71.  which  ligament  protect  uterus  from  prolapse  ?  
A.broad    
B.round  
C.uterosacral    
Answer  :  C    
 
72.  HIV  pregnant  women  something  drop  from  400  to  200  so  how  she  will  deliver?  
Spontaneous  vaginal  (  answer  )  
 
73.  Female  with  S&S  of  pregnancy  +right  lower  quadrant  pain  after  doing  US  there  is  ectopic  
pregnancy  and  the  pt  stable  what  will  you  do?    
Treat  medically  (  methotraxate  if  the  patient  stable  )  
 
74.  S&S  of  pregnancy  B-­‐HCG  negative,dx?  
Ovarian  torsion(  not  sure,  missing  details  )  
 
75.  what  is  the  role  of  OCP  in  protection  against  illegal  pregnancy  ?    
Prevent  fertilization    
 
76.  case  of  postpartum  hemorrhage  you  gave  oxytocin  no  stoppage  and  the  diagnosis  was  uterine  
atony,  next  step?    
Bimanual  compression  
 
77.  which  one  is  true  about  Pap  smear  ?    
Decrease  the  incidence  of  cervical  ca  dramatically  
 
78.  best  site  to  take  Pap  smear  ?    
Transformation  zone  
 
79.  In  Pap  smear  you  find  hyperplasia  in  ectocervix  ,  next  step?    
Colposcopy  directed  biopsy  
 
80.  correct  definition  of  recurrent  miscarriage?    
three  or  more  consecutive  pregnancy  losses  
 
81.  pregnant  e  bacterial  vaginosis  what  is  ttt?  
No  clindamycin  only  metronidazole  in  choices  
Metronidazole  is  the  most  common  and  preferred  antibiotic    
 
82.  A  young  woman  came  to  Primary  health  care  for  pregnancy  counseling,  she  had  chickenpox  
when  she  was  a  child.  What  will  you  do  to  her?  
A.  Varicella  immunoglobulin.  
B.  Rubella  antigen.  
C.  Give  MMR  vaccine  
Answer  :  B  
83.  CASE  about  abruptio  placenta  which  one  of  the  following  sign  and  symptoms  is  the  most  
common  or  the  most  serious  one  (I  am  not  sure)?  
A-­‐vaginal  bleeding  
B-­‐uterine  contraction  
C-­‐  Fetal  distress  (most  serious)  
Note:  the  choice  is  not  very  accurate  so,  make  sure  in  the  exam.  
Answer  :  A  
 
84.  Patient  in  labor  and  got  complicated,  will  go  for  c/s,  when  to  give  abx?  
A.  Preoperative  
B.  Intrapartum  
C.  Intraoperative  
Answer  :  A    
Note:  To  give  cefazolin  or  ampicillin  
https://www.uptodate.com/contents/cesarean-­‐delivery-­‐preoperative-­‐planning-­‐and-­‐patient-­‐
preparation#H9  
 
85  .Multiparous  ,38  wks,  in  labour,  90%  cervical  effacement,    4CM  dilatation  ,membranes  ruptured  e  
clear  fluid  ,On  CTG  ,FHR  dropped  from    140to  80,  the  pt  platelets=50,000,  what  type  of  anesthesia  is  
suitable?    
A.  GA  
B.  Epidural  
Answer:  A  
 
86.  MOA  of  metformin  in  PCOS?  
Reduces  insulin  resistance.  
 
87.  Dose  of  folic  acid  during  pregnancy  in  milligrams?  
A.  0.4    
B.  0.2    
C.  2  
D.  4  
No  risk  factor  :  0.4    
Risk  factor  (  NTD,  DM  )  =  4    
 
88.  Safest  anti  diabetic  med  during  pregnancy?  
A.  metformin    
B.  acarbose    
C.  sitagliptine    
D.  glypuride    
answer:A.  If  there  is  insulin  go  for  it  .  
 
89.  Lady  with  Hx  of  PCO  syndrome  the  best  management  is:                                                                                                                      
A.  Weight  reduction  *  first  line  *  
B.  Metformin  and  clomiphin  (  if  she  wants  to  get  pregnant  )    
C.  Weight  reduction,  danazol  and  metformin    
Did  not  find  clear  answer    
 
90.  Lesion  from  vulva  with  tree  like  shape?  
No  choises  
condyloma  acuminatum  valvua  ((  i  think  they  mean    cauliflower  o  answers  was  provided  I  tried  to  
search  no  answers  )    
 
91.  Female  come  for  follow  up  with  lesion  in  labia  majora  2  in  2  cm,  biopsy  was  taken,  what  is  the  
histopathology  of  this  biopsy?  
A.  Adenocarcinoma  
B.  Squamous  cell  carcinoma  
C.  Adeno  squamous  cell  carcinoma                  
 Answer  :  B    
92.  Female  come  to  the  clinic  with  plaque  in  labia  majora,  what  most  likely  to  be?    
A.  Basal  cell  carcinoma  
B.  Melanoma                                                  
C.  lupus  pernio  
Answer  :  B    
                                       
93.  Female  had  2  abortion  in  second  trimester,  last  one  done  for  her  D&C  diagnosed  as  cervical  
incompetence  now  pregnant  in  5  week  present  with  vaginal  bleeding  and  open  cervical  os  what  is  
the  cause  of  bleeding?  
A.  Luteal  phase  defect  
B.  Cervical  incompetence  (    second  and  third  trimesters)    
C.  Asherman  syndrome  
D.  Chromosomal  cause  
Answer  :  D  
https://www.uptodate.com/contents/miscarriage-­‐beyond-­‐the-­‐basics  
 
94.  Female  had  2  abortion  in  second  trimester,  last  one  done  for  her  D&C  diagnosed  as  cervical  
incompetence  now  pregnant  in  10  week,  vaginal  bleeding  stopped  and  closed  cervical  os.  what  to  
do  next?  
A.  Bed  rest  
B.  Cerclage  (  12-­‐14  weeks.)  
C.  Salicylate  
D.  Admission                                                  
Answer  :  A  
 
95.  Pregnant  in  37  week  come  complain  of  RUQ  pain,  headache  and  blurred  vision,  they  do  
emergency  c/s,  and  MgSO4  was  given,  one  houre  later  she  develop  tachycardia  130  and  BP  
110/74,  what  to  expect?                                      
A.  Hemorrhage  
B.  Mg  toxicity  
C.  Anesthesia  induced  hypotension  
Answer  :    B  
     
96.  Scenario  of  severe  preeclampsia  given  MGSO4  and  reflex  is  +1?  
 magnesium  sulfate  toxicity  
                                     
97.  Pregnant  lady  has    preeclampsia  started  on  mg  sulphate  and  hydralazine  then  RR  become  12  
what  to  give?  
A.  narcan  
B.  atropin                                                                                                                          
C.  ca  gluconate  
D.  naloxone  
Answer  :  C    
https://overdoseinfo.com/magnesium-­‐sulfate-­‐toxicity/  
98.  Primary  PPH  ,  give  the  pt  oxytocin  no  benefit,  what  is  second  step?  
A.  uterine  massage    
B.  ligation  internal  iliac  A  
C.  hysterectomy  
Answer  :  A  .    
 
99.  PPH  with  low  BP  and  tachycardia  the  1st  thing  to  do?  
A.  Ergotamine  
B.  Ringer  lactate  
C.  Normal  saline  with  something  
 
 
100.  Tumor  associated  with  polycystic  ovarian  syndrome?  
A.  Lung  
B.  Breast  
C.  Endometrium  
D.  Ovary            
Answer:  C  
 
101.  A  patient  about  40s  and  had  PCO  since  she  was  young,  now  she  has  vaginal  bleeding.  what  is  
the  most  likely  she  has?  
No  choices    
PCOS  associated  with  endometrial  hyperplasia  and  later  carcinoma  
 
102.  Female  with  PCOS  and  has  very  enlarged  ovaries,  on  examination  how  do  you  differentiate  
between  ascites  and  her  cystic  ovaries?  
A.  Fluid  thrill.  
B.  Dullness  in  the  flanks  and  tymany  in  the  midline.  
C.  Tympany  in  the  flanks  and  dullness  in  the  midline.  
Answre:  B  
 
103.  20  years  old  female  with  hirsutism,  acne,  menstrual  is  regular,  ovaries  US  shows  perl  string  
appearance.  what  is  the  dx:  
A.  Prolactinoma  
B.  PSOS  
C.  hypothyroidism  
Answer  :  B  (  perl  string  appernce  i  think  it  should  be  irregular  typo  )    
                                                                                                                                                           
104.  20  years  old  female  with  hirsutism,  acne,  menstrual  is  irregular,  ovaries  US  shows  perl  string  
appearance.  what  is  the  dx:  
A.  PCOS  
B.  Congenital  adrenal  hyperplasia  
Answer  :A  
 
105.  Girl  with  normal  menstruation  has  ovarian  mass  7  cm:                                                                                                                              
A.  Benign  teratoma  
B.  pco  
C.  something  malignant  
D.  Functional  cysts  
Answer  :D  
 
106.  Female  patient  with  gonorrhea.  What  to  rollout?  
A.  Herpes  
B.  Chlamydia                                                  
C.  Bacterial  vaginosis              
Answer  :  B        
                               
107.  Gonorrhea  commonly  found  in:  
Urethra  
Cervix  
Annswer  :B  
Diagnosis  is  by  testing  the  urine,  urethra  in  males,  or  cervix  in  females  
 
108.  Watery-­‐gray  vaginal  discharge,  fishy  odor,  some  itching  and  discomfort,  the  husband  uses  
condom.  The  most  likely  organism:  
A.  candida  
B.  bacterial  
Answer  :  B  
 
109.  60  YO  female  with  bloody  nipple  discharge,  O/E:  1  cm  mass  subaereolar.  What’s  the  best  
management:  
A.  excision  and  Bx  
B.  mammo  with  FNA.  
Answer  :  B  
110.  postpartum  came  with  discharge  with  other  symptoms.  I  
 
111.  30  years  old  female  with  ….  in  the  vagina  (i  forget),  she  has  high  FSH  and  LH?  
ovarian  failure                                                  
                   
112.  Patient  with  merorraghea  wha  to  give  her?  
Combined  oral  contraception    
 
113.  Inhibition  of  which  of  the  following  is  the  primary  action  of  oral  contraceptives?  
A.  Decrease  estrogen  to  prevent  the  ovulation  
B.  Decrease  GTRH  spur  at  the  mid  cycle  
C.  Increase  prolactin  
D.  Suppressing  the  release  of  gonadotropins  
E.  Spermatozoa  and  thickening  cervical  mucusa  
Answer  :  D  
 
114.  How    is  the  MOA  for  emergency  contraceptives?  
A.  Prevents  fertilization  
B.  Prevents  implantation  
C.  Prevents  anovulation  
D.  Delays  fertilization    
Answer:  B  
 
115.  A  patient  come  after  one  day  of  intercorse  afraid  to  get  pregnant.  What  to  give  her?  
emergency  contraceptives  
117.  How  ectopic  pregnancy  occurs  at  the  cellular  level?  
A.  Early  abscess  of  zona  pellucida  
B.  Persistence  of  zona  pellucida  
Answer:  A  (  i  think  they  mean  disappearance  =absent  )    
 
118.  Patient  with  constipation,  weight  loss  and  pelvic  mass.  What  will  help  in  reaching  the  Dx?  
A.  Pap  smear  
B.  Colposcopy  
Missing  information  wide  differential    
Ovarian  cancer  :  CA-­‐125  blood  test.  TVUS    
Symptoms  suggestive  of  cervical  cancer  :  colposcopy  .    
https://www.cancer.org/cancer/cervical-­‐cancer/detection-­‐diagnosis-­‐staging/how-­‐diagnosed.html  
 
119.  When  to  consider  prolonged  pregnancy?  
A.  40  weeks  
B.  41  weeks  
C.  42  weeks  
D.  43  weeks  
 Anwer  :  C  
120.  OCP  by  increase:  
A.  Estrogen    
B.  Progesterone    
C.  FSH    
D.  LH  
Answer  :  B  
 
121.  56  yrs  old  woman  with  uncontrolled  urination  upon  coughing,  sneezing  or  laughing,  Best  Mx  
measure  for  her  is:  
Kegel  Exercise  
 
122.  what  is  true  about  Pap  smear?  
A.  21  till  65  yrs  repeated  every  year  
B.  21  till  65  yrs  repeated  every  5  yrs  
C.  21  till  35  repeated  every  3  years  
D.  30  till  65  repeated  every  5  yrs    
Answer  :  D  every  5  year  if  (  pap+  HPV)  
 
123.  Female  has  genital  warts  over  the  past  years,  genital  warts  are  associated  with  what  ?  
hyperkeratosis    
 
124.  A  woman  in  labour,  everything  is  fine,  she  asked  for  Epidural  analgesia,  what  test  should  be  
requested?  
A.  Hb  
B.  PLT  count  
Answer  :  B  
 
125.  Best  way  to  diagnose  bacterial  vaginosis?  
gram  stain    
 
126.  23  yrs  old  female,  with  burning  upon  voiding  urine,  she  goes  frequently  to  bathroom  but  with  
little  amount  of  urine  voided,  she  does  not  have  suprapubic  tenderness  OE,  she  had  also  
hematuria.  What  to  do?  
A.  Urinalysis  and  culture  
B.  testing  for  Gonorrhea/chlamydia  
C.  Pylocyctogram  
Answer  :  unclear  question  but  usually  start  simple  and  noninvasive  to  diagnose  or  rule  out  most  
common  disease  such  as  urinalysis  and  culture  
 
127.  most  common  type  of  fibroid  
A.  submucous  
B.  intramural  
C.  subserous  
Answer:  B  
 
128.  multiparous  woman  term,  came  with  Progressive  contractions  and  cervical  dilatation,  
mentioned  that  her  membranes  ruptured  and  fluid  was  clear(    details  of  examination  plus"  uterine  
tenderness")    correct  action?  
Multiple  choices  and  one  of  them  was  give  intrapartum  abX  ✔  
Because  this  is  a  case  of  chorioamnionitis  with  possible  GBS  infection  so  treated  e  intrapartum  
antibiotics  
 
129  .40    yrs  old  multiparous  with  hx  of  CSCame  e  vaginal  bleeding,  which  was  painless,  she  is  38  
weeks,  US  examination  showed  low  lying  placenta  ,  which  of  the  following  would  make  the  doctor  
decide  CS  with  hysterectomy?  
A  .age  of  pt  
B  .Previous  CS  
C  .placenta  accreta    
D  .multiparity      
answer:C  
 
130  .A  woman  e  Ca  cervix  ,e  mets  beyond  uterus  and  cervix,  1st  group  of  LN  receiving  the  lymphatic  
drainage  is:  
A  .uterine  
B  .external  iliac  
C  .common  iliac  
D  .para  aortic  
Answer  :  B  
 
131.  Pregnant  lady  on  her  24th  week  of  gestation  with  a  significant  medical  history  of  DM  II  on  
insulin  with  nephritis,  chronic  HTN  controlled  on  medications.  O/E  her  fundal  height  was  25  cm,  
otherwise  unremarkable  PE.  What  complication  is  more  likely?    
A)  Pre-­‐Eclampsia.  
B)  Shoulder  dystocia.                                                                                                                                      
C)  Stillbirth.  
D)  Large-­‐for-­‐Gestational-­‐Age  infant.  
http://emedicine.medscape.com/article/1476919-­‐overview  
DM  can  increase  the  risk  of  preeclampsia  especially  with  nephritis  
Answer  :A  
 
132.  Preeclampsia  case  what  is  the  first  step  ?  
A.  Magnesium  sulphate  
B.  IV  hydralazine  
C.  Methyldopa  
Answer  :  sever/end  organ  damage  A    
 
133.  A  female  with  polycystic  ovarian  syndrome  noticed  hyperpigmented  skin  in  her  neck  and  
axilla,  what  is  this  abnormality  called?  
a.  Acanthosis  nigricans  
b.  Linea  nigra  
A  
 
134.  Female  patient  pregnant  present  with  abdominal  pain  and  vomiting.  Vital  sign  normal  except  
blood  pressure  160/100.  What  is  the  diagnosis?  
A.  Renal  failure.  
B.  Fetal  distress.  
(  no  preeclampsia  in  the  choices  )  sever  pre-­‐eclamsia  160/110    
 
135.  When  you'll  do  Alpha  fetoprotein  ?  (16-­‐18)    
A.  13  
B.  15  
C.  17  
D.  19  
Answer:  C  
 
136.  women  with  DUB  &  Endometrium  biopsy  shows:  endometrial  hyperplasia  This  caused  by?  
a)  Adrenal  hyperplasia  
b)  Liver  dysfunction                                                                                                          
c)  Local  genetic  mutation  in  the  endometrium                            
d)  Peripheral  percoser  converted  to  estrogen  
Answer  :  C  
 
137.  patient  with  high  grade  hyperplasia  with  atypia  and  total  hysterectomy?  
(I  do  not  know  what  is  the  question)    
 
138.  Patient  had  fibroid  3*4  cm  take  tamox  after  1  years  come  with  vaginal  bleeding  US  show  
fibroid  6cm  with  endometrial  hyperplasia  what  is  the  cause  of  bleeding?  
A.  Fibroid  
B.  Endometrial  hyperplasia  
Answer  :B  
 
139.  28  years  old  diabetic  female  DM1  controlled  by  insulin  she  is  married  and  wants  to  become  
pregnant.her  blood  glucose  is  well  controlled  and  she  is  asking  about  when  she  must  control  her  
metabolic  state  to  decrease  risk  of  having  congenital  anomalies:  
A.  before  conception  
B.  1st  trimester  
C.  2nd  trimester  
D.  3rd  trimester  
Answer  :  A  
140.  First  trimester  .with  H/O  GDM  in  last  pregnancy  3  yrs  back.  When  you  do  GTT?  
16  weeks  
   
If  there  is  a  risk  do  it  in  the  first  trimester  (less  or  equal  16)  
All  pregnant  24-­‐28  Weeks    
 
141.  Case  of  GDM  risk  in  future  develop  DM2?  
 
142.  Long  scenario  of  pregnant  women  admitted  due  to  hypertension,  proteinuria,  lower  limb  
oedema  fundal  level  not  corresponding  to  gestational  age.  What  the  risk  to  baby  to  develop  
IUGR???  
A.  elevated  maternal  liver  enzymes  (  ??  placental  insufficiency)    ?  
B.  oligohydramnios  
C.  polyhydromenous  
D.  GDM  
Answer  :B  (  I  asekd  dr.reem  alanzi    about  this  question  )    
SMLE  12  Page  437  Q  97  
 
143.  45  years  old  Female,  on  estrogen.  Complain  of  dysuria  for  the  last  year  frequency  low  steam  
void,  she  had  recurrent  UTI,  take  multiple  antibiotic  without  benefits,  Examination  shows  normal  
vagina/vulva/cervix?  and  tenderness  at  neck  of  bladder.  what  is  the  diagnosis?  
A.  Traumatic  urethritis  
B.  Interstitial  urethritis      
C.  DM  
D.  Candida  
Answer  :  B  
 
144.  A  wife  of  a  man  diagnosed  as  having  gonorrheal  infection  she  was  worried  about  
transmission  of  the  infection  from  her  husband  what  is  the  best  investigation  to  exclude  
gonorrhea  infection?    
Gram  stain  
 
145.  Lady  in  labor,  you  can  feel  nasal  bridge  and  orbital  ridge.  Which  one  compatible  with  vaginal  
delivery?  
A.  Mentoanterior  
B.  Mentoposterior  
C.  Transverse  mentoanterior  
answer:A  
 
146.  contraindications  for  breastfeeding?  
A.  Active  HCV.  (  IF  WITH  CRACKED  NIPPLE  )    
B.  Active  HIV.  
C.  Varicella  zoster    
Answer  :  b    
147.  disease  increases  the  mortality  in  pregnant  female?  
Pheochromocytoma  
Peripartum  cardiomyopathy  develops  in  the  last  weeks  of  pregnancy  until  6  weeks  postpartum  (  IF  IT  
WAS  one  of  the  option  )    
 
148.  Pregnant,  36  weeks,  present  with  agitation,  BP:  88/60,  fetal  distress,  what  is  the  diagnosis?  
A.  Pulmonary  embolism.  
B.  Amniotic  fluid  embolism  
C.  Abruption  (  BV  bleeding+  pain  )    
Missing  information  mostly  A  not  sure    .    
 
 
149.  Female  pregnant  with  small  fibroid  what  to  tell  her  about  the  risk:  
A.  Risk  of  preterm  labor  
B.  Degeneration  is  common    
C.  Mostly  asymptomatic  
Answer:  C  
 
150.  34  years  old  woman  ,Multiparous  ,  complain  of  heavy  menstruation  ,  upon  examination  :  
there  is  intramural  fibroid  
 
151.  Picture  of  CTG,  what's  the  dx?  
 
152.  Breastfeeding  mother  (they  gave  history  of  mastitis  and  was  treated  with  antibiotics)  after  3  
weeks,  lump  occurred  fluctuating,  not  tender,  what  is  the  most  likely  diagnosis?  
Breast  abscess  
https://emedicine.medscape.com/article/781116-­‐
clinical?pa=Rwlpdtd60tTvbZzGkjszR4uzFrShLqaKyXFvSgKPRiFt%2BR95nHSjnV9%2FkrGRFHQFfbcwf64
RVLoPX%2FSLPtDaPfEiL5fM42L%2B9xlMlua7G1g%3D  
 
153.  Pregnant  women  with  placenta  previa  what  is  the  risk  factor:  
A.  race  
B.  age    
C.  previous  placenta  previa  
Answer  :  C    
 
154.  Para2  woman  planned  with  her  husband  to  avoid  pregnancy  during  next  3  yrs  she  doesn’t  like  
to  use  IUDS  nor  OCP  What  should  you  tell  her  about  transdermal  contraceptive  
A.  It’s  more  likely  to  form  clots  more  than  OCP  
B.  It’s  easy  to  forget  changing  it  
C.  less  effective  than  IUD/OCP  
Answer  :  A  
 
155.  young  couples  trying  to  conceive  for  6  months,  both  are  healthy  
try  some  more.  
 
156.  Women  at  40  week  gestation  suddenly  develope  dyspnea  with  hypoxia  and  chest  pain:  
A.  Amniotic  fluid  embolism  
B.  Pulmonary  embolism    
C.  Myocardial  infarction    
Answer  :  B  
 
157.  Female  g3p2  20  week  ask  you  about  cancer  happen  during  pregnancy,  2  of  his  sister  develop  
cancer  during  pregnancy  and  died,  origin  of  cancer  
A.  Ovary  
B.  Breast  
C.  Cervix  
D.  Vulvar  
Answer:  B  
 
158.  Most  common  malpresentation:  
A.  Braw  
B.  Breech*  
C.  Face  
D.  Transverse  
Answer  :  B  
 
159.  case  scenario  about  female  with  past  hx  of  PROM  and  now  she  is  pregnant  asymptomatic  and  
ask  about  indication  of  screening  for  bacterial  vaginosis:  
A.  no  indication  
B.  screen  at  first  trimester  
C.  screen  at  second  trimester  
D.  screen  at  third  trimester.  
answer:A  
 
 
160.  MAFP  measure  in  which  week:  
A.  13  weeks  
B.  15  weeks  
C.  17  weeks*  
Answer  :  C  
15–18  weeks  
Not  sure  about  the  correct  answer  what  I  found  is  15-­‐18  week    
 
161.  Female  patient  with  hX  of  multiple  abortion  and  D&C    want  to  get  pregnant  what  is  the  
diagnosis:  
Asherman  syndrome  (  corrrect  )    
 
162.  Patient  with  two  time  ASCUS  pap  smear  what  do  next:  
Colposcopy  
 
163.  42  years  old  female  with  hx  of  hot  flushing  and  night  sweating  and  amenorrhea  what  the  
diagnosis?  
A.  Hypothyroidism  
B.  Congenital  adrenal  hyperplasia  
C.  Hyperprolactinemia    
Correct  :  C    
164.  Pregnant  women  during  labor  ,  Iv  oxytocin  was  given  the  CTG  show  variable  and  acceleration  
what  will  you  do?  
A.  Stop  oxytocin    
B.  Expectant  delivery  
C.  Change  mother  position  
Answer:b  
   
165.  Giving  birth  ,  cervix  6  cm  dilated  ,  fetus  is  left  occiput  posterior  ,  sign  of  molding  can  be  felt  
what  the  stage?  
First  (  correct  )    
   
166.  Daily  requirement  of  iron  in  pregnancy?  
14.7  mg/day  (  requirement  of  iron  )  
with  30  mg  to  60  mg  of  elemental  iron  (  dose)    
http://www.who.int/elena/titles/guidance_summaries/daily_iron_pregnancy/en/  
 
167.  On  examination:  her  cervix  is  dilated  by  3  cm  and  effaced  by  70%  and  fetal  presenting  part  at  
0  station.  After  6  hours  or  so,  her  cervix  is  dilated  by  7  cm  and  effaced  by  80%,  but  she  is  
complaining  of  tenderness  when  palpated  the  uterus,  her  temperature  is  38.  
what  is  the  best  management?  
A.  Give  intrapartum  antibiotics.  
B.  Emergency  cs  
C.  IV  antibiotics  ƒ  ampicillin  (2  g  IV  q6h)  and  gentamicin  (1.5  mg/kg  q8h)  ƒ  anaerobic  coverage  (i.e.  
clindamycin  if  C/S)  
answer:A  
https://www.uptodate.com/contents/group-­‐b-­‐streptococcus-­‐and-­‐pregnancy-­‐beyond-­‐the-­‐basics  
 
•  expedient  delivery  regardless  of  gestational  age  torronto  notes    
‫ ﻣﺪﺭرﻱي ﻭوﺵش َﺫذﺍا ﻳﯾﻤﻜﻦ ﺗﺴﺘﻔﻴﯿﺪﻭوﻥن ﻣﻨﻪﮫ ﺍاﺫذﺍا ﻣﺎ ﺍاﺳﺘﻔﺪﺗﻮﺍا ﺍاﻣﺴﺤﻮﻩه‬ :‫ ﺍاﺭرﻭوﻯى‬ ^    
 
168.  67  years  female  on  regular  follow  up  for  20  year  for  pap  smear  which  is  negative  regarding  
check  up  
A.  every  6months  
B.  every  year  
C.  every  3years  
D.  no  further  check  up  
answeR:  D  
 
169.  pt  hysterectomy  done  co  of  mild  pain  in  incision  site.  Hb  normal,  Temperature  37.5,  HTC  
normal,  Bp  110/70.  What  cause  to  be  still  in  hospital??  
A.  low  Hb  
B.  high  tempreture  
C.  high  HTC  
D.  because  not  oral  feeding  
Normal  temp  :  37ºC  (98.6ºF),  but  anywhere  between  36.5ºC  and  37.2ºC    
 Postpartum  fever  is  defined  as  a  temperature  of  38.7  degrees  C  (101.6  degrees  F)  or  greater  for  the  
first  24  hours  or  greater  than  38.0  degrees  C  (100.4  degrees  F)  on  any  two  of  the  first  10  days  
postpartum.  
I  asked  a  fellow  her  answer  was  B  But  I  not  convinced  by  her  answer  I  tried  to  look  for  criteria  of  
discharge  did  not  find  anything  useful  I  think  the  answer  is  D(  Dana  )  
 
170.  50  years  old  complain  of  2  months  of  amenorrhea  what  u  will  found  
A.  Increase  FSH,increase  LH  
B.  Decrease  FSH,decrease  LH  
C.  Increase  FSH,decrease  LH  
Answer  :A  
 
171.  Which  one  of  the  Contraceptive  drug  cause  hirsutism  “androgenic”  side  effects?    
progesterone  only  

 
172.  Female  with  developed  secondary  sex  character  sparing  axillary  and  pubic  hair  in  
investigation  there  was  high  testosterone,dx?  
Androgen  insensitivity  syndrome  
 
173.  Women  underwent  bilateral  salpingoophrectomy  and  hysterectomy,  she  developed  hot  
flushes.  You  want  to  give  her  Hormonal  replacement  therapy.    
What  will  you  give?    
A.  Cyclic  estrogen  and  progesterone    
B.  Continues  estrogen  and  progesterone    
C.  Transdermal  estrogen  patches    
D.  Levonorgestrel  IUD    
Answer  :c  (  NO  need  for  protective  effect  of  progestron  since  she  had  hysterctomy  )  but  estrogen  
patches  is  no  longer  giving  due  risk  of  DVT  and  high  estrogen.  Confusing  question  it’s  eather  A  or  C
   
 
174.  Hormone  replacement  therapy  increase  risk  of    
A.  Endometrial  cancer  
B.  Breast  cancer  
C.  Ovarian  cancer  
Answer  :  B  
 
175.  40  years  old  Patient  she  has  secondary  dysmenorrhea  and  heavy  bleeding  they  decided  to  do  
hysterectomy,  diagnosis  was  confirmed  with  a  histopathological  exam  of  the  tissue  of  
hysterectomy,  which  of  the  following  is  the  best  non  invasive  investigation  to  support  the  
suspected  diagnosis.  
A.  office  endometrial  biopsy    
B.  pelvic  MRI  
C.  pelvic  CT  
D.  pelvic  US  
Adenomyosis  :  answer  B  
 
176.  what  is  the  best  non-­‐invasive  method  to  diagnose  Adenomyosis  
A.  Office  sampling    
B.  Ultrasound    
C.  MRI  
Answer  :  MRI  (more  sensitive  and  specific)                                                                                                                                              
A  definitive  diagnosis  of  adenomyosis  can  only  be  made  from  histological  examination  of  a  
hysterectomy  specimen.  The  preoperative  diagnosis  is  suggested  by  characteristic  clinical  
manifestations  (ie,  menorrhagia  and  dysmenorrhea  with  a  uniformly  enlarged  uterus)  in  the  absence  
of  endometriosis  or  leiomyomas.                                                                                                                                    
Both  transvaginal  ultrasound  (TVUS)  and  magnetic  resonance  imaging  (MRI),  especially  T2-­‐weighted  
images,  are  increasingly  used  for  clinical  decision-­‐making.  
 
 
178.  Most  common  cause  to  perform  hysterectomy?    
Uterine  fibroid    
 
179.  Pap  smear  of    ASC-­‐US,  what  to  do  next?  
A.  Reassure    
B.  Hysterectomy    
C.  Repeat      
D.  Cone  biopsy    
Answer  :  b  (  if  21-­‐30  year:  repaat  )  if  >30  year  do  HPV    
 
180.  Pregnant  prolonged  labour  for  12hrs  asthmatic,  mitral  stenosis  what  is  the  indication  for  
forceps  delivery?  
A.  Asthma  
B.  Prolonged  labour  
C.  Mitral  stenosis  
D.  I  forgot  it  
Answer  :  C  (  either  prolonged  2nd  stage  or  maternal  cardiac  condition  ,  need  more  details  about  
which  stage  of  labor  )    
https://www.uptodate.com/contents/operative-­‐vaginal-­‐delivery  
 
181.  Pregnant  lady  on  20  weeks  has  active  herpes  what  you  gonna  do?                
A.  Give  acyclovir  
B.  Wait  until  2  week  and  C.S  election  gve  before  delivery.  
Answer  :A  
 
182.  Pregnant  with  UTI  what  is  the  complications  that  might  happen?  
A.  Low  birth  weight  
B.  Preterm  (  BV  )    
C.  Acute  pyelonephritis  
ANSWER  :  c    
183.  Elderly  women  with  vulvovaginal  lichen  planus  lesion  in  posterior  vaginal  fornix,  this  patient  
is  at  risk  to  develop  which  type  of  the  following  vaginal  cancer:  
A.  Squamous  cell  carcinoma    
B.  Adenocarcinoma  
C.  Squamous  adenocarcinoma  
Answer:  A  
 
184.  How  to  screen  for  thalassemia  in  pregnant  women:  
A.  US  on  12  weeks  
B.  Chorionic  villus  sampling  at  16  weeks    
C.  Triple  test  at  16  weeks  
D.  quadruple  test  at  15  weeks  
Note:  it’s  by  mniocentesis  –  this  is  done  from  15  weeks  of  pregnancy.  I  THINK  TYPO    
https://www.nhs.uk/Conditions/pregnancy-­‐and-­‐baby/Pages/screening-­‐sickle-­‐cell-­‐thalassaemia-­‐
pregnant.aspx  
 
185.  40  yrs  old  pregnant,  history  of  dead  upon  delivery  baby,  is  thinking  of  getting  pregnant;  &  
worries  that  the  same  will  happen  again  ,what  will  you  tell  her:  
A.  Sampling  of  amniotic  fluids  @  (  X  of  month)  
B.  Has  the  same  risk  as  the  rest.  
C.  Higher  risk  because  of  her  age  
D.  US  @  X  of  months  
Depend  on  number  A  should  be  more  than  16  weeks    
 
 
186.  OCP  side  effect  dry  skin  muscle  spasm,  deprration  
A.  Hypothyroidism  
B.  Hyperkalemia  
C.  Hypocalcemia  
ANSWER:  b  
187.  Ovarian  cancer  tumor  marker  ?  
A.  CA125  
B.  AFP  
C.  PS  
ANSWER  :  A  
 
188.  18  year  old  girl  with  primary  amenorrhea  Normal  pubic  hair  and  breast  development,  
Absence  of  uterus:  
A.  Klinefelter  syndrome  
B.  Mallrune  agencia  
C.  Turner  syndrome  
Answer  :B  
 
189.  Pregnant  at  8  weeks  and  had  previous  baby  with  down  syndrome,  she  wants  to  screen  for  
down  in  this  pregnancy,  you  booked  her  after  2  weeks  for  the  test.  What  should  you  write  in  the  
consent  form  as  a  complication  for  this  test?  
A.  Risk  of  miscarriage.  (my  answer  b/c  CVS  can  cause  miscarriage)  
B.  Rupture  amniotic.  
answeREr:  A  
 
190.  Case  of  premenstrual  symptoms,  what  to  give?  
 fluoxetine(  ssri  )    
 
191.  Ovarian  mass  measuring  5  cm  not  affecting  menstrual  cycle,  what  dx?  
A.  Follicular  cyst  (  simple  cyst)    
B.  Theca  lutein  (  lear,  straw-­‐colored  fluid)    
C.  Something  carcinoma  
NOT  SURE  !    
 
192.  What  is  the  most  common  birth  injury?  
A.  Clavicular  fracture  
B.  Shoulder  dislocation  
C.  Hip  dislocation  
D.  Femur  fracture  
Answer  :  A    
http://www.birthinjuryguide.org/birth-­‐injury/types/infant-­‐broken-­‐bones/  
 
 
193.  Case  of  G2P1  +0  (not  sure)  and  presented  with  lower  abdominal  pain  that  comes  and  goes  for  
10-­‐15  min  more  in  the  right  iliac  fossa.  Pregnancy  test  was  negative  (although  she  is  G2P1+0).  
What  is  the  diagnosis?  
A.  Acute  appendicitis  
B.  Ectopic  pregnancy  
ANSWER  :  A  ??  
 
194.  Menorrhagia  define  as?  How  much?  
Menorrhagia  is  defined  as  excessive  uterine  bleeding  occurring  at  regular  intervals  or  prolonged  
uterine  bleeding  lasting  more  than  seven  days.  Classically  more  than  80  ml/cycle.  
 
195.  Patient  with  elevated  CA125  levels,  what  is  she  in  risk  for?  
A.  Ovarian  germ  cell  tumor  
B.  Sex  cord  ovarian  tumor  
C.  Epithelial  ovarian  tumor  
Answer:  C  
 
196.  Patient  with  ovarian  mass.  What  is  the  best  screening  test:  
A.  Ultrasound  
B.  CA  125  
Answer:  B  
 
197.  Pregnant  lady  has  hypothyroidism,  how  much  do  you  have  to  increase  her  dose?  
A.  10%  
B.  20%  
C.  30%  
D.  40%  
Answer:  25-­‐50%  
 
198.  Amniocentesis  is  best  done  at  what  gestational  age:  
A.  10  Weeks  
B.  12  Week  
C.  14  Week  
D.  16  Week  
Answer:  D  
 
199.30  weeks  gestation  came  with  Bp=  162/95  urine  protein  +2.  Which  one  of  the  following  you're  
going  to  give?  
A.  Dexamethasone  
B.  Magnesium  sulphate  
C.  Methyldopa  
D.  Labetalol  
Answer:  B  
 
200.  Fetus  with  single  umbilical  artery.  What  could  be  the  cause  ?  
Answer:  The  occurrence  of  a  single  umbilical  artery  is  thought  to  be  due  to  secondary  atresia  or  
atrophy  rather  than  primary  agenesis  of  the  artery.  the  absence  of  the  left  umbilical  artery  is  much  
more  common  (~70%).  
 
201.  postmenopausal  women  taking  estrogen  and  progesterone,  Increase  risk  of  which  cancer?  
Answer:  Breast  cancer  
 
202.  What  is  the  preferred  pelvis  shape  for  vaginal  delivery?  
A.  Gynecoid    
B.  Android    
C.  Platypelloid    
D.  Anthropoid    
Answer:  A    
 
203.  Hymen  penile  penetration:    
Answer:  6  o’clock.  
Note:  The  question  is  not  clear  
 
204.  Pregnant  lady  with  history  of  2  SvD  with  normal  babies  of  3  kg.  The  baby  is  breech  and  the  
head  is  flexed  she  found  to  have  bicornuate  uterus  .  And  the  baby  weight  is  2kg  .  What  is  the  
contraindication  for  external  cephalic  version  ?  
A.  Baby  weight  
B.  Flexion  of  the  head    
C.  Bicornuate  uterus.    
D.  Hyperextended  head.  
Answer:  C  
 
205.  Pregnant  with  genital  warts  what  to  do?  
Answer:  Vaginal  delivery    
Source:  https://www.cdc.gov/std/treatment/2010/genital-­‐warts.htm  
 
206.  Placenta  previa  and  adherent  what  is  the  noninvasive  test  to  do?  
A.  MRI  
B.  Transvaginal  US  
C.  Doppler  ultrasound  
Answer:  B  
Explanation:  MRI  is  too  expensive    
 
207.  patient  in  postpartum  hemorrhage,  you  want  to  give  methylergonovine.  what's  the  relative  
contraindication  to  use  it:                                                                                    
A.  Asthma    
B.  Diabetes  
C.  Hypertension                          
Answer:  C                                                                                                          
 
208.  Most  common  way  of  HIV  vertical  transmission  and  …  ?  
A.  Placenta    
B.  Breastfeeding  
C.  Umbilical  cord  blood  contamination  
Answer:  A  
Explanation:  Exposure  through  pregnancy,  birth,  or  breastfeeding  —  Vertical  transmission  of  HIV  can  
occur  at  any  time  during  gestation  and  delivery,  and  through  breast  milk  in  the  postpartum  period.  
More  than  95  percent  of  HIV-­‐infected  children  worldwide  have  acquired  the  virus  via  vertical  
transmission.  The  rate  of  vertical  transmission  of  HIV  varies  from  approximately  20  to  30  percent  in  
the  absence  of  antiretroviral  therapy.  Although  it  is  known  that  HIV  can  be  transmitted  early  in  
gestation  in  utero,  most  transmissions  (50  to  80  percent)  are  believed  to  occur  during  the  time  
period  near  or  during  delivery  .  
reference:  uptodate  
 
209.  Mother  had  abortion  two  times  last  child  with  45X  monosomy,  recurrence  in  next  pregnancy?  
A.  40%    
B.  60%  
C.  70%  
D.  90%  
Answer:  Recurrence  of  TS  is  observed  in  1.4%,  which  represents  a  35-­‐fold  increased  probability  of  
having  a  second  child  with  TS  compared  to  general  population.    
Source:  https://www.ncbi.nlm.nih.gov/pubmed/21648298  
 
210.  Painless  Genital  ulcer  plus  inguinal  lymphadenopathy:  
Answer:  Syphilis    
 
211.  A  patient  with  painless  Genital  ulcer,  how  to  diagnose?  
Answer:  Dark  field  microscopy  
 
212.  Middle  age  women  trying  to  conceive  last  born  1  year  all  normal  what  you  check.    
Answer:  FSH?  
Note:  The  question  is  not  complete  
 
213.  Female  want  to  conceive  after  2  yrs,  what  best  ocs  to  give?  
 
214.  Young  lady  with  previous  hx  of  chlamydial  infection  tried  to  conceive  for  previous  14  months  
but  she  can't,  her  husband  has  normal  semen  analysis  and  she  didn't  have  any  hormonal  diseases,  
the  next  investigation  should  be:  
A)   MRI  
B)   US    
C)   Hysterosalpingography  
Answer:  C  
 
215.  40  year  old  with  heavy  bleeding  everything  excluded.  Even  D&R  normal.  Cause  of  bleeding?  
Answer:  Anovulatory  cycles.  
 
216.  Girl  18  years  come  with  no  menstrual  cycle  2  month  ago  she  is  not  sexually  active  and  on  
examination  she  feel  tenderness  on  abdominal  what  you  will  do  ?  
A.  Ultrasonography                                                                                  
B.  Do  pregnancy  test  
Answer:  B  
Explanation:  The  initial  laboratory  evaluation  (after  ruling  out  pregnancy)  for  women  with  secondary  
amenorrhea  should  include  follicle-­‐stimulating  hormone  (FSH),  serum  PRL,  and  thyroid-­‐stimulating  
hormone  (TSH)  to  test  for  POI,  hyperprolactinemia,  and  thyroid  disease,  respectively.  
 
217.  In  menopausal  lady,  diagnostic  test  
Answer:  FSH  
 
218.  Type  of  postpartum  hemorrhage  in  atony  of  uterus?    
Answer:  Primary  
   
219.  Postpartum  bleeding  gave  IVF  and  add  what?  
A.  Oxytocin  
B.  Ergotamine  
C.  Carbetocin                                                                                                          
Answer:  A  
 
220.  Regarding  preeclampsia,  first  symptom/  sign?  
Answer:  BP  ≥140  mmHg  systolic  and/or  ≥90  and  proteinuria      
 
221.  Pregnant  in  30  gestational  weeks,  pressure  was  high  unlike  previous  visites  was  normal.  No  
visual  disturbances  no  protein  in  the  urine.  What  is  the  diagnosis?  
A.  Gastinal  hypertension  
B.  Preeclampsia  
C.  Eclampsia  
D.  Superimposed  hypertension  
Answer:  A  
 
222.  Scenario  of  patient  with  preeclampsia  then  ask  what  is  the  most  important  test  should  be  
done  
A.  blood  pressure  monitoring  and  respiratory  rate  
B.  LFT  and  platelet          
C.  urinalysis  and  proteinuria  
D.  creatinine  and  BUN    
http://emedicine.medscape.com/article/1476919-­‐overview#a14  
Answer:  C  
Explanation:  since  the  scenario  didn’t  make  a  diagnosis  of  preeclampsia  there  for  urine  for  protein  is  
mandatory  in  addition  that  24  h  protein  in  urine  is  determine  the  severity  of  the  disease  
 
223.  Pregnant  Patient  with  herpes  what  is  best  management  ?  
Answer:  if  active  disease  on  40  weeks  gestation  delivers  CS  
     
224.  Best  dx  tool  for  female  with  irregular  massive  bleeding?  
Answer:  The  question  is  not  clear.  The  first  to  do  are  pregnancy  test  and  CBC.  The  history  will  direst  
the  other  test.    
 
225.  22  years  old  irregular  menses  w  abdominal  pain  started  7-­‐8  after  menstruation  ,  her  menses  
7-­‐9  days  ,  4-­‐5  intercourse  /week  protected  by  condom  ,  she  has  high  hygiene  with  Vaginal  douche  
monthly  after  menses  what  is  the  cause  of  symptom?                                                                                                                            
A.  Number  of  intercourse                                                                                                                            
B.  Vaginal  douche                        
C.  Condom  
Answer:  B?  
https://www.ncbi.nlm.nih.gov/pmc/articles/PMC2567125/  
 
226.  A  question  about  Tanner  stage  
Answer:  please  google  Tanner  scale  
 
227.  Pt  in  labour  6  cm  dilation,  fetal  CTG  acceleration  and  variable.  Next  step?  
Answer:  Question  is  not  complete,  probably  change  the  maternal  position.  
 
228.  Rupture  of  condom,  what  to  use  ?  
a)   Contraceptive  
b)   Douching    
c)   Tell  her  to  leave    
Answer:  A  
 
229.  34  period  day,  when  will  she  start  ovulating  ?  
Answer:  21  
   
230.  Multipara,  last  delivery  was  assessed  by  vacuum,  presented  with  bulging  mass  from  vagina.  
What  to  do  next:  
A.  Speculum  vaginal  examination  
B.  Retrograde  cystourethrogram  
Answer:  A  
 
231.  At  which  age  has  the  highest  risk  associated  with  chromosomal  abnormality:  
A.  >  30  years  
B.  >  35  years  
C.  >  40  years  
D.  >  45  years  
Answer:  D  
   
232.  Which  of  the  following  is  an  independent  risk  factor  that  will  increase  the  risk  of  infant  
getting  an  early  GBS  infection?  
A.  Intrapartum  fever  >39  
B.  Family  history  of  infant  with  GBS  
C.  Premature  rupture  of  membranes  for  more  than  18  hours  
 Answer:  C  
 
234.  Female  pt  with  symptoms  of  menopause  and  symptoms  of  depression  ,  what's  the  
treatment?  
A.  Estrogen  
B.  Progesterone  
C.  Paroxitine  
Answer:  For  post  menopausal  mood  lability/depression  we  can  use  HRT  with  or  without  SSRIs.  The  
best  choice  for  this  woman  is  combined  estrogen  and  progesterone.      
   
235.  34  wk  +  open  cervix  +  bulging  membrane  +  fetal  HR  ..  (forget  no.)  what  is  your  1st  
management  ?  
Answer:  question  is  not  complete,  is  the  fetus  in  distress?  
   
236.  What  is  true  about  liquid  based  cytology  for  Pap  smear:  
A.  The  brush  is  used  to  sample  the  endocervical  canal  
B.  The  sample  is  taken  from  the  transformation  zone  
C.  The  sample  is  taken  by  a  spatula  and  a  brush  
Answer:  A  
     
237.  A  woman  with  Cervical  cancer,  mets  beyond  uterus  and  cervix,  1st  group  of  LN  receiving  the  
lymphatic  drainage  is:  
A.  Uterine  
B.  External  iliac  
C.  Common  iliac    
D.  Para  aortic  
Answer:  A  
   
238.  For  who  should  we  measure  BP  for  24  hours  
A.  Confirm  compliance  of  medication  
B.  Elderly  with  palpitation  
C.  Resistant  HTN  on  medication  
D.  Suspected  preeclampsia  
Answer:  D?  
   
240.  The  cause  of  maternal  mortality  if  happened  during  pregnancy  ?  
A.  Syphilis    
B.  Toxoplasmosis    
C.  Biliary  cholestasis  
Answer:  ?  
   
241.  30  years  old  female  present  to  you  in  the  clinic  after  ROM  before  one  hour  which  was  clear  
fluid  P/E:  no  vesicles  in  the  vagina  medical  report  documents  as  pt  has  hx  of  recurrent  HSV  what  is  
your  management?  
A.  Proceed  to  C/S"  
B.  IV  acyclovir                                                                                                                                                          
Answer:  Perform  a  sterile  speculum  ex,  then  if  positive  PROCEED  TO  CS.  If  negative  SS  EX,  it's  
controversial..  either  SVD  or  CS.  
If  pt  is  not  in  labour  and  EX  is  normal,  but  with  known  hx  of  HSV,  obtain  a  weekly  cervical  culture.  
If  pt  is  presenting  with  active  lesions  but  not  in  labour,  give  IV  acyclovir.                                                                                                                                                
                                                                                                                                                                                                                                                                                                                           
242.  A  32  week  pregnant  lady  .  By  ur  exam  u  found  her  to  be  off  date.  What  serial  clinical  
assessment  u  will  do  over  the  next  2  weeks  ?  
A.  Maternal  weight  
B.  Cardiographs  
...  I  forgot  the  others  
                                                                                                                                                           
Answer:  The  question  is  not  complete.  The  3  basic  methods  used  to  help  estimate  gestational  age  
(GA)  are  menstrual  history,  clinical  examination,  and  ultrasonography.  
                                                                                                                                                           
243.  monochorionic  twins  in  27  week,  one  of  them  died,  wt  to  do  ?  
A.  wait  for  spontaneous  delivery  at  any  gestation    
B.  wait  till  34  week  and  deliver  
C.  give  steroid  and  deliver    
Answer:?  
                                                                                                                                                                                           
244.  Best  investigation  in  1st  trimester  
A.  CBC  
B.  Ultrasound      
Answer:  Question  is  not  clear  but  probably  CBC.  
245.  Women  is  bleeding  after  delivery,  and  can't  stop,  wt  to  do  immediately?  
A.  Hysterectomy.  
B.  Something  to  apply  pressure    
I  forgot  other  choices  
Answer:  Uterine  massage  
Immediately  commence  resuscitation.  Raising  the  legs  improves  venous  return  and  is  consistent  with  
the  positioning  used  to  diagnose  and  treat  the  underlying  causes  of  bleeding.  Administer  oxygen  and  
obtain  intravenous  access.  All  intravenous  lines  started  on  the  labor  ward  for  other  reasons  must  be  
placed  with  cannulas  of  sufficient  gauge  if  PPH  develops.  
                                                                                                                                                                                           
246.  Oby  surgery  clamb  the  artery  close  to  lateral  vaginal  wall  what  structure  may  be  injured:  
A.  Pudendal  nerve  
B.  Ureter  
Answer:  B  
 
 
247.  In  pudendal  nerve  block  which  of  the  following  will  not  be  affected?    
A.  Rectum  
B.  Vulva  
C.  Perineal  body  
D.  Urogenital  diaphragm  
Answer:  A  
   
248.  What  are  contraindications  of  instrumental  delivery?  
A.  Placenta  abruption  
B.  Breech  presentation  
C.  Cephalopelvic  disproportion                      
D.  Face  presentation  
Answer:  C  
   
249.  Smoking  pregnant  women  what  will  be  changes  in  her  child?  
Answer:  Intrauterine  growth  restriction  (small  baby).  
   
250.  Female  with  mass  protruding  from  vagina:              
Answer:  Utero  vaginal  prolapse                                                                                                                                                            
                                                                                                                                                           
251.  Female  post-­‐delivery  her  child  on  excessively  breastfeeding,  menstruation  resumed  after  
10th  month,  because  of  her  other  child  she  and  husband  ask  for  contraception,  what  will  you  
advise  them?  
A.  Depo  provera  
B.  OCP  
Answer:  A  
http://patient.info/doctor/postpartum-­‐contraception  
 
 
252.  Definition  of  recurrence  abortion  
A.  2  documented  consecutive    and  more  
B.  3  documented  consecutive  and  more  
C.  4  documented  consecutive  and  more  
Answer:  A  
 
253.  35  years  old  women  O/E  they  found  10*11  mm  raised  irregular  mass  on  lateral  aspect  of  the  
cervix..  what  to  do:  
A.  Excision  
B.  Taking  biopsy  from  the  mass                                                                                                                              
C.  Investigate  for  human  papillomavirus  
D.  Reassure  her  and  wait  for  pap  smear  result    
Answer:  B    
254.  Pregnant  in  labor  everything  normal  except  tender  cervix  during  examination  what  to  do?  
A.  Intravenous  AB                                                                                                                                                            
B.  Intra  labor  antibiotics  
Answer:  ?  
                                                                                                                                                     
255.  Pregnant  with  vaginal  bleeding  and  everything  was  normal  what  do  you  ask  pt  about?  
Last  sexual  intercourse.                                                                                                                                                  
Answer:  question  is  not  complete.  You  should  obtain  history  about  fetal  movements  and  previous  
US.  
 
256.  Female  patient  she  done  tube  ligation  4  years  back,  current  complain  is  she  has  period  which  
was  6  week  back  now  she  intermittent  spotting  of  blood  ,  V/E  there  is  no  blood  normal  cervix  and  
close  os  wt  the  next  step  ?                                            
A.  CT    
B.  Laparoscopy                                  
C.  Pregnancy  test                                                                                                                                
Answer:  Maybe  C  
                                                                                                                                                           
257.  Pregnant  at  the  at  the  time  of  delivery  was  given  epidural  when  she  was  6  cm  dilated  ,  later  
she  was  given  fentanyl!!  And  started  pushing  later  she  started  to  have  lightheadedness  and  she  
was  laying  on  her  side  what  will  you  do  for  her:  
A.  Ask  the  nurse  to  bring  forceps    
B.  Give  fentanyl  again  
C.  Give  ephedrine  
D.  Do  her  a  position  with  weird  name  
Answer:  D,  change  the  patient  position.  
   
258.    First  antenatal  visit  of  a  woman  in  her  10th  week  gestation  what  to  do?  
A.  Assess  fetal  size  or  like  that  
B.  Assess  risk  factors  
C.  Determine  the  fetal  age  or  like  that  
Answer:  B  
 
259.  In  regard  to  vulvar  cancer,  which  of  the  following  is  the  principle  in  diagnosing  it?  
A.  Clinical,  through  history  of  HPV.  
B.  Histopathological,  through  biopsy.    
C.  Radiological,  through  pelvic  US.    
D.  Irrelevant  choice.  
Answer:  B  
 
260.  Fallopian  tube  embryology:    
A.  Mesoderm  intermediate  cell.  
B.  Lateral  part  of  genital  ridge  
Answer:  A  
   
261.  Case  of  preeclampsia    (weight  gain,  proteinuria,  headache)  with  IUGR,    What  other  clinical  
finding  support  IUGR:  
A.  Oligohydramnios  
B.  Polyhydramnios  
C.  Liver  enzymes  
Answer:  A  
   
262.  Case  of  patient  with  recurrent  abortion  and  she  came  at  20  weeks,  cervix  length  was  30.  what  
to  do:  
A.  Circulage  
B.  Progesterone  
Answer:  B  
   
263.  Stillbirth  at  which  gestational  age?    
A.  20  
B.  25  
Answer:  B,  more  than  24  weeks.  
 
264.  Female  I  think  in  40  had  2  child  with  45  xo  next  child  percentage  to  have  it?  
A.  40  %  
B.  60  %  
Answer:  please  see  Q209  
   
265.  31  years  old  Female  came  to  do  pap  smear  and  HPV  test,  result  normal  before  1  years  done  
pap  smear  +  HVS  result  normal  when  to  do  test  again?  
A.  6  months  
B.  1  years  
C.  2  years  
D.  3  years  
Answer:  5  years,    
Source:  https://www.cancer.org/cancer/cervical-­‐cancer/prevention-­‐and-­‐early-­‐detection/cervical-­‐
cancer-­‐screening-­‐guidelines.html  
 
266.  Which  of  following  has  best  survival  on  COPD  patient?  
A.  Smoking  cessation  
B.  O2  supplement  
Maybe:  B  
 
267.  Female  has  4  intercourse  in  week  use  condom  also  her  husband  use  condom,  also  she  use  
douching  after  that  which  of  the  following  has  risk  to  health?  
A.  Use  of  condom  
B.  Douching  
C.  Number  of  intercourse  
Answer:  B  
 
268.  Female  pt  with  puritosis  and  enuresis,  with  pelvic  strengthening  exercises,  she  was  given  
duxbutine,  what  is  the  mechanisms  of  action:  
A.  Inhibit  muscarinic  cholinergic  
B.  Activate  acetylene  
C.  etc  
Answer:  ?  
                                                                                                                                                           
269.  Semen,male  Works  in  batteries  factory,  came  after  1  year  trying  with  his  wife  to  get  pregnant  
analysis:  oligospermia.  CBC  show  heavy  metals  on  RBC,  what  the  cause?                                                                                                            
A.  Nikle                                                                                                                  
B.  Lead  
Answer:  B  
                                                                                             
270.  Regarding  cervical  ca  screening:  
A.  Started  at  age  21  regardless  of  sexual  activity  
B.  HSV  +  Pap  smears  are  done  together  every  5  years  
C.  HSV  screening  is  an  alternative  to  Pap  smear.  
Answer:  A  
 
271.  When's  the  best  time  to  do  amniocentesis?  
A.  10  weeks  
B.  12  weeks  
C.  14  weeks    
D.  16  weeks    
Answer:  D  
 
272.  Ectopic  pregnancy  of  2.5*3  -­‐  hcg  5000  patient  stable  what  to  do  ?  
A.  Wait  
B.  Laparotomy  
C.  Laparoscopy  
D.  D&C  
Answer:  Methotrexate,  if  not  available  laparoscopy    
Source:  http://bestpractice.bmj.com/best-­‐practice/monograph/174/treatment/step-­‐by-­‐step.html  
 
273.  Female  nullipara  unable  to  conceive,  plethoric  face  and  purple  striae  at  abdomen  and  BP  
160/...,  hypokalemia,  what  also  she  has?  
A.  Acropachy  (  graves  )  
B.  this  is  cushing  ..  
Answer:  B  
 
274.  What  is  the  incidence  of  prolonged  pregnancy:  (  from  3  to  12)  
1-­‐5%  
10-­‐15%  
Answer:    As  many  as  10  percent  of  pregnancies  will  deliver  postterm.  
Source:  Uptodate  
 
275.  Female  pregnant  (  twin  )  GA  34  doctor  plan  for  CS  because  presentation  of  twin  A    may  cause  
fetal  complication,  what  is  the  presentation  ?  
A.  Cephalic  -­‐  breech  
B.  Transverse  -­‐  cephalic  
C.  Breech  -­‐  cephalic  
D.  Cephalic  -­‐  cephalic  
Answer:  B  
 
276.  Female  in  her  30s.  P3.  Complaining  of  amenorrhea  for  the  last  year  or  so.  Underwent  D&C  
after  2nd  pregnancy.  Labs  show  high  FSH  and  LH.  Low  estrogen.  
A.  Asherman  
B.  Premature  ovarian  failure.  
Answer:  B  
   
277.  Principle  treatment  of  urge  incontinence?  
A.  Medical  
B.  Surgical  
C.  Medical  and  surgical  
D.  Bladder  training  and  something  else  
Answer:  A  
 
278.  A  patient  who's  on  estrogen  therapy  is  complaining  of  dysuria,  urgency  and  frequency  for  a  
few  months.  She  took  antibiotics  multiple  times  but  it  didn't  work.  What  is  the  cause  of  her  
symptoms?  
A.  Bacterial  vaginosis    
B.  Candida  vaginitis    
Answer:  B  
 
 
279.  50  YO  female  with  urge  incontinence,  dry  vagina.  The  first  step  to  do:  
A.  urinalysis  and  culture  
B.  cystourethrogram  
C.  Bonney's  test  
Answer:  Probably  A    
 
280.  Male  with  pelvic  injury,  1st  thing  to  do:  
Answer:    Suprapubic  catheter.  
 
281.  Most  common  ovarian  cyst  is:  
A.  theca  lutein  
B.  follicular  
Answer:  B  
 
282.  The  day  of  ovulation  is  
A.  14  
B.  17  
C.  22  
Answer:  on  average  day  14  
 
283.  A  Lady  with  heavy  menstrual  bleeding  and  intermenstrual  bleed  with  no  Hx  of  pregnancy  or  
sexual  intercourse...  the  cause  is:  
A.  chronic  endometritis  
B.  anovulatory  cycles    
Answer:  B  
 
284.  Most  common  side  effect  of  IUD  ?  
Answer:  Menstrual  cycle  irregularities    
 
285.  PPH  with  low  BP  and  tachycardia  the  1st  thing  to  do  :        
A.  Ergotamine  
B.  Ringer  lactate  
C.  Normal  saline  with  something  
Answer:  C  
 
286.  Woman  P5,  all  svd,complains  of  (  case  scenario  about  uterine  prolapse  ),  what  test  to  do  ?  
A.  options  of  uro  gyne  tests  
B.  speculum    
Answer:  B  
 
287.  Patient  with  normal  menses  since  puberty  ,  normal  exam  except  ovarian  mass  
A.  Follicular  cyst  
B.  Ovarian  carcinoma            
C.  Endometrial  cancer  
Answer:  A  
 
288.  How  many  Barr  body  in  XXX  female:  
A.  1  
B.  2  
C.  3            
D.  4  
Answer:  C  The  bar  body  number  is  the  same  as  X    
 
289.  How  much  normal  blood  loss  during  menses?  
Answer:  60  (The  usual  amount  of  blood  loss  per  period  is  10  to  35  ml.)  
 
 
290.  Female  patient  in  her  late  40s,  she  has  children  and  does  not  want  to  get  pregnant  any  more.  
Presented  with  abdominal  pain,  she  has  previous  history  of  endometrioma  in  her  right  ovary  and  
it  was  removed.  She  did  imaging  and  it  showed  that  she  has  endometrioma  of  the  left  ovary.  What  
is  the  appropriate  management?    
A-­‐  Bilateral  salpingo-­‐oophorectomy  
B-­‐  removal  of  the  endometrioma    
Answer:  Most  likely  A  since  she  is  not  planning  to  conceive  and  failed  endometrioma  resection  
 
 
291.  Thalidomide  SE  in  pregnancy:    
Answer:  limb  defect  
 
292.  9  week  pregnant,  presented  with  bleeding,  cervix  was  dilated  1  cm  with  visible  tissues  
A.  Threatened  abortion  
B.  Incomplete  abortion  
C.  Complete  abortion  
Answer:  B  
 
293.  benign  ovarian  teratoma        
Answer  :  missed  Q  but  most  likely  benign  ovarian  teratoma  
 
294.  Female  with  HBV  and  had  done  pap  smear  (there  is  results).  What  is  the  cause?  
HBV  
HPV  
Answer:  The  question  is  not  clear  
                                   
295.  Pregnant  taking  iron  supplements  and  come  with  anemia  symptoms,  Hb  is  low,  what  is  the  
diagnosis?  
A.  Iron  deficiency  anemia    
B.  Thalassemia  
Answer:  Needs  further  details.  Maybe  B.  
 
296.  Pregnant  with  microcytic  anemia,  what  to  give  here?  
Answer:  Iron  
 
297.  When  to  give  antibiotic  for  patient  will  go  for  C/S?  
Answer:  Pre-­‐operation  
   
298.  Pregnant  had  PROM,  2  days  later  she  develop  fever  and  pain.  What  is  the  next  in  
management?  
A.  C/S  
B.  IV  Antibiotic    
Answer:  B  and  delivery    
 
●   Know  the  age  of  the  fetus  (e.g.  The  lungs  are  mature  by  36  weeks  before  that  we  give  
steroid).  
●   Know  the  phases  of  delivery,  stages  and  sections    
●   When  to  give  epidural,  pedundal  and  general  and  what  are  the  parts  blocked  in  each.    
●   gene  of  cervical  cancer  →  *see  the  image*  
http://m.cancer.org/cancer/cervicalcancer/detailedguide/cervical-­‐cancer-­‐what-­‐causes  
 
 
 
 
 
 
 
 
 
 
 
 
 
 
 
 
 
 
 
 
 
 
 
 
 
 
 
 
 
 
 
 

 General  Surgery  
   
   
Breast  and  Endocrine  
   
Q.  Patient  with  solitary  thyroid  nodule,  picture  of  hyperthyroidism,  what  is  the  most  
appropriate  next  step?  
A.  US  
B.  FNA  
Answer:B  
Ref:  https://emedicine.medscape.com/article/850823-­‐
overview?pa=OsGCL06RS84x%2Bx%2BlWevYeJ1avQalW6vjZFUwUEd8rFafzWuiuDz1vfrlWIllf
KfsX8MwC0EECwzp432Skuf9qw%3D%3D#a7  
   
   
Q.  When  FNAC  reveal  fibrocystic  change?  
Apocrine  metaplasia  
   
Q.  Female  work  as  an  actress,  present  with  mass  that  increase  before  menses,  after  taking  FNA  the  
result  is  yellow  stain  with  no  refilling  again  what  is  the  diagnosis?  
-­‐ANDI  (my  answer)  
-­‐phyllidus  tumor  
Breast  cyst  
(Cystic  change  were  not  provided  in  the  option)  
not  sure!  
   
   
Q.  yo  came  with  a  history  of  pre-­‐menstrual  multiple  breast  lumps.  She  said  it  goes  after  
menstruation  but  on  exam  she  was  found  to  have  multiple  lumps  with  one  dominant  lump  of  3  cm  
and  LN  exam  is  normal.  How  will  u  manange  :            
A.        Wait  for  the  the  next  menstrual  cycle  and  re-­‐evaluate  .                                                              
B.        FNA  
C.        Radilogy  and  mamogram  
Answer:  
   
Q.    Radical  mastectomy  with  pectoralis  major  removal,  which  movement  is  lost:  
A.  adduction  
B.  abduction  
Answer:  A  
   
   
Q.  Patient  with  solid  thyroid  nodule  2  cm  size,  euthyroid,  what’s  next?  
A.  FNA  
B.  CT  scan  
   
Answer:A  
   
Q.  40  years  old  female  with  unilateral  cystic  mass  with  no  lymph  node  involvement.  Next  
step?  
A.  Mammography  
B.  FNA  
C.  Excisional  biopsy  
   
Answer:  it  is  often  difficult  to  distinguish  a  cyst  from  a  solid  mass.  Ultrasonography  or  aspiration  
must  be  used  to  establish  a  definitive  diagnosis.  Cysts  require  surgical  biopsy  only  if  the  aspirated  
fluid  is  bloody  
Ref:  http://www.aafp.org/afp/2000/0415/p2371.html  
   
Q.  Patient  presented  with  signs  of  hypercalcemia,  his  labs  show  hypercalcemia  +  high  
Alkaline  phosphatase  (from  scenario  it  is  Hyperparathyroidism).  
The  clinical  presentation  in  the  question  will  lead  to  what?  
A.  Osteoporosis  
B.  Osteomalacia  
C.  Hyperparathyroidism  
   
Answer:C  ?  
   
   
Q.  Women  with  bilateral  breast  nipple  white  discharge  and  vision  disturbance  
Prolactin  level  is  high,  where  is  the  lesion?  
A.  Sella  turcica  lesion  
   
Answer:A  
   
   
Q.  Midline  swelling  moves  with  protruding  the  tongue?  
A.  Thyroglossal  cyst  
B.  Thyroid  nodule  
C.  Hygroma  
   
Answer:A  
   
Q.  Patient  presented  with  a  hard  Mass  on  the  outer  upper  area  of  the  breast  which  lymph-­‐
  node  you  have  to  examine?
  A.  Posterior  axillary
  B.  Anterior  axillary
  C.  Lateral  axillary
  D.  Medial  axillary
   
  Answer:
   
   
Q.  Case  of  bloody  nipple  discharge  (probably  asking  about  the  cause).  
A.  Intraductal  papilloma  
   
Answer:A  
   
Q.  Female  with  left  upper  outer  mass  in  the  left  breast,  which  lymph  node  group  will  be  
examined?  
A.  Pectoral  
   
Answer:  
   
Q.  21  y/o  Female  with  lump  in  breast,  what’s  the  best  method  for  evaluation?  
A.  Mammogram  
B.  US  
C.  FNA  
D.  Ductography  
   
Answer:C  
Ref:  https://www.mayoclinic.org/healthy-­‐lifestyle/womens-­‐health/in-­‐depth/breast-­‐
lump/art-­‐20044839?pg=2  
   
Q.  Patient  with  breast  mass,  with  calcification  in  the  MRI,  what  is  the  most  appropriate  
management?  
   
Answer:  not  sure!  
   
Q.  Dimpling  of  skin  in  breast  cancer  is  caused  by?  
A.  Cooper  ligament  
   
Answer:A  
   
Q.  Bilateral  breast  cancer  most  likely  cause?  
A.  Lobular  
   
Answer:  
   
Q.  Patient  in  her  40s  with  breast  mass,  what  will  be  your  next  step  in  her  management?  
A.  Mammogram  
B.  US  
C.  FNA  
   
Answer:A  
   
(Similar  question)  
Q.  41-­‐year-­‐old  Lady  has  breast  mass  since  2  weeks,  upon  examination  it  hard  and  fixed,  
what  is  "most  "  accurate  diagnostic  modality?  
A.  US  
B.  Mammography  
C.  Lactography  
D.  MRI  Breast  
   
Answer:D  ?  
   
Q.  Superolateral  breast  cancer  with  axillary  tail,  which  lymph  node  group  will  be  involved?  
A.  Anterior  group  
   
Answer:  
   
Q.  Patient  with  clear  nipple  discharge,  regular  menstrual  cycle,  how  you  will  investigate  her?  
A.  Prolactin  level  
   
Answer:  
   
Q.  Gene  of  ductal  carcinoma?  
 A.  P53  
   
Answer:A  
   
Q.  Breast  cancer  gene?  
A.  BRCA2  
   
Answer:A  
   
Q.  Thyroid  nodule  biopsy  found  medullary  (?),  what  is  the  diagnosis?  
A.  Papillary  cancer  
B.  Medullary  thyroid  cancer  
   
Answer:B  
   
Q.  Patient  with  bloody  stained  nipple  discharge,  you’re  suspecting  intraductal  papilloma.  
What’s  the  next  step?  
A.  Mammogram  
B.  FNA  
C.  Excision  
   
Answer:C  
Ref:  https://www.healthline.com/health/intraductal-­‐papilloma  
   
Q.  What’s  the  effect  of  taking  Tamoxifin  in  reducing  breast  cancer  in  high  risk  patients?  
A.  High  
B.  Low  
C.  Intermediate  
D.  No  effect  
   
Answer:  not  sure!  
   
   
Q.  Initial  investigation  of  a  single  thyroid  nodule?  
A.  FNA  
B.  CT  scan  
   
Answer:A  
Ref:  https://emedicine.medscape.com/article/850823-­‐overview#a7  
   
Q.  Female  treated  from  mastitis,  after  10  days  presented  with  painless  lump,  what  you  will  
do?  
   
Answer:?  
   
Q.  25  years  old  female  asking  about  self-­‐breast  exam,  when  it  should  be  done?  
A.  6-­‐7  days  after  the  cycle  
B.  3-­‐5  days  after  the  cycle  
C.  7-­‐10  days  after  the  cycle  
D.  14-­‐16  days  after  the  cycle  
   
Answer:B  
Ref:  https://medlineplus.gov/ency/article/001993.htm  
   
Q.  Female  with  bilateral  menstrual  breast  pain.  In  examination  there  is  bilateral  breast  
nodularity  and  mobile  mass  in  the  right  breast,  axillary  lymph  nodes  are  free,  what  to  do  
next?  
A.  Mammogram  then  US  
B.  Aspiration  and  cytology  
C.  Reevaluate  at  the  next  cycle  
   
Answer:  C  
http://www.aafp.org/afp/2005/0501/p1731.html  
   
Q.  Female  with  diffuse  thyroid  swelling  and  dominant  single  nodule,  lab  result  showed  
increased  T4  and  decreased  TSH,  what  will  you  do?  
A.  Radionuclide  scan  
B.  FNA  
C.  US  
D.  Thyroidectomy  
   
Answer:A  
   
   
Q.  Mobile,  firm  mass  not  related  to  menses,  what  is  the  diagnosis?  
A.  Fibroadenoma  
B.  Fibrocytic  change  
C.  İntraductal  carcinoma  
   
Answer:A  
   
Q.  After  aspiration,  when  is  a  breast  lump  safe  to  leave  alone?  
A.  There  is  minimum  blood  staining  of  aspirate  
B.  Clear  cyst  fluid  and  does  not  refill  
C.  Cyst  cells  with  hyperchromatic  nuclei  
D.  FNAC  suggests  fibro-­‐cystic  disease  
 
Answer:  B?  
   
(Similar  question)  
Q.  Patient  with  breast  mass,  which  of  the  following  indicates  that  the  mass  is  cystic  (or  no  
need  for  reevaluation)?  
A.  FNA  showing  clear  fluid  on  aspiration  and  disappearance  of  the  mass  after  
B.  Bloody  nipple  discharge  
C.  Clear  fluid  with  reappearance  of  the  mass  
   
Answer:A??  
   
Q.  Breast  cancer  prognosis  depends  on?  
A.  Number  of  axillary  lymph  nodes  involved  
B.  Estrogen  receptors  
C.  The  patient’s  age  
   
Answer:A  
   
Q.  Recurrence  of  breast  cancer  depends  on?  
A.  Number  of  the  involved  axillary  lymph  nodes  
B.  Estrogen  receptors  
C.  Progesterone  receptors  
   
Answer:A  
Ref:  https://www.mayoclinic.org/diseases-­‐conditions/recurrent-­‐breast-­‐cancer/basics/risk-­‐
factors/con-­‐20032432?p=1  
                                                                                                   
Q.  Treatment  of  multinodular  goiter?  
A.  Surgery  
B.  Radioiodine  
C.  Beta  blockers  
D.  Antithyroid  medications  
   
Answer:B??  
Ref:  https://www.healthline.com/health/multinodular-­‐goiter  
   
   
Q.  Patient  with  Lab  findings  of  hypothyroidism,  he  has  hoarseness  and  a  large  neck  swelling.  
O/E  lt  lobe  is  swollen  and  is  larger  than  the  right  one  
Best  treatment  is?  
A.  FNA  
B.  Thyroid  lobectomy  
C.  Radio  isotope  scan    
   
Answer:A  
   
Q.  Mammogram  can  detect  breast  cancer  before  clinical  examination  by  how  many  years?  
A.  1  
B.  2  
C.  3  
D.  4  
   
Answer:b  
Ref:  https://www.radiologyinfo.org/en/info.cfm?pg=mammo  
   
Q.  Tamoxifen  for  breast  cancer  patient,  she  has  metrorrhagia,  US  showed  thick  
endometrium,  what  to  do  next?  

A.  Endometrial  biopsy  

B.  CA125  

Answer:  not  sure!  

   

Q.  Lady  developed  postpartum  mastitis  (red  swollen  right  breast)  and  was  give  fluxacillin  for  
10  days.  A  few  weeks  later  she  presented  with  a  right  breast  hard  mass,  non-­‐tender.  What  is  
the  Dx?  
A.  Antibioma  
B.  Duct  ectasia  
   
Answer:  B  
   
   
Thoracic  Surgery  
   
Q.  Retrosternal  pain  and  barium  swallow  showed  esophageal  corkscrew  appearance,  what  is  
the  diagnosis?  
A.  Achalasia  
B.  Esophageal  cancer  
C.  Diffuse  esophageal  spasm  
   
Answer:  C  
   
   
Q.  Hyperthyroidism  presentation  but  with  bradycardia,  what  is  the  cause?  
A.  Multinodular  goitre  
B.  Hyperthyroidism  
C.  Hypothyroidism  
   
Answer:  not  sure!  
   
Q.  Patient  smoker,  alcohol  drinking  has  thyroid  nodules,  what  is  the  first  thing  that  you  do?  
                                                                                                                   
A.  Needle  aspiration  
   
Answer:  
   
   
Q.  During  a  surgery  in  the  diaphragm,  phrenic  nerve  is  dissected  at  which  level?  
A.  Anterior  to  scalenus  anterior  muscle  
   
Answer:A  
   
Q.  Which  organ  will  be  affected  with  injury  to  the  posterior  vegal  truck?  
A.  Bladder  
B.  Descending  colon  
C.  Esophagus  
   
Answer:C  ??  
   
   
   
Q.  Chronic  smoker,  presented  with  signs  of  lung  cancer,  what  is  the  origin  of  the  cancer?  
A.  Clara  
B.  Brush  cells  
C.  Goblet  
   
Answer:  b?  
   
Q.  In  the  OR  the  doctor  cut  the  vagus  nerve  by  mistake  while  doing  NISSIEN  fundoplication  
operation  for  treatment  of  chronic  GERD,  which  organ  will  be  affected  from  cutting  the  
nerve?  
A.  Esophagus  
B.  The  fundus  
C.  Urinary  bladder  
D.  Colon  
   
Answer:  
   
Q.  Patient  with  hoarseness,  what  is  your  first  next  step?  
A.  Laryngoscopy  
B.  Inner  ear  MRI  
   
Answer:  not  enough  options  but  I  will  go  with  A  
   
Q.  Patient  with  mass  in  the  throat,  what  is  your  management?  
A.  Surgery  plus  or  minus  radiotherapy  
   
Answer:  
   
Q.  Best  x-­‐ray  for  the  ribs?  
A.  Oblique  
   
Answer:A  
   
Q.  About  SVC  syndrome  and  most  likely  asking  about  the  site  of  the  lesion  in  the  
mediastinum.  
   
Q.  Patient  came  to  the  ER  with  congested  neck  veins,  difficulty  breathing  and  absent  breath  
sounds,  what  is  the  first  step  in  his  management?  
A.  Needle  thoracotomy  
   
Answer:  
   
                   
Q.  What  is  the  management  of  tension  pneumothorax?  
A.  Tube  thoracotomy  
B.  Needle  decompression  
   
Answer:B  
   
Q.  Patient  presented  after  MVA  with  unilateral  hyperresonance  and  mediastinal  shift.  X-­‐ray  
showed  hyperlucency,  what  is  the  1st  step?  
A.  100%  o2  
B.  Needle  thoracentesis  
   
Answer:B  
   
   
Q.  Lung  cancer  with  the  best  prognosis?  
A.  SCC  
B.  Adenosquamous  carcinoma  
C.  Adenocarcinoma  in  situ  
Answer:  C  ?  
   
   
Q.  Patient  came  to  ER,  complaining  of  hypotension,  tachycardia  and  hypercapnia,  
ECG  showed  arrhythmia,  what  is  your  immediate  action?  
A.  Needle  decompression  
B.  Pericardiocentesis  
C.  FAST  ultrasound  
D.  Thoracotomy  
   
Answer:B  ?  
   
   
Q.  You  found  nodule  6  cm  on  chest  xray  and  lab  test  show  hypercalcemia,  which  one  of  the  
following  cancers  is  associated  with  this  finding?  
A.  SCC  
B.  Adenocarcinoma  
   
Answer:A  
   
Q.  Cancer  in  the  middle  of  esophagus,  which  type?  
A.  Adenocarcinoma  
B.  Scc  
   
Answer:B  
Ref:  https://www.cancer.gov/types/esophageal/patient/esophageal-­‐treatment-­‐pdq  
   
   
Q.  Patient  diagnosed  with  Barrett's  esophagus  will  have?  
A.  Adenocarcinoma  
B.  Squamous  cell  carcinoma  
   
Answer:A  
Ref:  https://www.cancer.org/cancer/esophagus-­‐cancer/causes-­‐risks-­‐prevention/risk-­‐
factors.html  
   
Q.  In  non-­‐small  cell  lung  CA,  which  of  the  following  is  a  contraindication  to  do  surgery?  
A.  Stage  IA  
B.  Stage  IB  
C.  Peripheral  lung  lesion  
D.  SVC  obstruction  
                                                                                                                   
Answer:  not  sure!  
 Patients  with  stage  IIIB  or  IV  tumors  are  almost  never  surgical  candidates.  
   
                           
   
   
Urology  
   
Q.  Case  about  epididymo  orchitis,  what  is  the  treatment?  
   
Answer:  
Ceftriaxone  250-­‐500  mg  IM  once  plus.  
Azithromycin  1  g  PO  once  or.  
Substitute  doxycycline  100  mg  PO  BID  for  7-­‐14  days  for  the  azithromycin.  
Ref:  https://emedicine.medscape.com/article/2018356-­‐overview  
   
Q.  27  y  Man  with  progressive  enlarged  testis  no  tenderness  no  urinary  symptoms  no  sexual  hx  for  6  
months  (mostly  tumor):  
-­‐Us  and  surgical  referral  
-­‐Antibiotics                                                                                                        
-­‐Biopsy  
-­‐  NSAIDs  
Answer:C  
   
Q,  -­‐Stone  at  the  level  of  L3-­‐L4  on  x-­‐ray  
A-­‐  Ureter  
B-­‐  Minor  calyx  
C-­‐  Major  calyx  
D-­‐  Renal  pelvis  
Answer:A  
Ref:  http://www.startradiology.com/internships/General-­‐Surgery/Abdomen/Abdominal-­‐X-­‐ray/  
   
 Q.  principle  treatment  of  urge  incontinence?  
A-­‐medical  
B-­‐surgical  
C-­‐medical  and  surgical  ✔  
D-­‐  Bladder  training  and  something  else  
Answer:  C  
Ref:  https://emedicine.medscape.com/article/452289-­‐treatment  
   
   
Q.  What  is  the  best  management  of  pt  with  urge  incontence  ?  
Kegel  exercise  
Urethropexy  
NSAID  
Antibiotic  
Answer:  A  
Ref:  https://familydoctor.org/bladder-­‐training-­‐urinary-­‐incontinence/  
   
Q.  Which  one  of  the  following  cancers  directly  related  to  smoking?  
A.  Colon  
B.  Bladder  
C.  Testicular  
D.  Small  cell  lung  cancer  
   
Answer:B??  
   
Q.  Man  doing  proctectomy  I  think  and  he  came  later  with  metastatic  cancer,  what  is  the  
most  common  site  of  metastasis?  
A.  Vertebral  column  
B.  Lung  
C.  Kidney  
   
Answer:B  
Ref:  https://www.webmd.com/colorectal-­‐cancer/tc/colorectal-­‐cancer-­‐metastatic-­‐or-­‐
recurrent-­‐topic-­‐overview  
   
Q.  Male  with  lumbar  pain  extending  to  the  umbilical  region,  no  urinary  symptoms,  normal  
urine  analysis,  what  is  the  dx?  
A.  Epidydimo  orchitis  
B.  Pyelonephritis  
C.  Lumbar  muscle  spasm  
   
Answer:  not  sure!  
   
Q.  Patient  with  sudden  acute  testicular  pain,  what  is  the  most  appropriate  next  step?  
A.  Refer  to  surgery  
   
Answer:A  
   
Q.  Patient  with  renal  mass  (loin  pain  &  hematuria  &  weight  loss),  best  next  step  is?  
A.  CT  scan  
   
Answer:  
   
Q.  Which  part  of  the  male  urethra  is  more  likely  to  be  injured  with  catheterization?  
A.  Penile  
B.  Prostatic  
C.  Membranous  
D.  Middle  spongy  
   
Answer:C  
   
Q.  Patient  is  treating  from  erectile  dysfunction,  which  of  the  following  medications  can’t  be  
given  to  him  during  his  treatment?  
A.  Nitrates  
   
Answer:A  
   
Q.  Question  about  BPH,  hypertensive  but  already  on  medication.  What  is  your  management?  
TURPT  
Open  prostatectomy                                                                                                  
Alpha  blocker**  
Answer:  C  
   
Q.  Old  age  male  complaining  of  acute  fever,  dysuria,  lower  abdominal  pain,  rectal  exam  
showed  anterior  buggy  swilling,  what  is  the  diagnosis?  
A.  Acute  prostatitis  
B.  Chronic  prostatitis  
C.  BPH  
   
Answer:A  
   
Q.  Old  age  person  complaining  of  dysuria,  back  pain  ,investigations  showed  high  ALP,  very  
high  PSA  what  is  the  diagnosis?  
A.  BPH  
B.  Prostatic  cancer  
   
Answer:B  
   
   
Q.  Man  with  Premature  ejaculation.  What  to  give?  
A.  SSRI  
   
Answer:A  
   
Q.  Question  about  BPH,  hypertensive  but  already  on  medication.  What  is  your  
management?  
A.  TURPT  
B.  Open  prostatectomy  
C.  Alpha  blocker  
   
Answer:C  
   
   
Q.  Patient  with  filling  defect,  US  showed  echogenic  shadow,  which  type  of  stone?  
A.  Uric  acid  stone  
   
Answer:  
   
   


Q.  Ureteric  stone,  best  diagnostic  modality?  
A.  CT  without  contrast  
B.  CT  with  contrast  
C.  Renal  US  
   
Answer:A  
   
Q.  Old  man  with  difficulty  in  urination  and  back  pain  
ALP  high  
LFT  normal  
Dx?  
A.  BPH  
B.  Prostate  cancer  
C.  Orchitis  
   
Answer:  B  bone  mets  
   
Q.  Scrotal  swelling,  abdominal  pain,  exaggerated  bowel  sound,  irreducible  swelling,  next  ?  
A.  US  
B.  Urology  referral  
C.  Surgery  referral  
   
Answer:C  
   
   
Q.  70  Y/O  came  with  urinary  retention,1st  step  in  the  management?  
A.  Urinary  catheter  
   
Answer:A  
   
Q.  Most  common  presentation  of  renal  carcinoma?  
A.  Abdominal  mass  
B.  Hematuria  
   
Answer:B?  
   
Q.  Patient  with  recurrent  uti  presented  with  renal  stones,  what’s  the  organism?  
A.  Proteus  
B.  E.coli  
C.  Klebsiella  
   
Answer:  
Ref:  https://www.ncbi.nlm.nih.gov/m/pubmed/3026032/  
   
Q.  Patient  was  treated  for  renal  stone  before  ....they  attached  picture  of  X-­‐ray  which  I  did  
not  get  he  had  to  go  for  some  surgery  so  what  will  you  do
?  

A.  send  him  to  vascular  surgeon  for  follow  up  

Answer:A  

Hepatobiliary  
 
Q.  Pancreatic  trauma  case,  where  is  the  first  place  for  the  pancreatic  enzyme  to  collect  in?  
A.  Omental  bursa  
B.  Sub  hepatic  space    
C.  Sub  phrenic  space      
Answer:  A  
Note:  lesser  sac  is  the  same  as  omental  bursa.  
 
Q2.  Patient  with  severe  epigastric  pain  radiates  to  the  back,  labs  high  amylase,  ALP,  GGT,  
direct  bilirubin,  what  is  the  cause?  GGT  was  highly  elevated  more  than  ALP  
A.  Gallstones  
B.  alcoholic  pancreatitis    
Answer:  B  
 
Q.  You  did  DRE  and  found  prostate  gland  swelling,  which  lobe  of  prostate  has  the  
neoplasm!?  
A.     Anterior  
B.     Posterior  
C.     Medial  
Answer:  B    
 
Q3.  Migratory  thrombophlebitis  is  seen  in?  
A.  Pancreatic  cancer  
Answer:  A  
EXPLANATION:  
Trousseau's  sign  is  associated  with  gliomas,    pancreatic  or  pulmonary  malignancy  
(medscape)  
 
Q4.  Recurrent  cholecystitis  and  7  stones  treatment?  
A.  Cholecystectomy  
 
Answer:  A  
 
   
Q5.  Patient  with  upper  right  quadrant  pain  radiating  to  shoulder  with  nausea  and  vomiting  
and  she  had  same  attack  before,  ask  about  investigation?  
 
Answer:  most  likely  cholecystitis  and  US  is  the  investigation  of  choice.  
 
Q.  Patient  post  cholecystectomy  developed  unilateral  parotid  swelling,  saliva  was  cloudy,  
culture  was  negative,  What  does  he  have?  
A.  Sarcoid  granuloma.  

B.  Bacterial  sialadenitis  

C.  Sarcoma  

D.  Sjogren's  syndrome  

Answer:  B  

https://www.ncbi.nlm.nih.gov/pubmed/18949350  

Q.  New  onset  acute  pancreatitis,  which  type  of  nutrition  should  be  given?  
A.  TPN  
B.  Nasojejunal  tube  
Answer:  A  (NOT  SURE)  
 
 
 
Plastic  Surgery  
 
Q.  What  is  the  most  common  cause  of  death  in  flame  burns?  
A.  Hypovolemic  shock  
B.  Smoke  inhalation  
 
Answer:B  
 
Q.  Facial  suture  removal  day    
A.   3    
B.   5    
C.   7    
D.   10  
Answer:  B  
https://www.emedicinehealth.com/removing_stitches/page2_em.htm  
 
Q.  Patient  3  cm  lump  in  his  upper  back  ,  slowly  growing  for  years  .  Physical  examination  :  
compressible  ,  no  erythema  ,  there  a  punctum  in  the  middle  that  drain  white  foul  smelling  material  ,  
what  is  the  management  !?  
-­‐  Cryotherapy  
-­‐  Total  intact  resection✅✅✅  
-­‐  Antibiotic  and  resection  
Answer  :  Not  sure    
Epidermoid  cyst:  Incision  and  drainage  is  the  recommended  treatment  for  inflamed  epidermoid  
cysts,  carbuncles,  abscesses,  and  large  furuncles  
 
Q.  Pt  with  filling  defect  ,  us  ,  ecoech  shadow  ‫ ﻭوﺵش ﻧﻮﻉع ﺍاﻟﺴﺘﻮﻥن‬،٬ ‫ ﻧﻔﺲ ﺳﺆﺍاﻝل ﺍاﻟﻤﻠﺰﻣﻪﮫ ﺍاﻟﻲ ﺍاﺧﺘﻠﻔﻮ ﻓﻴﯿﻪﮫ‬ :    
A.   uric  acid  stone  .    
 
Q.  When  should  a  facial  suture  be  removed?  
A.  3  days  
B.  5  days  
C.  8  days  
D.  10  days    
 
Answer:  B  
 
Q.  Most  common  malignant  lesion  need  to  be  removed?  
A.  Erythema  gyratum  
B.  Erythema  migrans  
C.  Erythema  marginatum  
D.  Erythema  annulare  
 
Answer:  A    
https://emedicine.medscape.com/article/1081565-­‐clinical  
 
Q.  What  is  the  role  used  in  burns?  
A.  Role  of  9  
 
Answer:  A  
 
Q.  Patient  with  burn  over  the  anterior  trunk  and  circumferential  burn  on  upper  and  lower  
left  limbs.  The  weight  is  70  Kg,  Calculate  parkland’s  formula?  
 
Answer:  same  q  solved  in  13  booklet  with  different  statement  
The  role  is  parkland  formula  =  TBSA  of  burn%  x  weight  (kg)  x  4    
 
Q.  Tender,  hard  swelling  and  oozing  after  suturing  wound  in  ankle,  what  is  the  diagnosis?    
A.  Cellulitis  
B.  Abscess  
C.  Furuncle  
D.  Carbuncle  
 
Answer:  not  sure  
 
Q.  What's  the  first  step  in  managing  a  minor  burn?  
A.  Apply  butter  to  affected  area  
B.  Wash  with  tap  water  
C.  Apply  ice    
D.  Surgical  debridement  
 
Answer:  B  
First  aid  american  heart  association.    
 
Q.  Patient  with  lower  lip  lesion,  what  is  the  lymphatic  drainage?  
A.  Submental  
B.  Submandibular  
 
Answer:  A  
Reference:Snell  anatomy  
 
Q.  Signs  of  good  healing?  
A.  Epithelium  covering    
 
Answer:A  
 
Q.  Flat  reddish  skin  lesion  in  the  left  side  of  the  face  with  dilated  dermal  vessels,  what  is  the  
diagnosis?  
A.  Port  wine  stain  
B.  Cavernous  hemangioma  not  flat  
C.  Melanoma  not  red  
 
Answer:  A  
 
 
Q.  Leg  ulcer  with  rolled  outer  edge,  what  is  the  diagnosis?  
A.  SCC  
B.  Basal  cell  carcinoma  
Answer:  A  
Surgery  Recall    
 
Q.  Elderly  has  bedsore  that  invade  through  the  muscles,  what  is  the  stage?  
A.  1  
B.  2  
C.  3  
D.  4  
Answer:  D  
https://www.webmd.com/skin-­‐problems-­‐and-­‐treatments/pressure-­‐sores-­‐4-­‐stages#2-­‐5  
 
Q.  Patient  3  cm  lump  in  his  upper  back,  slowly  growing  for  years.  Physical  examination:  
compressible,  no  erythema,  there  a  punctum  in  the  middle  that  drain  white  foul  smelling  
material,  what  is  the  management?  
A.  Cryotherapy  
B.  Total  intact  resection  
C.  Antibiotic  and  resection  
Answer:B  (not  sure)  
Solved  before    
 

Q.  Child  with  cutaneous  hemangioma,  those  hemangiomas  could  be  found  in  which  organ?
  

A.  Liver  
B.  Spleen  
C.  Kidney  
Answer:  A  
 
Q.  Describe  neva,  What  is  the  evidence  of  naval  hyperplasia?  
A.  Change  in  the  color  
B.  Irregular  border  
C.  All  of  the  above  
 
Answer:  not  sure  
Colorectal  Surgery  
 
 
Q.  Colon  cancer  screen  recommended  grade  A,  which  age  group?  
A.  45-­‐65  
B.  50-­‐65  
C.  50-­‐75  
   
Answer:  C  
https://www.uspreventiveservicestaskforce.org/Page/Document/UpdateSummaryFinal/col
orectal-­‐cancer-­‐screening  
 
Q.  Young  pt  have  severe  pain  during  defection  followed  by  bleeding  and  relief  after  4  h:  
-­‐hemarroid  
-­‐anal  fissure      
Answer:  anal  fissure   (NOT  SURE)      
http://www.healthline.com/health/anal-­‐fissure  
 
Q.  Most  common  cause  rectal  bleeding  in  age  50  years?  
A.  Anal  fissure  
B.  Hemorrhoids    
The  answer  should  be  diverticulosis  BUT  if  the  q  is  only  containing  these  2  choices  
Then  more  information  needed  to  differentiate  between  A  and  B        
 
Answer:(NOT  SURE)  
 
Q.  What  is  the  best  screening  test  for  colorectal  CA?  (no  colonoscopy  in  choices)  
A.  Fecal  occult  blood  testing    
B.  Sigmoidoscopy  every  5  years  
C.  CT  colonography  
D.  Stool  DNA  
Answer:(not  sure),  could  be  A  because  sigmoidoscopy  cannot  detect  right  sided  of  the  colon  
where  42%  of  malignancies  are  found    
 
Q.  Colon  cancer  stage  B2  means?  
A.  No  LN  involvement  
 
Answer:  A  
Search    in  google  picture  for  AJCC  TNM  staging  system    
 
Q.  Which  of  the  following  is  at  a  high  risk  for  colorectal  carcinoma?  
A.  High  salt  diet  
B.  Smoker  
C.  Male  on  high  fat  diet  
D.  Gardner  syndrome  
 
Answer:D  
 
Q.  Which  one  of  these  polyps  have  potential  to  transform  to  malignant?  
A.  Hyperplastic  polyp(no  risk)  
B.  Adenoma(most  malignamt)  
C.  Peutz–Jeghers(low  malignant)  
D.  Juvenile  
 
Answer:  B  
 
Q.  k/c  case  of  chronic  constipation  complaining  of  pain  and  bleeding  during  defecation  the  
pain  persist  for  many  hours  what  is  the  diagnosis?  
A.  Thrombosed  hemorrhoid  
B.  Anal  fissure  
 
Answer:  A  or  B  not  clear  q  and  incomplete    
 
Q.  k/c  of  chronic  constipation  complain  of  pain  during  defecation  on  examination  there  was  
fluctuating  mass  in  the  perianal  area  what  is  the  treatment?  
A.  Sitz  bath  
B.  Incision  and  drainage  
 
Answer:  start  with  A  then  if  it  is  large  go  directly  to  B    
 
Q.  Case  about  thrombosed  pile  with  severe  pain,  used  warm  press,  sitzs  bath,  antibiotic  but  
still  pain  not  relieved,  what  to  do  next?  
A.  Evacuate  the  clot  
B.  Sitz  bath  
 
Answer:  A  
medscape  
 
Q.  Male  with  changed  bowel  habits,  LLQ  pain,  tenderness,  swelling  and  weight  loss,  what  is  
the  most  likely  diagnosis?  
A.  Diverticulosis  
B.  Diverticulitis  
C.  Cecum  cancer  
D.  Sigmoid  cancer  
 
Answer:  NOT  SURE    
 
Q.  Enlarge  medial  group  of  horizontal  -­‐inguinal  lymph  node  (superficial).  What  you  will  
check  1st?  
A.  Anal  canal  
B.  Muscle  of  thigh  
C.  Muscle  of  leg  
 
Answer:  A  (not  sure)  
They  receive  lymphatic  afferents  from  the  following:  
●   integument  of  the  penis  
●   scrotum  
●   perineum  
●   buttock  
●   abdominal  wall  below  the  level  of  the  umbilicus  
●   back  below  the  level  of  the  iliac  crest  
●   vulva  
●   anus  (below  the  pectinate  line)  
●   the  thigh  and  the  medial  side  of  the  leg  (the  lateral  leg  drains  to  the  popliteal  lymph  
nodes  first).  
wiki  
 
General  
 
24-­‐Case  of  necrotizing  fasciitis,  stain  showed  gram  positive  cocci  in  chains?    
1-­‐  ampicillin  and  gentamicin    
2-­‐  penicillin  and  clindamycin  (  I  think  correct)  
3-­‐  pepracillin  and  tazobactam  
4-­‐  imipenem  and  metronidazole  
Answer:  2  
 
Necrotizing  fasciitis  with  Gym  +ve  strept,  what’s  the  best  Abx  combination:  
A.  penicillin  and  clinda  
B.  piperacillin  and  tazobactam  
ANSWER:  A  
 
Case  of  necrotizing  fasciitis,  stain  showed  gram  positive  cocci  in  chains?  
1-­‐  ampicillin  and  gentamicin  
2-­‐  penicillin  and  clindamycin  
3-­‐  pepracillin  and  tazobactam  
4-­‐  imipenem  and  metronidazole  
Aaswer:  2  
BMJ  
 
23)  Stage  of  ulcer  reaching  fascia  and  extends  through  muscle:  
A.  I  
B.  II  
C.  III  
D.  IV  
Solved  previously  
 
Patient  with  stab  wound  injury  at  the  level  of  the  right    6  th  intercostal  space.  Which  will  be  affected?    
1-­‐  right  superior  lobe  
2-­‐  right  middle  lobe  
3-­‐  right  lower  lobe  
4-­‐  horizontal  fissure  
answer:D  
 
Q.  Patient  with  periumbilical  pain  with  anorexia  and  nausea,  labs  Increase  WBC,  dx?    
A.  Acute  appendicitis  
 
Answer:  A  
 
Q.  Which  organ  will  be  affected  if  you  ligate  internal  iliac  artery?    
A.  Ovary  
B.  Intestine  
C.  Bladder  
Answer:A  
 
Q.  Best  investigation  for  detection  of  retroperitoneal  hemorrhage?  
A.  Computed  tomography    
 
Answer:  A  
Previous  qs  
 
Q.  Patient  with  grade  1  splenic  injury,  what's  the  management?  
A.  Surgical  Rx  
B.  Observe  in  ICU  
C.  Observe  in  surgical  ward  
D.  Medical  Rx  
 
Answer:  i  could  not  find  the  CLEAR  answer  could  be    B  OR  C      
 
Q.  The  genetics  of  hereditary  chronic  pancreatitis?    
A.  Autosomal  dominant  
B.  Autosomal  recessive  
Answer:A  
https://www.ncbi.nlm.nih.gov/pmc/articles/PMC1774562/  
 
Q.  What  are  the  structures  that  could  be  injured  during  adrenalectomy?  
 A.  Right  side  will  injure  IVC,  left  side  will  injure  the  pancreas  
 
Answer:  ………...  
 
Q.  Appendicitis  histopathology?  
A.  Neutrophils  in  the  muscularis  layer    
 
Answer:A  
https://goo.gl/images/ktLXvY  
 
Q.  Open  cholecystostomy  incision?      
A.  Kochar  "subcostal"  incision    
Answer:A  
 
Q.  Irreversible  hernia  with  painful  abdomen,  what  it’s  called?  
A.  Strangulated  hernia  
Answer:  A  
 
 
Q.  Patient  presented  with  an  ulcer  on  sole  of  foot  surrounded  by  erythema,  what  is  the  best  
action?    
 
A.  Oral  Abx  and  discharge  the  patient  
B.  Reassurance    
C.  Admission  for  debridement  and  deep  tissue  culture  
Answer:C  
   
 
  Q.  Pathophysiology  of  acute  appendicitis
  A.  Appendicular  obstruction
 
Answer:  A  
Recall  surgery  
 
Q.  Patient  has  acute  pancreatitis  labs  shows  mild  elevation  of  TG  what  is  the  most  common  
cause  of  this  presentation?  
A.  Gallstones    
B.  Alcohol  intake    
C.  Hypertriglycerima    
 
Answer:  NOT  SURE  (incomplete)  
 
Q.  Most  common  cause  of  Meckel’s  diverticulum?  
A.  Terminal  duodenum  
B.  Terminal  jejunum      
C.  Terminal  ilium  
D.  Cecum    
 
Answer:C  
MTB  
 
  Q.  Pancreatic  cancer  gene?
  A.  K-­‐ras
answer  :A  
https://www.hopkinsmedicine.org/health/articles-­‐and-­‐answers/ask-­‐the-­‐expert/pancreatic-­‐
cancer-­‐experts-­‐answer-­‐commonly-­‐asked-­‐questions  
   
Q.  What  is  the  characteristic  sign  of  perforated  duodenal  ulcer  ?  
A.  Epigastric  pain  (symptom  not  sign)  
B.  Melena  
C.  Vomiting  
D.  Steatorrhea  
 
Answer:  A  
 
  Q.  Site  of  insertion  of  butterfly  needle?
  A.  2nd  intercostal  space  midclavicular  line
 
Answer:    may  be  wrong  q    
 
Q.  Patient  hospitalized  after  major  procedure  developed  a  small  pulmonary  embolism  which  
was  confirmed  by  CT.  What  is  the  best  drug  to  remove  the  embolus?  
A.  Heparin  
B.  Warfarin  
C.  Aspirin  
D.  Streptokinase  
 
Answer:A  
 
   
Q.  Patient  with  small  bowel  obstruction  after  small  bowel  surgery  one  year  ago,  what  is  the  
best  diagnostic  modality?  
A.  US  
B.  Barium  enema  
C.  Double  contrast  barium  meal  
D.  Small  bowel  barium  follow  through  
Answer:  B  
 
Q.  Patient  with  right  iliac  fossa  pain,  developed  vomiting  and  nausea  with  rebound  pain  
during  palpation,  what  is  the  most  likely  cause?  
A.  Diverticulitis  
B.  Acute  appendicitis  
C.  Crohn’s  disease  
 
Answer:  B  
 
Q.  Most  common  sign  of  lower  intestinal  obstruction?  
A.  Altered  bowel  motion  
B.  Absolute  constipation  
C.  Flatulence  
D.  Diarrhea  
 
Answer:  B  
medscape  
 
Q.  RTA  victim  e  closed  head  injury  and  LOC.  What  is  the  best  thing  to  do  ?  
A.  Intubation  and  ventilation  
B.  Check  the  pulses  
C.  Check  the  pupils  
D.  Check  the  airway  
 
Answer:  B  (not  sure)  
ATLS  
 
Q.  A  case  of  GERD  used  antacid  which  caused  constipation  
A.  Calcium  carbonate  
B.  Aluminum  hydroxide  
C.  Magnesium  hydroxide  
 
Answer:  B  
   
Q.  Patient  was  taking  a  drug  for  the  acidity  of  the  stomach,  now  complaining  of  
constipation,  what  is  that  drug?  
A.  Aluminum  hydroxide(constipation)  
 
Answer:  A  
Magnesium  hydroxide  (diarrhea)    
 
Q.  Old  age  patient  smoker  complains  of  white  colored  tongue  with  ulcers,  what  is  the  
diagnosis?  
A.  Squamous  cell  carcinoma  
 
Answer:A  
Squamous  cell  carcinoma  is  the  most  common  type  oral  cancer  in  smoker.  (wiki)  
 
Q.  What  to  do  when  there  is  a  problem  in  intubation?  
A.  Head  tilt  
B.  Jaw  thrust    
C.  Cricoid  pressure  
Answer:  incomplete  (is  it  trauma  patient  or  not)    
 
Q.  Elderly  has  abdominal  pain  and  vomiting,  vitally  stable.  on  abdominal  x-­‐ray  there  is  air  in  
rectum  and  dilated  loop.  what  is  the  management?    
A.  Nasogastric  tube  and  IV  fluid  NS  
B.  Narcotic  and  IV  antibiotics    
C.  Rectal  decompression  
Answer:  A(not  sure)  
http://teachmesurgery.com/general/presentations/bowel-­‐obstruction/  
 
Q.  Post  surgical  wound  infection  most  likely  from?  
A.  Hands  of  surgeons  
 
Answer:  q  incomplete  may  be  A    
 
Q.  Patient  treated  for  peptic  ulcer  with  triple  therapy  now  she  is  free  of  symptoms.  what  
you  will  do?  

A.  Nothing  
B.  H.  Pylori  antigen  test  
C.  Endoscopy  
 
Answer:  B    
https://www.uptodate.com/contents/helicobacter-­‐pylori-­‐infection-­‐and-­‐treatment-­‐beyond-­‐
the-­‐basics  
Q.  Patient  has  an  upper  abdominal  pain,
  (gastrin  and  pancreatic  enzymes  released,  and  
asking  about  the  diagnosis)?  

A.  Zollinger  Ellison  syndrome  

Answer:  A  

Medscape    
 
Q.  What  is  most  common  type  of  shock  associated  with  MVA:  
A.  Septic  
B.  Hemorrhagic  
C.  Neurogenic  
 
Answer:  B  
 
 
Q.  Young  male  have  epigastric  pain,  fullness,  nausea  postprandial  bloating  for  three  years,  
his  symptoms  appeared  after  he  stayed  in  India  for  one  year  for  studying,  symptoms  are  
worse  with  food.  What  is  the  diagnosis?  
A.  Functional  dyspepsia  
B.  H.pylori  infection  
C.  Esophageal  ulcer  
 
Answer:B  
                 
 
Q.  Presentation  of  erosion  of  gastric  ulcer  ?  
A.  Occult  bleeding  
B.  Epigastric  pain  with  generalized  peritonitis  
Answer:  not  clear  more  details  needed    
 
Q.  Old  patient  came  with  abd  pain  and  bloody  diarrhea  in  radiograph  showed  enlarge  
visceral  with  no  air  fluid  level,  what  is  the  diagnosis?  
A.  Ischemic  colitis  
Answer:.........  
                   
Q.  Extradural  hematoma  source  of  bleeding?  
  A.  Middle  cerebral  artery
  B.  Middle  meningeal  artery
  C.  Anterior  cerebral  artery
 
Answer:  B  
Step  up  to  medicine  page  571  
 
                         
Q.  Patient  with  duodenal  ulcer,  what  is  the  treatment?  
A.  Triple  therapy  
B.  Omeprazole  
 
Answer:q  is  unclear  if  h.pylori  suspected  go  with  A    
If  not  go  with  B    
 
Q.  Patient  with  post  disease  in  the  spine  got  abscess  in  the  anterior  chest  wall,  what  is  the  
route  of  transmission?  
A.  Anterior  cutaneous  
B.  Posterior  cutaneous  
 
Answer:  A  
Previous  bank  in  12  and  13  booklet    
 
 
Q.  Diabetic  patient  with  deep  ulcer  in  the  sole  of  foot  ask  about  management?  
A.  Admission  and  debridement  
 
Answer:    
Q.  Patient  have  MVA  come  to  ER  with  ulcerative  tissue  with  gas  gangrene  in  the  anterior  leg  
what  is  the  causative  organism?  
A.  C.  Perfringens    
B.  Staph  aureus  
 
Answer:  A  
http://www.msdmanuals.com/professional/infectious-­‐diseases/anaerobic-­‐
bacteria/clostridial-­‐soft-­‐tissue-­‐infections  
 
 
Pediatric  Surgery  
 
Q.  Contraindication  of  circumcision?  
A.  Hypospadias    
Answer:  A    
Toronto  note  16  edition    p1352  
 
Q.  Which  in  inguinal  canal  is  develop  from  the  external  oblique  muscle?      
External  spermatic  fascia              
Internal  spermatic  fascia  
Answer:  A  
Wikipedia  ,  just  write  external  spermatic  fascia    
Q.    How  to  manage  uncomplicated  sigmoid  vulvulus?            
a)  flatus  tub            
b)  Nasal  tube    
c)  Surgery  
 
Answer:  A  
Toronto  note  p445  
 
Q.  Paraphimosis  picture.    
Paraphimosis  occurs  when  the  foreskin  of  an  uncircumcised  or  partially  circumcised  male  is  retracted  
for  an  extended  period  of  time.  This  in  turn  causes  venous  occlusion,  edema,  and  eventual  arterial  
occlusion.  
 
Q.  Complication  of  PDA  ligation?  
A.  Injury  to  the  vagus  nerve  
B.  Injury  to  the  phrenic  nerve  
C.  Injury  to  the  recurrent  laryngeal  nerve  
 
Answer:  C  
https://www.ncbi.nlm.nih.gov/pubmed/16797086  
https://www.hindawi.com/journals/cripe/2017/2647353/  
 
Q.  Long  scenario  about  hydrocele  typical  :              
-­‐failure  of  obliteration  of  process  vaginalis  
           
 
Q.  What  is  the  most  common  urological  congenital  male  anomaly?  
A.  Hypospadias  
B.  Cryptorchidism    
C.  Phimosis    
Answer:  B  
Wikipedia    
 
Q.  How  to  diagnose  volvulus?  
A.  CT    
B.  Barium    
C.  MRI  
 
Answer:  B  
MTB    
 
Q.  Which  ligament  pass  through  inguinal  canal?  
A.  Round  
B.  Broad  
Answer:  A  
Repeated  in  previous  q  
Teachmeanatomy.info  and  wikipedia    
 
Q.  How  to  manage  uncomplicated  sigmoid  volvulus?    
A.  flat  tube    
B.  Nasal  tube  
C.  Surgery  
 
Answer:  the  answer  should  be  flatus  tube    
Toronto  note16  edition  page  445  
 
 
Q.  Child  with  hirschsprung's  disease,    asking  about  initial  diagnostic  test?  
A.  X-­‐ray  
Answer:  A  
MTB  3rd  edition    
 
Q.  Paraumbilical  hernia,  playing  basket?  (Maybe  they  were  asking  about  the  diagnostic  
modality)  
CT  abdomen  
Answer:  I  think  this  q  is  incorrect    
It  is  in  the  previous  bank  as  (boy  play  football  came  with  abdominal  pain  without  any  injury  
in  match  ,  physical  examination  was  tenderness  in  paraumbilical  area  what  do  you  do  next:  
(RECTUS  SHEATH  HEMATOMA)    
Answer:  could  be  recheck  or  CT  abdomen  (not  sure)  
Recheck  most  likely  because  it  is  clearly  related  to  exercise  not  something  else  serious.    
 
Q.    Initial  investigation  in  olive  mass  in  babies:  
A.  abdominal  X-­‐ray  
B.  abdominal  US  
C.  CT  
D.  barium  
Answer:  B  
MTB  3rd  edition    
 
Q.  painless  rectal  bleeding  on  investigation  you  diagnosed  the  case  as  meckel's  diverticulum  ,what  is  
the  commonest  site  for  it  ?    
Lower  ilium  
*  cecum  
*  duodenum          
*  jejunum              
Answer  :  ilium  (MD  is  commonly  present  90  cm  from  illio-­‐cecal  valve)  
 
 
 
 
 
 
 
 
 
 
 
 
 
 
 
 
 
 
 
 
 
 
 
 
 
 
 

Orthopedics  
   
Q1.  Patient  with  history  of  falling  on  an  outstretched  hand,  what  is  the  fracture?  
A.        Clavicle  
B.        Shoulder  
C.        Supracondylar  
D.        Colles  
Answer:  D  
   
Q2.  Patient  can’t  do  foot  dorsiflexion,  which  muscle  is  responsible?  
A.        Anterior  tibialis  
   
Answer:  A  
The  foot  and  ankle  dorsiflexors  include  the  tibialis  anterior,  the  extensor  hallucis  longus  (EHL),  and  
the  extensor  digitorum  longus  (EDL)  -­‐  Medscape.  
Q3.  Patient  have  fracture  in  his  tibial  it  is  opened  fracture  <1cm  ,  what  you  do  ?  
A.        Close  reduction  with  cast  
B.        Debridement  and  open  reduction  by  intramedullary  nail.  
C.        Give  him  AB  
   
Answer:  B  
Q4.  Unilateral  knee  swelling    and  pain,  knee  tap  labs:  cloudy    yellow  color,    mucoid,  WBC-­‐PMN  15  
(normal  less  than  200),  lymphocytes  80%,  what  is  the  diagnosis?  
A.        Gout  
B.        Septic  arthritis  
C.        Rheumatoid  Arthritis  
   
Answer:  B  
Q5.  An  athletic  guy  lifts    heavy  weights  as  usual,  this  time  he  experienced  sudden  pain  while  lifting  
the  weight,  examination  revealed  normal  range  of  motion  in  the  shoulder,  what  to  do?  
A.        Reassure  
B.        Diclofenac  
C.        Physiotherapy  
   
Answer:  B  
Q6.  Case  of  septic  arthritis  patient  started  on  oxacillin,  3  days  later  the  sensitivity  shown  up  and  it  
was  staph  aureus  resistance  to  cefoxitin,  what  is  your  management?  
A.      Stop  antibiotic  
B.      Continue  oxacillin  
C.        Start  vancomycin  
   
Answer:  C  
Q7.  Proximal  tibial  fracture  palpable  peripheral  pulse  ABI:  .85  what  to  do?  
A.        Angiography  
B.        Doppler  US  
C.        CT  
   
Answer:  A  
An  ABI  less  than  0.90  suggests  a  need  for  further  vascular  imaging:  angiography  in  a  stable  patient,  
and  operative  exploration  in  an  unstable  patient  -­‐  Medscape.  
   

Q8.  Male  patient  with  pain,  effusion,  erythema,  swelling  in  both  knee  joints,  ask  about  treatment?  
A.        Aspiration  and  antibiotics  
   
Answer:  A  
Q9.  Patient  with  multiple  fractures,  O/E  sky  blue  sclera,  X  ray  showed  multiple  healing  fractures  with  
callus  formation?  
A.        Osteogenesis  imperfecta  
   
Answer:  A  
Q10.  Old  patient  with  neck  pain  and  loss  of  sensation  in  the  left  arm,  best  step  in  Dx?  
A.      Spine  MRI  
   
Answer:  A  
Q11.  What  is  the  muscle  responsible  for  unlocking  the  knee?  
A.        Popliteus  
   
Answer:  A  
Q12.  Stab  wound  in  the  buttocks  there  is  defect  in  the  lateral  rotation  of  the  leg?  
A.        Gluteus  maximus  
B.        Adductor  longus  
   
Answer:  A  
Q13.  Commonest  cause  of  olecranon  bursitis?  
A.        Repetitive  trauma  
   
Answer:  A  
Q14.  Patient  complaining  of  pain  in  medial  epicondyle,  he  is  a  golf  player,  what  is  the  management?  
A.        Refine  golfer  
   
Answer:  ?  
RICE  (rest,  ice,  compression  and  elevation),  elbow  brace,  and  the  last  option  is  surgery:  
fascial  elevation  and  tendon  origin  resection  (FETOR)  
Q15.  Pain  in  the  wrist  with  positive  tinel's  sign  which  nerve  is  affected?  
A.        Median  nerve  
Answer:  A  
   
Q16.  Patient  with  thenar  muscle  atrophy  what  is  the  nerve  affected?  
A.        Median  nerve  
   
Answer:  A  
Q17.  Rickets  vs.  hypophosphatemia  
   
Answer:  
Rickets:  low  calcium,  low  vitamin  D,  high  PTH,  low  phosphorus  and  high  ALP.  
Hypophosphatemic  rickets:  
calcium  levels  may  be  within  or  slightly  below  the  reference  range;  alkaline  phosphatase  levels  will  
be  significantly  above  the  reference  range,  serum  parathyroid  hormone  levels  are  within  the  
reference  range  or  slightly  elevated,  while  calcitriol  levels  are  low  or  within  the  lower  reference  
range.  Most  importantly,  urinary  loss  of  phosphate  is  above  the  reference  range.  
   
Q18.  Pagets  vs.  osteoporosis  vs.  vit  d  deficiency  
   
Answer:  
-­‐  Paget’s  disease:  serum  calcium  and  phosphate  =  within  the  reference  range.  
Hyperuricemia,  serum  total  acid  phosphatase  is  high,  bone  specific  alkaline  phosphatase  is  high.  
   
-­‐  Osteoporosis:  levels  of  serum  calcium,  phosphate,  and  alkaline  phosphatase  are  usually  normal  in  
persons  with  primary  osteoporosis,  although  alkaline  phosphatase  levels  may  be  elevated  for  several  
months  after  a  fracture.  
Inadequate  vitamin  D  levels  can  predispose  persons  to  osteoporosis.  
DXA  scan  is  currently  the  criterion  standard  for  the  evaluation  of  BMD.  
   
-­‐  Vitamin  D  deficiency:  low  25-­‐hydroxyvitamin  D  or  25(OH)D.  
Q19.  Boutonniere  deformity  description?  
A.        PIP  flexion  and  DIP  hyperextension  
   
Answer:  A  
Q20.  Athlete  can’t  plantar  flex  the  foot,  where  is  the  injury?  
   
Answer:  could  be  achilles  tendon  injury,  they  present  with  weakness  in  foot  plantar  flexion.  
   
Q21.  Patient  can't  extend  wrist  and  finger,  nerve  injured?  
   
Answer:  Radial  nerve  
   
Q22.  Where  can  you  palpate  dorsalis  pedis  artery?  
A.        Lateral  to  the  extensor  hallucis  longus  
   
Answer:  A  
Q23.  Child  with  hip  pain,  x-­‐ray  was  normal  but  US  showed  fluid.  Lab  revealed  high  ESR,  and  CRP,  
otherwise  normal.  What  to  do?  
A.        MRI  
B.        CT  hip  and  pelvis  
C.          Aspiration  
   
Answer:  C  
Q24.  Athlete,  Heel  pain  in  the  morning,  what’s  the  dx?  
A.        Plantar  fasciitis  
   
Answer:  A  
They  present  with  sharp  heel  pain  worse  in  the  morning  when  first  getting  out  of  bed.  
Q25.  Posterior  shoulder  dislocation?  
   
Answer:  pain  with  flexion  +  adduction  and  internal  rotation  of  the  arm  +    posterior  dislocation  may  
leave  humeral  head  in  subacromial  position.  
   
Q26.  Treatment  of  spinal  stenosis?  
   
Answer:  analgesics,  anti  inflammatory,  physical  therapy,  and  lastly  surgical  decompression.  
Surgery  is  indicated  when  the  signs  and  symptoms  correlate  with  the  radiologic  evidence  of  spinal  
stenosis.  Generally,  surgery  is  recommended  when  significant  radiculopathy,  myelopathy  
(cervicothoracic),  neurogenic  claudication  (lumbar),  or  incapacitating  pain  is  present.  
   
Q26.  Perths  2  times  (xray/  scenario).  
   
Answer:  
Legg-­‐Calve-­‐Perthes  Disease  (Coxa  plana):  it  is  an  idiopathic  avascular  necrosis  of  the  proximal  
femoral  epiphysis  in  children.  4-­‐8  years  is  most  common  age  of  presentation,  male  to  female  ratio  is  
5:1,  up  to  75%  of  affected  patients  have  some  form  of  coagulopathy.  
Bone  age  is  delayed  in  89%  of  patients  
Younger  age  (bone  age)  <  6  years  at  presentation  is  most  important  good  prognostic  indicator.  
-­‐  Symptoms:  insidious  onset,  may  cause  painless  limp,  intermittent  hip,  knee,  groin  or  thigh  pain.  
-­‐  Signs:  hip  stiffness,  loss  of  internal  rotation  and  abduction,  gait  disturbance;  antalgic  limp  and  
Trendelenburg  gait.  
   
-­‐  X-­‐ray:  medial  joint  space  widening  (earliest)  from  less  ossification  of  head,  cresent  sign  
(represents  a  subchondral  fracture).  MRI  is  more  sensitive.  
   
-­‐  Treatment  is  nonoperative;  observation  alone,  activity  restriction  (non-­‐weight  bearing),  and  
physical  therapy  (ROM  exercises)  in  children  <  8  years  of  age  (bone  age  <6  years)  (they  don’t  benefit  
from  surgery).  
Operative:  femoral  and/or  pelvic  osteotomy  for  children  >  8  years  of  age.  
(Orthobullets).  
   
Q27.  Osgood  schlatter  disease.  
   
Answer:  
Osgood-­‐Schlatter  disease  (Tibial  Tuberosity  Avulsion  -­‐  Traction  Apophysitis).  
Anterior  knee  pain,  often  bilateral  (around  25-­‐50%  of  cases),  of  tibial  tuberosities  in  adolescent  
children  (10-­‐11  in  girls  and  13-­‐14  in  boys).  
Caused  by  repetitive  stress  from  the  quadriceps  tendon  pulling  on  the  tibial  tuberosities  during  rapid  
growth  spurts.  Sports  with  jumping,  running  and  kneeling  making  it  worse,  and  it  improves  with  rest.  
P/E:  pain  with  palpation  over  the  tibial  tuberosities,  and  reproduced  with  resisted  knee  extension.  
No  need  for  imaging.  
Treatment  is  conservative.  
(Master  the  Boards)  
   
Q28.  Patellar  chondromalacia  
   
Answer:  
Condition  characterized  by  idiopathic  articular  changes  of  the  patella.  more  commonly  grouped  
together  with  a  number  of  pathological  entities  known  as  "anterior  knee  pain"  or  "patellofemoral  
syndrome".  
Symptoms:  
-­‐  Diffuse  pain  in  the  peripatellar  or  retropatellar  area  of  the  knee  (major  symptom)  
-­‐  Insidious  onset  and  typically  vague  in  nature  
-­‐  Aggravated  by  specific  daily  activities  including:  climbing  or  descending  stairs,  prolonged  sitting  
with  knee  bent  (known  as  theatre  pain),  squatting  or  kneeling.  
P/E:  
-­‐  Quadricep  muscle  atrophy  
-­‐  Signs  of  patella  maltracking  
-­‐  Palpable  crepitus  
-­‐  Pain  with  compression  of  patella  with  knee  range  of  motion  or  resisted  knee  extension.  
Treatment:  rest,  rehab  and  NSAIDS  (mainstay  of  treatment  and  should  be  done  for  a  minimum  of  
one  year).  
Operative  treatment:  https://www.orthobullets.com/sports/3022/idiopathic-­‐chondromalacia-­‐
patellae  
(Orthobullets)  
   
Q29.  Medial  collateral  ligament?  
   
Answer:  you  can  test  it  with  valgus  stress  test.  
   

Q30.  question  about  a  boy  who  pushed  his  friend  from  the  back  10  hours  later  the  boy  came  to  ER  
complaining  of  severe  pain  what  is  the  ligament  affected?  
A.        Iliofemoral  
B.        Ischio…  
   
Answer:  ?  
Incomplete  question  
   
Q31.  Which  tendon  passes  through  lesser  sciatic  foramen?  
A.        Obturator  Internus  tendon  
   
Answer:  A  
   
Q32.  Diagnosis  of  osteoporosis    based  on  DXA  scan  ?  
A        -­‐3.5  
B.      2  
C.      3.5  
D.      2  
   
Answer:  less  than  -­‐  2.5  
   

Q33.  Treatment  of  elderly  with  osteoporosis?  


A.        Alendronate  
B.        Vit  D  
   
Answer:  A  
   
Q34.  Patient  with  lytic  lesion  in  the  femur,  his  prostate  test  came  normal.  Does  he  have  Paget’s  
disease  or  not?  
   
Answer:    I  have  no  idea  what  they’re  asking  about  
-­‐  Paget’s  disease:  serum  calcium  and  phosphate  =  within  the  reference  range.  
Hyperuricemia,  serum  total  acid  phosphatase  is  high,  bone  specific  alkaline  phosphatase  is  high.  
Q35:  Components  of  the  ankle  joint?  
A.        Distal  tibia,  fibula,  talus  
   
Answer:  A  
Q36.  Patient  cannot  move  her  hand,  where  is  the  defect?  
A.      Anterior  interosseous  
B.      Posterior  interosseous  
C.      Thenar  muscles  
   
Answer:  
Anterior  interosseous  compressive  neuropathy:  weakness  of  grip  and  pinch,  specifically  thumb,  
index  and  middle  finger  flexion,  patient  unable  to  make  OK  sign  (test  FDP  and  FPL),  pronator  
quadratus  weakness  shown  with  weak  resisted  pronation  with  elbow  maximally  flexed.  
(Orthobullets)  
   
Posterior  interosseous  compressive  neuropathy:  finger  metacarpal  extension  weakness  and  wrist  
extension  weakness;  inability  to  extend  wrist  in  neutral  or  ulnar  deviation,  the  wrist  will  extend  with  
radial  deviation  due  to  intact  ECRL  (radial  n.)  and  absent  ECU  (PIN).  
(Orthobullets)  
   
Thenar  muscle  action:  opposes  the  thumb,  by  medially  rotating  and  flexing  the  metacarpal  on  the  
trapezium.  
(Techmeanatomy)  
   
Q37.  Diabetic  patient  experiences  pain  with  stretching  the  second  and  third  fingers  and  she  cannot  
move  her  hand.  (picture  attached).  What  is  the  diagnosis?  
 Answer:  depends  on  the  picture  
   
Q38.  Post  trauma  patient  cannot  flex  the  distal  phalanx,  which  tendon  is  ruptured?  
 Answer:  Flexor  digitorum  profundus  
   
Q39.  When  the  blood  supply  in  the  superficial  palmar  arch  is  weak,  which  artery  is  responsible?  
   
Answer:  Ulnar  artery  
The  superficial  palmar  arch  is  formed  predominantly  by  the  ulnar  artery,  with  a  contribution  from  
the  superficial  palmar  branch  of  the  radial  artery.  
(Wikipedia)  
   
Q40.  Function  of  the  anterior  compartment  forearm  muscles?  
A.        Elbow  flexion  
B.        Elbow  extension  
C.        Hand  and  wrist  flexion  
D.        Hand  and  wrist  extension  
 Answer:  C  
   
Q41.  Picture  of  humeral  fracture,  then  the  patient  had  weakness  in  extension.  Which  nerve  in  
injured?  
   
Answer:  radial  nerve  
   
Q42.  Patient  with  hip  dislocation  and  the  leg  is  externally  rotated,  which  muscle  is  responsible  for  
the  external  rotation?  
   
Answer:  gluteus  maximus  
Q43.  13-­‐year-­‐old  boy  felt  down  and  came  with  forearm  swelling  and  pain.  X-­‐ray    pic  provided,  
something  special  on  x  ray  that  is  the  growth  plates  are  widely  separated?  
A.      Salter  Harris  fracture  
   
Answer:  A  
   
Q44.  Septic  arthritis,  culture  shows  gram  positive  cocci  in  clusters,  the  patient  was  given  Cloxacillin  
with  no  improvement  after  5  days,  what  will  you  do?  
   
Answer:  Start  Vancomycin  
Q45.  Squash  player  with  elbow  pain,  what  is  the  diagnosis?  
   
Answer:  lateral  epicondylitis  
   
Q46.  Old  female  with  back  pain  relieved  with  leaning  forward  and  walking  uphill,  peripheral  pulses  
are  intact,  what  is  the  diagnosis?  
   
A.          lumbar  spinal  stenosis  
   
Answer:  A  
Q47.  Case  of  carpal  tunnel  syndrome,  the  pt  is  a  typist  working  on  computer,  nerve  affected?  
A.        Median  nerve  
   
Answer:  A  
Q48.  Non  athlete  forcefully  plantar  flexed  his  foot  and  heard  a  click  sound  on  his  leg,  which  tendon  is  
affected?  
A.        Calcaneal  tendon  
   
Answer:  A  
Q49.  Which  muscle  would  be  completely  paralyzed  by  obturator  nerve  injury?  
A.        Gluteus  Maximus  
B.        Adductor  magnus  
C.        Adductor  longus  
   
Answer:  B&C  !!  (I  think  one  of  the  answers  was  added  by  the  one  who  wrote  the  question)  
Obturator  nerve  supplies:  external  obturator,  adductor  longus,  adductor  brevis,  adductor  magnus,  
gracilis  and  the  pectineus  (inconstant).  
(Wikipedia)  
Q50.  What  is  the  cause  of  carpopedal  spasm  ?  
A.        Low  Ca  
   
Answer:  A  
Acute  secondary  hypocalcemia  can  result  in  carpopedal  spasm,  muscle  twitching,  a  prolonged  QT  
interval,  and  positive  Chvostek  and  Trousseau  signs.  
(Medscape)  
Q51.  Drop  foot  &  loss  of  sensation  in  1st  &  2nd  digit,  which  nerve  is  affected?  
A.        Deep  peroneal  nerve  
   
Answer:  A  
Deep  peroneal  (fibular)  nerve  supplied  the  skin  of  the  adjacent  sides  of  the  big  and  second  toes.  
Q52.  Knee  pain  increase  by  walking  on  the  stairs  relieved  by  walking  on  flat  floor  this  pain  
aggravated  by  contracting  quadriceps  muscle,  what  is  the  cause?  
A.        Osteoarthritis  
   
Answer:    Patellar  femoral  syndrome    
 
Q53.  Presentation  of  posterior  hip  dislocation?  
   
Answer:  hip  and  leg  in  slight  flexion,  adduction,  and  internal  rotation.  
Q54.  Patient  got  numbness  and  electrical  like  pain  of  the  left  foot  up  to  knee,  so  at  which  level  is  the  
disc  ?  
A.        L3-­‐L4  
   
Answer:  L4-­‐L5  
   
Q55.  Patient  fall  down  over  his  leg,  x-­‐ray  shows  comminuted  fracture  of  the  tibia  and  fibula,  the  
dorsalis  pedis  pulse  is  normal,  sensation  normal  what  is  the  next  step?  
A.      Doppler  
B.      MRI  
   
Answer:  A  
Q56.  Most  common  nerve  injured  in  humerus  fracture?  
A.        Radial  nerve  
B.        Axillary  nerve  
C.        Ulnar  nerve  
D.        Median  nerve  
   
Answer:  B  
Q57.  What  is  the  type  of  wrist  joint?  
A.        Pivot  
B.        Hinge  
   
Answer:  Condyloid  joint  
Q58.  Humeral  neck/head  fracture,  which  nerve  would  be  affected?  
A.        Axillary  
B.        Radial  
C.        Ulnar  
D.        Median  
   
Answer:  A  
Q59.  Lachman  test  assess  which  of  the  following  structures?  
A.        Anterior  cruciate  ligament  
   
Answer:  A  
Q60.  Loss  sensation  of  the  anterior  and  posterior  aspect  of  the  hand,  which  nerve  is  affected?  
A.      Axillary  
B.      Radial  
C.      Ulnar  
D.      Median  
   
Answer:  C  
.  
Q61.  Loss  sensation  of  the  little  and  ring  finger,  which  nerve  is  affected?  
A.      Axillary  
B.      Radial  
C.      Ulnar  
D.      Median  
   
Answer:  C  
Q62.  Patient  complains  of  neck  pain  and  tingling  that  radiates  to  his  left  shoulder  and  arm  with  loss  
of  sensation  of  the  arm  following  the  pain  most  likely  dx?  
A.        Cervical  disc  prolapse  
   
Answer:  A  
Q63.  Girl  with  unilateral  knee  pain  and  high  fever,  no  hx  of  trauma,  knee  aspiration  results  showed  
high  wbcs,  what  to  do?  
A.      Surgical  consultation  and  IV  antibiotic  
B.      Oral  antibiotic  for  5  days  
C.      IV  antibiotic  for  2  weeks  
D.      Antipyretic  and  wait  for  lab  results  
   
Answer:  A  
   
Q64.  onion  skin,  periosteal  elevation,  progressive  pain  for  2  weeks?  
A.        Blood  culture  
B.        IV  antibiotics  
C.        MRI  
   
Answer:  C  
It’s  ewing’s  sarcoma  
Q65.  Patient  had  MVA  before  3  day  today  complaining  of  mild  neck  pain  with  stiffness?  Without  
neuro  deficit?  Initial  workup  
A.        X-­‐ray  
B.        MRI  
C.        CT  
   
Answer:  A  
   
Q66.  How  to  diagnose  scoliosis?  
A.        X-­‐ray  
B.        Adam’s  test  
C.        MRI  
   
Answer:  A  
   
Q67.  Which  degree  of  scoliosis  you  should  do  surgery?  
A.        10  
B.        15  
C.          5  
   
Answer:  cobb  angle  >  45°  
(Orthobullets)  
for  surgery  if  more  than  40-­‐45  or  symptomatic    
for  ortho  referral  if  more  than  15-­‐20  
 
Q68.  Wrist  pain  with  +ve  finkelstein's  test  and  –ve  phalen  test  how  to  treat?  
A.      Split  for  the  thump  
B.      Split  for  the  wrist  
   
Answer:  A  
First  line  treatment  of  De  Quervain's  tenosynovitis  is:  rest,  NSAIDS,  thumb  spica  splint,  steroid  
injection.  
(Orthobullets)  
   
Q69.  Elderly  male  with  history  of  3  fracture  in  2  years.  What  is  the  management?  
A.      Alendronate  
B.      Teriparatide  
C.      Selective  oestrogen  receptor  modulators  
   
Answer:  A  
Q70.  Fracture  of  the  head  and  neck  of  the  fibula,  which  nerve  is  injured?  
A.        Common  peroneal  
B.        Deep  perineal  
C.        Superficial  
   
Answer:  A  
   
Q71.  a  picture  of  complete  fracture  a  small  wound,  management?  
A.        Surgical  detriment  +  intramedullary  nail  
B.        Closed  reduction  +  external  fixation  
   
Answer:  A  
   
Q72.  Patient  post  motor  collision  3  days  back  came  with  neck  pain  which  started  early  as  mild.  Now  
he  has  neck  pain  and  stiffness  that  radiates  to  the  arms.  Which  one  of  the  following  is  most  likely  
diagnosis?  
A.        Disc  prolapse  
B.        Spondylosis  
C.        Subluxation  
   
Answer:  C  
Q73.  Picture  of  fracture  of  both  ulnar  and  radial,  management?  
A.      Warm  compression          
B.      Rest  and  reassurance  
C.      Reduction  and  casting  
   
Answer:  C  
Q74.  Best  way  to  prevent  fracture  in  osteopenic  postmenopausal  lady?  
A.        Daily  Vit  D  
   
Answer:  Bisphosphonate  or  calcium  +  vitamin  D  
http://clinicalevidence.bmj.com/x/systematic-­‐review/1109/overview.html  
   
Q75.  Old  patient  did  DEXA  scan  with  picture  of  osteoporosis  had  generalized  bone  pain  and  bone  
tenderness  and  high  Alkaline  phosphatase,  what  is  Dx?  
A.        Osteoporosis  
B.        Osteomalacia  
C.        Paget’s  disease  
D.        Metastasis  
   
Answer:  A  
Q78.  Back  pain  assessment  and  importance  of  onset  and  duration.  
A.      Short  onset  is  important  for  biological  origin.  
B.      Location  and  onset  are  important  for  the  biological  origin  
C.      Location  and  onset  are  important  for  the  outcome  
   
Answer:  A  (not  sure)  
Q79.  Features  of  osteoarthritis  in  x-­‐ray.  
   
Answer:  joint  space  narrowing,  osteophytes,  eburnation  of  bone,  subchondral  sclerosis,  and  
subchondral  cysts.  
(Orthobullets)  
   
Q80.  5-­‐year-­‐old  girl  diagnosed  perthes  disease,  treatment?  
A.      Surgery  
B.      No  weight  bearing  for  6  months  
   
Answer:  B  (see  above)  
Q81.  In  which  position  will  you  apply  a  splint  in  patients  with  carpal  tunnel  syndrome?  
A.      Dorsiflexion  
B.      Plantarflexion  
C.      Extension  
D.      Abduction  
   
Answer:  C  
Q82.  Young  boy  comes  with  knee  pain..whole  scenario  is  given..is  sporty..diagnosis  of  Osgood  
schlatter  is  made..the  knee  pain  is  due  to?  
A.      Traction  
B.      Tear  of  medial  ligament  
C.      Cruciate  lig  tear  
D.      Bursitis  
   
Answer:  maybe  A  
(The  pain  is  caused  by  stress  from  the  quadriceps  tendon  pulling  on  the  tibial  tuberosities  during  
rapid  growth  spurts).  
   
Q83.  Child  tripped  and  twisted  his  leg  now  he  can’t  walk,  what  is  the  cause?  
A.      Spiral  fracture  of  proximal  tibia  
B.      Spiral  fracture  of  femur  
C.      Chop  fracture  of  tibia  
   
Answer:  A  
Q84.  Tingling  and  numbness  of  the  ring  and  index  finger  worsen  with  elevation  of  the  arm  positive  
stress  test?  
A.      Carpal  tunnel  syndrome  
B.      Thoracic  outlet  obstruction  
C.      Ulnar  artery  thrombophlebitis  
 Answer:  B  
 
Q85.  Pain  at  the  midline  of  the  plantar  surface  when  walking  and  standing?  
A.      Plantar  fasciitis  
B.      Flexor  digitorum  longus  
C.      Flexor  hallucis  longus  
 Answer:  A  (not  sure)  
   
Q86.  Loss  of  adduction  of  the  fingers  which  nerve  injury?  
A.    Ulnar  
B.      Radial  
C.      Median  
D.      Axillary  
 Answer:  A  
 
Q87.  Patient  cannot  extend  his  leg  at  knee  joint  which  muscle  is  responsible?  
A.        Sartorius  
B.        Quadriceps  
C.        Biceps  femoris    
Answer:  B  
 
Q88.  Radius  +  ulna  fracture,  what  to  do  ?  
A.      Irrigation  
B.      Closed  reduction  
   
Answer:  you  need  to  know  the  type  of  the  fracture  +  the  location  of  the  fracture  rather  proximal  or  
distal  +  the  description  of  the  wound.  
 
Q89.  Case  about  patient  with  osteoarthritis  had  enlargement  of  the    distal  interphalangeal  joint  what  
is  it?  
A.      Heberden's  node  
B.      Bouchard  node  
   
Answer:  A  
   
Q90.  Type  of  hip  joint:  
A.        Ball  and  socket  joint  
   
Answer:  A  
   

Q91.  30-­‐year-­‐old  patient  presents  with  back  pain  and  tenderness  when  palpating  the  paraspinal  
muscle,  neurovascular  was  normal  and  ask  about  treatment?  
A.      Physiotherapy  
B.      Biofeedback  
C.      Surgery  
   
Answer:  A  
   
Q92.  Pain  in  snuff  box  most  likely  indicate  fracture  in  which  of  the  following?  
A.      Scaphoid  
   
Answer:  A  
   
Q93.  Boy  with  hip  pain  x  ray  shows  degenerative  and  collapse  of  femoral  neck  what  the  diagnosis?  
A.      Perthes  disease  
   
Answer:  A  
   

Q94.  2-­‐year-­‐old  child  fell  on  stairs,  presented  with  spiral  tibial  fracture  with  multiple  contusions  on  
various  healing  stages  what  you're  going  to  do?  
A.      Hospitalization  and  call  the  social  services  
B.      Referral  to  orthopedics  
C.      Cast  and  discharge  
   
Answer:  A  
Could  be  child  abuse  
   

Q96.  Case  of  osgood  schlatter  disease  came  with  bilateral  knee  swelling,  what  is  the  cause?  
A.      Osteochondritis  
B.      Hemorrhage  in  the  bursa  
C.      ACL  tear  
   
Answer:  A  (most  likely,  I  did  my  research  and  I  honestly  don’t  know  the  right  answer)  
Osteochondritis  
characterized  
knee,   the  medial   by  ddfissecans  
emoral   ciondyle  
egeneration   s  an  aintra-­‐articular  
nd  
is  rm osteochondrosis  
e-­‐calcification  
ost  commonly   rticular  coartilage  
of  aaffected.   f  unknown  
and  eutiology  
nderlying   that   is   In  the  
bone.  
The  pmatient  
effusion.  
also  
tenderness   ay  Ibf  e  aar  reports  
long  
leported.  
oose  the  
bvody  
ague,  
involved  
On  
is  ppphysical  
resent,  
oorly  
chondral  
leocalized  
mxamination,  
echanical  
surface.  
knee  
stymptoms  
A phe  
ain,  
 mpild  
atient  
as  
joint  
woell  
f  melocking  
aay  
ffusion  
s  mdemonstrate  
orning  
om
r  cay  
atching  
sbtiffness  
e  pqresent.  
uadriceps  
of  otr  
he  recurrent  
knee  
atrophy  
joint   or  
(American  family  physician)  

   

Q97.  Which  drug  of  the  following  used  in  the  treatment  of  osteoporosis  and  cause  constipation?  
A.      Bisphosphonates  
B.      Denosumab  
C.      Cinacalcet  
   
Answer:  A  
   
Q98.  Worker  with  elbow  pain,  exaggerated  by  hammer  use.  What  is  the  cause?  
A.      Lateral  epicondylitis  
B.      Biceps  tendonitis  
   
Answer:  A  
   
Q99.  Female  wearing  a  high  heel  falls  her  leg  moved  outward  (  or  something),  what  is  the  injured  
ligament?  
A.      Lateral  collateral  ligament  
B.      Deltoid  ligament  
C.      Anterior  cruciate  ligament  
   
Answer:  B  
An  eversion  ankle  sprain  is  a  tear  of  the  deltoid  ligaments.  
 
Q100.  Nerve  supply  of  the  calf  muscle?  
A.      Tibial  nerve  
B.      Femoral  
 Answer:  A  
   
Q101.  Type  of  joint  between  vertebra?  
A.      Cartilaginous  joint  
   
Answer:  A  
   
Q102.  Superficial  layer  of  the  sole  of  the  foot  had  transverse  cut,  which  structure  affected?  
A.      Tibialis  posterior  
B.      Abductor  hallucis  
   
Answer:  B  
   
Q103.  Injury  to  neck  of  fibula,  loss  of  dorsiflexion  and  there's  slight  eversion,  which  nerve  is  
affected?  
A.      Deep  peroneal  nerve  
B.      Common  peroneal  nerve  
   
Answer:  B  
   
Q104.  Long  scenario  about  median  nerve  injury.  What  is  the  abnormality  you  expected  to  see  in  
hand  ?  
A.      Ape  hand  
B.      Claw  hand  
C.      Wrist  drop  
   
Answer:  A  
   
Q105.  Injury  leads  to  loss  of  sensation  in  the  medial  2  fingers.  What  is  the  nerve  injured  ?  
A.      Ulnar  nerve  
B.      Median  nerve  
C.      Brachial  nerve  
Answer:  A  
   
Q106.  Female  typing  on  computer  giving  hx  of  carpal  tunnel  syndrome,  what’s  the  accurate  test?  
A.      Tinel’s  test  
B.      Compression  test  
   
Answer:  B  
   
Q107.  Patient  presented  with  bilateral  knee  pain  and  on  examination  you  found  joint  crepitus  and  
muscle  wasting  around  knee?  
A.      Rheumatoid  Arthritis  
B.      Osteoarthritis  
C.      Chronic  pain  syndrome  or  something  like  that  
   
Answer:  B  
   
Q108.  Player  develop  pain  with  movement,  got  better  with  rest.  What  is  the  diagnosis?  
A.      Plantar  fascitis  
B.      Calcaneus  bone  spur  
C.      No  tarsal  tunnel  in  the  choices  
   
Answer:  A  (could  be)  
You  cannot  judge  based  on  the  given  information  only.  
   

Q109.  Relation  of  femoral  vein  to  artery  


A.      Medial  
Answer:  A  
Medial  to  lateral:  vein,  artery,  nerve.  
   
Q110.  Which  of  these  joint  is  not  hinge  joint?  
A.      Knee  joint  
B.      Ankle  joint  
C.      Elbow  joint  
D.      Hip  joint  
   
Answer:  D  
A  hinge  joint  is  a  common  class  of  synovial  joint  that  includes  the  ankle,  elbow,  knee,  knee.  Jaw,  
Finger,  and  toe  joints.  Hinge  joint  are  formed  between  two  or  more  bones  can  only  move  along  one  
axis  to  flex  or  extend.  
Q111.  Boy  presented  with  limping  which  was  initially  painless  the  became  painful  relieved  by  
rest+xray?  
A.      AVN  
B.      Slipped  capital  femoral  fracture  
   
Answer:  A  
   
Q112.  Patient  fell  on  outstretched  arm  4  weeks  ago  presented  by  weak  abduction  and  external  
rotation,  xray  normal,  they  gave  a  picture  of  an  MRI?  
A.      Shoulder  dislocation  
B.      Impingement  syndrome  
C.      Rotator  cuff  injury  
   
Answer:  C  
The   presentation   suggests   rotator   cuff   injury   although   the   most   common   injuries   with   falling   on  
outstretched   arm   is   scaphoid   and   distal   forearm   fractures   +   the   MRI   attached   should   gives   you   a  
hint.  
   
Q113.  Blood  supply  of  post  compartment  of  the  leg?  
   
Answer:  popliteal  artery  is  the  continuation  of  the  femoral  artery  in  the  popliteal  fossa.  It  descends  
across  the  popliteus  muscle  and  at  its  lateral  border,  divides  into  the  anterior  and  the  posterior  tibial  
arteries.  
   
The  posterior  tibial  artery  gives  off  the  fibular  (peroneal)  artery  and  then  descends  in  the  deep  
posterior  compartment  of  the  leg,  accompanied  by  the  tibial  nerve.  It  passes  behind  the  medial  
malleolus  of  the  ankle  and  into  the  sole  of  the  foot  where  it  divides  into  the  medial  and  lateral  
plantar  arteries.  (Similarly,  the  tibial  nerve  divides  into  medial  and  lateral  plantar  branches  here.)  
   
The  fibular  (peroneal)  artery  is  the  muscular  artery  of  the  fibular  side  of  the  leg.  It  descends  near  the  
fibula,  within  the  substance  of  hallucis  longus  muscle,  in  the  deep  posterior  compartment.  It  also  
serves  as  a  large  collateral  vessel,  for  near  the  ankle  it  is  connected  by  a  horizontal  communicating  
branch  with  the  posterior  tibial  artery  and  by  a  perforating  ramus  with  the  anterior  tibial  artery.  
   
Q114.  70  years  old  female  with  left  hip  pain  while  using  stair,  on  examination  all  range  of  motion  is  
normal  except  abduction,  what  is  the  next  step:  
A.      Radiographs  
B.      NSAIDs  
C.      physiotherapy  and  analgesia  
   
Answer:  A  
   
Q115.  Patient  came  after  bone  fracture  how  to  check  the  bone  density  after  treatment?  
A.      Vitamin  D  
B.      Calcium                                                                                                      
C.        X-­‐ray  Pelvic  and  spine                                                                                                        
D.      High  energy  X  ray  (DEXA)  
   
Answer:  D  
   
Q116.  50  years  Female,  tall,  thin  with  no  comorbidity  her  mother  died  during  orthopedic  
surgery(replace  of  hip  joint),  she  asked  you  about  the  risk  ?  UNCLEAR  Q  
A.      Bone  something  
B.      Bone  something  with  exogenous  estrogen  
C.      Exogenous  estrogen  
   
Answer:  I  don’t  know  
   
Q117.  Athletic  run  3  k  and  suffered  from  leg  pain  not  relieved  by  NSAID  or  ice  bag,  x  ray  normal  ?  
A.      Stress  fracture  
B.      Osteosarcoma  
   
Answer:  A  
   
A118.  How  to  diagnose  rib  fracture?  
A.      PA  Chest  x-­‐ray                                                                                                        
B.      AP  Chest  x-­‐ray                                                                          
C.      Oblique  chest  x-­‐ray  
D.      Erect  position  x-­‐ray  
   
Answer:  C  
   
Q119.  Patient  have  back  pain,  lumbar  pain  and    tenderness,  have  high  renal  function  test,  high  IgG,  
low  IgM  and  IgA.  What  is  the  diagnosis?  
A.      Multiple  myeloma  
   
Answer:  
   
Q120.  Case  of  ESTABLISHED  OSTEOPOROSIS,  what  to  give  to  prevent  osteoporotic  fractures?  
A.        Calcitonin                                                                                              
B.        Bisphosphonate  
   
Answer:  B  
Bisphosphonates  are  of  proven  benefit  in  the  prevention  of  fragility  fractures.    In  addition  to  reduced  
fracture  risk,  interventions  can  improve  quality  of  life  and  reduce  mortality  in  patients  with  fragility  
fractures.Recommendations  for  administration  of  bisphosphonates  include  postmenopausal  women  
aged  65–75  years  if  osteoporosis  is  confirmed  by  dual-­‐energy  x-­‐ray  absorptiometry  scan,  and  older  
women  without  the  need  for  a  scan.  
(Medscape)  
   
Q121.  8  y/o  child  presented  with  hip  pain,  limping,  stiffness  for  the  past  5  months.  He  denied  any  
history  of  trauma.  O/E  there  was  a  fixed  flexion  deformity  of  30  degrees  with  limited  internal  
rotation.  Vital  signs    were  normal.  What  is  most  likely?  
A.      Developmental  Dysplasia  of  the  Hip  
B.      Slipped  Capital  Femoral  Epiphysis  
C.      Legg-­‐Calve-­‐Perthes  disease  
D.      Missed  septic  arthritis  
   
Answer:  C  
   
Q122.  Runner  felt  pain  anterior  to  the  heel  which  worsens  in  early  morning  but  gets  better  
throughout  the  day,  what  is  your  Dx?  
A.      Plantar  fasciitis  
B.      Calcaneal  heel  spur                                                                
C.      Calcaneal  Fracture  
   
Answer:  A  
   
Q123.  Atrophy  of  thenar  muscle,  sensation  is  intact,  which  nerve  is  affected?  
A.      Radial
  
B.      Ulnar  
C.      Median
  
Answer:  C  
   
Q124.  Osteoporosis  most  commonly  due  to?  
A.      Aging  
 Answer:  A  
   
Q125.  Patient  can't  extend  wrist  and  finger,  nerve  injured?  
A.      Radial  nerve  
Answer:  A  
   
Q126.  Child  had  trauma  then  developed  knee  pain  and  swelling  with  tenderness  on  passive  
movement,  next  step  ?  
A.        Blood  culture  
B.        Empirical  Abx  
C.        Knee  x  ray  
D.        Examination  of  synovial  fluid  
   
Answer:  C  
   
Q127.  Male  complaining  of  back  pain.  Imaging  shows  Bony  lytic  lesions  on  multiple  levels  with  moth  
eaten  appearance.  Diagnosis  by?  
A.        Serum  protein  electrophoresis  
B.        Bone  scan                                                                                                                    
Answer:  A  
   
Q128.  Weakness  in  gluteal  area,  the  artery  affected  is?  
A.      Femoral  
B.      Internal    iliac  
C.      External  iliac  
   
Answer:  B  
   
Q129.  Carpal  tunnel  syndrome,  cannot  move  his  fingers,  what  is  affected  muscle  ?  
A.        Palmar  interossei  
B.        Thenar  muscle      
   
Answer:  stupid  question  
The  nerve  affected  in  carpal  tunnel  syndrome  is  median  nerve,  which  innervates  the  thenar  muscles  
responsible  for  the  movement  of  the  thumb,  and  the  median  nerve  IS  THE  ONLY  NERVE  PASSING  IN  
THE  CARPAL  TUNNEL!  
+  Carpal  tunnel  patients  do  not  present  with  inability  to  move  the  fingers.  
The  palmar  interosseous  muscles  adduct  the  fingers  towards  the  middle  finger  +  they  are  supplied  
by  the  deep  branch  of  the  ulnar  nerve.  
Q130.  Closed  humeral  fracture  with  hand  drop,  what  is  the  type  of  nerve  injury  ?  
A.      Neuromatosis  
B.      Neuropraxia  
C.      Axonotmesis  
D.      Avulsion  of  radial  nerve  
   
Answer:  B  
   
Q131.  Case  of  winging  of  scapula,  what  is  the  origin  of  the  affected  nerve?  
A.      Root  of  brachial  plexus  
   
Answer:  A  
The  most  common  cause  of  scapular  winging  is  serratus  anterior  paralysis.  This  is  typically  caused  by  
damage  (i.e.  lesions)  to  the  long  thoracic  nerve.  
This  nerve  characteristically  arises  from  the  anterior  rami  of  three  spinal  nerve  roots:  the  fifth,  sixth,  
and  seventh  cervical  nerves  (C5-­‐C7).  
(Wikipedia)  
   
Q132.  Child  with  fever  and  bilateral  hand  joint  stiffness  along  with  tenderness,  Dx?  
   
Answer:  could  be  rheumatic  fever  
http://emedicine.medscape.com/article/236582-­‐clinical#b1  
Q133.  Most  common  shoulder  dislocation  in  epileptic  patients?  
   
Answer:  posterior  
   
Q134.  Stab  wound  at  post  triangle  of  neck.  Nerve  damage  presentation?  
A.        Unable  to  raise  arm  
   
Answer:  A  
I’m  not  sure,  but  with  injury  to  the  long  thoracic  nerve,  the  patient  won’t  be  able  to  raise  the  arm  
above  the  head.  
   
Q135.  What  is  the  most  common  bone  to  get  fractured  in  the  hand?  
A.        Scaphoid  
   
Answer:  A  
   
Q136.  Boy  complaining  of  leg  pain  and  externally  rotated,  scenario  typical  of  SCFE,  X-­‐ray  provided,  
clearly  showing  SCFE.  What  is  the  diagnosis?  
   
Answer:  Slipped  capital  femoral  epiphysis  
Presentation:  groin  and  thigh  pain,  limp  (antalgic  gait),  sometimes  knee  pain,  loss  of  hip  internal  
rotation,  abduction,  and  flexion,  abnormal  leg  alignment  (externally  rotated  foot  progression  angle),  
weakness  (thigh  atrophy).  
   
(Orthobullets)  
   

Q137.  Diabetic  presented  with  sudden  onset  of  middle  and  ring  finger  pain.  On  examination,  passive  
extension  radiates  the  pain  to  the  wrist.  No  history  of  trauma.  What's  the  treatment?  
A.        Colchicine  
B.        Cefazolin  
C.        Ibuprofen  
D.        Prednisolone  
   
Answer:  C  (not  sure)  
   
Q138.  Typical  Scenario  of  cervical  stenosis  with  an  MRI  picture  asking  about  the  diagnosis.  
Look  up  MRI  pic  of  cervical  stenosis  
   
Q139.  Patient  keeps  his  right  arm  extended  and  Lt  flexed  to  his  waist,  Dx?  
A.        Catatonia  
B.        Dystonia  
   
Answer:  B  (not  sure)  
   
Q140.  PAtient  with  25-­‐30%  lumbar  spinal  stenosis,  what's  the  treatment  ?  
A.      Surgery  
B.      Physiotherapy  
C.      Epidural  steroid  injection  
   
Answer:  B  (I  think)  

Treatment  of  lumbar  stenosis:  nonoperative  (first  line):  oral  medications,  physical  therapy,  and  
corticosteroid  injections.  

Modalities  include:  

●   NSAIDS,  physical  therapy,  weight  loss  and  bracing  


●   Steroid  injections  (epidural  and  transforaminal)  effective  and  may  obviate  need  for  surgery.  
Operative:  wide  pedicle-­‐to-­‐pedicle  decompression.  Indications:  

●   Persistent  pain  for  3-­‐6  months  that  has  failed  to  improve  with  nonoperative  management.  
●   Progressive  neurologic  deficit  (weakness  or  bowel/bladder).  
Wide  pedicle-­‐to-­‐pedicle  decompression  with  instrumented  fusion.  Indications:  
●   Presence  of  segmental  instability  (isthmic  spondylolisthesis,  degenerative  spondylolisthesis,  
degenerative  scoliosis).  
   
Q141.  Pronator  teres  syndrome?                                                                                                          
Answer:  medial  nerve  compression  at  elbow  
 
Q142.  Nerve  involved  in  tarsal  tunnel  syndrome?  
Answer:  tibial  nerve  
   
Q143.  years  old  lady  known  case  of  hypothyroidism  present  with  painful  movement  of  the  right  
shoulder  and  can't  raise  the  shoulder  due  to  this  pain,what's  the  most  likely  diagnosis?  
A.      Rotator  cuff  tear                                                                        
B.      Adhesive  capsulitis    
C.      Impingement  syndrome  
Answer:  B  
I  think  more  details  are  needed,  If  they  mentioned  having  weakness  I  would  choose  rotator  cuff  tear.  
                                                                                                                                                                                                                                           
Q144.  If  you  remove  the  pectoralis  major  muscle,  what's  gonna  happen?  
A.      Loss  of  arm  adduction  
B.      Loss  of  arm  abduction  
C.      Loss  of  arm  adduction  and  internal  rotation                                                                                                      
D.      Loss  of  arm  abduction  and  external  rotation  
 Answer:  C  
   
Q145.  Old  bilateral  deltoid  muscle  weakness  and  neck  stiffness  and  limiting  of  movement?  A.      
Brachial  plexus  neuropathy  
B.      Neck  strain  
 Answer:  cervical  spondylosis  
   
   
 

 ENT  
   
1:  A  little  girl  with  ear  discharge,  bulging  tympanic  membrane,  decreases  TM  vibration  and  
erythema  behind  the  ear.  External  ear  pinna  is  pushed  forward  and  inferior.  What  is  the  Dx?  
A-­‐Mastoiditis  
B-­‐Otitis  Media  
C-­‐Otitis  externa  
Answer:  A  (Key  words,  Pinna  is  pushed  forward  with  erythema  behind  the  ear)  
   
2:  Mastoditis  treatment?  
Answer:  Vancomycin  +  ceftriaxone  
   
4:  Patient  with  parotiditis  and  have  pain  with  eating  that  radiates  to  the  ear,  pain  by  which  
nerve"?  
A-­‐  Auriculotemporal  
Answer:  A,  If  not  choose  facial  nerve  
   
5:  Most  common  parotid  tumor  in  pediatric:  
Pleomorphic  adenoma  
   
Notes:  
Benign  -­‐  pleomorphic  adenoma  then  hemangioma,  
Malignant  -­‐  mucoepidermoid  tumor  
Head  and  neck  tumor:  HPV  is  a  risk  factor  
   
6:  Child  diagnosed  as  serous  OM,  what  is  the  best  management  to  relieve  his  symptoms?  
A-­‐  Myringotomy  
B-­‐  Myringotomy  +  Gommet  insertion  
   
Answer:  B  (https://emedicine.medscape.com/article/858990-­‐treatment)  
   
7:  Epiglottitis  treatment  
Answer:  Ceftriaxone  
   
8:  Thumb  sign  on  x-­‐ray  what  is  the  organism  ?  (Epiglottitis)  
Answer:  Haemophilus  influenza  
   
9:  Nerve  to  behind  ear  and  ...  part  of  auricle  
Answer:  Greater  auricular  nerve  
(It  supplies  the  skin  of  the  face  over  the  parotid  gland,  the  skin  over  the  mastoid  process  and  on  the  
posterior  surface  of  the  auricle.  Small  branch  pierces  the  auricle  to  supply  the  lobule  and  concha)  
https://radiopaedia.org/articles/greater-­‐auricular-­‐nerve  
   
10:  Tonsillitis  case,  which  antibiotics  will  you  use?  
A-­‐  Amoxicillin/  clavilonic  
B-­‐  Vancomycin  
C-­‐  Ciprofloxacin  
Answer:  A  
   
Notes:  Tonsillitis  presents  with  fever,  sore  throat,  foul  breath,  odynophagia,  dysphagia,  tender  
cervical  lymph  nodes  
   
11:  Loss  of  sensation  of  anterior  2/3  of  the  tongue?  
A-­‐  5  
B-­‐  7  
C-­‐  9  
D-­‐  12  
   
Answer:  A  
Loss  of  sensation  on  anterior  2/3  à    Trigaminal  (lingual  branch)  
Loss  of  taste  on  anterior  2/3  à  Facial  (chorda  tympani)  
Loss  of  sensation  and  taste  on  posterior  1/3  à  Glossopharyngeal  
Loss  of  motor  function  of  the  tongue  à  Hypoglossal  
Posterior  part  of  the  root  of  tongue  à    vagus  
   
12:  Man  had  accident  and  loss  of  test  in  the  anterior  two  third  of  the  tongue,  which  nerve  is  
affected?  
Answer:  Facial  nerve  
   
13:  Child  with  left  ear  pain  for  7  days,  had  previous  ear  infection  which  was  treated  with  
antibiotics,  otoscope  shows  bulging  of  tympanic  membrane.  What  is  the  treatment?  (otitis  media)  
A-­‐  Amoxicillin  
B-­‐  Amoxicillin  with  clavulinic  acid  
   
Answer:  B  
   
14:  20  Year-­‐old  patient  is  complaining  of  hearing  loss  in  one  ear,  she  also  mentions  that  she  can  
hear  her  own  voice  louder  in  the  affected  ear.  What  is  the  possible  cause?  
A-­‐  Otitis  media  
B-­‐  Otitis  externa  
C-­‐  Furuncle  in  ear  canal  
D-­‐  Superior  canal  dehiscence  
   
Answer:  D  (https://emedicine.medscape.com/article/857914-­‐clinical)  
   
15:  Most  common  cause  of  tinnitus
  
A-­‐  Chronic  salicylate  use  
B-­‐  Salicylate  poisoning  
C-­‐  Chronic  otitis  media
  
D-­‐  Sensorineural  hearing  loss
  
Answer:  D  (http://www.aafp.org/afp/2014/0115/p106.html)  

16:  Clear  case  of  atopic  patient  with  allergic  rhinitis,  symptoms  and  signs  were  given,  then  asked  
about  the  cells  responsible  for  his  allergy?  
A-­‐  Monocytes  
B-­‐  Macrophages  
C-­‐  Mast  cells  
D-­‐  Neutrophils  
 Answer:  C  

17:  Unilateral  nasal  obstruction,  with  foul  smelling  discharge  and  vestibule  erosion,  his  nasal  exam  
was  normal  ,what  to  do?  
A-­‐  X-­‐ray  head  and  chest  
B-­‐  Nasal  exam  under  GA  
C-­‐  ABx  
   
Answer:  A  (most  likely  foreign  body  insertion)  
   
18:  Which  one  of  the  following  drugs  causes  Vertigo?  
A-­‐  Streptomycin  
B-­‐  INH  
C-­‐  Ethambutol  
   
Answer:  A  (ototoxicity-­‐vestibular)  
   
19:  Cellulitis  in  the  face  of  infants  between  6  and  24  months  with  purple  discoloration  is  caused  
by:
  
A-­‐  GAS
  
B-­‐  SA  
C-­‐  HIB
  
D-­‐  Strep  pneumonia  
   
Answer:  C  (https://emedicine.medscape.com/article/214222-­‐overview#a4)  
   
20:  Most  common  cause  of  hearing  loss  ?  
A-­‐  SNHL  
B-­‐  Conductive  
   
Answer:  A  (http://www.aafp.org/afp/2012/0615/p1150.html)  
   
21:  Patient  has  loss  of  sensation  of  upper  lip  and  inferior  eye  lid,  what  nerve  is  affected?  
Answer:  Infra  orbital  nerve  
(Its  branches  supply  the  skin  of  the  medial  cheek,  lateral  nose  and  upper  lip,  the  mucosa  of  the  
anteroinferior  nasal  septum  and  oral  mucosa  of  upper  lip.)  
https://radiopaedia.org/articles/infraorbital-­‐nerve-­‐2  
   
22:  Patient  underwent  dental  procedure,  then  he  develop  numbness  on  upper  lip  what  is  the  
nerve:  
Inferior  orbital  nerve  
   
23:  child  brought  to  ER  with  barking  cough,  red  epiglottis,  thumb  sign  on  x  ray,  
Best  initial  management?  
A-­‐  Examination  of  epiglottis  
B-­‐  Endotracheal  intubation.  
C-­‐  Emergent  tracheostomy  
D-­‐  Nasopharyngeal  Tube  
Answer:  B  (https://emedicine.medscape.com/article/763612-­‐treatment)  
   
24:  Signs  of  epiglottitis,  what  is  the  diagnostic  test  ?  
A  -­‐Nasopharyngeal  soap  
B  -­‐X-­‐ray  
C-­‐  Direct  visualization  of  the  epiglottis  
   
Answer:  C  (https://emedicine.medscape.com/article/763612-­‐workup#c8)  
   
25:  Patient  with  submandibular  mass  that  becomes  painful  with  eating  and  chewing,  and  
disappears  after.  on  examination  there  is  a  hard  submandibular  mass.  Diagnosis?  
A-­‐  Acute  sialadenitis  
B-­‐  Submandibular  duct  calculus  
   
Answer:  B  (submandibular  sialadenosis)  (https://emedicine.medscape.com/article/882358-­‐
clinical#b5)  
   
26:  30  years  old  pregnant  with  hearing  loss  and  vertigo,  her  mother  had  the  same  presentation.  
What  is  the  cause:  
A-­‐  Acoustic  neuroma  
B-­‐  Otosclerosis  
   
Answer:  B  (https://emedicine.medscape.com/article/859760-­‐overview#a10)  
   
27:  Most  common  cancer  originates  from  which  paranasal  sinus?  
A-­‐  Ethmiod  
B-­‐  Frontal  
C-­‐  Sphenoid  
D-­‐  Maxillary  
   
Answer:  D  (https://emedicine.medscape.com/article/847189-­‐overview#a8)  
   
28:  Child  has  pale  edematous  nasal  what  to  do  (treatment  of  allergic  rhinitis)?  
Answer:  Intranasal  corticosteroids  
   
29  :  Most  appropriate  treatment  for  nasal  obstruction  ?  
A-­‐  antihistamine.  
B-­‐  Steroid  
   

 
 
30:  Patient  after  a  surgery  in  the  posterior  triangle  of  neck,  developed  loss  of  sensation  on  the  skin  
of  lower  mandible  and  Lower  part  of  the  ear,  what  nerve  is  injured?  
Answer:  Great  Auricular  nerve.  
   
31:  Patient  with  bad  breath  smell,  his  teeth  are  ok  and  he  expectorates  seed  like  particles,  what  is  
the  most  likely  cause?  
Crypt  tonsillitis  
   
32:  Patient  with  tonsillitis,  what  is  the  duration  of  antibiotics?  
A-­‐  5  
B-­‐  7  
C-­‐  10  
   
Answer:  10  days  
   
33:  Facial  nerve  affection,  normal  lacrimation,  abnormal  tympani  reflex  and  loss  of  taste,  site  of  
injury:  
A-­‐  Distal  to  tympani  chordi  
B-­‐  Proximal  to  tympani  chordi  
C-­‐  Distal  to  gbs  nerve  
D-­‐  Proximal  to  gbs  nerve  
   
Answer:  B  (If  the  lesion  is  proximal  to  the  geniculate  ganglion,  the  motor  paralysis  is  accompanied  by  
gustatory  and  autonomic  abnormalities.  Lesions  between  the  geniculate  ganglion  and  the  origin  of  
the  chorda  tympani  produce  the  same  effect,  except  that  they  spare  lacrimation.  If  the  lesion  is  at  
the  stylomastoid  foramen,  it  may  result  in  facial  paralysis  only.)  Medscape  
   
34:  Patient  complain  of  vertigo  for  22-­‐46  minutes  +  tinnitus  +  hearing  loss,  audiogram  show  low  
frequency  hearing  loss,  diagnosis?  
A-­‐  Meniere's  disease  
B-­‐  Acoustic  neuroma  
C-­‐  Vestibule  neuritis  
Answer:  A  
   
35:  Child  with  history  of  URTI  presented  now  with  bilateral  bulging  TM  and  pain  
A-­‐  observe  
B-­‐  Amoxicillin  45/kg  for  5  days  
C-­‐  Amoxicillin  90/kg  for  10  days  
Answer:  C  
   
36:  Moved  pinna  and  narrowed  external  canal,  what  is  the  diagnosis?  
A-­‐  Choleteatoma  
B-­‐  Otitis  externa  
C-­‐  Otitis  media  
Answer:  B  
   
37:  Old  patient  with  a  history  of  vertigo,  imbalance,  tinnitus  with  hearing  loss  what  is  the  next  
investigation?  
A-­‐  CT  temporal  
B-­‐  MRI  of  cerebelloponitine  angle  
C-­‐  Auditory  brainstem  response  
Answer:  B  (Acoustic  neuroma)  
                                                                                                                                                                               
38:  Scenario  about  hearing  loss  of  gradual  onset  in  one  ear  and  vertigo  .pic  of  bell's  palsy.  What  is  
the  diagnosis?  
Answer:    Acoustic  neuroma  
   
39:  young  boy  with  head  trauma  came  with  painful  swelling  in  nose.  on  examination  cystic  
swelling  on  both  sides  of  nasal  septum  was  seen,  what  will  be  your  action?  
A-­‐  Decongentant  
B-­‐  Antihistamine  
C-­‐  X-­‐ray  nasal  septum  
D-­‐  Inscision  and  drainage  
Answer:  D  (septal  hematoma)  
   
40:  Symptoms  of  allergic  rhinitis  and  asked  about  the  diagnosis?  
Answer:  sneezing,  nasal  obstruction,  tearing  and  itching.  
   
41:  What  is  the  most  common  cause  of  epistaxis  in  children?  
Answer:  Nose  picking,  foreign  body  
   
42:  Child  with  painless  hearing  difficulty  ,  tympainc  membrane  not  cleary  seen  
Answer:  Serous  otitis  media  
   
43:  Bilateral  mass  post  auricular  what  is  the  complications:  
A-­‐  Encephalitis  
B-­‐  Hearing  loss  
Answer:  A  
   
44:  40  years  old  man  complaining  of  vertigo  and  tinnitus  associated  with  nausea  and  vomiting,  the  
attack  of  vertigo  last  30  to  45  minutes  (not  sure  about  headache)  
Auditory  test  showed:  low  frequency  sensorineural  hearing  loss.  
What  is  the  most  likely  diagnosis?  
A-­‐  Meniere’s  disease  
B-­‐acoustic  neuroma  
C-­‐vestibular  disease  
D-­‐Paroxysmal  benign  vertigo  disorder  
   
Answer:  A    
45:  Child  with  barking  cough  what  finding  on  auscultation?  
Wheezing  because  of  secretion  
   
46:  Dysphagia  ,swollen  tonsils,  what  will  decrease  the  swelling  in  pharynges?  
A-­‐  Prednisolone  
B-­‐  Aspirin  
   
Answer:  A  
   
47:  Child  with  fever  and  swelling  of  the  ear  canal:  
Answer:  Otitis  externa  
   
48:  What  is  the  most  common  cause  of  hearing  loss  in  children?  
A-­‐  Eustachian  tube  dysfunction  
B-­‐  Prenatal  maternal  infection  
C-­‐  Otitis  media  with  secretion  
   
Answer:  C  
   
49:  17  year  boy  with  nasal  bleeding  several  minutes  ago.no  previous  h/o  bleeding  disorder,  now  
there  is  slow  bleeding  from  posterior  septum,  initial  action?  
A-­‐  Put  him  in  left  recumbent  position  
B-­‐  Posterior  nasal  packing  
C-­‐  Analgesic  and  vasoconstrictor  nasal  spray  
Answer:  B  ,  in  SMLE  12  the  answer  was  C    
   
50:  Patient  with  epistaxis.  On  examination  only  bleeding  point  was  Posterior  inferior  nasal  
septum,  which  vessel  is  affected?  
Answer:  Sphenopalatine  
   
51:  First  step  epistaxis?  
A-­‐  Nasal  packing  
B-­‐  Lying  side  ways  
C-­‐  Pinching  fleshy  part  together  
Answer:  C  
   
52:  Pt  woke  up  with  aphonia,  but  can  cough?  
A-­‐  Functional  aphonia
  
B-­‐  Laryngeal  cord  paralysis  
 Answer:  A    
53:  Patient  with  squamous  cell  carcinoma  on  his  lower  lip,  in  which  lymph  node  will  be  drained?  
A-­‐  Submandibule  
B-­‐  Submental  
Answer:  B  
   
54:  Patient  had  occipitofrontalis  paralysis  which  branch  of  facial  nerve  is  affected?  
A-­‐  Temporal  
B-­‐  Buccal  
Answer:  A  
   
55:  Case  of  bell’s  palsy,  which  nerve  will  be  intact?  
A-­‐  Massater  
B-­‐  Bucceneter  
C-­‐  Orbicularis  oris  
D-­‐  Orbicularis  oculi  
Answer:  A  (supplied  by  trigeminal)  (https://radiopaedia.org/articles/masseter-­‐muscle)  
   
56:  Child  swallowed  caustic  material  and  presented  with  drooling  of  saliva,  what  is  the  initial  step?  
A-­‐  Secure  airway  
B-­‐  Emergent  endoscope  
C-­‐  Charcol  
Answer:  A  
   
57:  Loss  of  forehead  muscle  action,  which  nerve  is  affected?  
Answer:  Fascial  nerve  
   
58:  Loss  of  gag  reflex  but  normal  uvula,  which  nerve  is  affected?  
A-­‐  glossopharyngeal  
B-­‐  Vegas  
Answer:  A  
   
59:  Lymph  node  of  the  forehead?  
Answer:  Submandibular  
   
60:  Complications  of  mastoiditis?  
Hearing  loss,  Facial  nerve  palsy,  Cranial  nerve  involvement,  osteomyelitis,  Petrositis,  Labyrinthitis,  
Gradenigo  syndrome  (otitis  media,  retro-­‐orbital  pain,  abducent  palsy)  Intracranial  extension  
(meningitis,  cerebral  abscess,  subdural  empyema)  sigmoid  sinus  thrombosis,  abscess  formation  
   
61:  Nasal  &  and  palate  dryness  due  to  obstruction  of?                                                                                                                  
-­‐  submandibular  gland  
-­‐  Submental  
-­‐  Ptergoplatine  
   
62:  Elderly  with  vasomotor  rhinitis  ?                                                                                                                                                            
●   Antihistamines  
●   Decongested  
●   Corticosteroid  
Mild  :  Antihistamine  and  saline  drop  
Sever  :  Steroid  and  anticholinergic  
Usually  as  the  optimal  tx  is  steroid  
http://www.mayoclinic.org/diseases-­‐conditions/nonallergic-­‐  rhinitis/diagnosis-­‐
treatment/treatment/txc-­‐20179179  
 
63:  New  born  delivered  by  forceps  and  affected  on  sternomastoid  foramen  :  
decrease  blood  supply  to  ear  ,  loss  sensation  of  face  same  side  ,  eye  opining  ,  loss  sensation  if  
anterior  2/3  
 
64:  Baby  ,  during  delivery  ,  there  was  stylomastoid  foramen  trauma  ,  what  features  will  be  during  
exams:  
A-­‐  loss  of  eye  close  
B-­‐  Loss  of  facial  sensation  
C-­‐  Loss  of  mastication  function  
Answer:  A  
   
65:  Which  nerve  supplies  biggest  part  of  the  tongue  and  covers  it  the  most?  
A  -­‐  trigeminal  
B  -­‐vagus  
C  -­‐glossopharyngeal  
Answer:  A  
   
66:  In  SNHL  where  is  the  pathology:  
Answer:  Hair  cells  
   
67:  Treatment  of  cholesteatoma?  
Answer:  Surgery    
   
Notes:                                                                                                                                                  
-­‐  Otosclerosis:  AD  
-­‐  Recurrent  URTI  with  psuedomonas  and  atypical  organism:  CF  
-­‐  Compression  on  maxillary  artery:  sphenopalatine  artery  affected  
-­‐  Thyroid  fascia:  pretrachial  fascia,  part  from  deep  cervical  fascia  
-­‐  Accessory  nerve:  in  post.  Triangle  
-­‐  triangle  between  hyoid  bone  and  two  ant.  Belly  of  digastric:  submental  
-­‐  External  laryngeal  nerve:  supply  cricothyroid  muscle  -­‐-­‐>  adduct  cord  to  phone  the  voice  
-­‐  Facial  N  injury  proximal  to  greater  petrosal  nerve  
-­‐  Loss  of  sensation  in:  lower  eyelid,  maxilla,  zygomatic  areas  -­‐-­‐>  maxillary  N  injury  which  give  inferior  
orbital  N  
-­‐  Upper  and  lower  lips  LN  drainage:  submandibular  
-­‐  LN  of  middle  of  face:  submandibular  
-­‐  LN  of  lateral  side,  includes  lateral  side  of  eyelid  and  eyebrow:  parotid  gland  
-­‐  Tip  of  the  tongue  and  center  of  the  lips:  submental  
   
 
 
 
 
 
 
 
 
 
 
 
 
 

Anesthesia  
 
1.          A  diabetic  patient  on  oral  hypoglycemic  drugs  is  going  into  surgery  what  to  do?  
a.     Give  insulin  
   
Answer:  IV  Insulin;  in  general  the  literature  suggests  keeping  glucose  levels  between  150  and  
200 mg/dL.  Both  DM  I  and  II  will  be  given  IV  insulin  with  different  rates  of  infusion.  
Reference:  https://www.ncbi.nlm.nih.gov/pmc/articles/PMC4452499/  
   
2.     Multiparous,  38  wks,  in  labour,  90%  cervical  effacement,  4CM  dilatation,  membranes  
ruptured  e  clear  fluid,  On  CTG,  FHR  dropped  from  140  to  80,  the  pt  platelets=50,000,  what  type  of  
anesthesia  is  suitable?  
a.          General  Anesthesia  
b.          Epidural  
c.          Pudendal  
   
Answer:  General  Anaesthesia;  Regional  anaesthesia  is  absolutely  contraindicated  in  case  of  low  
platelet  count  and  severe  coagulopathy.  However,  risks  and  benefits  of  regional  anaesthesia  should  
individually  be  assessed  in  patients  who  had  low  platelet  count  but  no  clinical  signs  of  coagulation  
disorders.  
References:  https://www.ncbi.nlm.nih.gov/pmc/articles/PMC4894094/  
   
3.     Local  anesthesia  SE  first  sign?  
a.     perioral  numbness.  
   
Answer:  Perioral  numbness;  SE  appears  1-­‐5  min  after  the  injection  which  begins  as  a  CNS  excitatory  
symptoms  “  circumoral/tongue  numbness,  Metallic  taste,  dizzy,  lightheadedness,  visual/audiotary  
disturbances,  Disorientation”    but  with  higher  doses  it  follow  a  rapid  CNS  depressing  status  “  
convulsion,  Coma,  Respiratory  arrest,  
References:  https://emedicine.medscape.com/article/1844551-­‐overview#a1  
   
4.     Anesthesia  in  asthmatic  
a.     ketamine  
Answer:  Ketamine  is  used  in  Asthma,  Hypovolemia,  major  trauma  because  it’s  a  sympathomimetic.  
References:  Toronto  note  2016  
5.      What  anesthetic  drug  can  act  as  analgesic  if  given  in  a  low  dose  
a.     Ketamine  
Answer:  Ketamine;  works  on  NMDA  receptor  and  subanesthetic  dose  will  potentiate  Opioid  
analgesia,  it  has  antitumor,  neuroprotective  and  anti-­‐inflammatory  properties.  
References:  https://www.ncbi.nlm.nih.gov/pmc/articles/PMC4258981/  
   
6.      Which  anesthetic  agent  is  100  times  stronger  than  Morphine?  
a.     Fentanyl  
   
Answer:  Fentanyl;  in  the  literature  its  said  the  Fentanyl  is  80  times  more  potent  than  morphine  and  
hydromorphone  is  20  times  more  potent  than  morphine.  
References:  https://palliative.stanford.edu/opioid-­‐conversion/equivalency-­‐table/  
   
7.     Fentanyl  in  patient  with  Absence  seizure:  increase  stimulation  of  GABA  leads  to  seizure  
   
Answer:  its  well  known  that  fentanyl  produces  an  epileptiform  activity  in  patients  with  epilepsy  
disorders;  the  proconvulsant  properties  of  it  affect  mu  and  delta  opioid  receptor  and  by  inhibiting  
GABAergic  interneurons  which  mediate  an  excitatory  effect  on  neurons  especially  in  the  epileptiform  
focus  zone.  
References:  http://onlinelibrary.wiley.com/doi/10.1046/j.1528-­‐1157.2001.18600.x/full  
   
8.     Drug  given  with  analgesia  to  decrease  its  side  effect  
a.     Metoclopramide  
   
Answer:  Metclopromide;  Dopmaine  2  Receptor  antagonist  and  act  on  the  chemoreceptor  zone  and  
gastric  motility.  
References:  https://www.ncbi.nlm.nih.gov/pmc/articles/PMC2464411/  
9.     Multiparous  woman  everything  was  fine,  except  the  FHR  dropped  from  140  to  80,  also  
mentioned  that  her  platelet  count  is  low,  what  type  of  anesthesia  to  give?  
a.     GA  
Answer:  General  Anaestheia.  
References:  https://www.ncbi.nlm.nih.gov/pmc/articles/PMC4894094/  
   
10.      A  pregnant  lady  was  provided  a  pudendal  nerve  block  as  an  analgesia,  which  structure  would  
be  fully  sensitive  and  not  blocked  by  the  analgesia?  
a.          Rectum  
b.     Perineal  body  
c.     Urogenital  diaphragm  
   
Answer:    Rectum;  Pudendal  nerve  block:  blocks  sensory  and  motor  innervation  to  the  Clitoris  “  
dorsal  nerve”,  skin  of  perineum  [labia  major/minora  and  vestibule]  “  perineal  branch”,  External  anal  
sphincter  and  perianal  skin  “  via  inf.  Hemorrhoidal  N.”  
References:  https://emedicine.medscape.com/article/83078-­‐overview#a1  
   
11.    Side  effect  of  morphine  ?  
a.     nausea  and  vomiting  
Answer:  nausea,  emesis,  constipation,  dizziness.  S/Sx  of  overdose  includes  [  meiosis,  fever,  HTN,  
decrease  responsiveness,  muscle  cramps/pain/stiff/spasms,  Sleepiness]  
References:  https://www.drugs.com/sfx/morphine-­‐side-­‐effects.html  
   
12.    Investigation  to  be  ordered  before  giving  epidural  analgesia?  
a.     Platelet  count  
   
Answer:  low  platelet  count  is  considered  as  a  CI  to  epidural  “  neuro-­‐axial  block”  and  it  must  be  R/O,  
having  plt.  Count  <  80,000  with  bleeding  is  considered  as  a  CI  but  if  without  bleeding  its  considered  
as  a  relative  CI.  
References:  https://www.ncbi.nlm.nih.gov/pmc/articles/PMC3417963/  
13.    In  order  to  increase  hyperbaric  in  cephalid  position  
a.          trendelenburg  position  
b.     Anti-­‐trendelenberg  
   
Answer:  Trendelenburg  position  is  used  to  extend  the  level  of  nerve  block  distribution.  
References:  https://academic.oup.com/bja/article/98/3/396/372828/Trendelenburg-­‐position-­‐with-­‐
hip-­‐flexion-­‐as-­‐a  
14.    pregnanat  women  during  labor  ,  spontaneous  rupture  of  membrane  ,  suddenly  baby  is  stress  
from  140  to  80  what  type  of  Anaesthesia  whould  you  use?  
a.     GA*  
b.     Pedundal  
Answer:  GA  is  usually  the  1st  choice  in  case  of  fetal  compromise.  
References:  
https://books.google.com.sa/books?id=L0shAwAAQBAJ&pg=PA187&lpg=PA187&dq=sudden+fetal+heart+rate+drop+type+of+anaesthesia
.&source=bl&ots=q0EutJSyzk&sig=YWMcE-­‐
hz75I06BkEOFiTyyFmv90&hl=ar&sa=X&ved=0ahUKEwjxmae244nXAhVEvBoKHa5iC9cQ6AEILzAB#v=onepage&q=sudden%20fetal%20heart
%20rate%20drop%20type%20of%20anaesthesia.&f=false  

   
15.    -­‐Problem  in  intubation  
a.          Head  tilt  
b.          Jaw  thrust  
c.          Cricoid  pressure  
Answer:  The  Q  doesn’t  have  sufficient  information.    Head  tilt  is  avoided  in  cervical  injuries  and  Jaw  
thrust  is  used  instead.  Cricoid  pressure  is  applied  in  case  of  risk  of  aspiration.  
References:  4th  year  med  school  course.  
   
16.    The  anesthetic  agent  of  choice  in  bronchial  asthma:  
a.     Nitric  oxide  
b.      Halothane  
c.      Cyclopropane  
d.     Chloroform  
Answer:  Halothane;  Propofol  and  ketamine  inhibit  bronchoconstriction,  decreasing  the  risk  of  
bronchospasm  during  anaesthesia  induction.  Propofol  yields  central  airway  dilation  and  is  more  
reliable  than  etomidate  or  thiopental.  Halothane,  enflurane,  and  isoflurane  are  potent  
bronchodilators  and  can  be  helpful  even  in  status  asthmaticus  
   
References:  https://www.ncbi.nlm.nih.gov/pubmed/17115010  
   
 
 
 
 
 
 

Dermatology  
   
   
1.          -­‐First  line  treatment  of  tenia  pedis:  
A-­‐Systemic  ketoconazole.  
B-­‐  Topical  terbinafine.  
C-­‐Topical  anti-­‐fungal  
   
Answer:  B/C  are  both  the  same  choose  which  ever  they  put  in  the  choices  
   
Most   dermatophyte   infections   can   be   managed   with   topical   treatments.   For   patients   with   limited  
tinea  pedis,  tinea  corporis,  or  tinea  cruris,  we  suggest  treatment  with  a  topical  antifungal  drug  with  
antidermatophyte   activity   rather   than   systemic   therapy   (Grade   1A).   Examples   of   effective   topical  
antifungal   agents   are   azoles,   allylamines,   ciclopirox,   butenafine,   and   tolnaftate.   Oral   antifungal  
therapy   is   used   for   extensive   infections   or   infections   refractory   to   topical   therapy.   Nystatin   is   not  
effective  for  dermatophyte  infections.  (See  'Tinea  pedis'  above  and  'Tinea  corporis'  above  and  'Tinea  
cruris'  above.)  Up  to  date  
   
   
2.           -­‐Patient  developed  cutaneous  lichen  planus  lesion  on  flexor  surfaces  of  the  elbow  bilaterally,  
with   no   mucosal   lesion   on   the   mouth   or   scalp,   which   one   of   the   following   statement   is   correct  
regarding  this  scenario:  
A-­‐  There  is  risk  for  malignancy  transformation.  
B-­‐  Automatic  resolution  over  time.  
C-­‐persistent  lesion.  
D-­‐  Relapsing  and  remitting  lesion.  
Answer:  B  
   
As  classical  cutaneous  lichen  planus  is  a  self-­‐limiting  disease,  routine  monitoring  is  usually  not  
required.  However,  patients  with  erosive  oral  or  genital  lichen  planus  need  to  be  monitored  
regularly  for  possible  squamous  cell  carcinoma  occurring  at  these  sites.  
   
In  cutaneous  disease,  lesions  typically  resolve  within  6  months  (>50%)  to  18  months  (85%);  chronic  
disease   is   more   likely   oral   lichen   planus   or   with   large,   annular,   hypertrophic   lesions   and   mucous  
membrane  involvement    http://emedicine.medscape.com/article/1123213-­‐overview  
In  contrast  to  cutaneous  LP,  which  is  self-­‐limited,  lichen  planus  lesions  in  the  mouth  may  persist  for  
many  years,  and  tend  to  be  difficult  to  treat,  with  relapses  being  common.  Atrophic/erosive  lichen  
planus  is  associated  with  a  small  risk  of  cancerous  transformation,  and  so  people  with  oral  LP  tend  to  
be  monitored  closely  over  time  to  detect  any  potential  change  early.  Sometimes  oral  LP  can  become  
secondarily  infected  with  Candida  organisms  Wiki  
   
   
3.          -­‐Which  one  of  the  following  patients  has  worst  prognosis  to  develop  Steven  Jonson:  
A-­‐HIV  patient  receiving  sulfa  group  medication  with  unknown  history  
B-­‐gouty  patient  with  known  sensitivity  to  allopurinol  
Answer:  B  
   
   
4.           -­‐A   Woman   diagnosed   previously   with   lichen   sclerosis   in   vulva,   she   has   mass   in   vulva   and  
biopsy  taken  ,  what  suspect  to  has  ?  
A.          Squamous  cell  carcinoma  
B.          Adenocarcinoma  
C.          Squamoadenocarcinoma  
D.        Melanoma  
        Answer  :A  
    LS   and   cancer   —   Women   with   LS   affecting   the   vulva   are   at   a   slightly   increased   risk   for  
developing    squamous  cell  skin  cancer.  Up  to  Date  
   
5.          -­‐Which  of  the  following  is  the  most  malignant  and  needs  to  be  removed  
A.          Erthyma  migrans    lyme  disease  
B.          Erythyma  marginatum    rhueamtic  fever  
C.          Erythema  gyratum  sign  of  underlying  lung  cancer  
D.        Erythema  annulare      
Answer:  C  
   
Erythema  gyratum  repens  often  precedes  the  detection  of  malignancy.  The  skin  eruption  is  present  
an   average   of   9   months   prior   to   the   diagnosis   of   malignancy,   with   a   range   of   1-­‐72   months.   In   a  
minority   of   patients,   erythema   gyratum   repens   occurred   simultaneously   with,   or   up   to   9   months  
after,  the  detection  of  the  neoplasm.  
   
6.          -­‐Patient  using  sulfa  drug  there  is  lesion  in  glans  of  penis  ask  about  descriptions?  
   
Answer:  The  characteristic  presentation  is  a  pruritic  or  burning,  sharply  circumscribed,  round-­‐to-­‐oval  
patch  with  violaceous  or  dusky  erythema.  Scaly,  erythematous  and  ulceration.  AAFP  
   
7.           -­‐Patient  came  with  lesion  in  the  labia  majora  0.5cm  x  0.5cm,  they  took  biopsy,  what  will  the  
histopathology  result  will  be?  
A-­‐Adenocarcinoma  
B-­‐  Squamous  cell  carcinoma  
C-­‐Adeno  squamous  cell  carcinoma                      
Answer:B  
   
8.           -­‐   Patient   came   with   plaque   in   the   labia   majora,   what   is   the   diagnosis?   (The   question   is  
incomplete  )  
A-­‐Basal  cell  carcinoma  
B-­‐Melanoma                                                
C-­‐Lupus  pernio  
   
Lupus  Pernio  –  no  because  its  Sarcoidosis    
Melanoma  a  lump  or  growth  in  the  vagina  
Basal  cell  is  unlikely  as  its  in  sun  exposed  area  
   
9.          -­‐  Patient  you  gave  her  retin  A  for  acne,  you  will  warn  her  from  what?  
A-­‐Increased  breast  tissue  
B-­‐Exposure  to  the  sun  
   
Answer:  B  
   
10.    -­‐  Rosacea  treatment?  
Answers:  Doxycycline  "tetracycline"    is  the  first  option  if  not  available  erythromycin  
   
11.     -­‐female   has   genital   warts   over   the   past   years,   genital   warts   are   associated   with   what?  
(Incomplete)  
A-­‐hyperkeratosis  à  is  the  description  of  the  wart  not  an  association  
   
The   most   common   type   of   eruption   is   a   morbilliform   (resembling   measles)   or   erythematous   rash  
(approximately   90%   of   cases).[4]   Less   commonly,   the   appearance   may   also   be   urticarial,  
papulosquamous,  pustular,  purpuric,  bullous  (with  blisters)  or  lichenoid.[3]  Angioedema  can  also  be  
drug-­‐induced  (most  notably,  by  angiotensin  converting  enzyme  inhibitors).    
   
   
12.    -­‐    An  athlete  with  itchy  pink  rash  between  his  thighs,  what  to  prescribe?  
A-­‐Tobical  Abx  
B-­‐Topical  steroid  
C-­‐  Topical  antifungal,  selenium  sulfide  (tinea  cruris)  
Answer:  C  
   
Tinea  cruris  is  usually  treated  with  topical  antifungal  agents.  Sometimes  hydrocortisone  is  added,  for  
faster   relief   of   itch.   Topical   steroids   should   not   be   used   on   their   own.   If   the   treatment   is  
unsuccessful,   oral   antifungal   medicines   may   be   considered,   including   terbinafine   and   itraconazole.  
Dermnetz  
   
13.    -­‐  Patient  on  multiple  drugs,  developed  violaceous  papule  and  eczematous  pattern  in  the  trunk,  
no  mucus  membrane  involving,  what  dx?  
A-­‐Lichenoid  eruption  
B-­‐Actinic  eruption  
C-­‐TEN  
D-­‐erythema  multiforme  
Answer:  A  
Extensive   rash   distributed   symmetrically   over   the   trunk   and   limbs   Photodistribution   –   the   rash   is  
predominantly   in   areas   exposed   to   the   sun   Rash   may   be   scaly   resembling   eczema   or   psoriasis  
Wickham   striae   are   usually   absent   Nail   and   mucous   membrane   (e.g.,   mouth)   involvement   is  
uncommon  (oral  lichen  planus)More  likely  to  resolve  leaving  marked  pigmentation  
   
14.    -­‐Leg  ulcer,  with  rolled  out  edge?  
A-­‐SCC  
B-­‐Basal  cell  
   
Answer:  A  
   
15.     -­‐Case   of   skin   disease   covering   the   head   and   extensor   surface   of   the   arms   with   silver   scaling  
that  bleeds  when  scratched.  Covering  around  15%  of  her  body  with  nail  pitting  (Psoriasis).  What  is  
the  most  appropriate  management?  
A-­‐Topical  steroid  
B-­‐PUVA  phototherapy  
C-­‐Methotrexate  
Answer:  C  
   
16.    -­‐Most  common  affected  site  in  lichen  planus?  
A-­‐Mouth  
B-­‐Hands  
Answer:  A  
   
17.    What  is  the  first  immune  responsible  agent  in  the  skin?  
A-­‐Keratinocyte  
   
Answer:A  
   
They  are  in  the  outer  most  part  of  the  epidermis  the  stratum  cornea  they  act  as  physical  barriers  and  
produce   antimicrobial   peptides   and   the   deeper   layer   keratinocytes   initiate   an   early   immune  
response  –  Journal  of  clinical  and  experimental  dermatology  
   
18.     -­‐Pt   with   large   painful   nodule   Over   the   nose   with   telangiectasia   over   the   face   (Picture   of  
rosacea)?  
a)Doxycycline  
b)Clindamycin  
c)topical  retinoic  acid  
d)topical  steroid  
Answer:  A  
   
19.     -­‐19   y/o   female   with   vitiligo   in   the   face   and   hand   the   size   is   increasing   despite   medical  
treatment  now  the  patient  wants  to  get  married  and  ask  you  what  to  do?  
A-­‐continue  medical  treatment.  
B-­‐stop  Medical  treatment  and  wait  
C-­‐skin  graft.  
D-­‐melanocyte  transfusion.  
   
Answer:  Most  Probably  A    
As  for  C  and  D  the  disease  has  to  be  stable  for  6-­‐12  months.  DermNet.org  
   
20.     -­‐picture   of   patient   with   wheal   in   fore   arm,   and   complain   of   hand   pain   after   bee   sting  
associated  with  enlarged  axillary  lymph  node  what  is  the  diagnosis?  
A-­‐Urticaris  
B-­‐lymphangitis  
 Answer:B  
 
 
21.    -­‐Acnes  with  pustules  and  nodules  what  type  of  acnes?  
A-­‐Inflammatory  
B-­‐obstructive  
Answer:  A  (100%)  
Obstructive  =  Comedonal  
Inflamatory=  All  other  types  
   
22.    -­‐  Most  common  type  of  eczema?  
A-­‐Cold  
B-­‐Soles  
C-­‐Atopic  Dermatitis  
D-­‐  Dermatographism  
Answer:  C  
Note  that  D  is  for  Urticaria  and  not  Eczema    
   
23.     -­‐A   patient   is   complaining   of   a   skin   mole,   his   father   had   them   as   well   and   remove   them   but  
didn't  know  if  its  benign  or  malignant,  the  patient  is  concerned  that  his  mole  is  malignant,  what  
will  suggest  the  malignancy?  
   
A-­‐Irregular  border  that  fades  in  color  to  be  normal  
B-­‐Equality  of  the  color  
Answer:  A  
   
24.     -­‐DM   patient   with   rash   on   thigh   folds,   diagnosed   as   eczema,   treated   with   azelic   acid,   topical  
steroid  and  Tacrolimus  with  no  improvement,  your  management?  
A-­‐topical  corticosteroid  again.  
B-­‐topical  anti  fungal.  
C-­‐oral  steroid.  
D-­‐powder.  
Answer:  B  
Patients   with   moderate   to   severe   atopic   dermatitis   that   is   not   controlled   with   optimal   topical  
therapy  may  require  phototherapy  or  systemic  immunosuppressant  treatment  to  achieve  adequate  
disease  control.  These  treatments  are  not  suitable  for  infants  and  young  children.  In  older  children  
and  adolescents,  they  should  be  used  when  other  management  options  have  failed  and  the  disease  
has  a  significant  impact  on  the  quality  of  life.  reference:  uptodate  
   
25.    -­‐Treatment  of  non-­‐inflammatory  acne  
A-­‐retinoid  acid  
B-­‐clindamycin  
C-­‐isosertonin  
D-­‐azelaic  acid  
   
Answer:A  
Source:  uptodate  
   
26.    -­‐  What's  the  treatment  of  cold  induced  urticaria  ?  
Answer  :  Antihistamine  
   
27.    -­‐  Old  lady  came  with  bilateral  lower  limb  erythema  and  crusting  (Incomplete)  
 Stasis  dermatitis  could  be  the  answer  if  there  is  history  of  venous  statis  or  its  risk  factors  
   
28.     -­‐Obese  man  develop  painful  lesion  on  gluteus  region,  with  initially  in  the  form  of  mild  lesions  
then  becomes  large  red  painful  (incomplete).  There  was  sinus  on  Examination?  
A-­‐Furunculosis  
B-­‐Hidradenitis  suppurativa  
   
Answer:  B  given  the  Obesity  and  location  
   
29.     -­‐46   year   old   female   came   to   the   clinic   complaining   of   hair   loss   on   the   central   area   of   the   scalp  
and  thinning  of  her  hair  over  the  past  few  years,  Dx?  
A-­‐Female  pattern  hair  loss  
B-­‐androgenic  alopecia  
Answer:  A  
   
30.    -­‐  A  clinical  Description  of  Psoriatic  lesions  without  Psoriasis  in  the  options  
A-­‐SLE  
   
Subacute  cutaneous  Lupus  has  a  variant  called  papulosquamous  which  may  resemble  psoriasis  –Up  
to  Date  
   
31.      -­‐Melanocyte  Producing  Cell  ?  
   
Corticotropes.  
   
32.    -­‐Itching  over  the  face,  Anti-­‐cubital    and  popliteal  fossa  ?  
Eczema  
   
33.    -­‐Nevus  creptus  something  like  this  (child  with  scalp  swelling  with  hair  loss  above  it)  Incomplete  
question  DDX  ?  
In  an  infant  or  young  child,  sebaceous  naevus  presents  as  a  solitary,  smooth,  yellow-­‐orange  hairless  
patch,  often  oval  or  linear  in  shape.Sebaceous  naevi  become  more  pronounced  around  adolescence,  
often  appearing  bumpy,  warty  or  scaly.  
   
34.    -­‐  Description  of  Herpes  simplex  genital  lesions                                      
Answer:  multiple  painful  fluid  filled  vesicles  
   
35.     -­‐  Scenario  of  patient  used  a  medication  for  depression  and  developed  rash  in  all  of  his  body?  
(Incomplete)  
 Answer:  Toxic  Epidermal  Necrolysis  (TEN)  
   
36.    -­‐Diaper  rash  with  satellite  lesion?  
Answer:  Candida  
   
37.     -­‐Typical   case   seborrheic   dermatitis   Greasy   lesion   on   baby   head   +   Picture   of   seborrheic  
dermatitis  
   
38.    -­‐Lesion  from  vulva  with  tree  like  shape  ?  
Answer:  Probably  Wart/HPV  
   
39.    -­‐Nodular  acne  with  pustule  type  of  acne?  
Answer:  Inflammatory  
   
   
40.     SLE+  rash  (papule,  multiple,  itchy,  burning  sensation,  erythematous)  resolve  after  24h  leaving  
hyperpigmentation?  
A.          -­‐discoid  rash  
B.          -­‐pityriasis  rosarea                                                                                                  
C.          -­‐urtecarial...  
Answer  is  acute  Cutaneous  Lupus  Erythematosus  (ACLE)  
http://emedicine.medscape.com/article/1065292-­‐overview  
Cutaneous  SLE  menifestation:  
·∙         Acute  cutaneous  lupus  erythematosus  (ACLE):  characterised  by  a  typical  butterfly  
pattern  malar  rash  involving  the  central  portion  of  the  face  and/or  a  more  generalised  
maculopapular  eruption.  ACLE  is  strongly  associated  with  SLE.  
·∙         Subacute  cutaneous  lupus  erythematosus  (SCLE):  characterised  by  a  non-­‐pruritic,  
non-­‐scarring  dry  rash.  SCLE  may  be  drug  induced  or  can  occur  in  patients  with  SLE,  
Sjogren's  syndrome,  and  complement  C2  deficiency.  
·∙         Chronic  cutaneous  lupus  erythematosus  (CCLE):  lesions  are  confined  to  the  skin  and  
oral  mucosa;  DLE  is  the  most  common  form.  
   
41.    Describe  neva,  What  is  the  evidence  of  neval  hyperplasia?                                                                                          
A.          Change  in  color  
B.          Irregular  border  
C.          All  of  the  above                                                                                                                                                
Answer:  C  
Asymmetry:  Unlike  common  moles,  atypical  moles  are  often  asymmetrical:  A  line  drawn  through  the  
middle  would  not  create  matching  halves.  
Border:  While  common  moles  usually  have  regular,  sharp,  well-­‐defined  borders,  the  borders  of  
atypical  moles  tend  to  be  irregular  and/or  hazy  —  the  mole  gradually  fades  into  the  surrounding  
skin.  
Color:  Common  moles  are  most  often  uniformly  tan,  brown  or  flesh-­‐  colored,  but  atypical  moles  
have  varied,  irregular  color  with  subtle,  haphazard  areas  of  tan,  brown,  dark  brown,  red,  blue  or  
black.  
Diameter:  Atypical  moles  are  generally  larger  than  6  mm  (1⁄4  inch),  the  size  of  a  pencil  eraser,  but  
may  be  smaller.  
Evolution:  Enlargement  of  or  any  other  notable  change  in  a  previously  stable  mole,  or  the  
appearance  of  a  new  mole  after  age  40,  should  raise  suspicion.  
                                                                                                                                                                                                                   
http://www.skincancer.org/skin-­‐cancer-­‐information/atypical-­‐  moles/warning-­‐signs-­‐and-­‐images  
                                                                                                                                           
42.    Diabetic  pt  complain  of  multiple  discharging  sinuses  in  the  back.  Dx:  
A-­‐Infected  lymphoma  
B-­‐Lymphangitis                                                                                                          
C-­‐Carbuncle                                                                                                                    
D-­‐Furuncle  
Answer:  C  
Carbuncle  :  is  an  abscess  and  its  associated  with  DM,  obesity  and  poor  hygeine.  essentially  ,  it’s  a  
bunch  of  furuncles  connected  under  the  skin.  
   
43.     Female  with  polycystic  ovarian  syndrome  noticed  hyperpegminted  skin  in  her  neck  and  axilla,  
what  is  this  abnormality  called?  
a.  Acanthosis  negricans  
b.  Linea  nigra                                                                                                                    
Answer:  A  
   
44.    A  case  of  xeroderma  pigmentosum  ,  what’s  the  mechanism  of  action  ?  
   
characteristic  dry,  pigmented  skin.  Xeroderma  pigmentosum  is  a  rare  disorder  transmitted  in  an  
autosomal  recessive  manner.  It  is  characterized  by  photosensitivity,  pigmentary  changes,  premature  
skin  aging,  and  malignant  tumor  development.,  These  manifestations  are  due  to  a  cellular  
hypersensitivity  to  ultraviolet  (UV)  radiation  resulting  from  a  defect  in  DNA  repair.  
http://emedicine.medscape.com/article/1119902-­‐overview  
   
45.     Patient   3   cm   lump   in   his   upper   back   ,   slowly   growing   for   years   .   Physical   examination   :  
compressible   ,   no   erythema   ,   there   a   punctum   in   the   middle   that   drain   white   foul   smelling  
material  ,  what  is  the  management  !?  
A-­‐Cryotherapy  
B-­‐Total  intact  resection  
C-­‐Antibiotic  and  resection  
Epidermoid  cyst:  Incision  and  drainage  is  the  recommended  treatment  for  inflamed  epidermoid  
cysts,  carbuncles,  abscesses,  and  large  furuncles  
   
46.    koebner  phenomenon  is  a  skin  lesions  appearing  on  lines  of  trauma.  
Induced  by:  skin  trauma.  
Associated  with:  psoriasis,  vitiligo  and  lichen  planus  
   
   
47.     Female   patient   will   get   married   had   bilateral   vitiligo   in   hands   since   3   years   lesion   increasing   in  
size,  want  to  get  rid  of  it,  most  appropriate  step?  
A-­‐Graft  
B-­‐Melanin  transfer  
C-­‐Continue  medication  
D-­‐Stop  medication  
Answer:  C  
   
48.    Most  common  type  of  physical  urticaria:  
A-­‐Water  
B-­‐Cold  
C-­‐Dematographisim  
D-­‐Cholinergic  
Answer:  C  (100%)  
   
49.    Smoker  developed  ulcer  lateral  to  his  tongue  rolled  out  edge  :                                                                                                      
a)  SCC                                                                                                                  
b)  basal  
Answer:  A        
   
50.     Old  patient  smoker  complains  of  white  colored  Plaque  over  the  tongue  with  ulcers  what’s  the  
diagnosis?  
Squamous  cell  carcinoma  
   
51.    Contraindicated  vaccine  in  dermatological  conditions  (eczema,  psoriasis)?  
A-­‐  Polio  
B-­‐  Measles  
C-­‐  Smallpox  
D-­‐  German  measles  (rubella)  
Answer:  C  (not  sure)  only  live  vaccine  of  the  lot  
   
52.    Eczema  with  topical  steroid  what  to  add:  
Answer:  Tacrolimus  
   
53.    Greasy  Scaly  rash  at  edge  of  forehead  and  over  cheeks  not  sparing  folds.  Treatment:  
Answer:  mupirocin  topical  
   
Topical  antibacterial:  indications  include  toxic  shock,  syndrome,  impetigo,  secondery  bacterial  skin  
infections  ,,...  u  may  see  it  under  the  name  bactroban  
   
54.    91-­‐Patient  came  with  buttock  lesion:  
A-­‐Basal  cell  carcinoma  
B-­‐Small  cell  carcinoma  
C-­‐Mycosis  fungoides  
Answer:  C  
   
Review  cutaneous  Tcell  lymphoma  
The  signs  and  symptoms  of  CTCL  vary  depending  on  the  type.  The  two  most  common  types  are  
mycosis  fungoides  and  Sézary  syndrome.  
Classic  mycosis  fungoides  is  divided  into  the  following  3  stages:  
·∙        Patch  (atrophic  or  nonatrophic):  Nonspecific  dermatitis,  patches  on  lower  trunk  and  
buttocks;  minimal/absent  pruritus  
·∙        Plaque:  Intensely  pruritic  plaques,  lymphadenopathy  
·∙        Tumor:  Prone  to  ulceration  
Sézary  syndrome  is  defined  by  erythroderma  and  leukemia.  Signs  and  symptoms  include  the  
following:  
·∙        Edematous  skin  
·∙        Lymphadenopathy  
·∙        Palmar  and/or  plantar  hyperkeratosis  
·∙        Alopecia  
·∙        Nail  dystrophy  
·∙        Ectropion  
·∙        Hepatosplenomegaly  may  be  present  
   

55.     92-­‐   Patient   with   Behçet   disease   with   erythema   nodosum   and   mouth   ucelrs,   not   responding   to  
topical  steroids  and  steroid  paste.  What  is  the  treatment?  
A-­‐azathioprine  
B-­‐  interferon  a  
C-­‐  immunoglobulin  
D-­‐??  
Answer:  A    
   
For  oral  and  genital  ulcerations,  topical  steroids  or  sucralfate  solution  are  first-­‐line  therapy  for  mild  
isolated  ulcerations.  Colchicine  has  also  been  used  to  prevent  mucocutaneous  relapse.  For  severe  
mucocutaneous  lesions,  systemic  corticosteroids,  azathioprine,  pentoxifylline,  dapsone,  interferon-­‐
alfa,  colchicine,  and  thalidomide  have  demonstrated  benefit.  
For  ocular  disease,  azathioprine  is  widely  accepted  as  the  initial  agent.  For  severe  eye  disease  
(significant  drop  in  visual  acuity,  retinal  vasculitis,  or  macular  involvement),  either  cyclosporine  A  or  
infliximab  may  be  used  in  combination  with  azathioprine  and  corticosteroids.  Interferon-­‐alfa,  alone  
or  in  combination  with  corticosteroids,  appears  to  be  a  second  choice  in  eye  disease.  
For  GI  lesions,  based  on  expert  opinion,  5-­‐ASA  derivatives,  including  sulfasalazine  or  mesalamine;  
systemic  corticosteroids,  azathioprine,  tumor  necrosis  factor–α  (TNF-­‐α)  antagonists,  and  thalidomide  
can  be  used.  
Arthritis  may  respond  to  prednisone,  local  corticosteroid  injections,  and  nonsteroidal  anti-­‐
inflammatory  drugs  (NSAIDs),  and  colchicine.  Interferon-­‐alfa,  azathioprine,  and  TNF-­‐α  blockers  may  
be  tried  in  rare  cases  of  patients  with  resistant,  prolonged,  and  disabling  attacks.  
Cutaneous  disease  with  erythema  nodosum  is  a  special  circumstance  and  may  be  treated  with  
colchicine  or  dapsone.  
CNS  disease  is  usually  treated  with  systemic  corticosteroids,  interferon-­‐alfa,  azathioprine,  
cyclophosphamide,  methotrexate,  and  TNF-­‐α  antagonists.  
Major-­‐vessel  disease  with  thrombotic  events  are  treated  with  systemic  anticoagulation  in  addition  to  
corticosteroids,  azathioprine,  cyclophosphamide,  or  cyclosporine  A.  Pulmonary  arterial  aneurysms  
are  treated  with  cyclophosphamide  and  corticosteroids.  
TNF-­‐α  antagonists  are  increasingly  used  and  have  become  standard  treatment  of  Behçet  disease  that  
is  inadequately  controlled  by  standard  immunosuppressive  regimens.  Infliximab  has  been  most  
widely  studied,  but  adalimumab  has  proved  successful  in  cases  refractory  to  both  conventional  
therapy  and  infliximab..  Etanercept  is  the  only  TNF  inhibitor  with  data  from  a  short  term  randomized  
controlled  study  with  proven  efficacy  in  suppressing  most  of  the  mucocutaneous  manifestations  of  
Behcet  disease  
Medscape  
   
56.     Patient  taking  sulfa  drug  developed  lesion  on  the  penis.  What  is  the  most  suitable  description  
of  the  lesion?  
Answer:  Erythematous  plaque,  blister  and  violaceous  lesion  
   
57.    After  sulfa  drug  the  patient  developed  penile  lesion  or  rash  (fixed  drug  eruption)  what  you  will  
find  (best  description  is)?  
A-­‐Ulceration  
B-­‐Blister  
C-­‐Vesicles  
Answer:  B  
   
58.  Patient  known  case  of  DM  come  with  postule  on  the  inner  fold  in  gluteal  region  they  did  
drainage  after  that  there  is  sinus    and  after  a  period  of  time  he  came  with  the  same  thing  what  is  
the  cause  ?  
A.                            1-­‐  furuncle  
B.                            2-­‐Carbuncle  
C.                            3-­‐  hidradenitis  suppurativa  
Answer:  C  
The  diagnosis  of  HS  is  based  upon  the  characteristic  clinical  manifestations;  biopsy  is  neither  
required  nor  diagnostic.  Specific  diagnostic  criteria  do  not  exist.  
There  are  three  main  diagnostic  features:  
●   Typical  lesions  —  multiple  deep-­‐seated  nodules  (blind  boils),  comedones,  and/or  fibrosis  
●   Typical  locations  —  bilateral  involvement  of  axillae,  groin,  inframammary  areas  
●   Relapses  and  chronicity  
         
59.  child  with  skin  rash  honey  cluster  (probably  means  crust)  Dx:  
impetigo  
   
60.  scenario  of  patient  used  a  medication  for  depression  and  developed  rash  in  all  of  his  body?  
(question  incomplete)?  Depends  on  what  manifestation  it  takes  
Answer:  Toxic  Epidermal  Necrolysis  (  TEN  )  
   
Stevens-­‐Johnson  syndrome  (SJS)  and  toxic  epidermal  necrolysis  (TEN)  are  severe  idiosyncratic  (not  
dose  dependant  and  variable)  reactions,  most  commonly  triggered  by  medications,  which  are  
characterized  by  fever  and  mucocutaneous  lesions  leading  to  necrosis  and  sloughing  of  the  
epidermis.  SJS  and  TEN  are  distinguished  chiefly  by  severity  and  percentage  of  body  surface  
involved.  
up  to  10  %  SJS  10-­‐30  overlap,  over  30  TEN  
   
61.  description  of  Herpes  simplex  genital  lesions  
Answer:  multiple  painful  fluid  filled  vesicles  
Remember:  S1  oral  S2  genital  
   
63.  pt  with  vitiligo  on  medications  for  3  yrs  no  response,  she  requested  a  surgical  TX  What  would  it  
be,  
Answer:  Melanocytes  transfer  
Remember:  it’s  the  treatment  of  choice  in  segmental  vitiligo  
N.B  could  be  correct  if  the  disease  is  stable  for  6-­‐12  months  
   
64.  What  is  the  tt  of  seborrhoeic  dermatitis?  
   
Several  randomized  trials  indicate  that  topical  corticosteroids  and  topical  antifungal  agents  are  
effective  for  treating  seborrheic  dermatitis  and  that  intermittent  topical  antifungals  may  prevent  
relapse.  However,  the  high  response  in  the  placebo  groups  suggests  that  frequent  shampooing,  or  
regular  use  of  emollients  may  also  be  beneficial  
   
65.  A  mother  brought  her  child  to  ED  with  itchy  skin  lesions  that  appeared  after  intake  of  some  
food.  The  lesions  appear  in  a  certain  area  and  stay  for  2  hours  then  disappear  then  appear  in  
another  area.  What  is  it  called?  
A.          Burrow  (rabies,  so  not  it)  
B.          Wheal  (urticaria  lesion)  
Answer:  B  
   
66.  19  y/o  female  with  vitiligo  in  the  face  and  hand  the  size  is  increasing  despite  medical  
treatment  now  the  pt  want  to  get  married  and  ask  you  what  to  do:  
A)continue  medical  treatment.  
B)stop  Medical  treatment  and  wait      
C)skin  graft.  
D)melanocyte  transfusion.  
Answer:  A  
   
67.  derma:  wheal,  what  is  the  type  of  urticaria  ?                                                                                                        
A-­‐cold  urticaria  
B-­‐solar  urticaria  
C-­‐  two  other  type  of  urticaria  I  forgot  
   
For  acute  urticaria,  the  main  consideration  involves  possible  precipitants,  such  as  the  following  :  
·∙        Recent  illness  
·∙        Medication  use  
·∙        IV  radiocontrast  media  
·∙        Travel  
·∙        Foods  
·∙        New  perfumes,  hair  dyes,  detergents,  lotions,  creams,  or  clothes  
·∙        Exposure  to  new  pets  (dander),  dust,  mold,  chemicals,  or  plants  
·∙        Pregnancy  (usually  occurs  in  last  trimester  and  typically  resolves  spontaneously  soon  
after  delivery)  
·∙        Contact  with  nickel,  rubber,  latex,  industrial  chemicals,  and  nail  polish  
·∙        Sun  or  cold  exposure  
·∙        Exercise  
·∙        Alcohol  ingestion  [3]  
Physical  urticaria  is  characterized  by  the  following:  
·∙        Blanchable,  raised,  palpable  wheals,  which  can  be  linear,  annular  (circular),  or  
arcuate  (serpiginous);  can  occur  on  any  skin  area;  are  usually  transient  and  migratory;  
and  may  coalesce  rapidly  to  form  large  areas  of  erythematous,  raised  lesions  that  
blanch  with  pressure  
·∙        Dermographism  or  dermatographism  (urticarial  lesions  resulting  from  light  
scratching)  
   
68.  Treatment  of  non-­‐inflammatory  acne  
A-­‐retinoid  acid  
B-­‐clindamycin  
C-­‐isosertonin  
D-­‐azelaic  acid  
Answer  :  topical  retinoid  acid  
Current  consensus  recommends  a  combination  of  topical  retinoid  and  antimicrobial  therapy  as  first-­‐
line  therapy  for  almost  all  patients  with  acne.  [3]  The  superior  efficacy  of  this  combination,  compared  
with  either  monotherapy,  results  from  complementary  mechanisms  of  action  targeting  different  
pathogenic  factors.  Retinoids  reduce  abnormal  desquamation,  are  comedolytic,  and  have  some  anti-­‐
inflammatory  effects,  whereas  benzoyl  peroxide  is  antimicrobial  with  some  keratolytic  effects  and  
antibiotics  have  anti-­‐inflammatory  and  antimicrobial  effects.  
   
69.  17  year  old  football  player,  hypopigmented  lesions  on  trunk  and  arm.  Treatment:  
A-­‐topical  abx  
B-­‐oral  abx  
C-­‐topical  steroid  
D-­‐selenium  sulphate  
Answer  :  D  
   
70.  School  boy  with  itchy  scalp,  10  other  classmates  affected,  diagnosis?  
A-­‐Pediculosis  capitis  
Answer  :  Pediculosis  capitis  (  i.e.head  louse)      
The  head  louse  (Pediculus  humanus  capitis)  is  an  obligate  ectoparasite  that  lives  on  human  beings  
and  feeds  on  human  blood.  Head  lice  infestation  (pediculosis  capitis)  mainly  affects  those  who  are  
socially  active,  particularly  young  children.  BMJ  
   
71.  patient  complains  of  skin  mole,  his  father  had  them  as  well  and  remove  them  but  didn't  know  
if  its  benign  or  malignant,  the  patient  is  concerned  that  his  mole  is  malignant,  what  will  suggest  
the  malignancy  ?  
A-­‐irregular  border  that  fades  in  color  to  be  normal  
B-­‐Equality  of  the  color  
No  other  choice  :  A  
ABCDE=  Assymetry.  Irregular  Borders,  abnormal  Color,  Diameter  and  Evoltion  
The  prototypical  melanoma  is  readily  diagnosable  by  the  ABCDE  approach,  based  on  its  asymmetry,  
irregular  border,  irregular  color,  large  diameter,  and  evolution.  However,  these  clinical  parameters  
are  largely  useless  in  three  instances,  as  follows:  
·∙        For  amelanotic  melanomas,  in  which  pigmentation  is  largely  or  entirely  absent  
·∙        For  desmoplastic  melanomas,  which  sometimes  manifest  without  an  associated  in  
situ  component  and  may  also  lack  clinical  pigmentation  
·∙        For  ulcerated  and  inflamed  melanomas,  especially  nodular  lesions,  which  may  
clinically  simulate  common  lesions  (eg,  basal  cell  carcinoma,  pyogenic  granuloma)  
because  of  masking  by  the  presence  of  ulceration  and/or  inflammation.  Medscape  
   
72.  Child  with  scaly  erethematous  plaques  with  follicular  hyperkeratosis  over  elbows  and  knees  .  
what  is  other  area  in  the  body  more  likely  to  be  affected  ?  
a-­‐  Eye                                                                                                                    
b-­‐  Adrenal  
c-­‐  Heart  
d-­‐  Kidney  
Answer:A  
   
73.  Picture  (  look  like  vesicle  )  start  as  1  only  then  spread  to  arm  legs  and  ..  with  lymph-­‐node  
enlargement  ?  
A-­‐herpes  simplex  virus  
B-­‐Dermatitis  herpetic  form                                                                                                                                
C-­‐Varicella  zoster                                                                                                                                        
INCOMPLETE  Q,  BUT  Verecila  zoster  is  the  closest  
 Answer:C  
 
74.  Patient  with  hx  if  allergy  and  pain  in  the  back  went  to  clinic  they  give  him  drug  (  i  forgot  the  
name  i  think  paracetamol)  then  he  came  next  day  with  viscles  in  back  from  midline  to  the  lateral  
side  ?  
A.  Xanthosis                                                                                                                          
B.  Tinia  corporus  
C.  Herpes  zoster  
Answer:  C  
                               
75.  Pinpoint  papule  in  the  face  of  baby  how  to  manage?  
Answer:  Reassure  
THIS  IS  KNOWN  AS  KERATOSIS  PILARIS???  
Incomplete  question…  can  be  milia,  baby  acne  or  miliaria  
   
76.  Baby  have  depigmentation  since  birth  and  photophobia  refractory  error  nystagmus  ,  What  the  
most  likely  complications?  
A.          -­‐brain    tumor  
B.          skin  cancer  
C.          renal  dysfunction  
Answer:  B  
This  a  case  of  Albinism,  complications  are:  
Skin  cancer,  sunburn                                              
Reduced  visual  acuity                                                                                                                    
Social  stigma  
http://emedicine.medscape.com/article/1200472-­‐followup#e2  
   
77.    Treatment  of  moderately  severe  acne  vulgaris:  
A.  topical  retinoid  
B.  tetracycline  
C.  isotretinoin  
Answer:  C  
In  2016,  the  American  Academy  of  Dermatology  (AAD)  issued  new  evidence-­‐based  guidelines  for  
treatment  of  both  adolescents  and  adults.  Recommended  treatments  include  topical  therapy,  
antibiotics,  isotretinoin,  and  oral  contraceptives.]  The  key  recommendations  include  the  following:  
·∙        Benzoyl  peroxide  or  combinations  with  erythromycin  or  clindamycin  as  
monotherapy  for  mild  acne;  benzoyl  peroxide  with  a  topical  retinoid  or  systemic  
antibiotic  therapy  for  moderate-­‐to-­‐severe  acne  
·∙        Topical  antibiotics  (eg,  erythromycin,  clindamycin)  are  not  recommended  as  
monotherapy  because  of  the  risk  of  bacterial  resistance  
·∙        Topical  retinoids  as  monotherapy  in  primarily  comedonal  acne,  or  in  combination  
with  topical  or  oral  antimicrobials  for  mixed  or  primarily  inflammatory  acne  
·∙        Topical  adapalene,  tretinoin,  and  benzoyl  peroxide  can  be  safely  used  to  treat  acne  
in  preadolescent  children  
·∙        Topical  dapsone  5%  gel  for  inflammatory  acne,  particularly  in  adult  females  
·∙        Systemic  antibiotics  are  recommended  for  moderate  and  severe  acne  and  forms  of  
inflammatory  acne  that  are  resistant  to  topical  treatments;  doxycycline  and  
minocycline  are  both  more  effective  than  tetracycline  
·∙        Topical  therapy  with  benzoyl  peroxide  or  a  retinoid  should  be  used  with  systemic  
antibiotics  and  for  maintenance  after  completion  of  systemic  antibiotic  therapy  
·∙        Monotherapy  with  systemic  antibiotics  is  not  recommended  
·∙        Systemic  antibiotic  use  should  be  limited  to  the  shortest  possible  duration;  to  
minimize  the  development  of  bacterial  resistance,  reevaluation  at  3-­‐4  months  
·∙        Use  of  oral  erythromycin  and  azithromycin  should  be  limited  to  those  who  cannot  
use  the  tetracyclines  (ie,  pregnant  women  or  children  aged  <8  y);  erythromycin  use  
should  be  restricted  because  of  its  increased  risk  of  bacterial  resistance  
·∙        Isotretinoin  is  recommended  for  severe  acne  or  moderate  acne  that  does  not  
respond  to  other  therapy;  low-­‐dose  isotretinoin  can  be  used  to  effectively  treat  acne  
and  reduce  the  frequency  and  severity  of  medication-­‐related  adverse  effects,  but  
intermittent  dosing  is  not  recommended;  all  patients  treated  with  isotretinoin  must  
adhere  to  the  iPLEDGE  risk  management  program;  patients  should  receive  routine  
monitoring  of  liver  function  tests,  serum  cholesterol,  and  triglycerides  at  baseline  and  
again  until  response  to  treatment  is  established,  but  routine  monitoring  of  complete  
blood  count  is  not  recommended;  patients  should  be  educated  about  the  potential  
risks  and  monitored  for  any  indication  of  inflammatory  bowel  disease  and  depressive  
symptoms  
   

78.  What’s  your  advice  to  SLE  pts  


A.  avoid  sun  exposure  
   
 
 
 
 
 
 
 
 
 
 
 
 
 
 
 
 
 
 

Ophthalmology  
   
1.     Patient  with  follicular  keratosis.  What  will  you  check?  
a-­‐Eyes  
Answer:  A  
Reference:  http://disorders.eyes.arizona.edu/category/clinical-­‐features/keratosis-­‐pilaris  
   
2.     Eye  endothelium  layer:  dry  cornea  (Incomplete)  
Explanation:  corneal  endothelium  is  responsible  for  removing  excess  water.  
   
3.     AIDS  retinitis:  CMV  
Explanation:  CMV  is  the  most  common  ocular  opportunistic  infection  in  patients  with  AIDS.  
Refrence:  Wills  Eye  Manual,  page  377  
   
4.     With  viral  conjunctivitis:  follicles  +  epithelial  nummular  keratitis  
Explanation:  
Ten  percent  of  patients  with  adenoviral  conjunctivitis  go  on  to  develop  corneal  involvement  
towards  the  end  of  week  2.  This  is  an  immune  keratitis  with  white  deposits  in  the  cornea,  
causing  significant  reduction  in  vision,  as  well  as  prominent  photophobia.  This,  ‘nummular  
keratitis’  requires  treatment  with  a  weak  topical  steroid  such  as  fluorometholone  (FML),  
and  therefore  requires  referral  to  an  ophthalmologist.  
Refrence:  https://www.foresteyesurgery.com.au/blog/files/viral-­‐conjunctivitis.php  
https://en.wikipedia.org/wiki/Nummular_keratitis  
   
                    5.          Orbital  pseudotumor  treatment:  
1.  Prednisone  80  to  100  mg  p.o.  q.d.  as  an  initial  dose  in  an  adult,  along  with  gastric  
prophylaxis  (e.g.,  ranitidine  150  mg  p.o.  b.i.d.).  Pediatric  dosages  typically  begin  with  1  
mg/kg/day  of  prednisone.  All  patients  are  warned  about  potential  systemic  side  effects  and  
are  instructed  to  follow  up  with  their  primary  physicians  to  monitor  blood  sugar  and  
electrolytes.  
2.  Low-­‐dose  radiation  therapy  may  be  used  when  the  patient  does  not  respond  to  sys-­‐  
temic  corticosteroids,  when  disease  recurs  as  corticosteroids  are  tapered,  or  when  
corticosteroids  pose  a  significant  risk  to  the  patient.  Radiation  therapy  should  only  be  used  
once  orbital  biopsy  has  excluded  other  etiologies.  
Refrence:  Wills  Eye  Manual,  page  158  
   
6.     Nystagmus:  mid  brain  
   
Peripheral  vestibular:  Horizontal  or  hori-­‐  zontal  rotary  nystagmus.  May  be  accom-­‐  panied  by  
vertigo,  tinnitus,  or  deafness.  May  be  due  to  dysfunction  of  vestibular  end  organ  (inner  ear  disease),  
eighth  cranial  nerve,  or  eighth  nerve  nucleus  in  brainstem.  Destructive  lesions  produce  fast  phases  
opposite  to  lesion.  Irritative  lesions  (e.g.,  Meniere  disease)  produce  fast  phase  in  the  same  direction  
as  the  lesion.  Vestibular  nystagmus  associated  with  interstitial  keratitis  is  called  Cogan  syndrome.  
   
Downbeat:  The  fast  phase  of  nystagmus  is  down  and  most  prominent  looking  down  and  to  the  right  
and  left.  Most  commonly,  the  lesion  is  at  the  cervico-­‐  medullary  junction  (e.g.,  Arnold–Chiari  
malformation)  or  a  manifestation  of  cer-­‐  ebellar  degeneration.  
   
Upbeat:  The  fast  phase  of  the  nystagmus  is  up.  If  present  in  primary  gaze,  the  lesion  typically  
involves  the  brainstem  or  anterior  vermis  of  the  cerebellum.  If  present  only  in  upgaze,  the  most  likely  
etiology  is  drug  effect.  
   
See-­‐saw:  One  eye  rises  and  intorts  while  the  other  descends  and  extorts.  Lesion  typically  involves  the  
parasellar  region  and  chiasm.  Typically  pendular  when  chiasmal  region  involved,  and  jerk  if  involving  
the  mid-­‐  brain.  One  proposal  suggests  a  unilateral  lesion  of  the  interstitial  nucleus  of  Cajal  or  its  
connections  are  responsible  for  this  nystagmus  subtype.  May  have  a  bitemporal  hemianopia  
resulting  from  chiasmal  com-­‐  pression.  May  be  congenital  or  associated  with  septo-­‐optic  dysplasia.  
   
Gaze  evoked:  Absent  in  primary  gaze,  but  appears  as  the  eyes  look  to  the  side.  Nystagmus  increases  
when  looking  in  the  direction  of  fast  phase.  Slow  frequency.  Most  commonly  the  result  of  alcohol  
intoxication,  sedatives,  cerebellar  or  brain-­‐  stem  disease.  
   
   
Convergence  retraction:  Convergence-­‐like  eye  movements  accompanied  by  globe  retraction  when  
the  patient  attempts  an  upward  saccade.  May  be  associated  with  limitation  of  upward  gaze,  eyelid  
retraction,  and  bilateral  mid-­‐dilated  pupils  that  react  poorly  to  light  but  constrict  better  with  
convergence.  Papilledema  may  be  present.  Usually,  a  pineal  region  tumor  or  other  dor-­‐  sal  midbrain  
abnormality  is  responsible.  
   
Periodic  alternating:  In  primary  position,  fast  eye  movements  are  in  one  direction  for  60  to  90  
seconds  and  then  reverse  direc-­‐  tion  for  60  to  90  seconds.  The  cycle  repeats  continuously.  Patients  
may  attempt  to  minimize  nystagmus  with  periodic  head  turning.  May  be  congenital.  Acquired  forms  
are  most  commonly  the  result  of  lesions  of  the  cervicomedullary  junction  and  posterior  fossa.  Other  
causes  include  MS,  medication  side  effects,  and  rarely  blindness.  
   
Refrence:  Wills  Eye  Manual,  page  282.  
   
7.     vertical  and  horizontal  nystagmus:  phencyclidine  (No  refrence  was  found)  
   
   
10.    Glaucoma  pt  +  COPD  has  cough:  
a.     because  of  timolol  drops  
Answer:  
Explanation:  Timolol  is  contraindicated  in  COPD  and  may  worsens  symptoms.  
   
11.    Drug  contraindicated  in  glaucoma:  
a.     Epinephrine  
Explanation:  Selective  alpha  agonist  (phenylepherine)  and  non  selective  alpha  
agonists  (epinephrine)  can  precipitate  angle  closure  glaucoma  in  patients  with  
narrow  angles  and  both  are  contraindicated  in  closed  angle  glaucoma.  
   
12.    Drugs  used  for  glaucoma  
Explanation:  
Common  drugs  used  in  glaucoma  include:  
Topical  beta  blockers,  topical  carbonic  anhyrdrase  inhibitors,  alpha  agonists  and  
prostaglandins  analougs.  
Prostaglandins  analougs  (like  latanoprost  “XalatanTM”)  are  usually  prescribed  as  first  line  
therapy  as  they  are  the  newest  with  the  least  side  effects.    
http://www.glaucoma.org/gleams/glaucoma-­‐medications-­‐and-­‐their-­‐side-­‐effects.php  
   
13.    Child  with  30  degree  esotropia,  whats  your  manegment?  
Accomodative  esotropia  is  managed  by  correction  of  hyperopia  by  glasses.  
Non  accomodative  esotropia  is  commonly  managed  surgically  
Refrence:  https://www.rcophth.ac.uk/wp-­‐content/uploads/2014/12/2012-­‐SCI-­‐250-­‐
Guidelines-­‐for-­‐Management-­‐of-­‐Strabismus-­‐in-­‐Childhood-­‐2012.pdf  
   
   
15.    Child  with  unilateral  eye  pain  and  inflammation  and  white  structure:  
                      Answer:  Retinoblastoma  
                      Explanation:  
                      A  malignant  tumor  of  the  retina  that  appears  as  a  white,  nodular  mass  that  breaks                      
through  the  internal  limiting  membrane  into  the  vitreous  (endophytic),  as  a  yellowish  subretinal  
mass  lesion  often  underlying  a  serous  retinal  detachment  (exo-­‐  phytic),  or  as  a  diffusely  spreading  
lesion  simulating  uveitis  (diffuse  infiltrating).  Iris  neovascularization  is  common.  Pseudohypo-­‐  pyon  
and  vitreous  seeding  may  occur.  Cataract  is  uncommon,  and  the  eye  is  normal  in  size.  May  be  
bilateral,  unilateral,  or  multifo-­‐cal.  Diagnosis  is  usually  made  between  12  and  24  months  of  age.  A  
family  history  may  be  elicited  in  about  10%.  
Refrence:  Wills  Eye  Manual,  page  177.  
   
16.    Abscent  red  eye  reflex  is  seen  in  ===>  retinoblastoma  
Causes  of  leukocoria  in  pediatrics  group:  congenital  cataract,  retinoblastoma,  retinopathy  of  
prematurity  (ROP),  toxocariasis,  coats  disease,  PFV/PHPV,  retinal  astrocytoma,  retinochoroidal  
coloboma,  retinal  detachment,  familial  exudative  vitreoretinopathy  (FEVR),  myelinated  nerve  fibers,  
uveitis,  incontinentia  pigmenti,  toxoplasmosis.  
Refrence:  Wills  Eye  Manual,  page  178  
   
   
17.    Characteristic  sign  of  retinoblastoma?  
Refrence:  https://emedicine.medscape.com/article/1222849-­‐clinical#b4  
   
18.    Retinoblastoma  case  that  affecting  the  child's  vision  but  MRI  should  intact  optic  
nerve  what  is  the  management  
a.     Chemotherapy  
b.     Radiation,  
c.     enncuulation  
                        Answer:  A  
 Explanation:  mainstay  treatment  of  retinoblastoma  without  optic  nerve  extension  is  
chemotherapy.    Read  about  retinoblastoma:  
https://www.aao.org/pediatric-­‐center-­‐detail/retinoblastoma-­‐2016  
   
https://emedicine.medscape.com/article/1222849-­‐
treatment?pa=8dtgBnfXs1A%2FLKJCYR4hr6MZiyqrlkdpQit9AxXgBnl%2BpH%2Bze2zxoyiiMU
mOyvMQJyGvMX%2Fu%2BWdIXoARf%2FT0zw%3D%3D  
   
21.    Pediatric  e  purulent  eye  discharge,  Culture  showed  gram  negative  diplococcic  How  to  
treat?  
a.     Iv  cephalosporin  
b.     Steroid  
c.     Topical  Abx  
Answer:    A    
Explanation:  Neonatal  Neisseria  Gonorrhoeael  conjunctivitis:  In  severe  cases  this  
can  cause  corneal  perforation.  Systemic  complications  include  rhinitis,  stomatitis,  
arthritis,  meningitis  and  septicaemia.  Due  to  increasing  resistance  to  penicillin  a  
systemic,  third-­‐  generation  cephalosporin  (ceftriaxone)  is  used  to  treat  the  
condition.  The  eye  must  be  kept  clean.  Topical  bacitracin  ointment  can  also  be  
given  but  sys-­‐  temic  treatment  is  the  most  important.  Refer  parents  to  a  sexually  
transmit-­‐  ted  diseases  clinic.    Refrence:  Lecture  notes,  11th  edition.  Page  114    
   
   
23.    A  pt  on  anti  TB  drugs  developed  eye  pain  
a.     Ethambutol  
Explanation:  A  
Retrobulbar  neuritis  resulting  in  blurred  vision  and  loss  of  red-­‐green  vision  occurs  
commonly  with  ethambutol  therapy  and  requires  careful  monitoring  of  visual  acuity  and  
color  discrimination.  Optic  neuritis  occurs  more  frequently  at  dosages  greater  than  15  
mg/kg/day.  Drug  therapy  should  be  discontinued  at  the  first  sign  of  vision  defects.  
https://www.drugs.com/sfx/ethambutol-­‐side-­‐effects.html  
   
24.    The  effect  of  anti  TB  drugs  on  the  eye  is  one  of  the  following  
a.     Bacterial  conjunctivitis,  
b.     Viral  conjunctivitis  ,  
c.     Glucoma  ,  
d.      Uveitis  
ANSWER:  D  
Explanation:  Rifabutin  can  cause  uveitits.  
http://eyewiki.aao.org/Drug_Induced_Uveitis  
Question  probably  written  wrong.  
If  asking  about  ocular  manifestations  of  TB>  tuberculous  uveitis.  
If  asking  about  ocular  complications  of  anti  TB>  ethambutol  causes  optic  neuritis.  
   
 25.    Case  with  conjunctivitis  and  the  eyelash  turned  inwards.  
a.          entropion  
Explanation:  
1-­‐Trichiasis  is  a  condition  in  which  eyelashes  grow  in  a  posterior  direction  toward  the  corneal  
surface.  
2-­‐Distichiasis  is  the  growth  of  lashes  from  the  meibomian  gland  orifices  (extra  line  of  eyelashes)  
3-­‐Entropion  is  an  inward  turning  of  the  eyelid  margin  and  appendages  such  that  the  pilosebaceous  
unit  and  mucocutaneous  junction  are  directed  posterior  towards  the  globe.  
   
26.    Water  discharge  +  red  eye?  
a.     Viral  conjunctivitis  
Explanation:  Watery  discharge  can  be  a  sign  for  dry  eyes  or  viral  conjunctivitis.  In  
Viral  KC,  follicular  reaction  can  be  seen  on  slit  lamp  examination,  and  a  pre  
auricular  lymph  node  might  be  present.  A  history  of  upper  respiratory  tract  
infection  might  preceed  ocular  sumptoms.  
Read  about  dry  eye:  https://emedicine.medscape.com/article/1210417-­‐workup#c8  
   
   
27.    Post  eye  surgery  complication  
a.     endophthalmitis  (Infection  in  the  eye)  
Answer:  The  most  serious  and  feared  complication  after  ocular  surgeries  is  
endophthalmitis.  
   
28.    -­‐Patient  presented  with  eye  redness  and  problem  in  vision  just  during  exams  his  
visual  acuity  is  6/6  and  normal  eye  exam  what  does  he  have:  
                        a.          Astigmatism  
                        b.          Myopia  
                        c.          Hypermetropia  
                        d.          Anisometropia  
                        Answer:  C  
                        Refrence:  http://eyewiki.aao.org/Hyperopia  
   
29.    -­‐Patient  presented  with  unilateral  esotropia  what's  the  management  
                      a.          Glasses  
                      b.          Corrective  surgery  
   
If  the  case  was  accomodative  esotropia,  it  is  treated  by  glasses  (Hypermetropia  correction)  
Most  cases  of  non  accomodative  esotropia  are  treated  by  surgical  correction.  
For  further  information:  https://www.rcophth.ac.uk/wp-­‐content/uploads/2014/12/2012-­‐
SCI-­‐250-­‐Guidelines-­‐for-­‐Management-­‐of-­‐Strabismus-­‐in-­‐Childhood-­‐2012.pdf  
   
30.    -­‐Patient  with  increase  ICP  what  nerve  would  u  examine  before  the  CT  scan  
a.     Optic  nerve  
b.     Abducens  nerve  
c.     Facial  nerve  
Answer:  A  
   
   
31.    Pt  came  to  you  after  trauma  c/o  loss  of  the  abduction  of  (left  or  right  eye).  So  which  cranial  
nerve  affected  
a)  III  
b)  IV  
c)  V  
d)  VI  
Answer:  D  (Abducense  nerve  innervates  LR  muscle  which  is  responsible  for  abduction)  
   
32.    pt  eye  drop  and  eye  goes  to  medial  side  what  nerve  injury  when  try  to  close  the  eye:  
A.    3  
B.    7  
C.    4  
Explanation:  3rd  nerve  palsy  presents  with:  ptosis,  lateral  and  downward  gaze.  
 
 

 
33.    Lacrimal  gland  mass  causes  proptosis  in  which  direction?  
a.     In  and  down  
b.     Out  and  down  
Answer:  A  
Explanation:  lacrimal  gland  masses  displace  the  eyes  inferonasally.  
http://webeye.ophth.uiowa.edu/eyeforum/cases/235-­‐Adenoid-­‐cystic-­‐carcinoma-­‐
lacrimal-­‐gland.htm  
   
 34.    -­‐Patient  with  conjunctivitis,  which  one  of  the  following  will  make  you  refer  to  
ophthalmology?  
a.          Bilateral  conjunctivitis  
b.          Photophobia  
c.          Mucopurulent  discharge  
d.          Itching  
Answer:  B  (Couldn’t  find  a  source)  
   
35.    -­‐Clear  ophtha  question  about  herpetic  keratitis  (dendrites  with  fluoroscien)  
   
36.    Man  has  eye  pain,  seeing  halos,  headache,  many  GI  symptoms,  eye  examination:  red  and  
injected  vessels.  What  is  the  diagnosis?  
a.          Digoxin  toxicity  
b.          Angle  closure  Glaucoma  
Answer:  B  
Explanation:  Acute  angle  closure  glaucoma  presents  with  an  acute  onset  of  red  injected  painful  eye,  
blurry  vision,  halos  around  light,  mid-­‐dilated  pubil,  N\V  and  headache.  
https://emedicine.medscape.com/article/1206956-­‐clinical  
   
37.    -­‐Case  of  foreign  body  in  the  eye,  successfully  remove,  what  is  next  ?  
a.          Topical  steroid  
b.          Topical  antibiotic  
Answer:  B  
Explanation:  
Following  removal  of  the  conjunctival  foreign  body,  two  drops  of  a  topical  broad-­‐spectrum  antibiotic  
drop  should  be  placed  in  the  affected  eye.  If  there  is  no  corneal  abrasion  or  significant  inflammation  
of  the  eye,  no  further  treatment  or  follow-­‐up  is  necessary.  The  patient  should  be  instructed  to  return  
if  the  foreign  body  sensation  returns  or  if  any  symptoms  of  pain,  redness,  or  visual  changes  occur.  
https://emedicine.medscape.com/article/1844102-­‐overview#showall  
   
38.    HIV  pt  came  for  ophthalmology  follow  up  you  found  cotton  wool  spots  in  the  eye,  what  is  the  
cause  of  this  condition  in  this  pt:  
a.          HIV  
b.          EBV  
Answer:  A.  
Explanation:  HIV  retinopathy.  Source:  (Wills  Eye  Manual,  Page  300)  
   
40.    (Pic)  of  swollen  upper  eyelid,  no  discharges,  hx  of  eye  redness,  tearing  in  the  morning,  
decrease  vision,  no  itching,  temp:  38.1  
a.          Stye  
b.          Chalazion  
c.          Vernal  conjunctivitis  
d.          Orbital  cellulitis  
Answer:  D  
   
   
   
Q.  What  is  the  treatment  of  orbital  cellulitis?  
IV  ceftriaxone  with  Vancomycin  
Answer:  Broad-­‐spectrum  intravenous  antibiotics  to  cover  Gram-­‐positive,  Gram-­‐negative,  and  
anaerobic  organisms  are  recommended  for  48  to  72  hours,  followed  by  oral  medication  for  at  least  1  
week.  Ee  currently  recommend:  
   
Ampicillin-­‐sulbactam  3  g  i.v.  q6h  in  adults;  300  mg/kg  per  day  in  four  divided  doses  in  children,  
maximum  daily  dose  12  g  ampicillin-­‐sulbactam  (8  g  ampicillin  com-­‐  ponent).  
or  
Piperacillin-­‐tazobactam  4.5  g  i.v.  q8h  or  3.375  g  q6h  in  adults;  240  mg  of  piperacillin  
component/kg/day  in  three  divided  doses  in  children,  maximum  daily  dose  18  g  piperacillin.  
For  adults  who  are  allergic  to  penicillin  but  can  tolerate  cephalosporins,  use  vancomycin  as  dosed  
below  plus:  
Ceftriaxone  2  g  i.v.  q.d.  and  metronida-­‐  zole  500  mg  i.v.  q6–8h  (not  to  exceed  4  g  per  day).  
Source:  Wills  Eye  Manual  (Page  182-­‐183)  
   
41.    Patient  has  painful  big  mass  in  the  lower  eyelid  beside  upper  nose,  what  is  your  management?  
a.          topical  steroid  
b.          Surgical  drainage  
c.          oral  antibiotics  
Answe:  B  
Explanation:  In  general,  dacryocystitis  is  a  surgical  disease.  Surgical  success  rates  in  the  treatment  of  
dacryocystitis  are  approximately  95%.  Acute  cases  are  best  treated  surgically  after  the  infection  has  
subsided  with  adequate  antibiotic  therapy.  
For  acute  dacryocystitis,  an  external  dacryocystorhinostomy  is  preferred  after  several  days  of  
initiating  antibiotic  therapy.  Rarely,  dacryocystorhinostomy  must  be  performed  during  the  acute  
phase  of  the  infection  to  facilitate  clearing  of  the  infection.  Reference:  Medscape.  
   
42.    Blue  corneal  picture  =  stained  by  fluorescein  stain  to  detect  corneal  abrasion  or  foreign  bodies  
in  the  eye.  Treatment  of  abrasion  is  topical  cycloplegic,  topical  antibiotic  and  topical  analgesia.  
Reference  http://www.aafp.org/afp/2013/0115/p114.html  
   
   
43.    Eye  nail  abrasion,  rx:  
a.            anti  viral  drops  
b.            antiviral  ointment  
c.            contact  lens  
d.            steroids  (no  sure)  
Explanation:  Treatment  of  abrasion  is  topical  cycloplegic,  topical  antibiotic  and  topical  analgesia.  
Reference  http://www.aafp.org/afp/2013/0115/p114.html  
   
   
44.    Corneal  abrasion?  
a.     Fitting  lens  
b.     Steroids  
c.     Antiviral  
 Question  is  incomplete.  
Steroids  can  be  given  with  severe  keratitis  but  herpes  keratitis  should  be  ruled  out  first.  
In  abrasions,  steroids  are  not  given.  Lenses  are  not  used  in  cases  of  abrasions  and  ulcers.  
Treatment  of  abrasion:  topical  cycloplegic,  topical  antibiotic  and  oral  analgesia.  
   
More  information  for  corneal  abrasion:  
   
1.  Antibiotic  
—Noncontact  lens  wearer:  Antibiotic  oint-­‐  ment  (e.g.,  erythromycin,  bacitracin,  or  
bacitracin/polymyxin  B  q2–4h)  or  antibi-­‐  otic  drops  (e.g.,  polymyxin  B/trimethoprim  or  a  
fluoroquinolone  q.i.d.).  Abrasions  sec-­‐  ondary  to  fingernails  or  vegetable  matter  should  be  covered  
with  a  fluoroquinolone  drop  (e.g.,  ciprofloxacin,  moxifloxacin)  or  ointment  (e.g.,  ciprofloxacin)  at  
least  q.i.d.  
—Contact  lens  wearer:  Must  have  antipseu-­‐  domonal  coverage.  May  use  antibiotic  oint-­‐  ment  or  
antibiotic  drops  at  least  q.i.d.  
   
2.  Cycloplegic  agent  (e.g.,  cyclopentolate  1%  to  2%  b.i.d  or  t.i.d.)  for  traumatic  iritis  which  may  
develop  24  to  72  hours  after  trauma.  Avoid  steroid  use  for  iritis  with  epithelial  de-­‐  fects  because  it  
may  retard  epithelial  healing  and  increase  the  risk  of  infection.  Avoid  use  of  long-­‐acting  cycloplegics  
for  small  abra-­‐  sions  to  allow  for  faster  visual  recovery.  
   
3.  Patching  is  rarely  necessary.  Patching  may  be  helpful  for  comfort,  but  DO  NOT  patch  if  the  
mechanism  of  injury  involves  veg-­‐  etable  matter,  fingernails,  or  if  the  patient  wears  contact  lenses.  
Be  careful  that  the  patch  is  properly  placed  so  that  the  upper  lid  is  totally  prevented  from  opening  as  
this  can  cause  a  serious  abrasion.  
   
4.  Consider  topical  nonsteroidal  anti-­‐inflamma-­‐  tory  drug  (NSAID)  drops  (e.g.,  ketorolac  0.4%  to  0.5%  
q.i.d.  for  3  days)  for  pain  control.  Avoid  in  patients  with  other  ocular  surface  disease  and  in  
postoperative  patients.  Oral  ac-­‐  etaminophen,  NSAIDs,  or  narcotics  (in  severe  cases)  can  also  be  
used  for  pain  control.  
   
5.  Debride  loose  or  hanging  epithelium  be-­‐  cause  it  may  inhibit  healing.  A  cotton-­‐tipped  applicator  
soaked  in  topical  anesthetic  (e.g.,  proparacaine)  or  a  sterile  jewelers  forceps  (used  with  caution)  may  
be  utilized.  
   
6.  No  contact  lens  wear.  Some  clinicians  use  bandage  contact  lenses  for  therapy.  We  rare-­‐  ly  do  
unless  the  size  of  the  abrasion  and  dis-­‐  comfort  warrants  it  and  there  is  poor  heal-­‐  ing  in  the  absence  
of  infection.  If  a  bandage  contact  lens  is  used,  patients  should  use  prophylactic  topical  antibiotics  
(e.g.,  poly-­‐  myxin  B/trimethoprim  or  a  fluoroquinolone  q.i.d.)  and  should  be  followed-­‐up  daily  for  
evaluation  and  contact  lens  replacement.  Refrence:  Wills  Eye  Manual,  page  17.  
   
45.    Picture  of  corneal  ulcer  that  is  showing  dendritic  lesion.  What  other  feature  is  associated  with  
it?  
a.          Scleritis  and  episcleritis  
b.          Chorditis  and  retinitis  
c.          Optic  neuritis  
d.          Hypoesthetic  cornea  (decrease  the  corneal  sensation  of  pain)  
Answer:  D  
Explanation:  Herpes  keratitis  results  in  decreased  sensation  over  the  cornea  
   
46.    How  is  corneal  ulcer  diagnosed?  
a.     Slit  lamp  
b.     Fluorescein  dye  
Answer:  B  
   
47.    What  dye  is  used  in  corneal  ulcer  (pic  of  blue  color):  
a.     Fluorescein  dye  
b.     Rose  Bengal  dye  
c.     Geimsa  
d.     Lissman  Green  B  stain  
Answer:  A  
   
48.    Corneal  ulcer  treatment:  
a.            oral  ABx,  cycloplegic  
b.          ABx  ointment,  
c.          cotton  bud  debridement....  
d.          (Not  sure  about  choices)  
Answer:  B  
https://emedicine.medscape.com/article/1195680-­‐treatment  
   
49.    Female  with  corneal  abrasion,  normal  visual  acuity  normal  IOP,  by  examination  the  lower  
eyelid  rolled  in,  diagnosis:  
a)Uveitis  
b)Glaucoma  
c)Entropion  
d)Ectropion  
Answer:  C  
51.    History  of  URTI  then  developed  watery  eye  
a.     Viral  conjunctivitis  
Answer:  A.  
Explanation:  URTI  caused  by  adenovirus  infection,  followed  by  adenovirus  
conjunctivitis.  
   
52.    -­‐Redness  in  the  eye  with  presence  of  cat  in  the  home  
a.     allergic  conjunctivitis  
Answer:  A  
53.    Complication  of  cataract  surgery  
a.     endophthalmitis  
https://www.webmd.com/eye-­‐health/cataracts/extracapsular-­‐surgery-­‐for-­‐
cataracts  
Answer:  A.  Endophthalmitis  is  the  most  serious  and  feared  side  effect  post  occular  surgeries.  
Other  early  complications  post  cataract  surgery:  cystoid  macular  edema,  retinal  detachment,  corneal  
edema,  hyphema.  
Late:  Glaucoma,  dislocated  IOL,  staining  of  the  lens  capsule,  retinal  detachment,  ptosis.  
   
54.    Pediatric  patient  with  strabismus,  what  is  the  complication  
a.     amblyopia  
Answer:  A  
   
55.    Superior  oblique  muscle  movement  
a.          Medially  down  
b.          Medially  up  
Explanation:  Superior  oblique  action:  intorsion,  abduction  and  downward  gaze  
The  righ  answer  shouold  be  Lateral  and  down.  
   
56.    Infant  since  birth  unilateral  red  eye  associated  with  continuous  tearing  
a.          Congenital  glucoma  
b.          Chlamydia  
c.          Gonorrhea  
Answer:  A  
Explanation:  The  primary  symptoms  of  primary  congenital  glaucoma  are  epiphora,  photophobia,  and  
blepharospasm.  Reduced  vision  can  also  occur  from  corneal  edema  or  progressive  myopia  and/or  
astigmatism.  
Signs  include:  epiphora,  conjunctival  erythema,  corneal  enlargement,  corneal  clouding,  Haab  striae,  
abnormally  deep  anterior  chamber,  myopia  and/or  astigmatism,  and  enlarged  optic  nerve  cupping.  
http://eyewiki.aao.org/Glaucoma,_Congenital_Or_Infantile  
   
58.          Case  of  acute  angle  closure  glaucoma,  what  is  the  first  step  in  management?  
a.          Acetazolamide  
b.          Timolol  
c.          Topical  steroids  
Answer:  A  
Explanation:  
 Acetazolamide  should  be  given  as  a  stat  dose  of  500  mg  IV  followed  by  500  mg  PO.  A  dose  of  a  
topical  beta-­‐blocker  (ie,  carteolol,  timolol)  will  also  aid  in  lowering  IOP.  
https://emedicine.medscape.com/article/798811-­‐treatment#d10  
   
60.    Ophthalmology  ex  for  DM2  patient  every  
a.          6  months  
b.          12  months  
c.          24  months  
d.          36  Months  
Answer:      B                                                    
   
62.    Regarding  distichiasis  definition?  (multiple  defintions  and  u  choose  the  most  accurate)  
Explanation:  Distichiasis  is  the  growth  of  lashes  from  the  meibomian  gland  orifices,  which  can  irritate  
and  damage  the  corne  (it  results  in  an  extra  raw  of  eyelashes)  
   
 
 
 
 
 
 
 
 
 
64.    Lady  with  mucopurulent  eye  discharge  when  she  woke  up  she  her  eye  was  stuck  she  couldn't  
open  it.  On  exam,  cornea  was  normal,  red  conjunctiva.  What  is  the  Dx?  
a.          Viral  
b.          Bacterial  (correct)  
Answer:      B,  in  viral  it  is  watery  discharge.                                                    
   
65.    Visual  acuity  200  of  one  eye  +  Esotropia,  other  normal.  What  is  the  intial  thing  to  do?  
a.     Fundoscopy  
b.     Muscle  resection  
Answer:  this  is  a  case  of  Amblyopia,  patching  should  be  started.  
   
66.    Progressive  painless  vision  loss.  Drusen  spots  and  central  scotoma,  what  is  the  diagnosis:  
a.     Macular  degeneration  
Answer:    A                                                    
   
Explanation:  In  age  related  macular  degenaration,  extracellular  breakdown  deposits  called  “drusen”  
develop  in  Bruch’s  membrane  causing  blockage  of  the  drainage  to  the  choroidal  bed.  Findings  
include:  localized  retinal  atrophy  and  pigmentary  changes  in  the  macula  that  correlate  with  poor  
central  vision.  The  visual  loss  occurs  slowly,  however,  and  takes  many  years  to  progress.  
Source:  Wills  Eye  Manual  (Page  322-­‐323)  
   
67.      Picture  of  one  eye  conjunctivitis,  how  to  prevent  the  infection  for  the  other  eye?  
a.     Eye  drop  
b.     Eye  ointment  
c.     Hand  washing  
Answer:  C  
   
   
73.    Patient  has  painful  big  mass  in  the  lower  eyelid  beside  the  upper  nose,  what  is  your  
management?  (Dacrocystitis)  
a.     Topical  steroids  
b.     Surgical  drainage  
c.     Oral  antibiotics  
Answer:      C      
Explanation:  surgical  intervention  is  made  when  the  acute  episdode  has  resolved.  
Source:  Wills  Eye  Manual  (Page  145-­‐146)                                                
   
74.    Patients  has  decreased  vision  loss  and  went  to  optometrist  and  got  new  glasses,  one  
week  he  comes  back  with  decreased  vision  with  the  new  glasses  he  was  referred  to  
ophthalmologist  due  to:                                                                                                                                                                                            
a.          Cataract  formation  
b.          Increase  glucose  in  lens  
c.          Increase  sorbitol  in  lens  
Answer:      C      
Explanation:    Rapid  changes  in  refraction  indicates  osmotic  changes  in  the  lens  by  
sorbitol.  (Couldn’t  find  a  source)                                        
   
75.    Pt  wear  contact  lens  then  c/o  pain  in  rt  eye  and  discharge  what  is  the  organism  ?  
a.     H.influenza  
b.     Acanthameba  
c.     Nesseria  meningitis  
d.     Pnumococcal  Chlymedia      
Answer:      B  
Explanation:  
Pseudomonas  aeruginosa  is  the  most  commonly  recovered  causative  organism  in  contact  lens-­‐
related  disease,  followed  by  Gram-­‐positive  bacteria,  fungi  and  Acanthamoeba.                                                                                                                            
https://www.ncbi.nlm.nih.gov/pmc/articles/PMC3272197/  
   
76.    Pt  with  myopia  and  chorioretinal  degeneration,  this  is:                                                                            
a.     Physiological  
b.     Pathological  
Answer:  B  
Explanation:  myopia  severe  enough  to  cause  retinal  complications  is  called  
pathological  myopia.  
Source:  http://eyewiki.aao.org/Pathologic_myopia_(myopic_degeneration)  
   
77.    Picture  of  upper  eyelid  swelling,  and  they  gave  history  of  eye  pain  and  tearing  what  
else  could  be  found:  
a.          Uveitis  
b.          Discharge  
c.          Scleritis  
Answer:  Uveitis  and  scleritis  don’t  cause  upper  eye  lid  swelling.  Discharge  could  be  the  answer  if  this  
was  a  case  of  orbital  cellulitis.                                                                                                                                                      
78.    Pt  was  taking  anti  TB  meds  and  now  he  presented  with  red  eye  and  pain  along  with  
photophobia  what  the  dx?  
a.          Bacrtial  conjunctivitis  
b.          Viral  conjunctivitis  
c.          Uveitis  
d.          Optic  neuritis  
Answer:      D                                                                                                                                                                                                                                    
https://www.drugs.com/sfx/ethambutol-­‐side-­‐effects.html  
                                                                                                                                                               
79.    An  old  man,  presents  to  the  ER  with  the  complaint  of  painful  Rt  eye  associated  with  
nausea.  O/E  the  Rt  eye  was  injected,  with  semi-­‐  dilated  pupil.  Lt  eye  was  normal.  What  is  
the  correct  Dx  of  these?  
a.     Uveitis.  
b.     Primary  open-­‐angle  glaucoma.  
c.          Acute  angle-­‐closure  glaucoma.  
d.     Irrelevant  choice.  
                                                           
Answer:      C      
   
80.    Child  exposed  to  fingernail  injury,  what  this  patient  may  have?  
a.          Purulent  tearing  
b.          Photophobia  
c.          Vision  loss  
Answer:      B  
   
81.    Same  as  previous  scenario,  but  asking  about  management:  
Answer:      Topical  antibiotics  
   
Q.  Mother  have  a  photophobia  and  severe  eyes  pain  after  she  received  accidentally  
fingernail  trauma  by  her  daughter  how  to  manage?  
Answer:      Topical  antibiotics  
   
82.    Best  treatment  for  ptosis:  
a.          Muller  resection  
b.          Levator  muscle  resection  
c.          Frontalis  suspension  
Answer:      C        
If  there  is  levator  dysfunction>  frontalis  suspension.  
Answered  by  a  resident.  
   
83.    Old  age  HTN,  DM  had  painless  vision  loss  examination  show  drusen  diagnosis?  
a.          Central  retinal  vein  occlusion  
b.          Central  retinal  artery  occlusion  
c.          Macular  degeneration  
Answer:      C  
   
84.    Patient  with  red  eye+  history  of  cough,(picture  of  viral  conjunctivitis  attached).How  
to  prevent  transmission?  
Answer:      Hand  washing  
                                                                   
85.    Patient  with  red  eye,  cillary  flush,  photophobia,  dendritic  lesion  in  the  cornea:                                                                            
   
a.          HSV  of  eye  
b.          dentric  lesion  .  
                       Answer:      A  
                                                                                                                                                                               
86.    Long  scenario  about  glaucoma,  pain,  congestion  of  cornea  -­‐  vomiting,  photophobia,  
what  is  the  treatment?  
a.          acetazolmide.  
b.          Corticosteroids  
c.          Antibiotics  
Answer:      A  
   
87.    Decrease  vision  &  painful  eye  movement?  
Answer:      Optic  neuritis  
   
88.    What  cause  color  vision  loss?  
Answer:      Optic  neuritis  
   
89.    Young  healthy  pt  e  red  eye  circumcorneal  congestion,  IOP  30,  dilated  pupil,  what  best  
drug?  
a.          Latanoprost,  
b.          something  b  blocker,  
c.          acetazolamide,  ...  
d.        
Answer:      C  
                      Acute  angle  glaucoma  Treatment:  
beta  blocker  
pilocarpine  eye  drop  
Mannitol  
azetolazmide  
These  all  used  for  glaucoma  
   
91.    Eldely  Patient  with  progressive  vision  loss  for  6  m,  came  to  ER  with  painfull  red  eye,  with  
tearing  and  white  color.  IOP  is  high  What  is  the  cause:  (Scenario  not  complete)  
a.          Malignant  glaucoma  
b.          Adcanced  rubeosis  iridis  
Malignant  glaucoma:  AKA  aqueous  misdirection  syndrome.  Happens  post  op.  Presents  with  acute  
angle  closure  and  high  IOP  that  is  resistant  to  treatment.  
Rubeosis  iridis:  history  of  retinal  ischemia  (PDR,  CRAO,  CRVO)  and  vessels  on  iris  causing  angle  
closure.  
92.    Treatment  of  severe  ptosis  no  other  details?  
A.  Frontalis  suspension  
B.  Levator  resection  
Levator  dysfunction  >  frontalis  suspension.  
 
93.    Patient  with  decrease  color  vision?  
A.     Optic  atrophy  
B.     Iridocyclitis  
C.     Forget  other  options  
Answer:      A  
Color  vision  function  of  optic  nerve  so  look  for  optic  nerve  pathology  like  optic  neuritis.  
Cataracts/diabetic  retinopathy  DO  NOT  cause  color  blindness.  
   
Q.Patient  presented  with  loss  of  color  vision..  what's  the  diagnosis?  
Answer:      Optic  atrophy  
http://emedicine.medscape.com/article/1217760-­‐overview  
   
94.    Child  with  30  degree  esotropia,  whats  your  management?  
A.                            Orthoptics  
B.                            surgery  
C.                            Prism  
30  degrees  of  esotropia  is  a  large  angle  so  surgery  (but  only  if  it  says  medial  rectus  recession).  
Answered  by  resident.  
   
                                 95.  Brown  dicoloration  of  eye  then  disappeared.  What  is  the  drug  :  
A.     Vanco  
B.     Metacloprmide  
C.     Lanatoprost  
Answer:  C  
   
96.    Patient  with  COPD  has  glaucoma.  What  to  give?  
A.     Topical  temolol  
B.     Topical  something  olol  
C.     Oral  Acetazolamide  
Answer:    C  
   
97.    -­‐  Drugs  used  for  glaucoma:  Page  500  schedule  
   
98.    pt  came  for  ophthalmology  check  up,  has  optic  disc  cupping.  
                                      Tonometry  showed  high  IOP.  What  would  u  tell  this  pt:  (Incomplete)  
A.     1/  tonometry  is  sufficient  
B.     2/do  check  up  for  blood  related  members  
C.     3/interventions  may  reverse  these  changes  
Tonometry  is  not  sufficient  (may  need  fundoscopy).  Screening  relatives  not  typically  recommended.  
Glaucoma  changes  are  irreversible.  Answered  by  resident.  
   
99.      Four  to  five  cases  about  bacterial  and  viral  conjunctivitis  
Viral>  watery  discharge  with  history  of  URTI.  
Bacterial>  mucopurulent  discharge.  
Allergic>  watery  discharge  and  itching.  
   
100.                            Case  of  Wilson  disease  with  eye  manifestations.  What  to  give?  
Answer:  Penicillamine  
Eye  manifestation  in  Wilson’s  >  Kayser  Fleischer  ring.  
   
101.      Patient  with  headache  and  his  eyes  pushed  outward.  What’s  the  management?  
A.     Head  MRI  
B.     Corticosteroids  
Scenario  incomplete.  
If  suspecting  orbital  cellulitis  (with  fever,  visual  disturbance  and  restricted  motility)  >  CT  scan  is  
sufficient.  
If  suspecting  intracranial  tumor  >  MRI.  
   
102.  SCENARIO  of  ptosis  (no  other  information  available)  there  is  elevator  muscle  dysfunction,  
what  is  the  management?  
A.     1-­‐mullurectomy  
B.     2-­‐levtor  resection  
C.     3-­‐frontalis  suspension  
D.   4-­‐levator  muscle  aponeurosis  
Answer:  C  
Levator  dysfunction>  frontalis  suspension.  Answered  by  resident.  
Note:  To  answer  this  question  we  have  to  know  three  variables  which  are:  
1-­‐  Age  of  the  patient  (congenital,  pediatric  or  older)  
2-­‐unilateral  or  bilateral  
3-­‐severity  of  the  symptoms  (mild,  severe)  and  here  is  the  summary:  
If  it  congenital  mild  unilateral,  no  severe  ptosis  or  symptoms  first  option  is  Levator  resection  BUT  if  
the  patient  is  old  or  the  ptosis  is  bilateral  or  severe  ptosis  go  and  do  frontalis  suspension  
Please  go  and  
   
103.  Patient  k/c  of  HF  &  schizophrenia  on  medication,  came  with  brownish  discoloration  
in  retina  &  brown  vision..  what's  the  drug?  
Answer:  Thioridazine  
   
106.  Pseudotumor  of  the  eye    >  steroids  
Pseudotumor  cerebri:  usually  obese,  presenting  with  headache  and  visual  disturbance.  
Associated  with  6th  nerve  palsy.  Findings:  High  ICP  and  papilledema.  Treatment:  steroids.  
   
109.      picture  of  HSV  on  cornea:  
A.     episcleritis  
B.     scleritis  
C.     uveitis  
D.   Corneal  sensation  
Answer:D  
   
111.  Side  effect  of  topical  steroids:  
A.     cataract  
B.     glaucoma  
Both  correct  but  glaucoma  more  concerning.  
   
112.  Child  came  to  eye  evaluation  the  right  eye  20/20  the  left  eye  20  /200  ,  the  left  eye  
turne  in  and  the  eye  seems  crossed,  no  restricted  movement  in  the  eye  examination  of  
extraocular  muscles  normal.  what  is  the  diagnosis:  
A.     Congenital  cataract.  
B.     Nystagmus  
C.     strabismus  
Answer:  C  
   
113.  Long  scenario  of  open  angle  glucoma  asking  waht  is  the  mechanism  due  to  
obstruction  of  aqueous  drainage  within  the  trabecular  meshwork  and  its  drainage  into  
the  Canal  of  Schlemm??  
   
114.  45  y  farmer  presented  complaining  of  dry  eye  he  has  smoked  2  pack/day  for  20yrs  
what  is  the  management  besides  artificial  tearing  
A.     a)advise  him  to  exercise  (  blinking  exercises)  
B.     b)stop  smoking  
C.     c)wear  glasses  
D.   d)all  of  the  above  
Answer:  B  
Smoke  is  irritant  to  eye.  Smokers  twice  as  likely  to  experience  dry  eye.  
   
115.                            Trauma  to  the  face  that  lead  to  enucleation  of  one  eye.  Patient  came  later  
with  pain  in  the  other  eye.  What  it  the  pathophysiology?  
Answer:  Release  of  sequestered  antigen  
case  of  sympathetic  ophthalmia  
   
116.                            Patient  with  ptosis  and  eye  looking  down  and  out.  What  is  affected?  
A.     3rd  
B.     4th  
C.     3th  and  6th  
D.   3rd  and  4th  
Answer:  A  
   
117.                            Case  of  angle  closure  glaucoma  and  asked  about  the  diagnosis  
The  same  case  exactly  but  asked  about  treatment?  IV  acetazolamide  and  4%  pilocarpine  
Complication  of  cataract  surgery?  
Answer:    Endophthalmitis  
   
118.                            Which  one  of  the  following  drugs  is  contraindicated  to  be  given  in  Acute  
closure  angle  glaucoma?  
 Any  mydriatic  (ex:  atropine)  will  exacerbate  crisis.  
   
Q.  In  open  angle  glaucoma,  which  drug  is  contraindicated?  
   
119.                              Bronchocostrictive  disease  with  acute  angle  closure  glaucoma.  What  are  you  
going  to  give?  
A.  Timolol  
B.  Acetazolamide  
Answer:  B  
   
120.                            Adult  Case  of  eye  watery  discharge,  associated  with?  
A.     Dust  and  pollen  will  increase  symptoms  
B.     Retinal  exam  will  show  diabetic  and  hypertension  changes  
C.     Something  about  glaucoma  
Answer:  A  If  asking  about  allergic  conjunctivitis.  
   
121.                            Picture  of  glaucoma,  What  to  do  with  it  ?      
Inform  blood  relative  
   
122.                            Retnoblastoma  causes?  
A.          Strabismus  
B.          Squent  
C.          Leucocoria  (white  pupil)  
Answer:  C  
   
123.    Man  has  Eye  pain,  seeing  halos  ,  headache  ,  many  GI  symptoms  ,  Eye  examination  :  
red  and  injected  vessels  .  What  is  the  diagnosis?  
a-­‐  digoxin  toxicity  
b-­‐  Angle  closure  Glaucoma  
answer  B  
   
124.                            Clinical  Features  of  Angle  Closure  Glaucoma:  
•  red,  painful  eye  =  RED  FLAG  
•  unilateral,  but  other  eye  increased  risk  
•  decreased  visual  acuity,  vision  acutely  blurred  from  corneal  edema  
•  halos  around  lights  
•  nausea  and  vomiting,  abdominal  pain  
•  fixed,  mid-­‐dilated  pupil  
•  corneal  edema  with  conjunctival  injection  
•  marked  increase  in  IOP;  may  be  noticeable  even  to  palpation  (>40  mmHg)  
•  shallow  anterior  chamber  ±  cells  in  anterior  chamber  
Toronto  Note  
   
126.                            Patient  presented  with  history  of  right  eye  pain  and  decrease  vision  ..  flurocin  
staining  showed  (dendrites)..  wht’s  the  diagnosis?  
Answer:  herpes  keratitis  
http://emedicine.medscape.com/article/1194268-­‐overview  
   
128.  Target  diastolic  Bp  in  HTN  retinopathy:  
80-­‐90  in  2  days  
   
129.  Patient  came  to  ER  due  to  car  accident  had  DM  and  HTN    and  loss  of  vision  in  the  
peripheral:  
A.     Cataract  
B.     Glaucoma  
C.     RD  
Answer:  C  
   
130  A  question  with  a  picture  attached  showing  the  everted  lower  eyelid,  what  is  it  called?  
Answer:  Ectropion  
   
131.                            Which  of  the  following  can  be  managed  with  trabeculectomy?  
A.                            Angle  closure  glaucoma  
B.                            Open  angle  glaucoma  
Answer:B  
   
132.  Patient  with  bilateral  exophtalmous,  pulsatile,  with  bruit  on  auscultation:  
A.     hyperthyroidism  
B.     cavernous  sinus  thrombosis  
C.     carotid-­‐cavernous  fistula  
Answer:  C  
   
133.  picture  of  corneal  ulcer  that  is  showing  dendritic  lesion.  What  other  feature  is  associated  with  
it?  
A.     Scleritis  and  episcleritis  
B.     Chorditis  and  retinitis  
C.     Optic  neuritis  
D.   Corneal  hypoasthesia  
Answer:  D  
   
134.  Pt  with  unilateral  viral  conjunctivitis  then  became  bilateral.  What’s  your  advice  to  avoid  
spread  of  the  infection?  
A.  Isolate  yourself  from  others.  
   
135.  Patient  on  neostigmine  but  further  evaluation  showed  drop  eyelid  ?                      
Stop  neostagmin  (  my  answer  )  Add  pyrdostagmin                                                                                                                                        
This  a  case  of  ocular  MG  so  the  tx  is  Immune  suppression  with  steroids  is  often  the  main  therapy.  
https://www.ncbi.nlm.nih.gov/pubmed/22037997  
   
136.  One  with  eye  trauma,  the  first  thing  you  do  is  to  r/o:  
A.  foreign  body  
B.  keratitis  
C.  conjunctivitis  
Answer:  A  
   
137.  Hypertensive  patient,  showing  change  in  the  optic  disc  what  is  your  next  step  in  
management?  
A.     Referral  to  ophthalmology  
B.     Treat  her  
Answer:  A  
   

Psychiatry  
   
   
1-­‐Father  with  Schizophrenia  have  child  percentage  of  affected?  
A-­‐5%  
B-­‐10%  
C-­‐15%  
D-­‐  20  
Answer  Is  B  10%  
Reference  FA  step  one  2016  
   
2-­‐ADHD  child  ,  can't  involve  for  anything  for  long  time  ,  like  in  school  or  even  when  he  
playing  a  video  games  he  can't  finish  it  and  searching  for  something  else  to  do  and  so  
on,  which  type  of  ADHD  he  has?  
A-­‐  inattentive  
B-­‐  impulsive  
C-­‐  hyperactive  impulsive.  
Answer  is  A  
Three  types  of  ADHD:  
1-­‐inattention:  getting  distracted,  having  poor  concentration  and  organizational  skills  and  have  
trouble  focusing  on  a  single  task  
2-­‐impulsivity:  interrupting,  taking  risks  
3-­‐hyperactivity:  never  seeming  to  slow  down,  talking  and  fidgeting,  difficulties  staying  on  task  
   
3-­‐Patient  have  depression  and  came  to  the  clinic  complaining  of  decreased  libido  which  
drugs  you  will  shift  your  patient  to  ?  
A-­‐  paroxtein  
B-­‐  TCA  
C-­‐  amytriptaline  
Correct  answer  most  likely  one  of  the  three  highlight  with  red  :  
   
Consistent  evidence  shows  that,  with  the  exception  of  bupropion  (Wellbutrin),  trazodone  (Desyrel)  
and  nefazodone  (Serzone),  antidepressant  medications  may  cause  a  decline  in  libido  or  sexual  
functioning  despite  improvement  of  depression  
Ref:  http://www.aafp.org/afp/2000/0815/p782.html  
   
4-­‐questions  about  post  traumatic  stress  disorder,  I  can't  remember  exactly  what  was  about  
Post-­‐traumatic  stress  disorder  (PTSD)  is  a  mental  health  condition  that's  triggered  by  a  terrifying  
event  —  either  experiencing  it  or  witnessing  it.  Symptoms  may  include  flashbacks,  nightmares  and  
severe  anxiety,  as  well  as  uncontrollable  thoughts  about  the  event  
PTSD  symptoms  are  generally  grouped  into  four  types:  intrusive  memories,  avoidance,  negative  
changes  in  thinking  and  mood,  and  changes  in  physical  and  emotional  reactions.  Symptoms  can  vary  
over  time  or  vary  from  person  to  person  
   
   
5-­‐21  years  old  patient  with  known  case  of  depression,  has  been  found  on  the  floor  
unconscious  with  empty  pill  bottle  ,  patient  was  obtunded  ,  dilated  pupil  and  
unreactive  bilaterally  and  other  symptom(  I  can  not  remember)  
What  is  the  medication  the  patient  most  likely  on?  
A-­‐Sertraline  
B-­‐  Fluoxetine  
C-­‐  Other  SSRI  
D-­‐  Amitriptyline  
Answer  is  D  
 Ref:  https://emedicine.medscape.com/article/819204-­‐clinical#b4  
   
6-­‐which  one  of  the  following  SSRI  has  the  greatest  risk  in  pregnancy?  
A-­‐  Escitalopram  
B-­‐  Fluoxetine  
C-­‐  Paroxetine  
D-­‐  Sertraline  
Answer  C  
Ref:  https://www.drugs.com/pregnancy/paroxetine.html  
   
   
7-­‐2  year  and  half  kid  with  long  scenario  about  impulsive  behavior  and  communication  
impairment  with  other  and  the  distinct  point  of  the  question  is  echolalia  (which  is  
repetition  of  other  words)  what  is  the  diagnosis:  
A-­‐  ADHD  
B-­‐ASPENGER  SYNDROME  
C-­‐  AUTISTIC  DISORDERS  
D-­‐  ……………………………..  
B  is  a  part  of  C  ,  need  more  information  .  
   
8-­‐  there  was  question  about  ADHD  diagnostic  criteria  according  to  ICD10  CLASSIFICATION,  
but  I  cannot  remember  the  answers:  
But  it  was  like  this  :(  number  of  feature  and  the  description  of  it)  for  example:  
A-­‐  3  hyperactive,  3  inattentive,  and  other  
B-­‐  2  hyperactive,  3  inattentive  
C-­‐  2  hyperactive,  2  inattentive  
   
Answer:  
At  least  6  inattentive  +3  hyperactive  +  1  impulsive  
   
Ref:  https://adhd.org.sa/en/adhd/resources/diagnosing-­‐adhd/diagnostic-­‐criteria/icd-­‐10-­‐criteria/  
   
9-­‐  pt  suddenly  feel  the  environment  is  strange  ?  
 A-­‐  depersonalization  
B-­‐  derealization  
Answer  B  
   
Derealization  (sometimes  abbreviated  as  DR)  is  an  alteration  in  the  perception  or  experience  of  the  
external  world  so  that  it  seems  unreal.  Other  symptoms  include  feeling  as  though  one's  environment  
is  lacking  in  spontaneity,  emotional  colouring,  and  depth.  It  is  a  dissociative  symptom  of  many  
conditions  
https://www.mayoclinic.org/diseases-­‐conditions/depersonalization-­‐derealization-­‐
disorder/symptoms-­‐causes/syc-­‐20352911  
   
   
10-­‐  child  always  alone  ,  dose  not  have  toys  ,  doesn’t  play  with  others  ,  what  is  the  
relation  cause  or  something?  
A-­‐  intelligence  
B-­‐  interpersonally  
Answer:  B  ?  
   
11-­‐23  yrs  old  pt  has  hallucinations,  delusions,  for  1  month,  the  doctor  diagnosed  him  as  
a  case  of  schizophrenia,  Which  of  the  following  is  against  the  diagnosis  of  schizophrenia?  
A.          1/  age  of  the  pt  
B.          2/hallucinations  
C.          3/  delusions  
D.        4/  duration  of  symptoms  
Answer  is  D  
To  diagnose  schizophrenia  at  least  duration  should  be  6  month  
   
12-­‐pt  think  he  has  cancer  went  to  6  doctors  and  examination  was  normal,  but  still  he  think  
he  got  cancer,Dx?  
Hypochondriasis  
   
13-­‐  7  yrs  old  boy  with  clear  manifestation  of  ADHD  which  of  the  following  drugs  might  be  
given?  
A-­‐Atomaxatine  
Answer  is  A  
   
Drugs  can  be  used  :  
amphetamine    and  methylphenidate  Stimulants  usually  reduce  hyperactivity  and  impulsivity  and  
improve  focus.  
Or  Atomoxetine  
Or  Clonidine  and  guanfacine  
https://www.webmd.com/add-­‐adhd/tc/attention-­‐deficit-­‐hyperactivity-­‐disorder-­‐adhd-­‐medications  
   
14-­‐  definition  of  brief  psychosis  and  Schizophrenia/  schizoaffective(  duration)  
Schizophrenia  :  People  with  this  illness  have  changes  in  behavior  and  other  symptoms  -­‐-­‐  such  as  
delusions  and  hallucinations  -­‐-­‐  that  last  longer  than  6  months.  It  usually  affects  them  at  work  or  
school,  as  well  as  their  relationships.  
Schizoaffective  disorder:  People  have  symptoms  of  both  schizophrenia  and  a  mood  disorder,  such  as  
depression  or  bipolar  disorder.  
Schizophreniform  disorder:  This  includes  symptoms  of  schizophrenia,  but  the  symptoms  last  for  a  
shorter  time:  between  1  and  6  months.  
Brief  psychotic  disorder:  People  with  this  illness  have  a  sudden,  short  period  of  psychotic  behavior,  
often  in  response  to  a  very  stressful  event,  such  as  a  death  in  the  family.  Recovery  is  often  quick  -­‐-­‐  
usually  less  than  a  month.  
   
15-­‐Case  of  paralysis  of  the  upper  limbs  (and  I  guess  lower  limbs)  ..  Dx:  conversion  
disorder.  
Conversion  disorder  is  a  mental  condition  in  which  a  person  has  blindness,  paralysis,  or  other  
nervous  system  (neurologic)  symptoms  that  cannot  be  explained  by  medical  evaluation  
   
16-­‐  A  young  woman  came  with  low  mood,  insomnia,  and  features  of  depression  with  heavy  
menses.  These  symptoms  occur  9  days  prior  to  her  menses  and  disappear  the  second  day  of  her  
menses.  How  would  you  manage  her?  
A⁃  SSRI.  
B⁃  Estrogen.  
C⁃  Progesterone  
Answer  is  A  
   
 The  core  symptoms  of  premenstrual  syndrome  (PMS)  include  affective  symptoms  such  as  
depression,  irritability,  and  anxiety,  and  somatic  symptoms  such  as  breast  pain,  bloating  and  
swelling,  and  headache.  The  symptom(s)  must  impair  functioning  in  some  way  and  must  remit  at  
menses  or  shortly  thereafter.  Premenstrual  dysphoric  disorder  (PMDD)  is  a  more  severe  form.  
We  recommend  selective  serotonin  reuptake  inhibitors  (SSRIs)  as  first-­‐line  therapy  for  women  with  
premenstrual  symptoms.  Ref:  uptodate  
https://www.uptodate.com/contents/treatment-­‐of-­‐premenstrual-­‐syndrome-­‐and-­‐premenstrual-­‐
dysphoric-­‐
disorder?source=search_result&search=premenstrual%20syndrome&selectedTitle=1~150#H101868
128  
   
17-­‐define  ADHD  :  
   
Attention  deficit  hyperactivity  disorder  (ADHD)  is  a  disorder  that  manifests  in  childhood  with  
symptoms  of  hyperactivity,  impulsivity,  and/or  inattention.  The  symptoms  affect  cognitive,  
academic,  behavioral,  emotional,  and  social  functioning  
   
18-­‐female  pt  with  depression  and  suicidal  attempt,  stable  for  3  months  on  paroxetine  and  
now  she  is  pregnant,  what's  next?  
A-­‐continue  paroxitine  and  monitor  depression  
B-­‐stop  paroxitine  due  to  premature..  
C-­‐stop  paroxitine  due  to  it  damage  the  baby  
D-­‐continue  and  add  valium  
Most  likely  C  
   
Paroxetine  actually  carries  a  category  D  rating  from  the  U.S.  Food  and  Drug  Administration  (FDA)  for  
safety  in  pregnancy  
Some  SSRIs  are  generally  considered  a  safe  option  for  pregnant  women,  but  Paxil  already  carries  a  
warning  label  cautioning  against  use  during  pregnancy  due  to  the  potential  for  fetal  heart  defects.  
http://www.aafp.org/afp/2012/0415/p747.html  
   
19-­‐  pt  with  symptoms  of  schizophernia  for  3  months  and  then  return  to  normal.  what's  the  
Diagnosis?  
A-­‐Schizophernia  
B-­‐Schizoaffective  
C-­‐Schzioniform  
D-­‐breif  psychosis  
Answer  is  C  
   
20-­‐Anti  psychotic  causes  weight  gain?  
   
Weight  gain  and  metabolic  effects  are  the  most  prominent  side  effects  of  SGAs.  Clozapine  and  
olanzapine  are  especially  associated  with  these  problems  
Olanzapine  is  the  most  common  one  .    Uptodate  
   
-­‐21-­‐    most  common  anti  psychotic  causing  dystonia?  
Among  the  FGAs,  the  high-­‐potency  drugs  fluphenazine,  haloperidol,  loxapine,  pimozide,  and  
thiothixene  are  usually  associated  with  the  highest  risk  of  extrapyramidal  symptoms  
   
Haloperidol  is  the  most  common  one.    uptodate  
   
22-­‐  Antipsychotic  drug  causing  QT  prolongation?  
Prolongation  of  the  QT  interval  tends  to  be  mild  with  SGAs  but  somewhat  greater  with  iloperidone  
and  ziprasidone  than  with  other  agents.  uptodate  
   
-­‐23-­‐Atypical  Antipsychotic  least  to  cause  EPS?  
   
 risperidone  carries  the  highest  risk  of  EPS  (8  to  25  percent  in  adults)  
least  risk:  Quetiapine,  iloperidone,  pimavanserin,  and  clozapine  are  the  preferred  agents  in  patients  
at  high  risk  for  EPS.  uptodate  
   
   
24-­‐  Antiparkinsonism  which  is  hepatotoxic:  
Answer:  tolcapone  
   
Tolcapone  has  been  reported  to  cause  serum  aminotransferase  elevations  above  3  times  the  upper  
limit  of  normal  in  1%  to  5%  of  patients.  
Ref:  https://livertox.nih.gov/Tolcapone.htm  
   
25-­‐TCA  side  effect?  
3  C  (  cardiotoxicty  +  Coma+  convulsions)  +  Anticholinergics  
   
26-­‐TCA  most  associated  with  weight  gain?  
Answer:  Amitriptyline  
The  cyclic  antidepressants  block  histamine  receptors  and  cause  sedation,  increased  appetite  leading  
to  weight  gain,  confusion,  and  delirium.  The  most  potent  antihistaminic  drugs  are  maprotiline  and  
the  tertiary  tricyclics  amitriptyline,  doxepin,  and  trimipramine  
Ref:  
https://www.uptodate.com/contents/tricyclic-­‐and-­‐tetracyclic-­‐drugs-­‐pharmacology-­‐administration-­‐
and-­‐side-­‐
effects?source=search_result&search=tricyclic%20antidepressants%20and%20weight%20gain&selec
tedTitle=1~150#H26?  
   
   
27-­‐Q  about  MAOI  phenelzine,  HTN  with  cheese?  
MAOI  if  mixed  with  tyramine  conaiants  products  like  “  cheese  “  it  may  cause  HTN  crisis  
   
28-­‐  Nocturnal  enuresis  and  depression?  
Imipramine  (TCA)  
   
29-­‐T-­‐half  of  SSRI?  
Uptodate  says:  
The  half-­‐life  for  fluoxetine  ranges  from  1  to  3  days,  and  for  its  metabolite  norfluoxetine,  4  to  16  days.  
Fluvoxamine  has  a  half-­‐life  of  approximately  15  hours.      
   
https://www.uptodate.com/contents/selective-­‐serotonin-­‐reuptake-­‐inhibitors-­‐pharmacology-­‐
administration-­‐and-­‐side-­‐effects?source=search_result&search=T-­‐
half%20of%20SSRI&selectedTitle=1~150#H399779781  
   
30-­‐A  pt  with  depression  on  medications  found  comatose  with  empty  pill  bottle  beside  her  
On  invx;  ABG  showed  Metabolic  acidosis,  what  drug  overdosed?  
A-­‐Aspirin  
B-­‐  SSRI  
Answer  is  A  
Ref:  https://emedicine.medscape.com/article/1009987-­‐clinical  
   
31-­‐clear  scenario  about  depression  (low  mood  lack  of  sleep,,,,  what  is  deficient  in  this  
patient?  
serotonin  
   
32-­‐a  pt  presented  to  psychiatry  department,  the  pt  says  that  some  people  know  what  he  is  
thinking  about  and  his  thoughts  are  known  by  others  as  if  distributed.  This  type  of  thought  is?  
Answer:  thought  broadcasting  
   
 In  psychiatry,  thought  broadcasting  is  the  belief  that  others  can  hear  or  are  aware  of  an  individual's  
thoughts.  This  differs  from  telepathy  in  that  the  thoughts  being  broadcast  are  thought  to  be  
available  to  anybody.  Thought  broadcasting  can  be  a  positive  symptom  of  schizophrenia.  
   
33-­‐18  yrs  old  female,  e  hx  of  amenorrhea  for  6  months,  low  BMI,  but  she  thinks  that  she  is  
fat  and  has  to  lose  weight.  OE  (  increased  hair  distribution  in  her  body  plus  other  
findings)  
What  is  the  diagnosis?  
Answer:  Aneroxia  Nervosa  
   
34-­‐best  and  rapid  management  for  specific  anxiety?  
 A.  Benzodiazipines  
 B.  Sertraline  
 C.  Imipramine  
 D.  Bupropion  
Answer  is  A  
   
Long  Term  CBT  or  SSRI  
Short  term  BENZO  or  beta  blocker  .       Ref:  FA  step  one  2016  
   
35-­‐Best  drug  for  short  term  Mx  of  GAD  without  causing  dependence  or  addiction;  
1/alprazolam  
2/sertraline  
3/flouxetine  
4/Bupropion  
Answer:  D  (  I  think  it  is  a  misspelling  and  it  should  buspirone.  )  
BUSPIRONE  —  The  azapirone  buspirone  has  been  shown  in  clinical  trials  to  reduce  symptoms  of  
anxiety  in  patients  with  generalized  anxiety  disorder  (GAD),  offering  similar  efficacy  to  
benzodiazepines  without  the  risk  of  dependence.  Buspirone  is  thought  to  affect  the  serotonergic  
system  via  blockade  of  5HT1A  autoreceptors.  Buspirone  can  be  used  as  monotherapy  (in  the  
absence  of  comorbid  major  depression)  or  for  augmentation  at  doses  of  10  to  60  mg/day  
Reference:  Uptodate    
   
36-­‐A  child  always  kicking  his  mother,  shouting  at  her,  not  responding  to  her  commands  at  
all,  he  was  separated  from  her  after  she  was  divorced,  what  is  the  problem  here?  
1/depersonalization  
2/derealization  
3/anxiety  from  separation  
   
answer:C  “By  resident”  
correct  answer  is  Adjustment  
Adjustment  disorder  is  an  unusually  strong  or  long-­‐lasting  reaction  to  an  upsetting  event.  The  
triggering  event  might  be  a  divorce,  a  death  in  the  family,  moving  to  a  new  home,  starting  a  different  
school,  a  break  up,  or  a  big  life  disappointment.  A  child  with  the  disorder  will  have  a  hard  time  
coping  with  his  emotions  and  may  become  depressed  or  anxious,  exhibit  hostility,  pick  fights,  or  
refuse  to  go  to  school  
Ref:  https://childmind.org/guide/guide-­‐adjustment-­‐disorder/what-­‐is-­‐adjustment-­‐disorder/  
   
37-­‐Female,  known  case  of  schezophrenia,  came  to  ER  with  suspeciousness,  upon  examination,  
she  was  starring  at  a  person  and  saying,  *you  can't  kill  me*  ,  what  does  she  have  ?  
*Derealisation  
*Concrete  thinking  
*hallucination  
Most  likely  Delusion  
   
38-­‐An  elderly,  known  case  of  Alzheimer's  disease,  developed  hallucinations,bizarre  
behaviors  and  became  aggressive,  what  drug  to  add?  
1/Haloperidol  
2/Resperidone  
3/Chloropromazine  
4/Amytreptline  
Answer  is  B  
   
Atypical  antipsychotics  are  the  most  widely  used  class  of  psychotropic  medications  in  the  treatment  
of  AD  psychosis  and/or  behavioral  and  psychological  symptoms  of  dementia.  A  meta-­‐analysis  of  
atypical  antipsychotics  for  aggression  and  psychosis  in  AD  suggests  that  both  risperidone  and  
olanzapine  help  reduce  aggression  and  risperidone  helps  reduce  psychosis.9  
http://www.psychiatrictimes.com/geriatric-­‐psychiatry/managing-­‐psychosis-­‐patients-­‐alzheimer-­‐
disease/page/0/3  
   
39-­‐pt  came  with  symptoms  of  depression.  when  the  doctor  asked  him  about  psycosis  symptoms  
he  left  the  room.  what  does  this  represent?  
A-­‐  acting  out  
B-­‐  Intellectual  
C-­‐  Siblimation  
D-­‐  Interjection  
Answer  is  A  
   
40-­‐scenario  of  anorexia  nervousa,  admitted  to  ER,  she  
she  denies  eating  and  says  she's  not  hungry,  BMI  11.3  
What  most  likely  to  find  in  her  labs  :  
-­‐  inc  K  
-­‐  Dec  Creatnin  
-­‐  Other  labs  
   
Read  about  it  here  :  https://emedicine.medscape.com/article/912187-­‐workup#c9  
   
41-­‐Young  age  female  known  case  of  Bulemia  nervosa  came  to  the  ER  after  vomiting,  what  do  
you  expect  to  see  in  her  lab:  
A-­‐Hypokalemia  
B-­‐Elevated  liver  enzymes  
Answer  is  A  
   
•  Hypokalemic  hypochloremic  metabolic  alkalosis:  Observed  with  vomiting  
•  Acidosis:  Observed  in  cases  of  laxative  abuse  
   
42-­‐Patient  admitted  to  er  ,  upon  examination:  dental  smth?  
A-­‐Anorexia  nervosa  
B-­‐bulemia  nervosa  
Answer  is  B  
   
43-­‐anti-­‐psychotic  cause  constipation?  
Antipsychotic-­‐related  constipation  is  a  common  and  serious  adverse  effect  ,  especially  for  people  
taking  clozapine  
Ref:  http://www.cochrane.org/CD011128/SCHIZ_drug-­‐treatments-­‐constipation-­‐caused-­‐
antipsychotic-­‐medications  
   
44-­‐Dementia  rx?  
Cholinesterase  inhibitors.  These  medications  —  including  donepezil  (Aricept),  rivastigmine  (Exelon)  
and  galantamine  (Razadyne)  —  work  by  boosting  levels  of  a  chemical  messenger  involved  in  memory  
and  judgment  
   
45-­‐Repetitve  idea  scenario  pt  Knows  this  is  wrong?  
 obsession  
   
46-­‐psychotic  pt  on  neuroleptic  drug  the  side  effect?  
A-­‐seizure      
B-­‐akathisia  
Answer  is  B  
   
SE:  Acute  dystonia-­‐  Akathsia-­‐parkinsonism-­‐Tardive  dyskinesia.      FA  step  one  
   
   
47-­‐pt  with  Poor  self-­‐hygiene  ,  echolalia  what's  the  treatment?  
A.        Carbamazepine  
B.        SNRI  
C.          Lithium  
Answer:  
Incomplete  scenario,  most  likely  antipsychotic,  
   
48-­‐Female  fear  of    elevator  ,  which  of  the  following  is  the  best  treatment  ?  
A-­‐Flooding  
B-­‐imipramine  
C-­‐propranolol  
D-­‐psychoanalysis  
Answer  is  D  
   
49-­‐Pt  with  hallucination  and  low  self  esteem  ,  what's  the  treatment  ?  
A-­‐Antipsychotic  
B-­‐CBT  
C-­‐Antipsychotic  and  CBT  
answer:C    
   
50-­‐Female  pt  with  symptoms  of  menapose  and  symptoms  of  depression  ,  what's  the  treatment?  
A.        Estrogen  
B.        Progesterone  
C.          Paroxetine  
 Answer:  
   
51-­‐Impaired  focus  on  the  current  function  with  disruption  of  vitals  physical  activities.  
What  condition  have  these  characteristics?'  
1-­‐  OCD  
2-­‐  dissociative  disorder                                                                  
3-­‐  major  depression  
4-­‐  psychosis  
Answer  :  most  likely  major  depression  
   
52-­‐Prevention  of  dementia?  
A-­‐leisure  exercises  
B-­‐medication  
C-­‐just  rehab  like  option  
   
Answer  is  A  
We  encourage  all  patients,  especially  those  with  early  dementia  and  those  with  risk  factors  for  
dementia,  to  maintain  or  increase  physical  activity  and  exercise  as  long  as  there  are  no  
contraindications.  Similarly,  we  encourage  cognitive  leisure  activities  and  social  interaction  for  as  
long  as  these  are  feasible.  However,  we  recognize  that  these  lifestyle  factors  remain  unproven  as  a  
means  of  preventing  dementia  
Ref:  
https://www.uptodate.com/contents/prevention-­‐of-­‐dementia#H18  
   
53-­‐autism  case  the  child  eats  papers.  what  do  ?  
Answer:  Behavioral  therapy  
   
54-­‐Elderly  man  admitted  for  pneumonia,  developed  fluctuating  level  of  consciousness  and  
severely  disturbed  sleep/wake  cycle,  how  to  treat?  
a.  Regular  Haloperidol  until  symptoms  abate  
b.  Regular  Risperidone  until  symptoms  stop  
c.  Isolate  him  in  a  dark  quite  room  
d.  Allow  a  relative  to  stay  
Answer:  Most  likely  C  
   
   
55-­‐  Leaden  paralysis  in?  
A.  conversion  disorder.  
B.  personality  disorder.  
C.  somatization  
D.  schizophrenia.  
Regarding  these  options,  could  be  A  
-­‐         Leaden  paralysis  is  one  of  the  feature  of  atypical  depression  
-­‐         Conversion  disorder  is  a  mental  condition  in  which  a  person  has  blindness,  paralysis,  or  
other  nervous  system  (neurologic)  symptoms  that  cannot  be  explained  by  medical  evaluation  
   
56-­‐CAGE  questionnaire?  
Have  you  ever  felt  you  needed  to  Cut  down  on  your  drinking?  
Have  people  Annoyed  you  by  criticizing  your  drinking?  
Have  you  ever  felt  Guilty  about  drinking?  
Have  you  ever  felt  you  needed  a  drink  first  thing  in  the  morning  (Eye-­‐opener)  to  steady  your  nerves  
or  to  get  rid  of  a  hangover?  
   
57-­‐Disease  associated  with  clinical  dementia:  
A.  b12  deficiency  
B.  Parkinson’s  
Answer  is  A  
   
58-­‐Teenage  Pt.  Got  a  bad  grade  in  an  exam  and  feels  guilty,  he  try  to  explain  to  everyone  
he  meets/  talk  with  that  the  exam  was  poorly  written&  its  not  his  fault  that  he  got  a  
bad  grade.  what  does  this  represent?  
-­‐  intellectualization  
-­‐  sublimation  
Answer:  Both  choices  are  wrong.  I  think  the  right  answer  is  one  of  the  missing  choices,  if  
rationalization  was  one  of  the  choices  then  I  think  it  would  an  appropriate  answer.    
   
-­‐Intellectualization:  avoiding  negative  feeling  by  excessive  use  of  intellectual  functions  and  by  
focusing  on  irrelevant  details  (e.g.  physician  dying  from  colon  cancer  describes  the  pathophysiology  
of  the  disease  in  detail  to  his  12  y/o  son.)  
-­‐  sublimation:  replacing  an  unacceptable  wish  with  a  course  of  action  that  is  similar  to  the  wish  but  
does  not  conflict  with  one's  value  system.  (e.g.  Teenager's  aggression  toward  his  father  is  
redirected  to  perform  well  in  sports)  
-­‐Rationalization:  proclaiming  logical  reasons  for  actions  performed  for  other  reasons,  usually  to  
avoid  self-­‐blame.    (e.g.    after  getting  fired,  claiming  that  the  job  was  not  important  anyway).  
Reference:  FA,  FA  psychiatry.  
   
Feeling  hopeless  is  another  form  for  expressing  (or  indicates  or  assess)  which?  
-­‐  suicidal  ideation  
-­‐  Depression  
-­‐  Anxiety  
Answer:  A  
Reference:  SMLE12  
   
what’s  the  most  common  sign  (presentation)  of  depression?  
A.  late  morning  awaking  
B.  loss  of  eye  contact  
Answer:  Could  not  find  a  source  about  the  most  common  sign  of  depression.  Probably  the  question  
is  not  written  correctly,  see  the  next  question.  
   
What  is  associated  with  major  depression?  
-­‐  loss  of  eye  contact  
-­‐  Late  awakening  from  sleep  
-­‐  Hallucination  
Answer:  
MDD  DSM-­‐5:  
Must  have  at  least  five  of  the  following  symptoms  (must  include  either  number  1  or  2)  for  at  least  a  
2-­‐week  period:  
1-­‐          Depressed  mood  of  the  time  
2-­‐          Anhedonia  (loss  of  interest  in  pleasurable  activities)  
3-­‐          Change  in  appetite  or  weight  (increase  or  decrease)  
4-­‐          Feelings  of  worthless  or  excessive  guilt  
5-­‐          Insomnia  or  hypersomnia  
6-­‐          Diminished  concentration.  
7-­‐          Psychomotor  agitation  or  retardation  (i.e  restlessness  or  slowness  )  
8-­‐          Fatigue  or  loss  of  energy  
9-­‐          Recurrent  thoughts  of  death  or  suicide  
   
Hypersomnia:  refers  to  symptoms  of  excessive  quantity  of  sleep,  reduced  quality  of  wakefulness  and  
sleep  inertia/sleep  drunkenness  (i.e.  impaired  performance  and  reduced  alertness  after  awakening).      
They  often  complain  of  non-­‐restorative  sleep,  automatic  behaviors  (routine  behavior  performed  
with  little  to  no  recall),  and  difficulty  awakening  in  the  morning.  
Reference  :  FA  psychiatry  
   
schizophrenic  complaint  on  medication  came  with  metabolic  acidosis  what  is  the  drug?  
-­‐Olanzapine  
-­‐Clozapine  
-­‐haloperido  
Answer:    C  (Not  sure).  
The  only  thing  I  found  is  that  patients  receiving  haloperidol  had  significantly  higher  blood  lactate  
levels  than  patients  receiving  olanzapine  
   
Probably  the  question  was  not  written  correctly  and  original  question  was  asking  about  metabolic  
syndrome  instead  of  metabolic  acidosis,  and  in  that  case  the  answer  would  be  Olanzapine  (  both  
clozapine  and  olanzapine  are  known  to  cause  it  but  it  is  much  more  common  with  olanzapine).  
   
Haloperidol  has  the  least  potential  to  cause  metabolic  syndrome.  Clozapine  and  risperidone  also  
have  the  potential  to  cause  metabolic  syndrome  but  have  a  lower  potential  to  do  so  as  compared  
with  olanzapine.    References:  https://www.ncbi.nlm.nih.gov/pmc/articles/PMC3136014/  
https://www.ncbi.nlm.nih.gov/pubmed/21328719  
   
Antipsycotic  drug  causes  eye  pigmentation?  
Chlorpromazine  causes  corneal  and  lenticular  deposits  
Thioridazine  causes  Retinal  deposits  
Reference:  FA  psychiatry  
   
2  cases  about  panic  disorder  (ttt  and  dx)  
DSM-­‐5:  
-­‐  Recurrent,  unexpected  panic  attacks  without  identifiable  trigger.  
-­‐  One  or  more  panic  attack  followed  by  >1  month  of  continuous  worry  about  experiencing  
subsequent  attacks  or  their  consequences  and/or  a  maladaptive  change  in  behavior(e.g.  avoidance  
of  possible  triggers)  
-­‐  Not  caused  by  another  mental  disorder,  substance  abuse  or  medical  condition.  
Reference:  FA  psychiatry  
   
In  panic  disorder,  how  to  manage?  
Answer:  If  a  single  panic  attack  is  the  diagnosis,  a  
benzodiazepine  is  the  treatment.  If  panic  disorder  is  the  diagnosis,  then  pick  the  SSRI.  
Reference  :  MTB2  
   
definition  of  delusion?  
Delusions  are  fixed,  false  beliefs  that  remain  despite  evidence  to  the  contrary  and  cannot  be  
accounted  for  by  the  cultural  background  of  the  individual.  
Reference:  FA  psychiatry  
   
adjustment  disorder?  
DSM-­‐5:  
1-­‐          Development  of  emotional  or  behavioral  symptoms  within  3  months  in  response  to  identifiable  
stressful  life  event.  These  symptoms  produce:  
-­‐Marked  distress  in  excess  of  what  would  be  expected  after  such  an  event.  
-­‐  significant  impairment  in  daily  functioning.  
2-­‐  Symptoms  are  not  those  of  normal  bereavement  
3-­‐  symptoms  resolve  within  6  months  
“  in  adjustment  disorder,  the  stressful  event  is  not  life  threatening.  In  post-­‐traumatic  stress  disorder,  
it  is.”  
Reference:  FA  psychiatry  
   
   
2  q  about  SE  of  antidepressant  Tca  and  SSri?  
TCA:  
-­‐Antihistamine:  weight  gain  and  sedation  
-­‐  Antiadrenergic:  orthostatic  hypotension,  reflex  tachycardia,  arrhythmia  (Wide  QT,QRS  and  PR  
interval)  
-­‐Anticholinergic:  Dry  mouth,  blurred  vision,  urine  retention  ,  constipation,  exacerbation  of  narrow  
angle  glaucoma.  
SSRi:  
GI  disturbances:  Nausea  and  diarrhea.  
Insomnia  and  vivid  dreams  
Sexual  dysfunction  :  decreased  libido  ,  anorgasmia  ,  delayed  ejaculation.  
Restlessness.  
Reference:  FA  psychiatry.  
   
what  is  the  least  fatal  dose  of  Tca  
A.        500  mg  
B.        1g  
C.          2g  
D.        5g  
   
Answer:  B  
Reference:  Comprehensive  Emergency  Mental  Health  Care  By  Joseph  J.  Zealberg,  Alberto  B.  Santos,  
Jackie  A.  Puckett  
   
Patient  had  history  of  pancreatic  cancer  on  chemotherapy  then  improved  completely  came  to  
doctor  concerning  about  recurrence  of  cancer  and  a  history  of  many  hospital  visits.  This  
patient  has:  
a)  Malingering  
b)  somatization  
c)  Factitious  
d)  Conversion  
Answer:  A  
History  of  many  hospital  visits  in  a  patient  with  cancer  (in  this  case  pancreatic  cancer)  and  the  
patient  is  claiming  that  he  has  recurrence  would  let  us  think  that  he/she  is  seeking  narcotics.  Based  
on  this  information  the  answer  would  be  A.    “This  is  my  personal  explanation”.  
   
Patient  consciously  fakes,  profoundly  exaggerates  or  claims  to  have  a  disorder  in  order  to  attain  a  
specific  2ndary  (external)  gain  (e.g.  avoiding  work,  obtaining  compensation).  Poor  compliance  with  
treatment  or  follow-­‐up  of  diagnostic  tests.  Complaints  cease  after  gain  (vs.  factitious  disorder).  
   
Factitious:  patients  with  factitious  disorder  intentionally  falsify  medical  or  psychological  signs  and  
symptoms  in  order  to  assume  the  role  of  sick  patient.  
Malingering:  Intentional  reporting  physical  or  psychological  symptoms  in  order  to  achieve  personal  
gain.      Reference:  FA  ,  FA  psychiatry  
   
patient  with  irregular  menses  BMI  was  16  she  think  that  she  is  fat  and  want  to  lose  
weight  what  the  diagnosis?  
-­‐  bulimia  nervosa  
-­‐  anorexia  nervosa  
Answer:  B  
Reference:  FA  psychiatry  
   
18  yrs  old  female,  e  hx  of  amenorrhea  for  6months,  low  BMI,  but  she  thinks  that  she  is  
fat  and  has  to  lose  weight.  OE  (  increased  hair  distribution  in  her  body  plus  other  
findings)
What  is  the  diagnosis?  
anorexia  nervosa  
“repeated”  
Reference:  FA  psychiatry  
   
what  is  best  to  describe  post  traumatic  syndrome?  
A)  disorientation  and  dissociation  
B)  Flashback  and  arousal  
Answer:B  
Reference:  FA  psychiatry  
   
outcome  of  45  y/o  pt  with  recurrent  schizophrenia  on  maintenance  therapy?  
A)  complete  remission  
B)  70%  will  have  good  quality  of  life.  
C)  one  third  of  patients  will  have  good  prognosis  
   
Answer:  C  
Even  with  medication  40-­‐60%  of  patients  remain  significantly  impaired  after  their  diagnosis,  while  
only  20-­‐30%  function  fairly  well  in  the  society.  About  20%  of  patients  with  schizophrenia  attempt  
suicide.  
   
Reference:  FA  psychiatry  +  BMJ/schizophrenia  prognosis  
   
Child  with  rapid  blinking  of  the  eye  communicate  with  his  parents  during  it  what  is  the  
diagnosis  all  are  normal?  
A)TIC  
B)  petit  mal  seizure  
C)  tourette  syndrome.  
Answer:  A  
Reference  :  FA  psychiatry  
   
4  years  old  have  decrease  in  head  growth  ,  has  weird  hand  movement  (  wiringing)  ,  lost  
expressive  end  receptive  language  skills  ,  lost  his  interest  in  his  social  environment  
.  what  is  the  diagnosis?  
-­‐  Autism  
-­‐  Mental  retardation  
-­‐  Rett  syndrome  
Answer:C  
Reference:  FA  psychiatry  
   
   
4y/o  child  doesn’t  speak  except  mama  papa  response  will  to  verbal  command  his  older  
brother  had  the  same  problem  when  he  was  same  age  he  is  shy  what  is  the  diagnosis:  
A)Autsim  
B)  development  language  disorder  
Answer:  B  
   
Reference:  
https://www.uptodate.com/contents/expressive-­‐language-­‐delay-­‐late-­‐talking-­‐in-­‐young-­‐children  
   
http://www.childspeech.net/u_iv_e.html  
   
Cancer  associated  with  depression:  
A)  pancreas  
B)  prostate  
C)  kidney.  
D)  Liver.  
 Answer:  A  
Reference:  FA  psychiatry  p.37  
https://www.ncbi.nlm.nih.gov/pmc/articles/PMC4356432/  
   
What  is  the  percentage  of  a  child  to  have  schizophrenia  if  his  father  has  the  disease?  
A.   5%,  
B.    10%,  
C.    15%,  
D.    20%  
Answer:  B  
10%  FA  step1,  12%  in  FA  psychiatry,  10%  in  http://schizophrenia.com/family/FAQoffspring.htm  
.the  average  risk  for  first  degree  relatives  of  a  person  with  schizophrenia  is  9%.(  The  Encyclopedia  
of  Schizophrenia  and  Other  Psychotic  Disorders).  
   
A  female  patient  was  hearing  voices  coming  out  of  refrigerator  and  microwave  telling  
the  food  is  poisoned  what  does  she  has?  (  auditory  hallucinations)  
Reference:  Toronto  notes    
   
best  and  rapid  management  for  specific  anxiety?  
 A.  Benzodiazepines  
 B.  Sertraline  
 C.  Imipramine  
 D.  Bupropion  
Answer:  A  
Reference:  FA  psychiatry  
   
Best  drug  for  short  term  Mx  of  GAD  without  causing  dependence  or  addiction;  
1/alprazolam  
2/sertraline  
3/fluoxetine  
4/Bupropion  
   
Answer:  D  (  I  think  it  is  a  misspelling  and  it  should  buspirone.  )  
BUSPIRONE  —  The  azapirone  buspirone  has  been  shown  in  clinical  trials  to  reduce  symptoms  of  
anxiety  in  patients  with  generalized  anxiety  disorder  (GAD),  offering  similar  efficacy  to  
benzodiazepines  without  the  risk  of  dependence.  Buspirone  is  thought  to  affect  the  serotonergic  
system  via  blockade  of  5HT1A  autoreceptors.  Buspirone  can  be  used  as  monotherapy  (in  the  
absence  of  comorbid  major  depression)  or  for  augmentation  at  doses  of  10  to  60  mg/day  
Reference:  Uptodate    
   
Married  man  feeling  anxiety,  irritability,  weight  loss  and  trouble  sleeping.  He  has  no  
marital  problems  and  no  external  cause  for  his  anxiety.  What  is  the  most  likely  
diagnosis?  
GAD  
MDD  
Schizophrenia  
Answer:  A  
Reference:  Toronto  notes  
   
Newly  Married  couple  came  to  psychiatry  clinic.  The  wife  is  complaining  that  her  
husband  goes  back  home  multiple  times  to  check  if  they  locked  the  door  and..  (other  OCD  
symptoms).  The  husband  does  not  deny.  Diagnosis?  
OCD  
Answer:  A  
 Reference:  Toronto  notes  
   
Man  with  Premature  ejaculation.  What  to  give?  
SSRI  
Reference:  Toronto  notes  
   
Anxiety  medication  that  does  not  cause  dependence?  
-­‐One  of  the  benzodiazepines  
-­‐An  SSRI  
-­‐Buspirone  
Answer:  C  
Reference:  
 BUSPIRONE  —  The  azapirone  buspirone  has  been  shown  in  clinical  trials  to  reduce  symptoms  of  
anxiety  in  patients  with  generalized  anxiety  disorder  (GAD),  offering  similar  efficacy  to  
benzodiazepines  without  the  risk  of  dependence.  Buspirone  is  thought  to  affect  the  serotonergic  
system  via  blockade  of  5HT1A  autoreceptors.  Buspirone  can  be  used  as  monotherapy  (in  the  
absence  of  comorbid  major  depression)  or  for  augmentation  at  doses  of  10  to  60  mg/day  
Reference:  Uptodate    
Man  with  symptoms  of  sadness,  grief,  wife  died  two  months  ago.  Dx?  
-­‐MDD  
-­‐Bereavement  
Answer:  B  
 Reference:  Toronto  Notes  
   
Treatment  of  OCD?  
Increase    sertonine  availability  
Decrease  sertonine  availability  
Increase  sertonine  secretion  
Decrease  sertonine    secretion  
Answer:  A  
Reference:  FA  psychiatry  
   
Elderly  take  haloperidol,  and  came  to  ER  afebrile  rolled  eye.  what's  the  Dx?  
Neuroleptic  syndrome  
Dyskinesia  
Antonia  
   
Answer:  The  correct  answer  is  dystonia  
Reference:  Toronto  notes  
   
Clozapine  used  for  which  childhood  psychiatric  disorder?  
a-­‐  Schizophrenia  
b-­‐  Bipolar  
c-­‐  Mania  
 Answer:  A  
   
Reference:  https://www.ncbi.nlm.nih.gov/pubmed/18220495  
http://www.jaacap.com/article/S0890-­‐8567(09)61687-­‐7/abstract  
   
Female  diagnosed  with  depression  on  medication  and  has  constipation  Which  of  the  
following  is  the  drug  used  
a-­‐  TCA  
b-­‐  SSRI  
c-­‐  SSNRI  
Answer:  A  
Reference:  FA  psychiatry  
   
what's  the  first  line  treatment  of  depression?  
SSRI  
   
Reference:  Toronto  notes  
   
What  is  the  first  line  in  the  management  of  depression  in  children?  
A-­‐  Fluoxetine  
B-­‐  Imipramine  
C-­‐  MOAI  
Ans:  A  
   
Reference:  Toronto  notes  
   
58  years  old  man  is  complained  of  tiredness  and  fatigue,  and  after  his  wife  died  with  MI  when  she  
was  67  years  old,  his  symptom  increased  and  new  symptoms  are  (insomnia,  there  is  death  
thoughts  and  lack  of  concentration  in  work  and  loss  of  interest),  no  
loss  of  appetite.  Your  diagnosis  ?  
-­‐dysthermia.  
-­‐major  depressive  disorder.  
-­‐bereavement.  
Answer:  B  
5  symptoms  (including  loss  of  interest  “anhedonia”)  
MDD  DSM-­‐5:  
Must  have  at  least  five  of  the  following  symptoms  (must  include  either  number  1  or  2)  for  at  least  a  
2-­‐week  period:  
1-­‐          Depressed  mood  of  the  time  
2-­‐          Anhedonia  (loss  of  interest  in  pleasurable  activities)  
3-­‐          Change  in  appetite  or  weight  (increase  or  decrease)  
4-­‐          Feelings  of  worthless  or  excessive  guilt  
5-­‐          Insomnia  or  hypersomnia  
6-­‐          Diminished  concentration.  
7-­‐          Psychomotor  agitation  or  retardation  (i.e  restlessness  or  slowness  )  
8-­‐          Fatigue  or  loss  of  energy  
9-­‐          Recurrent  thoughts  of  death  or  suicide  
   
Reference:  FA  Psychiatry    
   
Drug  that  maintain  treatment  of  bipolar  disorder  ?  
Lithium  
SSRI  
Answer:  A  
Reference:  Toronto  notes  
   
   
   
   
Patient  when  presenting  a  topic  in  a  meeting  started  to  have  palpitations,  anxiety,  she    avoids  to  
give  presentations  so  not  to  experience  the  previous  feeling?  
Social  phobia  
Post-­‐traumatic  stress  disorder  
Specific  phobia  
Panic  disorder  
Answer:  A  
Reference:  FA  psychiatry  
   
Best  treatment  for  hallucinations  is?  
antipsychotics.  
Reference:  Toronto  notes  
   
Kid  having  problem  with  eye  contact  and  has  weird  behiavor?  
Autistic  disorder  
Autism  spectrum  disorder  is  characterized  by  impairments  in  social  communication/interaction  and  
restrictive  behaviors/interests.  Problems  with  social  interaction  and  communication  like  (inability  to  
hold  conversation  and  decreased  eye  contacts).  Restricted  repetitive  patterns  of  behavior,  interest  
and  activities  like  (  preoccupation  with  unusual  objects  ,  rigid  thought  patterns,  hand  flapping..etc)  
Reference:  FA  psychiatry  
   
Psychiatric  drug  cause  dystonia  ?  
Olanzapin  
Clozaoin  
Respiridone  
   
Answer:  If  the  missing  choice  is  1st  generation  antipsychotic  then  it  is  the  right  answer.  
Between  these  choices,  C  is  the  most  appropriate  answer.  
Reference:  Manual  of  Basic  psychiatry.  
   
 Patient  complaining  of  multiple  symptoms  for  2  yrs  examination  and  labs  normal  ?  
A)  Somatization  
B)  Conversion  disorder  
C)  Ocd  Schizophrenia  
   
Answer:  A  
Reference:  FA  psychiatry  
   
Pt  came  to  ER  he  said  (his  neighbor  want  to  kill  him)  and  he  hearing  his  neighbor  voice,  what  is  dx?  
Schizophrenia  
Reference:  Toronto  notes  
   
Parents  of  child  noticed  that  he's  sitting  alone,  not  developing  well,  playing  with  his  toy,  and  have  
repetitive  behavior  what's  ur  dx:  
A-­‐  Global  developmental  delay  
B-­‐  Autism  
Ans:  B  
Reference:    FA  psychiatry  
   
What  is  the  treatment  of  generalized  anxiety  disorder?  
1-­‐tricyclice  antidepressants  
2-­‐benzodiazepines  (used  in  acute  attack)  
3-­‐ssri  
4-­‐buspirone  
Answer:  C  
Reference:  FA  psychiatry  
   
Patient  with  dysmenorrhea,  breast  tenderness  and  mood  symptoms.  what's  the  treatment?  
Progesterone  
SSRI  
Reference:  MTB2  
   
Patient  his  family  member  died  from  3  weeks  he  has  hallucinations  and  acting  lost.  
what's  the  Dx?  
schizophrenia
  
brief  psychotic  disorder  
schizoaffective  disorder  
Answer:  B  
Brief  psychotic  disorder:  
Patient  with  psychotic  symptoms  as  in  schizophrenia,  however  the  symptoms  last  up  to  1  month.  It  
may  be  seen  in  reactions  to  extreme  stress  such  as  bereavement,  sexual  assault….  etc.  
   
DSM-­‐5  schizoaffective  disorder:  
-­‐Meet  the  criteria  for  either  Major  depressive  or  manic  episode  which  psychotic  symptoms  
consistent  with  schizophrenia  are  also  met.  
-­‐  delusions  or  hallucinations  for  2  weeks  in  the  absence  of  mood  disorder  symptoms  (this  criterion  is  
necessary  to  differentiate  schizoaffective  disorder  from  mood  disorder  with  psychotic  symptoms).  
-­‐  mood  symptoms  present  during  the  majority  of  psychotic  illness.  
   
Reference:  psychiatry  FA  
   
Obseesive  complussive  disorder,  what  is  the  mechanism  of  action  of  the  drug  used  to  treat  it?  
Serotonin  reuptake  inhibitor/increase  serotonin  availability  
Reference:  Psychiatry  FA    
   
A  female  pt  had  been  isolating  herself,  and  thinks  people  are  chasing  her,plus  she  avoids  eye  
contact  ?  
 Psychosis/  schizophrenia  
Reference:  Toronto  notes  
   
Pt  visits  psychiatrist  and  the  do  noticed  depressive  symptoms,  while  taking  hx,  she  was  
not  responding,  then  stopped  visiting?  
There  is  another  form  of  the  question:  patient  came  with  symptoms  of  depression,  when  doctor  
asks  him  questions  about  psychosis,  the  patient  left  the  hospital,  what  does  that  mean?  
A)  Acting  out  
B)  intellectual  
C)  sublimation  
D)  interjection  
Answer:  A  
Acting  out:  Expressing  unacceptable  feelings  and  thoughts  through  actions.  
Reference:  FA  
   
Your  friend  avoids  you,  what  type  of  personality?    
Answer:  Avoidant  
Avoidant  personality:  Patient  with  avoidant  personality  disorder  have  a  pervasive  pattern  of  social  
inhibition  and  an  intense  fear  of  rejection.  They  will  avoid  situations  in  which  they  may  be  rejected.  
Their  fear  of  rejection  is  so  overwhelming  that  it  affects  all  aspects  of  their  lives.  They  avoid  social  
interactions  and  seek  jobs  in  which  there  is  little  interpersonal  contact.  
Reference:  FA  psychiatry  
   
A  5  yr  old  boy  who  is  shy  and  can  only  say  mama,  baba,  his  older  brother  was  like  him?  
A)Autsim  
B)  development  language  disorder  
Answer:B  
Reference:  
https://www.uptodate.com/contents/expressive-­‐language-­‐delay-­‐late-­‐talking-­‐in-­‐young-­‐children  
   
http://www.childspeech.net/u_iv_e.html  
   
Most  common  risk  factor  in  schizophrenia  ?  
both  parents  have  it  
   
I  could  not  find  a  source  about  the  most  common  risk  factor  of  schizophrenia  and  I  think  that  the  
question  was  asking  about  the  most  significant  risk  factor  instead  of  most  common,  in  which  the  
answer  will  be  Monozygotic  twins  or  if  the  both  parents  have  schizophrenia  
-­‐Genetic:  40%  concordance  in  monozygotic  (MZ)  twins;  46%  if  both  parents  have  
schizophrenia;  10%  of  dizygotic  (DZ)  twins,  siblings.        Reference:  Toronto  notes  
   
Man  with  erectile  dysfunction,  but  in  morning,  normal  erectile  fuction,  where  to  refer  ?  
cardio,  
neuro,  
psychological  
Answer:  C  
   
Reference:  Toronto  notes  
   
Lady  with  low  mood,  insomnia,  poor  appetite  for  5  months.  Her  symptoms  get  worse  during  
the  premenstrual  period.  The  symptoms  do  NOT  resolve  after  menstruation.  What  is  the  
management?  
A-­‐Reassure  
B-­‐Give  fluoxetine  for  premenstrual  syndrome  
C-­‐Give  antidepressant  for  postmenstrual  syndrome  
D-­‐Refer  for  psychological  evaluation  
Answer:  Mostly  D  by  exclusion.  Definitely  not  reassurance.  the  symptoms  usually  resolve  after  
menstruation  in  PMS  and  there  is  nothing  called  postmenstrual  syndrome.  
   
Most  potency  antipsychotic  drug?  
Haloperidol  
I’m  not  sure  if  it  is  the  most  potent  or  not.  But  1st  generation  antipsychotics  are  classified  based  on  
their  potency:  High  potency  antipsychotics  include  haloperidol  ,  fluphenazine  ,  trifluoperazine  and  
pimozide..     Reference:  FA  psychiatry  
   
Patient  complain  of  chest  pain,  vertebral  pain,  a  lot  of  complain  with  pain  also  has  past  history  of  
complain  of  pain  and  nausea  and  vomiting  with  irregular  menses  weakness  in  left  leg.  Malingering  
and  factious  ruled  out,  diagnosis?  
A.        Pain  disorder  
B.        Somatization  disorders  
Answer:  B  
Pain  disorder  removed  from  DSM-­‐5.  
Reference:  FA  psychiatry  
   
Patient  had  social  phobia,  avoid  gathering,  on  work  had  palpitation  chest  pain  before  
meeting  since  5  years  treatment?  
A.          Sartoline  
B.          Buporin  
C.          Azla  (  benzo)  
D.        Beta  blocker  
Answer:  Sertraline  
Reference:  FA  psychiatry  
   
Women  is  anxious  ,  irritable  ,  insomina  for  2  months?  
A.          Major  depression  
B.          GAD  
C.          Social  phobia  
Answer:  B  
   
Erectile  dysfunction?  Options>  multiple  drugs  including  Mertazepine  
Answer:  Mertazepine  does  not  cause  erectile  dysfunction  ,  choose  SSRI  instead.  
   
Cancer  patient  on  SSRI,  anti  cancer  drug,  .  Feverish.  Agitated  but  alert.  (+Mydriasis?)  
and  splenomegaly  on  examination?  
A.          1.Neuroleptic  malignant  syndrome  
B.          2.Serotonin  syndrome  
C.          3.Side  efffect  of  anti  cancer  drug  
Answer:  B  
Reference:  FA  psychiatry  
   
Which  of  the  following  has  a  poor  prognosis  for  schizophrenia?  
A.          acute  onset  
B.          adolescents  age  onset  
C.          family  history  
D.        panic  attack  during  psychosis  
Answer:  C  
Reference:  FA  psychiatry  
   
A  patient  with  chronic  back  pain  for  which  she's  taking  NSAID  for  more  than  1  year.  No  history  of  
trauma  and  her  sphincter  function  is  normal.  She's  also  complaining  of  mild  epigastric  discomfort.  
She's  concerned  about  her  financial  status,  What  would  you  give  her?  
1)  escitalopram  
2)  amitryptaline  
3)  gabapentin  
Answer:  B  
Reference:  BMJ/Back  pain  
   
A  patient  came  with  aggressive  hostile  and  violent  behavior.  What  are  the  neurotransmitters  
responsible  for  his  behavior?  
1)  high  serotonin  
2)  low  serotonin  
3)  high  endorphins  
4)  low  endorphins  
Answer:  2  
A  deficit  in  serotonin  has  been  theorized  to  have  a  primary  role  in  causing  impulsivity  and  
aggression.  
Reference:  https://en.wikipedia.org/wiki/Aggression  
   
 
 
 
 
 
 
 
 
 
 
 
 
 
 
 
 
 
 
 
 
 
 
 
 
 
 
 
 
 
 
 
 
 
 
Basic  Science    
   
1.          Child  with  lactic  acidosis  +  hyperammonemia  ?  
a)     -­‐pyruvate  dehydrogenase  deficiency.  
b)     -­‐pyruvate  carpoxylase  deficiency.  
ANSWER:  A  
REFERENCES:  https://www.ncbi.nlm.nih.gov/pubmed/4050860  
   
·∙              Child  with  high  lactic  acid?  
Answer:  Pyruvate  dehydrogenase  deficiency  
https://emedicine.medscape.com/article/948360-­‐overview  
   
 ●   Scenario   of   a   child   with   hyperammonemia  
and  lactic  acidosis.  Which  enzyme  deficiency  
results  in  this  disorder?  
1.          Pyruvate  dehydrogenase  
2.          Pyruvate  carboxylase  
Answer:  A  
 
   
 
 
 
 
 
 
 
 
 
 
 
 
 
 
 
2.          Dizygotic  twins?  
a)          Dichorionic  diamniotic  regardless  to  the  sex  
b)          Dichorionic  monamniotic  same  sex.  
ANSWER:  A  dizygotic  twin  pregnancy  always  results  in  dichorionic  -­‐  diamniotic  pregnancies.    
REFERENCES:  https://radiopaedia.org/articles/dizygotic-­‐twin-­‐pregnancy-­‐1  
   
3.          Oxybutynin  mechanism  of  action?  
a)          antagonist  muscrinic  
b)     agonist  nicotinic.  
ANSWER:    A.    anticholinergic  -­‐    antimuscarinic  “  M1,M2,M3”  antagonist.  
REFERENCES:  https://en.wikipedia.org/wiki/Oxybutynin  
   
4.          Bisphosphonate  drugs  mechanisms,  and  then  asked  which  one  ?  
 ANSWER:  Bone  stabilizer—bisphosphonate;  pyrophosphate  analog;  reduces  hydroxyapatite  crystal  
formation,  growth,  and  dissolution,  which  reduces  bone  turnover  
REFERENCES:  step  up  to  step  1.  
   
5.          Which  one  of  the  choice  secrets  gulcocorticod  hormones?  
A-­‐Reticularis  
B-­‐Glomerulosa  
C-­‐Medulla  
D-­‐Faciculata  
   
ANSWER:  D.  Glucocorticoids  are  chiefly  produced  in  the  zona  fasciculata  of  the  adrenal  cortex,  
whereas  mineralocorticoids  are  synthesized  in  the  zona  glomerulosa.  
REFERENCES:  https://en.wikipedia.org/wiki/Glucocorticoid  
   
6.          case  scenario  about  patient  with  hypertention  ,  labs  result  hypernatremia  hypokalemia  
and  asking  about  cell  responsible  for  that?  
a)          Reticularis  
b)          Glamerulosa  
ANSWER:  B  
REFERENCES:  https://en.wikipedia.org/wiki/Mineralocorticoid  
   
   
7.          Patient  complain  of  abdominal  pain  I  think  with  diarrhea  ,  also  his  wife  noticed  he  had  
SOB  and  tightness.  Doctor  order  5-­‐hydroxyindoleacetic  acid  in  urine  Which  cell  
responsible?  
A.chromaffin  cell  
B.Enterocell  
C.Lympho  cell  
D.Goblet  cell  
ANSWER:  A.  [carcinoid  syndrome  ]  
 5-­‐HIAA  is  the  main  metabolite  of  serotonin  and  is  frequently  produced  by  carcinoid  tumors  of  the  
enterochromaffin  cells  of  the  small  intestine.  
REFERENCES:  https://en.wikipedia.org/wiki/5-­‐Hydroxyindoleacetic_acid  
   
8.          UTI  case  with  gram  negative  indole  positive:  
A)          E.  coli  
 
 
 
 
9.  female  on  anti  dyslipidemia  drug  present  with  flushing  what  is  the  drug  :  
A.  Niacin  
B.  Atorvastatin  
Answer:  A  
References:  https://www.healthline.com/nutrition/niacin-­‐flush#section2  
   
   
10.  what  is  the  MOA  of  glipizide  ?  
A.  Increase  insulin  secretions  from  pancreas  
   
Answer:  
Sulfonylureas  likely  bind  to  ATP-­‐sensitive  potassium-­‐channel  receptors  on  the  pancreatic  cell  
surface,  reducing  potassium  conductance  and  causing  depolarization  of  the  membrane.  
Depolarization  stimulates  calcium  ion  influx  through  voltage-­‐sensitive  calcium  channels,  raising  
intracellular  concentrations  of  calcium  ions,  which  induces  the  secretion,  or  exocytosis,  of  insulin.  
References:  https://www.drugbank.ca/drugs/DB01067.  
   
11.when  the  body  exposed  to  ionized  radiation  what  the  effect  of  radiation  on  DNA?  
a)          break  DNA  strains  
b)          affecting  DNA  pyrimidine  
 Answer:  
   
   
 13.  which  of  the  following  is  synthesized  in  nucleolus?  
a)          rRNA  
b)          mRNA  
c)          tRNA  
Answer:  rRNA  
References:  https://www.ncbi.nlm.nih.gov/pubmed/18712681  
   
14.  what’s  the  duration  of  Nitrofurantoin  regimen  in  recurrent  UTI?  
Answer:    If  unrelated  to  sexual  intercourse,  consider  a  six-­‐month  course  of  low-­‐dose  nitrofurantoin.  
References:  https://patient.info/doctor/recurrent-­‐urinary-­‐tract-­‐infection  
   
15.  which  diuretic  can  cause  renal  stone?  
   
   

   
 
   
16.lip  swelling  for  3  years  deficiency  in  which  enzyme?  
a)          C1  esterase  inhibitor  
b)          Acute  urticaria.  
c)          Ascher  syndrome.  
   
Answer:    C1  esterase  inhibitor  deficiency.  
References:  https://emedicine.medscape.com/article/1048887-­‐differential#1  
   
17.old  man  with  abnormal  hand  and  head  movement  he  toke  drug  1  day  before  what  is  the  drug:  
Answer:  Metoclopramide  
References:  https://www.drugs.com/metoclopramide.html  
   
18.pt  with  neck  rigidity  and  abdominal  muscle  regidity  (clear  extraparamidal  
manifestations),  which  drug  can  cause  these  symptoms?  
   
Answer:  Metoclopramide;  extrapyramidal  syndrome  (EPS)/acute  dystonic  reactions  (at  least  25%).  
References:  https://www.drugs.com/sfx/metoclopramide-­‐side-­‐effects.html  
   
19.  which  drug  if  you  stopped  abruptly  it  will  cause  hypertensive  crisis?  
Answer:  Sympatholytic  antihypertensive  are  commonly  known  to  cause  rebound  HTN;  beta  blockers,  
alpha  blockers,  Clonidine,  labetalol.  
References:  http://www.ld99.com/reference/old/text/2878909-­‐814.html  
   
20.Which  cell  produce  melanocyte  releasing  hormone??  
Answer:  Melanocyte  stimulating  hormones  is  secreted  from  the  pars  intermedia  from  the  
melanotrophs  cells  which’s  a  type  of  chromophobes  cells.  
   
   
22.which  one  of  the  following  is  found  in  the  nucleus?  
 mRNA  in  nucleus  
 
23.Which  of  the  following  made  in    nucleolus  ?  
 rRNA  in  Nulcleolus    
 
#  for  Q22-­‐23;  no  options  were  provided  but  check  the  table  for  
relevant  
Information  related  two  both  structures.  
   
   
   
   
   
Q.  bond  between  r-­‐RNA  &  m-­‐RNA  ?  
   
Q.  what  is  the  bond  between  mRNA  and  tRNA?  
 Hydrogen  bond  
   
24-­‐what  is  the  longest  phase  in  the  cellular  division?  
a)          Anaphase  
b)          Prophase  
c)          Interphase.  
Answer:  Interphase.The  G1  phase,  S  phase,  and  G2  phase  are  
collectively  known  as  interphase.  Cells  spend  90  to  95  percent  of  the  
time  in  interphase,  where  DNA  is  synthesized  and  the  cell  doubles  in  
mass  before  mitosis  begins.  
Reference:  https://www.thoughtco.com/understanding-­‐the-­‐cell-­‐
cycle-­‐373391  
   
25-­‐source  of  energy  that  transfer  molecules  in  and  out  nucleus?  
a)          GTP  in  cytoplasm  
b)          ATP  cytoplasmic.  
c)          GTP  nuclear.  
   
Answer:  ATP  Cytoplasmic      Molecule  movement  requires  energy  in  order  to  move  solute  against  its  
electrochemical  gradient.  There  are  two  ways  in  which  a  flow  of  energy  can  be  coupled  to  
transporters:  Primary  active  transport  requires  energy  is  provided  by  cytoplasmic  atpase  and  
Sodium,  potassium—atlases  (Na,  K—atpase)  is  present  in  plasma  membranes  
References:  http://www.biology-­‐online.org/9/3_movement_molecules.htm  
   
26.on  OR  the  doctor  cut  vagus  nerve  by  mistake  while  doing  NISSIEN  fundoplication  
operation  for  ttt  of  chronic  GERD,  what  organ  will  be  affected  from  cutting  the  nerve?  
A-­‐esophagus,  
B-­‐fundus,  
C-­‐urinary  bladder.  
D-­‐colon.  
Answer:  A.  
   
27.Postroir  vagal  trunk  supplies?  
 Answer:  posterior  vagal  trunk  are  branches  of  the  posterior  vagal  trunk  which  supply  the  stomach.  
References:  https://en.wikipedia.org/wiki/Posterior_gastric_branches_of_posterior_vagal_trunk  
   
28.what  is  the  organ  affected  if  you  ligate  anterior  iliac  artery?  
a)          ovary  
b)          intestine,  
c)          bladder.  
Answer:  bladder  necrosis;    It  has  been  reported  that  there  exist  some  complications  after  IIAL  such  
as  urinary  bladder  necrosis,  peripheral  nerve  ischaemia  or  gluteal  compartment  syndrome.  
References:  https://www.ncbi.nlm.nih.gov/pmc/articles/PMC4105656/  
   
29.  Which  OCP  causes  hyperkalemia?                                                                        
a)          Estradiol-­‐levonorgestrel                                                                                                  
b)          Estradiol-­‐drosperirenone  
   
Answer:  Estradiol-­‐drosperirenone  ;  is  an  oral  contraceptive  (OC)  that  possesses  unique  
antimineralocorticoid  activity.  
References:  https://www.ncbi.nlm.nih.gov/pmc/articles/PMC3265420/  
   
   
31.difference  between  Hb  and  HbA1C  ?  
Glycosylation  of  something  
   
32.which  type  of  cells  will  be  high  in  viral  infection?  
Lymphocyte  
 

 
   
   
   
   
33.Which  muscle  would  be  completely  paralyzed  by  obturator  nerve  injury?  
A.          Gluteus  maximus  
B.          adductor  magnus  
C.          adductor  longus  
   
Answer:  adductor  longus.;  obturator  nerve  motor  supply  is  gracilis,  adductor  longus  and  brevis.  
References:  https://www.gpnotebook.co.uk/simplepage.cfm?ID=x20120114173923202976  
   
36.action  of  metformin  on  muscles?  
a)          Decrease  glucose  uptake,  
b)          increase  glocuneogenesis,  
c)          increase  release  of  glucose,  
d)          increase  acid  oxidation  
   
Answer:  increase  acid  oxidation  
;  the  MOA  of  metoformin  is  not  fully  understood  yet  but  in  vivo  studies  have  showed  that  
Extracellular  accumulation  of  the  drug  in  the  muscles  is  responsible  for  its  hypoglycaemic  affect.  
suppresses  lipid  accumulation  in  skeletal  muscle  by  promoting  fatty  acid  oxidation.  
References:  
 https://www.ncbi.nlm.nih.gov/pubmed/7988785.  
https://en.wikipedia.org/wiki/Metformin#Mechanism_of_action.  
https://www.ncbi.nlm.nih.gov/pubmed/25016691.  
   
   
40.Patient  has  hemorrhage,  baroreceptor  activated  which  lead  to  increase  heart  rate,  what  
of  those  will  be  decrease?  
a)          Ventricular  contractility  
b)          Venous  capacity  
c)          Coronary  blood  flow  
Answer:  Venous  capacity.  
References:  
https://www.google.com.sa/url?sa=t&rct=j&q=&esrc=s&source=web&cd=1&cad=rja&uact=8&ved=
0ahUKEwi09cfWhJvXAhVCExoKHUxJB-­‐
MQFggtMAA&url=http%3A%2F%2Fmediwikis.com%2Fwiki%2Findex.php%2FBaroreceptor_Reflex_a
nd_Response_to_Haemorrhage&usg=AOvVaw0Rxp86Y_cyoZsJzQjV2CTs  
   
41.  RTA  victim,  hypotensive,  tachycardic,  tachypnic,  activation  of  baroreceptors  will  
result  in  which  of  the  following?  
   
 

 
   
References  :http//:advan.physiology.org/content/40/3/283  
   
42.How  hyperglycaemia  affect  wound  healing?  
a.          Decrease  immune  system.  
b.          Dec  phagocytosis.  
c.          Stimulates  bacteria  growth.  
Answer:  C.  
References:  http://www.ncbi.nlm.nih.gov/pubmed/16006275  
   
43.Where  can  you  palpate  dorsalis  pedis  artery?  
Answer:  lateral  to  the  extensor  hallucis  
longus.  
   

   
   
   
45.S&S  of  anxiety  and  ask  about  deficiency?  
The  answer  was  Serotonin.      
   
 

 
   
   
46.Non-­‐steroidal  and  Anti-­‐inflammatory?  
 Answer:  Ibuprofen  
   
47.pudendal  nerve  block  which  area  is  not  affected?  
Answer:  Pudendal  nerve  block:  blocks  sensory  and  motor  innervation  to  the  Clitoris  “  dorsal  nerve”,  
skin  of  perineum  [labia  major/minora  and  vestibule]  “  perineal  branch”,  External  anal  sphincter  and  
perianal  skin  “  via  inf.  Hemorrhoidal  N.”  
References:  https://emedicine.medscape.com/article/83078-­‐overview#a1  
   
49.which  nerve  is  responsible  for  sensation  (not  taste)  of  anterior  two  
thirds  of  the  tongue?  
a)          5  
b)          7  
c)          9  
d)          12  
Answer:  Trigeminal  nerve.  
References:  
   
 
50.Loss  of  anterior  2/3  tongue  taste?  
Answer:  Facial  nerve  
   
51.Loss  of  posterior  1/3  tongue  sensation  what  is  the  nerve?  
Answer:  Glossopharyngeal  
   
53.disease  of  spine  got  anterior  chest  abscess  which  nerve  carry  the  
infection:  
A-­‐  lateral  cutaneus  
B-­‐  anterior  cutaneus  
C-­‐  posterior  cutaneus  
D-­‐  diaphragmatic  
Answer:  Anterior  cutaneous.  Tuberculous  exudates  from  the  spine  generally  follow  the  course  of  
anterior  cutaneous  or  lateral  cutaneous  branch  of  the  intercostal  nerve  to  produce  cold  abscess  in  
the  parasternal  line  or  midaxillary  line,  respectively.  
References:  http://www.atmph.org/article.asp?issn=1755-­‐
6783;year=2012;volume=5;issue=2;spage=142;epage=144;aulast=Das  
   
54.in  patient  post  mastoctomy  they  do  for  her  reconstrtion  from  the  rectus  muscle  what  is  
the  vesles  that  they  should  take  with  it?  
a)          1.superior  epegastric  artery  
b)          2.inferior  epigastric  artery  
 Answer:    Inferior  epigastric  artery.  
References:  https://www.ncbi.nlm.nih.gov/pmc/articles/PMC3015273/  
   
55.which  of  the  following  when  it  doesn't  undergo  Heterogeneity  it  causes  gastric  cancer?  
Answer:  E  cadherine      gene.;  loss  of  function  of  E-­‐cadherin  (CDH1)  has  been  linked  with  diffuse  
gastric  cancer  susceptibility.  
References:  https://www.ncbi.nlm.nih.gov/pubmed/14630673                                
   
56.which  of  the  following  has  a  Side  effect  of  causing  angioedema?  
 
   
 

 
   
   
57.  which  is  responsible  for  apoptosis?  
Answer:  P53  pathway.  
 

 
   
58.Protein  enter  in  proteasome  by  what  ?  
A-­‐  N  terminal  single  
B-­‐  N  terminal  double  
C-­‐  S  terminal  single  
D-­‐  S  terminal  double  or  C  
   
Answer:  N  terminals  single.  
   
59.How  does  the  protein  enter  the  peroxisomes?  
a)          Folded  with  the  help  of  C-­‐terminal  
b)          Unfolded  with  the  help  of  T-­‐terminal  
c)          Folded  with  the  help  of  T-­‐terminal  
d)          Unfolded  with  the  help  of  C-­‐terminal  
Answer:  D.  
   
60.which  decrease  the  risk  of  some  cancers?  
a)          Vit  D  
b)          Fibers                                          
c)          Salt  
Answer:  Vitamin  D.  
   
61.scenario  of  pt.  who  his  freind  pushed  him  from  behind,  what  is  the  ligament  that  prevent  
overextension  of  hip?  
Answer;  Iliofemoral  ligament  
   

   
 62.How  ectopic  pregnancy  occurs  at  the  cellular  level?  
A.    early  Disappearance  of  zona  pellucida.  
B.    Fertilization  at  ampulla  tube.  
C.    Persistence  of  Zona  pellucida.  
D.    Fast  division  of  blastomere  
Answer:  A.  
References:  https://www.ncbi.nlm.nih.gov/pmc/articles/PMC3276879/  
   
63.which  Muscle  is  passing  through  the  lesser  sciatic  foramen?  
Answer:  Obturator  internus  muscle.  
   

   
   
   
64.CSF  is  circulating  in  which  space?  
Answer:  Subarachnoid  space.  
   
65.which  vitamin  help  in  absorbtion  of  non  heme  iron?  
Answer:  Vitamin  C.  
   
65.pt  with  vomiting  and  hyperventilation.  what  do  expect  to  find  in  him?  
Answer:  vomiting  causes  metabolic  alkalosis,  hyperventilation  causes  respiratory  alkalosis.4  
   
66.  What  is  usually  given  with  analgesics  to  reduce  side  effects?  
Answer:  Metoclopramide  
   
67.What  moves  the  molecules  in  cell  in  and  out  of  the  nucleous?  
a.  ATP  cytoplasmic  
b.  ATP  nuclear  
c.  GTP  cytoplasmic  
d.  GTP  nuclear  
Answer:  A  
   
68.about  action  or  Side  effects  of  atropine?  
Answer:  These  include  palpitation,  dilated  pupils,  difficulty  in  swallowing,  hot  dry  skin,  thirst,  
dizziness,  restlessness,  tremor,  fatigue  and  ataxia.  
References:    https://www.rxlist.com/atropine-­‐drug.htm                                                                                                                
   
69.  what's  the  mechanism  of  insulin  resistance  in  obesity?  
Answer:    Mitochondrial  Over  activation.  

   
References:  https://www.ncbi.nlm.nih.gov/pmc/articles/PMC3936017/  
   
70.Insulin  resistance  in  type  2  DM,  how  does  it  occur?  
Answer:  Evidence  is  provided  showing  that  increased  plasma  FFA  levels  produce  insulin  resistance  
dosage  dependently,  acutely,  and  chronically.  
References:  https://www.ncbi.nlm.nih.gov/pubmed/11727400  
   
   
71.what  forms  the  ankle  joint?  
Answer:    Tibia,  Fibula  and  Talus  bones.    

   
   
72.  the  blood  in  the  superficial  palmar  arch  is  low.  the  problem  in  which  artery?  
Answer:  Ulnar  Artery.  
   
73.pt  can't  move  her  hand.  where  is  the  problem?  
A)          ant.  interossi  
B)          or  post.  interossi  
C)          or  theaner  
   
Answer:  the  patient  is  presenting  with  Radial  nerve  injury  and  the  post.interosseous  nerve  supplies  
the  post  compartment  of  the  forearm”  extension”  
   
74.diabetic  pt  with  pain  when  she  tries  to  stretch  2nd  and  
3rd  finger  (and  i  think  she  
can't  move  her  hand).  there  was  pic  in  the  exam.  what's  the  
Daignosis?  
A)          diabetic  dactylitis  
B)          dupturen  contracture  
   
Answer:  Diabetic  Dactylitis.  
 
75.pt  had  trauma  then  can't  flex  the  DIP.  where  is  the  lesion?  
ANSWER:    Flexor  digitorm  profundus  injury.  “  in  PIP  
immobility  the  FD  superficilias  is  injuried”  
   
76.Action  of  Anterior  compartment  of  the  forearm  muscle  ?  
Answer:  In  general,  muscles  in  the  anterior  compartment  of  the  forearm  perform  flexion  at  the  wrist  
and  fingers,  and  pronation.  
References:  http://teachmeanatomy.info/upper-­‐limb/muscles/anterior-­‐forearm/  
   
77.  pic  of  humerus  bone  fracture  with  weakness  in  wrist  extension,  which  nerve  is  injured?  
Radial  nerve  injury.  
   

   
   
   
   
78.pt  can't  extend  wrist  and  finger  ,  nerve  injured?  
Answer:  Radial  nerve.  
   
79.Patientwith  trauma,  femoral/hip  fracture.  Left  leg  short  and  laterally  rotated.  What  muscle  is  
responsible  for  lateral  rotation:  
A-­‐gluteus  maximus  
B-­‐rectus  femorus  
 C-­‐gracilus  
D-­‐adductor  magnus  
Answer:  a  The  lateral  rotators  are:  the  superior  gemellus,  inferior  gemellus,  obturator  externus,  
obturator  internus,  quadratus  femoris,  gluteus  maximus  and  the  piriformis.  References:  
Http://radiopaedia.org/articles/gluteal-­‐muscles  
   
   
80.pt  has  BP  of  160  over  something.  what's  the  cause?  
A)          change  in  paroreceptor  sensitivity  
B)          increase  peripheral  vascular  resistance  
Answer:  B.  
 
 
 
   
81.pt  in  shock.  what  will  happen?  
a)          decrease  coronary  perfusion  
b)          decrease  in  venous  capacitance  
 Answer:  insufficient  information.  

   
   
   
83.  pt  with  erectile  dysfunction  and  taking  a  medication  for  that.  which  of  the  following  
is  contracindicated  for  this  patient?  
Answer:  nitrate  
   
84.pt  with  hyperthyroidism  came  with  red  eye.  which  medication  can  cause  this?  
Answer:  Propylthiouracil.  
References:  https://www.drugs.com/sfx/propylthiouracil-­‐side-­‐effects.html  
   
85.Blood  supply  to    AV    and    SA    node  in    the  heart  ?  
Answer:  Right  coronary  artery,  
   
   

 86.Hinge  joint?  
   
   
   
 
87.Most  common  SE  of  atropine?  
Most  common  side  effects  from  Rxlist.com.  
●   dry  mouth,  
●   blurred  vision,  
●   sensitivity  to  light,  
●   lack  of  sweating,  
●   dizziness,  
●   nausea,  
●   loss  of  balance,  
●   hypersensitivity  reactions  (such  as  skin  rash),  and  
●   rapid  heartbeat  (tachycardia).  
   
88.  Inhaled  antiviral  works  by?  
   
   
   
   
   
   
89.Bacteria  sexual  behaviour?  
Answer:  Bacterial  conjugation.  
   
90.Ribavirin  SE?  
A)  anemia  
B)  renal  damage  
C)  Hepatic  damage.  
   
Answer:  Haemolytic  anaemia;  haemolytic  anemia,  decreased  hemoglobin,  insomnia,  dyspnea,  lack  
of  concentration,  emotional  lability,  and  irritability.    
   
91.  Side  effects  of  Anti-­‐TB.  
-­‐  Anti  TB,  causing  seizure:  isoniazid  
-­‐  Anti  TB,  causing  parasthesia  or  nerve  problem:  isoniazid  Rx:  pyrodixine.  
 -­‐  Anti  TB,  causing  visual  problem:  ethambutol  
-­‐  Anti  TB,  causing  vertigo  or  ear  problem:  streptomycin  
-­‐  Anti  TB,  causing  red  urine:  rifampicin  
   
92.  Thalidomide  SE  in  pregnancy?  
Answer:  limb  defect.  
References:    http://onlinelibrary.wiley.com/doi/10.1002/bies.200900103/pdf  
   
93.Anti  thyroid  causing  pancytopenia  (Agrnanulocytosis)?  
Answer:  Propylthiouracil  and  methimazole  both  can  produce  thrombocytopenia.  
References:  https://www.ncbi.nlm.nih.gov/pubmed/22049174  
   
94.Anti  thyroid  used  in  pregnancy?  
Answer:  Both  methimazole  (MMI)  and  propylthiouracil  (PTU)  may  be  used  during  pregnancy;  
however,  PTU  is  preferred  in  the  first  trimester  and  should  be  replaced  by  MMI  after  this  trimester.  
References:  https://www.ncbi.nlm.nih.gov/pubmed/21389085  
   
95.Drug  contraindicated  in  glaucoma?  
Antihistamines.  H2  receptor  agonists(cimetidine),  adrenergic  agonists.  Epinephrine,  ipratropium  
bromide.  
References:  https://www.reviewofoptometry.com/ce/meds-­‐that-­‐dont-­‐mix-­‐with-­‐glaucoma-­‐patients  
   
96.  Drug  for  HTN  reduces  heart  rate  and  peripheral  resistance?  
Answer:  Carvediolol;      is  a  beta-­‐adrenergic  receptor  blocking  ability  decreases  the  heart  rate,  
myocardial  contractility,  and  myocardial  oxygen  demand.  Carvedilol  also  decreases  systemic  vascular  
resistance  via  its  alpha-­‐adrenergic  receptor  blocking  properties.  
   
97.Drug  for  HTN  reduced  preload  and  cause  vasodilatation?  
Answer:  ACE’I,  
References:  http://cvpharmacology.com/vasodilator/ACE  
   
98.drug  associated  with  Hyperpigmentation?  
   
 Answer:  amiodarone.  
   
 

 
 
99.  Orlistat  mechanism  of  action?  
 Answer:  Orlistat  works  by  inhibiting  gastric  and  pancreatic  lipases,  the  enzymes  that  break  down  
triglycerides  in  the  intestine.  When  lipase  activity  is  blocked,  triglycerides  from  the  diet  are  not  
hydrolyzed  into  absorbable  free  fatty  acids,  and  instead  are  excreted  unchanged.  Recently  it  was  
found  to  inhibit  the  thioesterase  domain  of  fatty  acid  synthase.  
References:  https://en.wikipedia.org/wiki/Orlistat#Mechanism_of_action  
   
   
100.Aspirin  SE?  
 Answers:  frequently  reported  side  effects  include:  dyspepsia,  epigastric  distress,  heartburn,  and  
nausea.  
References:  https://www.drugs.com/sfx/aspirin-­‐side-­‐effects.html  
   
   
KHALID  ALOMAR  
   
-­‐  Aspirin  in  kids?  (fulminant  hepatitis  and  cerebral  edema)?  
Answer:  Reye  syndrome  
   
-­‐  Post  menopausal  hormonal  therapy  SE?  
Answer:  breast  cancer  
   
-­‐  Bisphosphonate  inhibits  osteoclast  activity?  
Answer:  alendronate  
Reference:  https://en.wikipedia.org/wiki/Alendronic_acid  
   
-­‐  Bisphosphonate  works  by  ADP?  
Answer:  clodronate,  etidronate  and  tiludronate  
Reference:    https://en.wikipedia.org/wiki/Bisphosphonate#Mechanism_of_action  
   
-­‐  Med  for  osteoprosis  causes  heartburn?  
Answer:  risedronate  
Reference:    http://reference.medscape.com/drug/actonel-­‐risedronate-­‐342835#4  
   
-­‐Drug  for  asthma  decreases  secretion  more  than  bronchodilatation?  
 Answer:  ipratropium  bromide  
 Reference:  https://en.wikipedia.org/wiki/Ipratropium_bromide  
   
-­‐  Hg  F  inducer?  
a-­‐  hydroxyurea,  
b-­‐  Na  butyrate,  
c-­‐  decitibine  
d-­‐  5-­‐azacytidine  
 Answer:    A  
   
drug  that  cause  vertical  and  horizontal  nystagmus?  
Answer:  phencyclidine  
Reference:    https://www.uptodate.com/contents/phencyclidine-­‐pcp-­‐intoxication-­‐inadults?  
source=search_result&search=Phencyclidine&selectedTitle=1~62  
   
Perinoeal  artery?  branch  from  post.  Tibial  artery,  a  branch  from  popliteal  artery  
 Answer:  ??  
Perineal  artery  is  a  branch  from  internal  pudendal  artery  which  is  a  branch  from  internal  iliac  artery.  
   
-­‐    facial  veins  (via  the  superior  and  inferior  ophthalmic  veins)  as  well  as  the  
sphenoid  and  middle  cerebral  veins.  They,  in  turn,  empty  into  the  inferior  petrosal  
sinuses,  then  into  the  internal  jugular  veins  and  the  sigmoid  sinuses  via  the  superior  
petrosal  sinuses  
 Answer:  ??  
   
-­‐  Radial  pulse:  
Answer:  lateral  to  flexor  carpi  radialis  
   
-­‐  Fingers  adduction:  
Answer:  deep  ulner  nerve  
   
-­‐  Fibger  abduction:  
Answer:  deep  radial  nerve  
   
-­‐  Fingers  adductors:  
Answer:  palmar  interosseous  
   
-­‐  finger  abductors:  
Answer:  dorsal  interosseous  
   
-­‐  Blood  supply  of  hand:  
Answer:  superficial  arch;  ulnar  mainly  +  radial.  
                          Deep  arch;  radial  mainly  +  ulnar  
   
-­‐  Pain  during  hand  typing:  decrease  blood  in  ulnar  artery  "carpal  tunnel  syndrome;  
median  Nerve"  
Answer:  median  nerve  
   
-­‐  Nail  fold  area  supplied  by:  
answer:  median  N  
   
-­‐  Blood  of  distal  palmar  aspect:  
Answer:  proper  palmar  digital  artery,  from  common  palmar  
digital  artery.  A  
   
-­‐  Pronator  teres  syndrome:  
Answer:  median  N  
   
-­‐  Can't  flex  distal  phalanx:  
Answer:  rupture  profundus  tendon  
   
-­‐  Pain  with  rising  arm  and  numbness  in  ulnar  distribution:  
Answer:  thoracic  outlet  syndrome  
   
-­‐  Falling  on  outstretched  hand:  
Answer:  colle's  fracture  
   
-­‐  Hip  and  knee  flexion:  
Answer:  sartorius  muscle  
   
-­‐  Gluteal  artery  branch  from  
Answer:  internal  iliac  
   
-­‐  Obturator  nerve:  
Answer:  Adductor  longus  
supplies  all  adductors;  adductor  longus,  brevis,  gracilis  ..  etc  +  
sensation  in  medial  aspect  of  thigh  
   
-­‐  Internal  iliac  artery:  
Answer:  Bladder?.  
gives  the  inferior  and  superior  gluteal  artery  to  gluteus  
muscles,  and  obturator  artery  to  adductor  groups.  Otherwise,  other  muscles  in  the  lower  
limb  from  the  femoral  artery  
   
-­‐  Strongest  hip  ligament:  
Answer:  iliofemoral  
   
-­‐  Patient  after  MVA,  walking  bending  to  unaffected  side  (positive  trendelburg  test):  
Answer:  injury  to  sup.  Gluteal  nerve  which  supplies  gluteus  medius  and  minimums  
   
-­‐  Post.  Compartment  of  leg  BV:  
Answer:  posterior  tibial  artery  
   
-­‐  Ant.  Compartment  of  leg  BV:  
Answer:  anterior  tibial  artery.  
   
-­‐  Posterior  leg  compartment:  
Answer:  tibial  N  
   
-­‐  Anterior  leg  compartment:  
Answer:  common  peroneal  which  gives  deep  perpneal  
   
-­‐  Foot  Dorsal  flexion:  
Answer:  tibial  muscle  
   
-­‐  Foot  Plantar  flexion  "standing  on  toe":  
Answer:  plantaris  
   
-­‐  Patient  can't  planter  flex  his  foot  after  injury:  
Answer:  calcaneal  tendon  rupture  "achilles  
tendon"  
   
-­‐  Football  player  cant'  extend  big  toe:  
Answwer:  deep  muscle  under  foot  (turf  toe)  
   
-­‐  Fibular  head  fracture:  
Answer:  common  peroneal  nerve;  drop  of  foot  
   
-­‐  First  thing  to  be  injured  in  palmar  foot:  
Answer:  abductor  hallucis  longus  M  
   
-­‐  Flat  foot:  
problem  with  tibialis  posterior  tendon  
   
-­‐  Tongue  muscle:  originates  from  occipital  
Answer:  
Mandible,  
hyoid  bone,  
styloid  process  
of  temporal  
bone.  
   
 
 
   
 

 
 
 
 
-­‐  3rd  layer  of  skull:  
Answer:  aponeurosis  
Reference:  Medscape.  
   
-­‐  Compression  on  maxillary  artery:  
Answer:  sphenopalatine  artery  affected  
   
-­‐  Thyroid  fascia:  
Answer:  pretrachial  fascia,  part  from  deep  cervical  fascia  
   
-­‐  Accessory  nerve:  
Answer:  in  post.  Triangle  
   
-­‐  LP  puncture:  
Answer:    injury  to  ligamentum  flavum  
   
-­‐  triangle  between  hyoid  bone  and  two  ant.  Belly  of  digastric:  
Answer:  submental  
   
-­‐  External  laryngeal  nerve:  
supply  cricothyroid  muscle  -­‐-­‐&gt;  adduct  cord  to  phone  the  
voice  
Answer:  supply  cricothyroid  muscle  
   
-­‐  Loss  of  smell  sensation;  
Answer:  temporal  lobe  
   
-­‐  Facial  N  injury  
Answer:  proximal  to  greater  petrosal  nerve  
(Q  from  SMLE13:  Loss  of  taste  sensation  of  the  anterior  2/3  of  the  one  side  of  the  tongue  and  
decreased  hearing  on  the  ear  on  the  same  side)  
   
-­‐  Loss  of  sensation  in:  lower  eyelid,  maxilla,  zygomatic  areas  -­‐-­‐&gt;  maxillary  N  
injury  which  give  inferior  orbital  N  
Answer:  inferior  orbital  N  
   
-­‐  Foramen  between  3rd  and  4th  ventricle:  
Answer:  aqueduct  of  sylvius  
   
-­‐  Upper  and  lower  lips  LN  drainage:  
submandibular  
Answer:  submental  and  submandibular  

   
 
-­‐  LN  of  middle  of  face:  
Answer:  submandibular  
   
 
 
 
 
 
 
 
 
-­‐  LN  of  lateral  side,  includes  lateral  side  of  eyelid  and  eyebrow:  
Answer:  parotid  gland  
   
-­‐  Tip  of  the  tongue  and  center  of  the  lips:  
Answer:  submental  
Reference:  http://teachmeanatomy.info/neck/vessels/lymphatics/  
   
-­‐  Follicular  cell  of  ovary:  
Answer:  cortical  cord  
References:  Reference:  Langman’s  Medical  Embryology  
https://en.wikipedia.org/wiki/Sex_cords  
   
-­‐  Phrenic  nerve  injury:  
increase  heart  rate  
Answer:  phrenic  nerve  injury  causes  hiccups  problem  and  difficulty  breathing.  
   
-­‐  Cardiac  notch:  
Answer:  4th  rib  
   
-­‐  Injury  in  forth  intercostal  space:  
Answer:  horizontal  fissure  
   
-­‐  Thoracocentesis:  
between  posterior  axillary  line  and  midline  ;  between  7  and  9  intercostal  
Answer:  between  7th  and  9th  intercostal  space  and  posterior  and  mid  axillary  line.  
Reference:  Medscape.  
   
-­‐  Needle  thoracostomy:  
Answer:  2nd  intercostal,  mid  axillary  line  
   
-­‐  Chest  tube:  
Answer:  4th  intercostal,  anterior  axillary  line  
   
-­‐  Adrenalectomy:  
Answer:  right  side  will  injure  IVC,  left  side  will  injure  the  pancreas  
   
-­‐  Internal  iliac  artery  injury:  
Answer:  bladder  is  affected  
   
-­‐  Best  x-­‐ray  for  rib:  
Answer:  oblique  view  
   
-­‐  Meckel's  diverticulum:  
Answer:  in  lower  ilium  
   
-­‐  Crohn:  
Answer:  distal  ilium  
   
-­‐  UC:  
Answer:  rectum  
   
-­‐  Celiac:  
Answer:  proximal  small  intestine  
   
-­‐  Appendicitis  histopathology:  
Answer:  neutrophils  in  muscularis  layer  
   
-­‐  Tenia  coli:  
Answer:  in  muscularis  externa  layer  
   
-­‐  H.  Pylori:  
Answer:  stimulate  gastrin  cell  
   
-­‐  Intrinsic  factor  for  vit  B12:  
Answer:  from  parietal  cell  
   
-­‐  in  pt  with  congested  neck  vein  and  tumor  (SVC  syndrome):  
in  middle  mediastinum  
Answer:  could  be  superior  or  middle  mediastinum.  
   
-­‐  Sugar  pentose:  
Answer:  arabinose  
   
-­‐  Melanin  pigment:  
 corticotrope  
Answer:  Corticotrophs  mainly  secrete  melanocyte-­‐stimulating  hormone.  
Melanocytes  secrete  melanin.  
   
-­‐  Rhesus  autoimmunity  in  pregnancy:  
Answer:  hypersensitivity  type  2  
   
-­‐  phosphoenolpyruvate  to  pyruvate  and  generation  of  ATP:  
Answer:    by  pyruvate  kinase  
   
-­‐  From  pyruvate  to  lactic  acid:  
 Answer:  lactic  dehydrogenase  
   
-­‐  Accumulation  of  lactic  acid:  
Answer:  pyruvate  carboxylase  deficiency  
   
-­‐  Phenylketonuria:  
Answer:  phenylalanine  hydroxylase  deficiency  
   
-­‐  Strongest  layer  of  vein:  
answer:  adventitia  
   
-­‐  Juxtaglomerular  apparatus  origin:  
Answer:  macula  densa  cell.  
   
-­‐  Alveoli  part  contacting  blood:  
Answer:  saccular  phase.  
   
-­‐  Phase  of  alvuloendothelium  part  get  matured:  
Answer:  canalicular  phase  
   
-­‐Amyotrophic  lateral  sclerosis:  
Answer:  anterior  Horn  
   
-­‐  Minimal  change  glomerulonephritis:  
Answer:  Light  micro:  normal,  electron:  fusion  of  foot  
process  
   
-­‐  Hepatic  cell  secrete  extra  matrix  component:  
Answer:  stellate  cell  
   
-­‐  Ever  smoker:  
Answer:  100  sig  per  life  
   
-­‐  Biopsychological  therapy:  
Answer:  Holistic.  
   
-­‐  Wild  mushroom:  
Answer:  inhibits  RNA  polymerase  II  
   
-­‐  X  fragile  syndrome:  
Answer:  macrogonadism  
   
-­‐  Comes  with  turner:  
Answer:  hypothyroidism  
   
-­‐  Vit  C  def  affect  which  stage  of  wound  healing:  
Answer:  collagen  synthesis  or  proliferative  stage.  
Reference:  http://www.surgerysupplements.com/the-­‐role-­‐of-­‐vitamin-­‐c-­‐in-­‐wound-­‐healing/  
   
-­‐  of  embriology  i  cant  remember  (failure  of  spiral  )  ??  
Answer:  Transposition  of  great  vessels  
   
Which  of  the  following  results  from  malrotation  of  septum,  truncus  arteriosus?  
Answer:  Transposition  of  great  vessels  
   
Protein  after  transfer  sequence  what  is  the  last  destination  ??  
Answer:  endoplasmic  reticulum  
   
an  athlete  came  with  xanthametaous  lesion  on  his  Achilles'  tendon,  otherwise  normal,  
Genetic  defect  on  which  of  the  following  
Answer:  LDL  receptors  ✔  
   
pt  heared  about  incretin  on  TV  and  wishes  to  know  more  about  its  action  
Answer:  increases  insulin  secretion  
   
a  pt  with  muscle  weakness,  apart  from  hypotension  and  bradycardia,  his  examination  was  
normal,  
His  lab  tests  (  high  K+,  low  NA,  low  Chloride,  high  urea)  
What  is  the  etiology  behind  his  presentation?  
a-­‐  hyponatremia  
b-­‐  hyperkalemia  
c-­‐  low  chloride  
d-­‐  uremia  
Answer:    b  
   
pt  after  a  surgery  in  the  posterior  triangle  of  neck,  developed  loss  of  sensation  on  
the  skin  of  lower  mandible  and  Lower  part  of  the  ear,  
Nerve  injured  is  
Answer:  Great  Auricular  nerve.  
   
clear  case  of  Carpal  tunnel  syndrome,  the  pt  is  a  typist  working  on  computer,  nerve  
affected?  
Answer:  Median  nerve✔  
   
During  a  surgery  in  the  diaphragm,  
Phrenic  nerve  is  dissected  at  which  level  
Answer:  anterior  to  sclaneous  anterior  muscle  
   
non  athlete  forcefully  planter  flexed  his  foot  and  heard  a  click  sound  on  his  leg  ,  
What  tendon  affected?  
Answer:  calceneal  tendon  
   
The  effect  of  anti  TB  drugs  on  the  eye  is  one  of  the  following:  
a-­‐          bacterial  conjunctivitis  
b-­‐          viral  conjunctivitis    
c-­‐          glaucoma    
d-­‐          uveitis  
Answer:  d  
   
an  RTA  victim  was  brought  to  hospital  with  multiple  injuries,  transferred  to  ICU.  
There,  he  developed  multi  organ  failure,  
The  phosphate  level  in  his  blood  is  raised,  what  is  the  cause  of  his  multi  organ  
failure?  
a-­‐  heart  
b-­‐  lungs  
c-­‐  kidneys  
d-­‐  liver  
Answer:  a  
   
the  transmission  of  maternal  antibodies  to  the  fetus  in  pregnancy  is  a  way  of  
a-­‐  active  artificial  Immunity  
b-­‐  passive  artificial  immunity  
c-­‐  passive  natural  immunity  
d-­‐  active  natural  immunity  
 Answer:  c  
   
clear  case  of  atopic  pt  e  allergic  rhinitis,  symptoms  and  signs  were  given,  then  asked  
about  the  cells  responsible  for  his  allergy  
a-­‐          monocyte  
b-­‐          macrophage  
c-­‐          mast  cells  
d-­‐          neutrophils  
Answer:  c  
   
lymphatic  drainage  of  teste  
Answer:  para  aortic  LN  
   
75.pt  had  trauma  then  can't  flex  the  DIP.  where  is  the  lesion?  
ANSWER:    Flexor  digitorm  profundus  injury.  “  in  PIP  immobility  the  FD  superficilias  is  injuried”  
   
76.Action  of  Anterior  compartment  of  the  forearm  muscle  ?  
Answer:  In  general,  muscles  in  the  anterior  compartment  of  the  forearm  perform  flexion  at  the  wrist  
and  fingers,  and  pronation.  
References:  http://teachmeanatomy.info/upper-­‐limb/muscles/anterior-­‐forearm/  
   
77.  pic  of  humerus  bone  fracture  with  weakness  in  wrist  extension,  which  nerve  is  injured?  
Radial  nerve  injury.  
   
Q.  A  woman  e  Ca  cervix,  e  mets  beyond  uterus  and  cervix,  
1st  group  of  LN  receiving  the  lymphatic  drainage  is:  
a-­‐          uterine  
b-­‐          external  iliac  
c-­‐          common  iliac  
d-­‐          para  aortic  
Answer:  a  
(para  uterine  -­‐>  obturator  -­‐>  internal  iliac  -­‐>  common  iliac  -­‐>  external  iliac  -­‐>  para  aortic)  
References:    https://www.uptodate.com/contents/invasive-­‐cervical-­‐cancer-­‐epidemiology-­‐risk-­‐
factors-­‐clinical-­‐manifestations-­‐and-­‐
diagnosis?source=see_link&sectionName=ROUTES%20OF%20SPREAD&anchor=H1087317#H108731
7  

   
 
-­‐   Flouroquinolones  mode  of  action:  
Answer:  DNA  gyrase  inhibitors  
   
Typical  scenario  of  Acute  appendicitis,  (periumbilical  pain  radiating  to  RIF,  pt  has  
aneroxia,  nausea  and  fever,  what  is  the  pathophysiology  ?  
Answer:  Appendiceal  obstruction  
   
Side  effect  of  morphine  is  
Answer:  nausea  and  vomiting  
   
What  is  the  most  common  part  of  urethra  could  be  injured  during  catheterization  ?  
a-­‐          Membranous  urethra  
b-­‐          Prostatic  urethra  
c-­‐          Penile  urethra  
Answer:  a  
   
Baby  ,  during  delivery  ,  there  was  stylomastoid  foramen  trauma  ,  what  features  will  be  
during  exams:  
a-­‐  loss  of  eye  close  
b-­‐  Loss  of  facial  sensation  
c-­‐  Loss  of  mastication  function  
 Answer:  a  

   
Q.  Which  of  the  following  integrates  glucose  and  fatty  acid  metabolism  ?  
a-­‐  pyruvate  
b-­‐  Citrate  
c-­‐  Carboxylase  
 Answer:  ??  acetyl-­‐CoA.  
References:  https://en.wikipedia.org/wiki/Acetyl-­‐CoA  
https://www.docsity.com/en/answers/biochemistry/which-­‐following-­‐metabolite-­‐grates-­‐glucose-­‐
fatty-­‐metabolism/193124/  
   
patient  with  CAD  ,  prescribe  for  her  drug  lower  lipid  .  Present  with  facial  flush  ,  
what  is  the  drug  ?  
a-­‐  statin  
b-­‐  Nicain  
c-­‐  Fibrate  
Answer:  b  
Reference:    https://en.wikipedia.org/wiki/Niacin  
   
pt  on  antithyroid  meds,  then  developed  fever.  What  does  he  use?  
Answer:  Methimazol  
   
How  to  exclude  fictitious  fever?  
a  -­‐Blood  culture  
b  -­‐Urine  analysis  
c  -­‐Heart  rate  
d  -­‐Rheumatoid  factor  
 Answer:  c  
   
pt  with  Carpopedal  spasm?  
Answer:  low  ca  
   
Good  healing?  
Answer:  epithelium  covering  
   
-­‐Metformin  MOA?  
Answer:    decrease  liver  glucose  production  
   
-­‐PTU  MOA?  
 Answer:  block  release  of  thyroid  hormones  
   
Drop  foot  &  loss  of  sensation  in  1st  &  2nd  digit.  which  nerve  is  responsible?  
Answer:  Deep  pernoneal  nerve  
   
pt  with  End  Stage  Liver  Disease  and  yeast.  Which  antifungal  you  should  use?  
a-­‐          fluconazole  
b-­‐          capsufungin  
 Answer:  b  
Reference:  http://www.medscape.com/viewarticle/775172_4  
   
Overlflow  incontinance  Rx  Moa?  
Answer:  activate  muscarinic  receptors  (muscarinic  agonist  )  
Reference:    https://en.wikipedia.org/wiki/Overflow_incontinence  
   
which  one  will  protect  aganst  dental  carries?  
Answer:  Fluride  supplmenets  
   
which  is  true  about  Parkinson  case?  
Answer:  low  dopamine  
   
which  is  true  about  case  of  Parkinson  
Answer:  problem  in  substantia  nigra  
   
   
how  Ocp  prevents  pregnancy?  
a-­‐          Relase  estrogen  that  prevent  ovulation  
c-­‐          relase  prolactin  that  prevent  ovulation  
d-­‐          Stop  spermatozoa  and  i  increase  thickness  of  cervical  mucosa  
Answer:  a  
Reference:  https://en.wikipedia.org/wiki/Combined_oral_contraceptive_pill  
   
Hormone  replacement  therapy?  
Answer:  increase  risk  of  endometrial  ca  
   
Pt  got  numbness  and  electrical  like  pain  of  the  left  foot  up  to  knee  ,  so  at  which  
level  is  the  disc?  
Answer:  L3-­‐L4  
   
   
Gout  meds  MOA:  
Answer:  
-­‐Allopurinol  +  febuxostat  =  xanthine  oxidase  inhibitor  
-­‐pegloticase  =  recom.  uricase  that  catalyzes  uric  acid  to  a  more  water  sol.  Matter  
-­‐probenecid:  inhibits  reabsorption  in  pct,  but  can  lead  to  uric  acid  stones  
-­‐colchicine  =  microtubule  polymerization  inhibitor  
   
MVA  injury  to  jugular  Forman  which  muscle  will  be  still  functioning  ?  
a-­‐sternoclecomastiod  
b-­‐sternoglossal  
c-­‐sternpphygenas  
d-­‐trapazius  
 Answer:  b  Styloglossus  
   
Q.  You  gave  a  patient  a  drug  and  ask  him  to  come  back  to  check  PT  and  INR  what  was  the  drug  ?  
a-­‐Asprin  
b-­‐Warfarin  
c-­‐enxoparin  
d-­‐UF  heparin  
Answer:  b  
   
MOA  of  emergency  contraceptives?  
a-­‐          delay  fertilization  
b-­‐          delay  implantation  
 Answer:  a  
Reference:  https://en.wikipedia.org/wiki/Emergency_contraception  
   
-­‐  Which  one  of  the  following  GIT  cell  secret  defensins:  
a-­‐paratial  cell  
b-­‐chief  cell  
c-­‐paneth  cell  
d-­‐plasma  cell  
Answer:  c  
Reference:    https://www.ncbi.nlm.nih.gov/pmc/articles/PMC4073591/  
   
type  of  wrist  joint?  
a-­‐PIVOT  
b-­‐HINGE  
Answer:    ellipsoid  (condyloid).  
   
BRUNNER's  gland  location?  
Answer:  submucosal  duodenum    
Reference:    https://en.wikipedia.org/wiki/Brunner%27s_glands  
   
anti  diabetic  medication  working  through  activation  enzyme  AMP  protein  kinase?  
A-­‐glipizide  (sulfonylurea)  
B-­‐tolbutamide  
C-­‐pioglitazone  
D-­‐biguanide  or  metformin  
Answer:  d  
Reference:  https://en.wikipedia.org/wiki/Metformin  
   
loss  of  sensation  of  posterior  &  anterior  aspect  of  the  hand  ;  which  never  is  affected?  
a-­‐  radial  
b-­‐  Medial  
c-­‐  Ulnar  
d-­‐  Axillary  
Answer:  d  
   
 Loss  sensation  of  the  little  &  ring  finger  which  nerve  is  affected?  
a-­‐  radial  
b-­‐  Medial  
c-­‐  Ulnar  
c-­‐          Axillary  
Answer:  c  
   
Q  about  OCP  and  side  effects  of  dry  skin,  muscle  spasm  ,  and  depression?  
a-­‐          hypothyroidism  
b-­‐          Hyperkalemia  
c-­‐          Hypocalcemia  
Answer:    a  
   
Q.  which  contraceptive  has  a  side  effect  of  increasing  facial  hair  and  causing  (i  guess  
acne?)?  
 Answer:  no  choices.  
   
Co-­‐receptor  of  HIV  ?  
Answer:  CXCR4  
   
To  collect  blood  sample  we  need  how  much  blood  for  one  sample?  
a-­‐          3-­‐5  ml  
b-­‐          8-­‐  10  ml  
Answer:??  Differ  according  to  the  test  and  the  age  of  the  patient.  
   
What’s  minimum  volume  of  blood  to  do  a  culture:  
10_20  ml  
Answer:  ??  
5  ml  for  adult,  1  ml  for  for  pediatric.  
References:  http://d2xk4h2me8pjt2.cloudfront.net/webjc/attachments/71/6e4b1cf-­‐blood-­‐culture-­‐
collection-­‐and-­‐volume-­‐huddle-­‐tool.pdf.  
https://www.slideshare.net/doctorrao/role-­‐of-­‐specimen-­‐collection-­‐in-­‐microbiology  
   
vit  c  difeciency  which  affected  in  stages  of  wound  healing?  
Answer:  Collagen  synthesis  
 
case  scenario  regarding  patient  known  case  of  epilepsy,  he  take  anti-­‐epileptic  drug  
for  long  time  and  now  he  complain  of  increased  hair  in  his  body  and  gum  bleeding,  what  
do  you  think  of  the  following  medication  he  is  on?  
Answer:  phenytoin  
   
CD40L  defect?  
Answer:  hyper  IgM  syndrome  
Reference:    https://en.wikipedia.org/wiki/Hyper_IgM_syndrome  
   
Blouting  and  gases  after  eating  ice  cream  and  dairy  product.  Enzyme  deficint?  
a-­‐          Lastase  
b-­‐          Pancreatic  alpha  amylase  
c-­‐          Sacrase  
d-­‐          Isomaltose  
Answer:  lactase  
   
Dideoxynucletide,  used  in  what  ?  
a-­‐  PCR  
b-­‐  Western  blot  
c-­‐  DNA  sequencing  
d-­‐Southern  blot  
 Answer:  c  
   
Which  one  of  the  following  follows  a  mendellian  inheritance?  
Answer:  Thalassemia  
   
What  will  increase  in  ancylostoma  duodenale  ?  
Answer:  eosinophils  
Reference:    https://emedicine.medscape.com/article/218805-­‐workup  
   
scenario  of  mother  how  had  a  previous  pregnancy  with  down  syndrome  and  I  think  it  asked  about  
why  or  how  it  happen?  
Answer:  nondisjunction  
   
loss  of  sensation  over  shoulder  which  part  of  pleura  will  be  affected?  
a.                 Mediastinal  
b.                 Pleural  
c.                 Vesceral  
Answer:  a  
   
Why  aspirin  causes  hypererythema  ?  
Answer:  no  choices  
   
example  of  hinge  joint?  
Answer:  elbow  
   
example  of  Condyloid  joint?  
Answer:  wrist  
   
type  of  joint  between  vertbra?  
Answer:  Cartilaginous  joint  
   
which  layer  of  scalp  contain  bv  and  nerves?  
Answer:  no  choices.    
2nd  layer,  connective  tissue=  superficial  fascia.  
   
Case  scenario  with  Lactic  acidosis  and  Hyperalinen.  Multivitamin  B  was  given  with  no  
response:  
What  to  give:  
a.  Thiamin  
b.  Pyruvate  kinase  
c.  Pyruvate  dehydrogenase  
d.  Pyruvate  carboxylase  
Answer:  d  
   
Which  one  of  the  following  is  associated  with  malignancy?  
a.  Streptococcus  bovis  
b.  Staph  aureus  
c.  Pseudomonas  
Answer:  A  
References:  https://www.hindawi.com/journals/ijmicro/2011/792019/  
   
Q.  Male  got  bee  sting  and  his  wife  is  searching  for  epinephrine  pin.  What  is  the  mechanism  of  
action  of  this  drug  (No  mast  cell  or  histamin  in  the  options):  
Answer:  no  choices.  Cross  reactivity  with  the  cardiac?  
   
Q.  Scenario  of  a  patient  whose  thirsty  with  low  urine  output  because  of  ADH  secretion.  
(They  attached  a  picture  of  a  nephrons  with  number  labels  on  each  segment  and  asked  to  
choose  which  area  does  ADH  work  on).  
Answer  is  collecting  duct  (which  was  labeled  by  number  5  in  the  picture)  

   
   
   
What  is  the  first  immune  responsible  agent  in  the  skin?  
Answer:  Keratinocyte  
   
loss  of  adduction  of  the  fingers  which  nerve  injury:  
a-­‐    ulner  
b-­‐    Radial.  
c-­‐    Median  
d-­‐  Axillary.  
Answer:  a  
   
contracting  muscle  compared  to  relaxing  muscle:  
a-­‐    more  pyruvate  converted  to  lactate.  
b-­‐  
Answer:  a  
   
which  enzyme  deficiency  will  result  in  converting  pyruvate  to  lactate:  
A)  Pyruvate  kinase.  
B)  pyruvate  carboxylase.  
C)  Pyruvate  dehydrogenase  
Answer:  b  
   
the  maximum  accepted  level  of  LDL  in  mmol/l?  
Answer:    4.12  

   
patient  can  not  extend  his  leg  at  knee  joint  which  muscle  responsible  ?  
a-­‐          sarturius  mucsle  
b-­‐          quadricbles  
c-­‐          biscepts  femoris  
Answe:  b  
   
posterior  triangle  trauma,  pt  complain  of  hearing  loss  which  nerve  is  affected:  
a-­‐          greater  occipital  
b-­‐          lesser  occipital  
c-­‐          great  auricular  
Answer:  ??  
   
OCP  work  by  increase:  
a-­‐estrogen  ,  
b-­‐progesterone,  
c-­‐FSH  
d-­‐LH  
Answer:  most  likely  b.  
   
Inhibition  of  which  of  the  following  is  the  primary  action  of  oral  contraceptives?  
A-­‐Spermatozoa  and  thickening  cervial  mucous  
B-­‐Implantation  
C-­‐  Gondatropic  hormone  release  and  ovulation  
Answer:  C  
   
pt  on  opioid,  contraindication  with  these  drug  is?  
a-­‐NSAID  
b-­‐aspirin  
c-­‐laxative  
d-­‐coxen...  
Answer:  ?  
   
Patient  with  dm,  which  one  of  the  following  is  contraindicated?  
a-­‐  losartan  
b-­‐  Nifedipine  
c-­‐  Hydrazine  
Answer:  c?  
Reference:  https://en.wikipedia.org/wiki/Hydrazine_sulfate  
   
How  heparin  in  high  dose  cause  hyperthermia  
a-­‐  inc  metabolism  
b-­‐  uncupling  oxidation  phosphrylation    
c-­‐  Increase  liver  metabolism  
d-­‐  Increase  energy  
Answer:  b?  
   
contraindicated  hypoglycemic  drug  in  diabetic  patient  with  heart  disease?  
a-­‐  pioglitazone  
b-­‐  sitagliptin  
Answer:  a?  
Reference:  https://www.uptodate.com/contents/drugs-­‐that-­‐should-­‐be-­‐avoided-­‐or-­‐used-­‐with-­‐
caution-­‐in-­‐patients-­‐with-­‐heart-­‐failure  
https://www.medscape.com/viewarticle/579822_2?pa=9SK0rtUK%2BCkGjM0fP6yZGm5ZplE3h%2Bx
vCZ7C5bPWTGIqqtp5ScqVT7b0XmUJeZrIs7CF3wx2Tu1U792SxywYLg%3D%3D  
https://www.ptcommunity.com/news/20170422/sitagliptin-­‐januvia-­‐linked-­‐increase-­‐heart-­‐failure-­‐
hospitalizations  
   
case  scenario  about  patient  after  dental  extraction  c/o  of  loss  of  sensation  below  the  
eye  and  upper  lip  ,  I  think  also  cheek  which  nerve  was  injured  ?  
Answer:  infraorbital  
   
which  medication  decrease  the  effect  of  OCPs?  
Answer:  anti  epileptic  
   
case  scenario  about  patient  with  basal  skull  fracture  what  expected  nerve  injury?  
a-­‐  Olfactory  
b-­‐  optic  
C-­‐  oculomotor  
Answer:  a  
   
type  of  hip  joint?  
Answer:  ball  and  socket  joint  
   
case  scenario  regarding  breast  mass  and  asking  about  which  lymph  node  must  be  assist  
first?  
a-­‐  anterior  axillary  lymph  node  
b-­‐  posterior  
c-­‐  medial  
d-­‐  lateral  
Answer:  a  
   
child  present  with  malaise,  history  of  meningitis  last  week  treated  with  iv  ab.  
Lab  :  hb:  low  
Rbc:  low  
What  antibiotic  he  used  ?  
a-­‐  chloramphinicol  
b-­‐  Erythromycin  
Answer:  a  
   
Hepatotoxic  alzehaimer  drug?  
Answer:  Tacrine  
   
Differentiate  between  types  of  gram  positive?  with  catalase  
 Answer:?  
   
Forceps  delivery,  Truma  behind  ear,  What  happen  to  him?  
a-­‐loss  sensation  
b-­‐hearing  loss  
Answer:  ?  facial  nerve  
   
Role  of  metformin  in  PCOS?  
Answer:  reduces  insulin  resistance.  
   
pt  complain  recurrent  renal  stone  on  htn  medication  to  reduced  calcium  excretion  and  
cause  gout?  
a-­‐Hydrochlorithiazide  
b-­‐fursemide  
Answer:  a  
   
main  lateral  rotator  muscle  of  hip?  
Answer:  Gluteus  maximus  
   
Patient  k/c  of  HF  &  schizophrenia  on  medication  ,came  with  brownish  discoloration  in  
retina  &  brown  vision..  what's  the  drug  ?  
Answer:  Thioridazine  
*  Chlorpromazine—(C)orneal  deposits;  
*  Thioridazine—re(T)inal  deposits  
   
   
Qs  about  alternative  cell  reservoir  for  HIV?  
a-­‐CD8  
b-­‐Monocyte-­‐Macrophage  
Answer:  b  
Reference:    https://www.ncbi.nlm.nih.gov/pmc/articles/PMC4947377/  
   
Latency  period  in  HIV,  cell  responsible?  
Answer:  CD4  Cells  
Reference:    http://www.canfar.com/news/stages-­‐hiv-­‐infection  
   
rRNA  in  which  cell  ?  
a-­‐Basophils  
b-­‐Monocyte  
 Answer:  a?  
   
injury  in  the  sole  ,1st  structure  injured?  
Answer:  Abdactor  hallusis  longus  
   
side  effect  of  amitryptaline?  
a-­‐Dystonia  
b-­‐Diarrhea  
Answer:    a  
   
typical  scenario  of  mysthenia  Gravis  (fatigabality  and  weakness  worsen  at  the  end  of  
the  day,What  is  the  pathophysiology?  
a-­‐  viral  
b-­‐bacteria  
c-­‐drug  induced  
d-­‐Autoimmune  
Answer:  d.  
   
another  scenario  about  Mysthenia  graves,Part  affected?  
Answer:  Acetylcholine  receptors  
   
a  physician  wants  to  decrease  the  recurrence  of  UTI,  
Best  measure  is  ?  
Answer:  (decrease  PH  ,  increase  urea  and  increase  urine  osmolarity)  
   
relation  of  saphinous  vien  to  mallelus?  
Answer:  anteromedial  
   
Function  of  anterior  compartment  of  forearm  muscles  ?  
a-­‐  flexion  of  elbow  
b-­‐  extension  of  elbow  
c-­‐  flexion  of  hand  and  wrest  
d-­‐  extension  of  hand  and  wrest  
Answer:  c  
   
A  case  of  GERD  used  antacid  which  caused  constipation  
a-­‐Calcium  carbonate  
b-­‐Aluminum  hydroxide  
c-­‐Magnesium  hydroxide  
Answer:  b  
   
Patient  presented  to  er  with  agitation,  poor  hygiene  and  suspicions  er  dr  referred  him  
to  psychiatry  what's  the  dx  
a-­‐Acute  confusional  state  
b-­‐Schizophrenia  
c-­‐Psychosis  
Answer:  b?  
   
Patient  with  increase  ICP  what  nerve  would  u  examine  before  the  CT  scan?  
a-­‐II  
b-­‐III  
c-­‐IV  
d-­‐X  
Answer:  a  
   
Patient  on  TB  medication  developed  numbness,  what  would  u  give  him?  
Answer:  Vitamin  B6  (pyridoxine)  
   
adult  male  had  an  accident  to  the  lateral  side  of  his  fibula,  next  day  the  patient  is  
complaining  of  drop  of  his  foot  and  no  inversion,  diagnosis?  
a-­‐  deep  peronial  injury  
b-­‐  common  peronial  injury  
Answer:  if  no  eversion  the  answer  is  b.  
   
Which  one  of  the  following  drugs  causes  Vertigo?  
a-­‐Streptomycin  
b-­‐INH  
c-­‐Ethambutol  
Answer:  a  
   
Gout,  which  of  the  following  will  be  inhibited  by  the  drug  you  will  give?  
a-­‐Xanthine  oxidase  
b-­‐PRPP  
Answer:  a  
   
Which  of  the  cells  is  responsible  for  tissue  destruction  in  IBD?  
a.B  lymphocytes  
b.T  lymphocytes  
c.  NK  cells  
Answer:  b  
   
Virulence  factor  of  streptococcus  ?  
a-­‐M-­‐  protein  
b-­‐Capsule  
c-­‐Endotoxin  or  exotoxin  
d-­‐phosphatase  
Answer:  a.  
Reference:    https://en.wikipedia.org/wiki/Virulence_factor  
   
What  is  the  mechanism  of  diarrhea?  
a-­‐Decrease  fluid  intake  
b-­‐Decrease  fluid  absorption  
c-­‐Increase  fluid  intake  
d-­‐Increase  fluid  absorption  
Answer:  b  
   
Embryo  origin  of  right  atrium  ?  (no  options)  
Answer:  Rt  sinus  veinous  
   
Strongest  part  of  vein  (no  options)  
Answer:  Tunica  adventitia  
   
Relation  of  femiral  vein  to  artery  (no  options)  
Answer:  Medial  
   
-­‐Nerve  injury  with  humerus  fracture  (no  options)  
Answer:  Radial  N  
   
Ligament  pass  through  inguinal  canal  
a-­‐Round  
b-­‐Broad  
Answer:  a  
   
When  you  examine  superior  inguinal  LN  You  should  examine  ?  
Answer:  anal  canal  
   
Enlarge  medial  group  of  horzintal  -­‐inguinal  lymphnode  (  superficial).  What  you  will  
check  1st?  
Answer:  no  choices.  
   
garlic  odor  organism?  
a)  Anal  canal  
b)  Muscle  of  thigh  
c)  Muscle  of  leg  
Answer:  ??  
   
a  man  after  MVA  ,  can  not  raise  his  arm  above  horizontal  plane  and  has  winging  scapula  
.  Localise  the  injury  in  the  brachial  plexus?  
a-­‐  roots  
b-­‐  Posterior  cord  
c-­‐  Lateral  plexus  
d-­‐  Medial  
Answer:  a  
   
What  is  the  enzyme  used  glycolysis?  
a-­‐  pyrovate  kinase  
b-­‐  Pyrovate  carboxylate  
c-­‐  Glucose  phosphatase  
Answer:    a  
   
Nerve  responsible  for  posterior  compartment  of  the  leg?  
a.  Sciatic  nerve  
b.  Tibial  nerve  
c.  Fibular  nerve  
Answer:  b  
   
Loss  of  forehead  muscle  action  what  is  the  nerve?  
Answer:  Fascial  nerve  
   
Loss  of  gag  reflex  but  normal  uvula:  
a.  glossopharyngeal  
b.  Vegas  
Answer:  a  
   
Pt  had  occipitofrontalis  paralysis  which  branch  of  facial  nerve  is  affected?  
a.  Temporal  
b.  Buccal  
 Answer:  a  
   
Known  asthma  has  whitish  rash  easly  removed  in  mouth  .  which  antiasmatic  cuse  this?  
a)  Cromolyn  sodium  
b)  Betamethasone  
c)  Albutmerol  
Answer:  b  
   
carpal  tunnel  syndrome  ,  can’t  move  his  fingers  .  what  is  affected  muscle?  
a-­‐                      palmar  interosssi  
b-­‐                      thenar  muscle  
Answer:  b  
   
Closed  humerou  fracture  with  hand  drop  .  what  is  the  type  of  nerve  injury?  
a-­‐  neuromatosis  
b-­‐  neuropraxia  
c-­‐  axonotmesis  
d-­‐  avulsion  of  radial  nerve  
Answer:  b  
   
Eldery  has  bed  sore  that  invade  through  the  muscles  ,  what  is  the  stage?  
A)1  
B)2  
C)3  
D)4  
Answer:  d  
   
Female  taking  paracetamol  500mg  bid  daily  presented  with  liver  enzymes  elevated  and  
hepatomegaly  *  
a-­‐  alpha  1  antitrypsin  deficiency  
b-­‐  liver  damage  due  to  paracetamol  
c-­‐  hepatitis  
d-­‐  Glutathione  depletion  
Answer:  d  
   
nerve  supply  of  calf  muscle?  
a-­‐*  tibial  nerve  
b-­‐*  Femoral  
Answer:  a  
   
Which  nerve  supplies  biggest  part  of  the  tongue  and  covers  it  the  most  ?  
A  -­‐  trigeminal  
B  -­‐  vagus  
C  -­‐glossopharyngeal  
Answer  :  A  
   
which  of  these  components  is  acidophilic  
a-­‐corticotrophic  
b-­‐trophoyrophic  
c-­‐lactotrophic  
d-­‐gonadotrophic  
Answer:  c?  
   
Acidophil:  one  of  the  hormone  -­‐  producing  acidophilic  cells  of  the  adenohypophysis;  type  include  
corticotrophic,  lactotrophic,  lipotrophs,  and  somatotrophs.  also  called  alpha  cell  and  
A  cell.  
Reference  :  Dorland's  medical  dictionary  
   
Patient  have  MVA  come  to  ER  with  ulceraiton  tissue  with  gas  gangrene  in  the  anterior  
leg  what  is  the  MO?  
a-­‐C.  Perfiringes  
c-­‐          staph  aures  
Answer:  a  
   
Superior  oblique  muscle  movement?  
a-­‐Medially  down  
b-­‐Medially  up  
Answer:  a  
   
child  pt.  came  with  scenario  of  chest  infection  ,  first  day  of  admission  he  treated  
with  cefotaxime,  next  day,  pt  state  became  bad  with  decrease  perfusion  and  x-­‐ray  show  
complete  rt.  Side  opcifaction  +  hydrothorax    ,  causative  organism  :  
a-­‐Strepto.  Pnem  
b-­‐Staph.  Aureus  
c-­‐Hemophilus  influenza  type  b  
d-­‐Pseudomonas  
Answer:  d  
   
COPD  exacerbation  by  Infection,  patient  has  fever  and  greenish  sputum,  what  is  the  most  
likely  microorganism?  (  no  pseudomonas  in  the  answers)  
a-­‐staph.  Aureus  
b-­‐streptococcus  pneumonie  
c-­‐mycoplasma  pneumonia  
d-­‐homophiles  influenza  
Answer:  d  
   
What  drug  is  likely  to  cause  heat-­‐stroke  as  it  inhibits  sweating?  
Answer:  Hyoscamine  Sulfate  
   
foul  smelling  urine  in  a  7  y/o  child  with  fever  and  lower  abdominal  pain?  
a-­‐E.Coli  
b-­‐Protues  
c-­‐Klebsiella  
Answer:  a?  
   
-­‐  Gram  -­‐ve,  lactose  non-­‐ferminting  oxidase  +ve?  
Answer:  Psudomonas  
   
most  common  electrolyte  disturbance  in  a  patient  with  digoxin?  
Answer:  K  
   
patient  with  superficial  temporal  artery  bleeding,  blood  will  be  collected  in?  
Epicranial  aponeurosis  
Answer:  ?  
   
Drug  which  decreases  HR  and  Pre  and  after  load?  
a-­‐Carvidilol  
b-­‐Nifidepine  
c-­‐Amlodepine  
Answer:  a?  
Reference:  https://en.wikipedia.org/wiki/Carvedilol  
   
-­‐  Long  sinario  about  median  nerve  injury.  What  is  the  abnormality  you  expected  to  see  
in  hand  ?  
A)  ape  hand  
B)  Clwe  hand  
C)  Wrist  drop  
Answer:  a  
   
-­‐  Injury  leads  to  loss  of  sensation  in  the  medial  2  fingers.  What  is  the  nerve  injured  
?  
A)  Ulnar  nerve  
B)  Median  nerve  
C)  Brachial  nerve  
Answer:  a  
   
-­‐What  is  biochemical  deficiency  in  respiratory  distress  syndrome  in  neonate!?(no  
surfactant  in  answers)  
a-­‐  shingomyelin  
b-­‐Phosphate  
c-­‐dodecylbenzenesulfonate  
Answer:  c  
Refrence:  https://en.wikipedia.org/wiki/Sodium_dodecylbenzenesulfonate  
   
History  of  URTI  then  developed  watery  eye?  
 Answer:  Viral  (adeno)  
   
Weakness  in  gluteal  area  ,  the  artery  affected  is  ?  
a-­‐Femoral  
b-­‐Intrenal  iliac  
c-­‐External  
 Answer:  b  
   
Old  pt  take  antiviral  that  taken  by  inhalation?  
Answer:  **Zanamivir  
   
Culture  showed  non  fermenting  gram  negative  bacilli  what  is  organism  (no  options)  
***correct  answer  one  of  the  following  
1-­‐                      Moraxilla  
2-­‐                      Legionella  
3-­‐                      Pseudomonas  
4-­‐                      Bordetella  
   
which  of  these  joint  is  not  hinge  joint?  
a)  Knee  joint  
b)  Ankle  joint  
c)  Elbow  joint  
d)  Hip  joint  
Answer:  d  
   
-­‐  Patient  in  ICU  had  onset  of  ventilator  associated  pneumonia  culture  showed  gram  
negative,  oxidase  positive  and  non  lactose  fermenting  organism  
A-­‐  Pseudomonas  
B-­‐  Ecoli  
C-­‐  Klebsiella  
Ans:  a  
   
Patient  with  recurrent  uti  presented  with  renal  stones  whats  the  organism  
A-­‐  Proteus  
B-­‐  Ecoli  
C-­‐  Klebsiella  
Answer:  A  
   
Long  scenario  about  patient  come  to  ER  with  chest  pain  radiat  to  left  arm  then  get  
coma  and  death  what  you  will  find  in  the  brain  ?  
a-­‐  necrosis  because  of  left  middle  artery  
b-­‐red  nearon  degeneration  in  hippocampus  
Answer:  a??  
   
During  bypass  heart  surgery  the  assistance  asking  about  origin  of  right  coronary  
DOMINANT  artery  ?  
a-­‐  posterior  interseptal  
b-­‐anterior  septal  
c-­‐  marginal  
d-­‐  cercumflex  
Answer:  a  
Refrence:  https://en.wikipedia.org/wiki/Right_coronary_artery  
   
old  patient  known  to  have  acquired  immunodeficiency  presented  with  cough  and  night  
sweat  ,  he  did  mentoux  test  which  was  negative  but  culture  was  positive  for  
tuberculosis  what  is  most  likely:  
a-­‐  culture  is  false  positive  
b-­‐  mentoux  is  false  negative  
c-­‐  mentoux  is  not  a  screening  test  for  TB  
d-­‐  the  patient  should  be  screened  with  heaf  test.  
Answer:  b  
   
What  is  the  pathology  in  Huntington  syndrome  ?  
1-­‐something  related  to  stratum  whith  nerotransmitter  deffect  
Answer:  ?  
Early  damage  is  most  evident  in  the  striatum,  
Reference:  https://en.wikipedia.org/wiki/Huntington%27s_disease  
   
Case  of  side  effect  of  ACEI?  
Answer:  cough  
   
You  did  DRE  and  found  prostate  gland  swelling  ,  which  lobe  of  prostate  has  the  
neoplasm!?  
a-­‐  anterior  
b-­‐  Posterior  
c-­‐  Medial  
Answer:  b  
   
food  poisoning  case,  4  family  members  ate  from  a  restaurant,  they  developed  diarrhea  
and  vomiting  and  remit  after  24  hrs  Culture  showed  gram  positive  bacilli?  
a-­‐salmonella  
b-­‐shigella  
c-­‐  Staph  Aureus  
d-­‐Bacillus  ceres  
Answer:  d  
   
where  surgeon  well  do  vagus  nerve  crush  ?  
Answer:  
   
heart  duct  emperionic  origination?  (Specific  name  not  Mesoderm)  
Answer:  
   
picture  of  ring  cell  stage  of  malaria.  Asked  the  stage  ang  malaria  type?  
Answer:  
   
picture  of  histopath  of  tumor  with  stary  sky  pattern?  
Answer:  EBV  
   
case  of  wegeners  but  asked  about  the  basic  pathology  of  disease?  
Answer:  
   
A  patient  ate  a  wild  mushroom.  Which  of  the  following  will  be  inhibited?  
a-­‐RNA  polymerase  I  
b-­‐RNA  polymerase  II  
c-­‐RNA  polymerase  III  
d-­‐DNA  Gyrase  
Answer:  b  
   
bipolar  disorder  develop  symptoms  of  hypothyroidism.  Which  medicine  responsible?  
 Answer:  no  choices.  lithium  
   
what  organism  can  cause  paralysis  ??  
Answer:  botolinium  
   
scenario  about  female  sexually  active  and  came  with  symptom  i  forgot  ,  in  lab  results  he  
mentioned  gram  negative  diplococci  and  asked  about  the  diagnosis  ??  
Answer:  N.  Gonorrhea  
   
a  drug  which  stops  sweating?  (no  option  as  hydrosamine)  
a-­‐cimtidine  
b-­‐odensteron  
c-­‐and  two  more  which  were  ending  with  chloride  
Answer:  ?  Aluminum  chloride.  
   
man  taking  diuretics  have  muscle  weakness  diarrhea  irritability?  
a-­‐hypokalemia  
b-­‐hyperkalemia  
c-­‐hyponatermia  
d-­‐hypocalcemia  
Answer:  b  
   
Mother  with  Rh  -­‐ve  and  a  father  with  Rh  +ve,  what  the  propility  of  having  Rh  +ve  child?  
a-­‐25%  
b-­‐50%  
c-­‐75%  
d-­‐100%  
Answer:  b  
   
Phyenytoin  side  effect?  
Answer:  Gum  hyperplasia  
   
Long  thoracic  nerve  root  number?  
Answer:  no  choces.  C5,C6,C7  
   
During  fights,  what  system  is  activated?  
Answer:  Sympathetic  
   
A  female  with  immunity  disorder,  where  is  the  defect?  
a-­‐IL-­‐2  
b-­‐IL-­‐3  
c-­‐IL-­‐6  
Answer:  ?  
   
Nitrofuratonin  side  effect?  
Answer:  no  choices  
vaginal  itching  or  discharge  
Reference:  https://www.drugs.com/nitrofurantoin.html  
   
Overdosed  drug  caused  resp  depression?  
Answer:  
   
Overdosed  lead  to  comatose?  
Answer:  no  choices.  Opioids?  
   
Overdosed  lead  to  dilated  pupils?  
Answer:  
   
A  truma  case  with  facial  fractures,  his  eyes  (  something)  what  type  of  reaction?  
Answer:  
   
Muscles  responsible  for  unlocking  the  knee?  
Answer:  Poplitus  
   
Mechanism  of  warfarin  ?  
Answer:  no  choices.  
   
Most  common  cutaneous  manifestation  of  antimalarial  medications?  
a-­‐Pruritus  
b-­‐Pigmenation  
c-­‐Photosensitivity  
d-­‐Generalized  yellow  discoloration  of  skin  
Answer:  b  
   
What  is  a  relative  contraindication  of  Methylergometrine?  
a-­‐Asthma  
b-­‐DM  
c-­‐HTN  
 Answer:  c  
Reference:  https://en.wikipedia.org/wiki/Methylergometrine  
   
Difference  between  HbA1C  and  non-­‐modified  hemoglobin:  
Answer:  glycosalation  of  valine  
Reference:  https://books.google.com.sa/books?id=AQ0vvBFg790C&pg=PA243&lpg=PA243&dq=non-­‐
modified+hemoglobin+to+HbA1C&source=bl&ots=C04hA8qNFv&sig=zHSFT97YYOrLB9IuigHseYjxRqU
&hl=ar&sa=X&ved=0ahUKEwip6uSF1arXAhWJMBoKHeEBDmoQ6AEIUzAJ#v=onepage&q=non-­‐
modified%20hemoglobin%20to%20HbA1C&f=false  
   
hook  worm,  how  to  cause  anemia?  
a-­‐destructin  rbc  
b-­‐send  toxins  to  bone  marrow  
c-­‐compete  with  host  on  b  12  
Answer:    a  
Reference:  https://en.wikipedia.org/wiki/Hookworm_infection  
   
patient,  with  diarrhea  ,  flaglated  protozoa,  how  it  cause  diarrhea  ?  
a-­‐increase  screation  of  fluids  
b-­‐kills  normal  flora  
c-­‐coats  the  small  bowl  
d-­‐absorption  is  prevented  
Answer:  a  
Reference:  https://emedicine.medscape.com/article/176718-­‐overview#a3  
   
Lymph  node  of  the  forehead  ?  
Answer:  no  choices.  preauricular  
   
Cat  bite,  which  organism:  
Answer:  Pasteurella  multocida  
   
Chronic  smoker,  presented  with  signs  of  lung  cancer,  what  is  the  origin  of  cancer?  
a-­‐    Clara  
b-­‐    Brush  cells  
c-­‐    Goblet  
 Answer:  a  
Reference:  https://www.ncbi.nlm.nih.gov/pmc/articles/PMC4860431/  
   
Patient  travelled  to  Africa  where  TB  bovine  is  endemic  there,  what  prevent  him  from  
receiving  BCG  vaccine,  deficiency  in  what:  
a-­‐  IFN  gamma  
b-­‐  IFN  alpha  
c-­‐  IL  4  
d-­‐  IFN  beta  
Answer:  a  
   
Most  common  carpal  bone  involved?  
Answer:    Scaphoid  
   
Female  presented  with  hand  joints  swelling,  she  were  informed  by  physician  that  she  has  
bone  loss,  she  is  angry  about  her  permenant  bone  loss  
What  is  the  explanation  of  bone  loss:  
a-­‐  Increase  pressure  in  joint  space  
b-­‐  Material  secreted  from  synovial  fluid  
c-­‐  Drug  induced  menopause  
Answer:  
   
Which  drug  will  decrase  basline  and  contact  induced  acid  secretion  from  stomach:  
a-­‐  Ranitidine  
b-­‐  PPI  (not  sure)  
-­‐  Other  choices  that  I  don’t  remember.  
Answer:  b?  
   
 Known  case  of  sarcoidosis,  presented  with  signs  of  anemia.  From  investigation  he  has  
iron  deficiency  anemia.  What  is  the  pathophysiology:  
a-­‐  Decreased  hepcidin  
b-­‐  Increased  hepcidin  
Answer:  b  
Increase  hepcidin  associated  with  IDA.  
   
wernicke's  area  injured,  which  type  of  aphasia?  
Answer:  no  choices  
fluent  aphasia  
   
Female  150k.g.  Weight  and  height  160  according  to  BMI  ?  
a-­‐I  Obesity  
b-­‐II  obesity  
c-­‐III  obesity  
 Answer:  c  
   
Patient  complain  of  abdominal  pain  I  think  with  diarrhea,  also  his  wife  noticed  he  had  
SOB  and  tightness.  Doctor  order  5-­‐hydroxyindoleacetic  acid  in  urine  Which  cell  
responsible?  
a.  chromaffin  cell  
b.  Enterocell  
c.  Lympho  cell  
d.  Goblet  cell  
Answer:  a  [carcinoid  syndrome  ]  
   
Degree  for  screening  of  abdominal  aortic  aneurysm  for  pt  age  70  y/o  who  never  smoked  ?  
a-­‐A  
b-­‐B  
c-­‐C  
d-­‐D  
Answer:  c  

   
 
What  is  biochemical  defect  in  x  linked  agammagloubinemia  !?  
 Answer:  no  choices  
the  white  blood  cell  formation  processdoes  not  generate  mature  B  cells,[2]  which  manifests  as  a  
complete  or  near-­‐complete  lack  of  proteins  called  gamma  globulins,  including  antibodies,  in  their  
bloodstream.  
   
Child  with  repeated  polymicrobial  chest  infection  ,  skin  test  positive  for  candida  
antigen  Blood  test  all  normal  except  high  IgG  or  IgM  !?  Low  lymphocyte.  Lymph  node  
biopsy  showed  :  rudimentary  germinal  centres  .  What  is  the  pathophsyioloy  of  this  
disease!?  
Answer  :  the  diagnosis  is  Bruton  Agammaglobulinemia  
Answer:  no  choices.  
   
Goblet  cells  are  mostly  found  in                                                                                                
1-­‐  stomach  
2-­‐  duodenum  
3-­‐  ascending  colon  
4-­‐  rectum                      
Answer:  ascending  colon  
The  colon  contains  a  larger  percentage  of  goblet  cells  than  the  small  intestine  
 Reference:  https://www.ncbi.nlm.nih.gov/pmc/articles/PMC2933006/  
   
about  RAST  test  measure??    
Answer:  no  choices                
A  radioallergosorbent  test  (RAST)  is  a  blood  test  using  radioimmunoassay  test  to  detect  
specific  IgE  antibodies,  to  determine  the  substances  a  subject  is  allergic  to.  This  is  
different  from  a  skin  allergy  test,  which  determines  allergy  by  the  reaction  of  a  
person's  skin  to  different  substances.  
   
Beta  Thalassimia?                                                                                                
a)Point  mutation  
b)  Deletion  
c)Insertion  
d)Fragment  shaft  
answer:  a  
   
Most  common  known  side  effect  of  beclometasone  in  children?                            
a-­‐Excitable  behaviour!  
b-­‐labail  mood  
c-­‐growth  retardation  
d-­‐intraoccular  Hypertension  
Answer:  c?  
Reference:  https://www.drugs.com/sfx/beclomethasone-­‐side-­‐effects.html  
   
1st  responsible  for  Bp  regulation?                                                                                        
a-­‐  heart  
b-­‐  aorta  
c-­‐  arterioles                                              
d-­‐  capillaries  
 Answer:  b  
   
case  of  bell's  palsy,  which  nerve  will  be  intact                                    
a-­‐  massater  
b-­‐bucceneter  
c-­‐  orbicuaris  oris  
d-­‐  orbicuaris  
Answer:  a  
   
Side  effect  of  levodopa  ?                                                                                                                                        
Htn  “  cuz  hypo  not  hyper”                                                                                                                                    
n/v                                                                                    
answer:  nausea  and  vomiting  
   
what's  the  effect  of  niacin?  
a-­‐dec  LDL  
b-­‐Dec  TGD  
c-­‐  inc  HDL  
Answer:  c  
   
Metformin  action  is  enhanced  in  by  which  enzyme?                                                                                                                  
Answer:  activation  of  AMP-­‐activated  protein  kinase  
   
Pronator  teres  syndrome  ?                                                                                    
Answer:  Medial  nerve  compression  at  elbow  
   
about  final  destination  of  proteins  in  cell  cycle?                              
a-­‐Anaphase  
b-­‐Prophase  
c-­‐Cytokinesis                                        
Answer:  GOLOGI  APPARTUS  OR  TELOPHASE!  
                                                                                                                                           
What  structure  holding  Uterus?                                                              
Answer:  uterosacral  ligaments  
   
pt  has  urine  culture  MRSA  he  is  on  antibiotic  after  fewa  day  develop  redness  on  face  
,neck  what  is  antibiotic?  
a-­‐pencillin  
b-­‐vancomycin  
 Answer:  b  
   
Which  one  of  these  vaccines  taken  by  intranasal  route?                              
a-­‐Zanamavir                                                                                
b-­‐Oseltamivir  
Answer:  a  
   
Nasal  &  and  palate  dryness  due  to  obstruction  of  ?                                                    
a-­‐  submandibular  gland  
b-­‐  Submental  
c-­‐  Ptergoplatine  
Answer:  a  
   
Which  of  chemodrugs  will  cause  HTN  +  skin  problem  +  pulmonary  fibrosis  ?  
a-­‐  bleomycin  
b-­‐  Methotrexate  
Answer:  a  
   
Patient  on  anti  epileptic  and  Heart  failure  medication  presented  with  signs  of  drug  
overdose  and  ECG  showing  dysarethmia  ?  
a-­‐  digoxin  
b-­‐  SSRI                                                                                                              
c-­‐  Quinine  
Answer:  a  
   
Child  with  autoantibodies..  chances  of  getting  dm1?                    
a-­‐40  %  
b-­‐60  %  
c-­‐80  %  
d-­‐100  %  
Answer:?  
   
Nerve  involved  in  tarsal  tunnel  syndrome?  
Answer:  TIBIAL  NERVE  
   
Oby  surgery  clamb  the  artery  close  to  lateral  vaginal  wall  what  structure  maybe  
injured?  
a-­‐Peudendal  nerve  
b-­‐ureter  
Answer:  b  
   
Long  scenario  about  hydrocele  typical?                                                                                                    
Answer:  failure  of  obliteration  of  process  vaginalis  
   
pt  eye  drop  and  eye  goes  to  medial  side  what  nerve  injury  when  try  to  close  the  eye?  
a-­‐3  
b-­‐7  
c-­‐4  
Answer:  a?  
   
HSP  disease  what  is  the  immunoglobulin  responsible?              
a-­‐IgG  
b-­‐IgM  
c-­‐IgA  
d-­‐IgE  
Answer:  
   
which  in  inguinal  canal  is  develop  from  the  external  oblique  muscle?                                                                                              
a-­‐External  spermatic  fascia                                                                                                              
b-­‐Internal  spermatic  fascia  
Answer:  a  
Reference:  https://en.wikipedia.org/wiki/External_spermatic_fascia  
   
A  pt  with  hypoK  ,  HypoCL  and  hypoNa  and  HyperHCO3  i  think  with  some  symptoms  ,  what's  the  
1ry  defect  ?  
A.  NaCl  
B.  H  excertion                                                                                                            
C.  H  absorption  
D.  k  
Answer:  a  
bartter  syndrome,  NaCl  reabsorption  defect.  
   
type  of  innate  mechanism  for  extracellular  bacteria?  
Answer:  no  choices  
                                                                                                                                                                               
primary  immunodeficiency  that  expose  the  pt  to  recurrent  of  viral  and  molds  infection?  
a-­‐  T  cell  
b-­‐  B  cell                                                                
c-­‐  complement  defi  
Answer:  a  
   
20  yrs  male  with  hx  of  hematuria  proteinuria  6  gm,  ......  what  you  will  see  in  renal  
biopsy  (  hx  of  post  strptococcus  glomerulonephritis)?                                                                                                            
a-­‐  membrane                                        
b-­‐  Minimal                                                                                                                                
c-­‐  Post  infection  
Answer:  c?  
   
renal  vein  come  from  which  part  of  inferior  vena  cava?                                
Answer:  Subcardinal  
   
18  years  old  male  hx  of  RTA  presented  with  basal  skull  
Fracture  Reaching  to  the  jugular  foramen  ,  Nerves  were  injured  .  Which  of  the  following  
muscles  will  not  be  affected  ?  
A-­‐sternocleidomastoid  muscle.  
B-­‐hyoglossus  muscle  
C-­‐stylopharyngus  muscle  
D-­‐trapezius  muscle  
Answer:  B  :  Hyoglossus  muscle  
   
if  you  remove  the  pectoralis  major  muscle  ,  what's  gonna  happen?  
a-­‐loss  of  arm  adduction  
b-­‐loss  of  arm  abduction  
c-­‐loss  of  arm  adduction  and  internal  rotation  
d-­‐loss  af  arm  abduction  and  external  rotation  
Answer  :  C  
   
Patient  have  bilateral  thin  walled  parenchyma  of  kidney?                        
Answer:  (PCKD)  
   
Child  with  DM  type  1  what's  the  Mechanism  of  action  of  the  disease  ?  
a-­‐  Triglyceride  uptake  
b-­‐  Liver  increase  of  fatty  acid  
Answer:  
   
A  case  of  xeroderma  pigmentosum  ,  whats  the  mech  of  action  ?  
Answer:  defect  in  DNA  repair.  
   
Which  of  the  following  genes  is  associated  with  atheroscelerosis,  but  linked  with  
polygenetic  Alzheimer's  disease  as  well?  
A)  Melyiod  protien  B.  
B)  Apolipoptotein  E.                                            
C)  Presenilen  1.  
D)  Presenilen  2  
Answer:    b  
   
Fallopican  tube  embryology?  
 Answer:  mesoderm  intermidite  cell  >  latral  part  of  genital  ridge  
   
Patient  with  gullain  barre,  what  cell  is  affected  ?  
Answer:  schwan  cell  
   
U  prescribe  roaccutane  =  retinoids  for  women  ,  what  is  the  serious  side  effect?  
Answer:  birth  defect  
   
Facial  nerve  affection,  normal  lacrimation,  abnormal  tympani  reflex  and  loss  of  taste,  
site  of  injury:  
a-­‐Distal  to  tympani  chordi  
b-­‐Proximal  to  tympani  chordi  
c-­‐Distal  to  gbs  nerve  
d-­‐Proximal  to  gbs  nerve  
Answer:  b  
   
HDL  has  protective  mechanism  for  coronary  artery  disease  by  which  enzyme?  
Answer:  antioxidant  enzyme  paraoxonase  (PON)  1  
   
Patient  had  DM  with  HTN  had  renal  transplant  after  1  month  show  rejection  biopsy  show  
hla  class  1.  Which  cell  responsible?  
a-­‐Macrophage  
b-­‐Cd  4  
c-­‐Cd8  
d-­‐Nk  
Answer:  c  
Reference:  https://en.wikipedia.org/wiki/Human_leukocyte_antigen  
   
8-­‐Most  specific  for  AIDS:
  
A)  night  sweat
  
B)  generalized  LAP
  
C)  opportunistic  infections  
Answer:  b  
   
Man  lost  smell,  which  lobe  is  affected  
A)  parietal  
B)  temporal  
C)  frontal  
D)  occipital  
Answer:  b  
   
 Man  with  stroke  and  vision  loss,  affected  lobe:  
A)  parietal  
B)  temporal  
C)  frontal  
D)  occipital  
Answer:  d  
   
Latency  period  in  HI,  cell  responsible?  
Answer:  CD4+  T  cells  with  a  memory  
   
Brown  dicoloration  of  eye  then  disappear  ..  What  is  the  drug  :  
a-­‐Vanco  
b-­‐Metacloprmide  
c-­‐lanatoprost  
Answer:  c  
   
What  type  of  skin  reaction  in  tb  reaction?  
A-­‐1  
B-­‐2  
C-­‐3  
D-­‐4  
Answer:    d  
Reference:  https://en.wikipedia.org/wiki/Type_IV_hypersensitivity  
   
Tyrosine  kinase  inhibitors  work  against?  
a.Epidermal  growth  factor  
b.Endodermal  growth  factor  
 Answer:  a  
   
Patient  with  cardiomyopathy  and  heart  failure,  biopsy  showed:  central  necrosis  surrounded  by  
inflammatory  cells  and  large  histiocytes  (  Anitschkow  cell)  What  is  the  most  likely  diagnosis?  
a-­‐  rheumatic  fever  
b-­‐  myocardial  sarcoidosis  
Answer:  1  
   
Chemotherapy  medication  that  acts  on  cell  cycle  something  ,skin  rash,  pulmonary  fibrosis?  
a-­‐  methotrexate  
b-­‐  bleomycin  
c-­‐  cisplatin  
Answer:  b  
   
Which  beta  blocker  has  the  most  alpha  adrenergic  effect?  
a-­‐  labetalol  
b-­‐  nadolol  
c-­‐  pindolol  
Answer:  a  
   
Drug  that  is  given  in  patient  with  heart  disease  that  inhibits  the  entrance  of  calcium,  which  lead  to  
decrease  in  the  myocardial  excitable  something?  
a-­‐  diltiazim  
b-­‐  hydralazine  
c-­‐  nitroglycerin  
Answer:  a  
   
Mechanism  of  action  of  heparin?  
a-­‐  reversible  bind  to  antithrombin  3  
b-­‐  inhibit  factor  V  
c-­‐  inhibit  factor  ii,  vii,  9,  and  10  
d-­‐  
Answer:  1  
   
How  many  Barr  bodies  in  XXX  female?  
a)  1  
b)  2  
c)  3  
d)  4  
Answer:  2  (Barr  body  is  the  inactive  X  chromosome  )  
   
Scenario  of  a  patient  with  colorectal  cancer.  Diagnosed  to  have  adenocarcinoma,  he  did  some  
tests  which  showed  high  tyrosine  kinase.  Which  of  the  following  do  u  expect  to  be  high?  
a)  endo  dermal  growth  factor  
b)  epidermal  growth  factor  
c)  coupled  G  protein  receptor  
d)  platelet  growth  factor  
Answer:  
   
Scenario  of  a  patient  who  had  jaundice  after  flu  like  symptoms.  Indirect  bilirubin  is  high,  direct  is  
normal  and  total  bilirubin  is  high.  AST,  ALT  and  GGT  normal.  What  is  the  gene  mutation  
responsible?  
a)  TPEN  
b)  p53  
c)  APC  
d)  ??  (Probably  the  right  answer)  
Answer:  
   
Teratogenic  effect  of  Thalidomide  
 Answer:  meromelia  
   
Inhibitor  of  uptake  of  which  of  the  following  should  be  given  to  the  patient?  (what  pt,  question  is  
missing  info)  
a-­‐  GABA  
b-­‐  DOPA  
c-­‐  Serotonin  
Answer:  ?  
   
Trypanosoma  cruzi  works  on  or  something  like  that  on  which  receptor  (at  cellular  level):  
A.  IL6  
B.  IL10  
Answer:  ?  
   
MOA  of  statin:  
Answer:  HMG  CoA  reductase  inhibitor  
 
 
 
 
 
 
 
 

Genetics  
   
1.     Marffan  syndrome  descriptions  long  arm  and  ask  about  mood  on  inheritance?  
Answer:  AD.  
   
2.     Pt  with  brown  macules  on  skin  with  axillary  fleckring  dx  is  NF,  what  is  the  mode  of  
inheritance?  
 Answer:  Autosomal  Dominant.  
   
3.          Tall  &  thin  boy  with  joint  laxity  also  his  mother  has  same  features  what  is  mode  of  
inheritance?  
Answer:    the  features  describes  marfan  synbdrome  and  the  mode  of  inheritance  is  Autosomal  
dominant.  
   
4.     pt  with  tender  flank  and  mutation  in  chromosome  16  what  mode  of  inheritance?  
Answer:    Autosomal  Dominant  Polycystic  Kidney  disorder  [PKD  1/polycystin  1  
PKD2/polycystin  2]  
   
5.     SCA  mode  of  inheritance?  
Answer:    autosomal  recessive  
References:  https://ghr.nlm.nih.gov/condition/sickle-­‐cell-­‐disease#inheritance  
   
6.     48.type  of  inheritance  of  Fanconi  anaemia?  
Answer:    Autosomal  recessive  genetics;  FA  is  primarily  an  autosomal  genetic  disorder.  This  means  that  
two  mutated  alleles(one  from  each  parent)  are  required  to  cause  the  disease.  The  risk  is  25%  risk  that  
each  subsequent  child  will  have  FA.  About  2%  of  FA  cases  are  X-­‐linked  recessive,  which  means  that  if  the  
mother  carries  one  mutated  Fanconi  anemia  allele  on  one  X  chromosome,  a  50%  chance  exists  that  male  
offspring  will  present  with  Fanconi  anemia.  
References:  https://en.wikipedia.org/wiki/Fanconi_anemia  
   
7.     ehlers-­‐danlos  what  is  the  type  of  transmission??  
-­‐  Autosomal  dominant.  
   
8.     Male  present  with  multiple  maculs  5-­‐15  mm  ,  and  axilary  frickling  ,  his  wife  is  
pregnant  ,  what  is  the  mood  of  inheritance?  
a-­‐  Autosoma  
b-­‐  X-­‐linked  
c-­‐  Mitochondrial  
 Answer:  a  
   
9.     Mode  of  inhertance  FAP?  
Answer:    no  choices.  Autosomal  dominant?  
   
10.    Wiskott-­‐Aldrich  Syndrome  mode  of  inheritance?  
Answer:  X-­‐linked  recessive  
   
11.    type  of  inheritance  of  intrahepatic  cholestatic?                          
a-­‐AR                    
b-­‐  X  linked  
c-­‐  mitochondrial  
Answer:  a  
Reference:  https://ghr.nlm.nih.gov/condition/progressive-­‐familial-­‐intrahepatic-­‐
cholestasis#inheritance                    
   
12.    Burkitt  lymphoma  gene?  
Answer:    Myc  gene  is  normally  found  in  ch.8  is  transferred  to  ch.14.  in  Africa  its  associated  with  EBV.  
References:  https://www.ncbi.nlm.nih.gov/books/NBK22257/  
   
13.    38.A  child  from  Ghana  with  a  neck  mass,  Biopsy  showed  starry  sky  appearance,  KI  67  was  
positive,  Which  genetic  change  is  true?  
Answer  burkitts  lymphoma,  Ch.14  and  Myc  gene  translocation.  
   
14.    Parents  brought  their  son  with  cystic  fibrosis,  asking  about  the  chance  of  his  daughter  
to  be  carrier:  
a-­‐          1:2  
b-­‐          2:4  
c-­‐          1:25  
d-­‐          1:3  
Answer:    b  
   

   
 
15.    where  is  the  gene  of  Neurpfibromatosis?  
Answer:  17q11.2  
   
16.    -­‐  RCC  gene*  (  as  presentation  of  hematuria,  wt  loss  ,  loin  pain  )?  
Answer:  VHL  
 *Gene  for  renal  cell  carcinoma.  
   
17.    -­‐  Breast  ca  gene?  
Answer:  BRACA2  
   
18.    Breast  cancer  gene  (no  BRCA)*?  
a-­‐          her2  
b-­‐          CA125  
c-­‐          P53  
Answer:  a  
   
19.    Case  of  RA  asking  about  genetics?  
Answer:  HLA  B27  
   
20.    Scenario  with  a  patient  who  came  with  fever  and  altered  level  of  consciousness.  Labs  
show  high  urea  and  creatinine,  low  platelets  and  anemia  (Scenario  of  TTP).  What's  the  
gene  mutation  responsible?  
a.  Cardiolipins  
b.  ADAMTS13  
c.  Glycoprotein  
Answer:  b  
   
21.    Pancreatic  cancer  which  gene  associated  with?  
Answer:  no  choices  
BRCA1,  BRCA2,  PALB2,  CDKN2A,  ATM,  STK11,  and  the  genes  linked  to  Lynch  syndrome  (MLH1,  
MSH2,  MSH6,  PMS2,  and  EPCAM).  
   
22.    cardiomyopathy  mutation?  
Answer:  no  choices  
Common  Genes:  
TTN  gene  (dilated),  
MYH7,  MYBPC3,  TNNT2,  TNNI3  (hypertrophied).  
   

 
 
References:  https://ghr.nlm.nih.gov/condition/familial-­‐dilated-­‐cardiomyopathy  
https://ghr.nlm.nih.gov/condition/familial-­‐hypertrophic-­‐cardiomyopathy#genes  
   
23.    mother  had  abortion  two  times  last  child  with  45X  monosomy,  recurrence  in  next  
pregnancy?  
a-­‐          %30  
b-­‐          %50  
c-­‐          %70  
d-­‐          %90  
Answer:  a  
   
24.    Familial  adenomatous  polyposis  gene?  
 Answer:  The  human  APC  gene  is  located  on  the  long  (q)  arm  
of  chromosome  5  in  band  q22.2  (5q22.2)  
   
25.    gene  associated  with  type  1  DM  
a-­‐DR2  
b-­‐DR4  
c-­‐DR6  
d-­‐DR7  
Answer:  b.  
   
26.    Most  common  mutation?  
 Answer:  trisomy  21  
   
27.    Marfan  syndrome  features  of  the  mother  and  her  boy:  what  is  the  probability  of  the  
children  to  have  it?  
a-­‐25%  
b-­‐50%  
c-­‐75%  
d-­‐100%  
Answer:  b  
   
28.    X  linked  is?  
a-­‐AD  
b-­‐Multifactorial  
 Answer:?  
   
29.    HCM  which  gene  has  the  worst  prognosis  (no  options)                                  
Answer:  if  itʼsHCM  hypertrohic  cardiomyopathy  gene  assoc  MYH7,  MYBPC3,  TNNT2,  and  TNNI3  
   
30.    Patient  with  tetanus  and  hypocalcemia  presented  with  pcp,  aspiragillus  pneumonia,  and  
other  infections,  what  is  the  cause?  
Answer:  Deletion  of  chromosome  22  (digoarge)  
   
31.    Patient  with  jaundice  ,  labs  shows  high  indirect  bilirubin  and  total  bilirubin,  was  
about  the  gene  or  the  mutation?  
Answer:  no  choices  
Crigler-­‐Najjar  syndrome  type  II  is  caused  by  homozygous  or  compound  heterozygous  
mutation  in  the  UDP-­‐glucuronosyltransferase  gene  (UGT1A1;  191740)  on  chromosome  2q37.  
Mutations  in  the  same  gene  cause  Gilbert  syndrome  (143500)  and  Crigler-­‐Najjar  
syndrometype  I  (218800).  
1  )  Crigler-­‐Najjar  syndrome  is  inherited  in  an  autosomal  recessive  pattern,  which  means  
both  copies  of  the  UGT1A1  
2  )  Gilbert's  syndrome  (  asymptomatic  usually  )    is  due  to  a  mutation  in  the  UGT1A1  
gene  
   
32.    Gene  mutation  found  in  non-­‐small  cell  lung  cancer?  
-­‐Answer:  EGFR  mutations  &  ALK  alterations  
   
33.    Genotype  of  mild  thalassemia:  
A.  a0  -­‐  -­‐  /  -­‐  -­‐  
B.    a0  -­‐  -­‐  /  -­‐  a  
C.    a0  -­‐  -­‐  /  a  a  
D.  a+  -­‐  a  /  a  a  
Answer:  c?  
A:  one  gene-­‐>  sever,  B:  two  gene  -­‐>  moderate.  
   
34.    male  e  hemophilia  a  and  female  carrier  what  %  their  child  will  have  hemophilia?  
Answer:    50%  
The  chance  for  a  carrier  mother  to  deliver  a  baby  with  haemophilia  is  producing  50%  carriers  and  
this  chance  is  the  same  in  all  pregnancies  even  if  she  had  a  previous  child  with  haemophilia.  
References:  https://www.hog.org/handbook/section/2/how-­‐hemophilia-­‐is-­‐inherited  
   
   
Inheritance  questions:  
-­‐  fanconi  anemia:  AR  
-­‐  Progressive  familial  intrahepatic  cholestasis:  AR  
-­‐  Willson:  AR,  chromosome  14  
-­‐  Cystic  fibrosis:  AR,  chromosome  7,  long  q  arm  
-­‐  Congenital  chloride  diarrhea:  AR  
-­‐  Congenital  liver  fibrosis:  ass.  With  AR  polycyctic  kidney  disease  
-­‐  Von  willebrand  disease:  AR  
-­‐  Alpha  and  beta  thalasemia,  and  SCA:  AR  
-­‐  Otosclerosis:  AD  
-­‐  Polycystic  kidney  adult:  AD,  chromosome  16  
-­‐  Heridtary  spherocytosis:  AD  
-­‐  Marfan  syndrome:  AD  
-­‐  Hereditary  chronic  pancreatitis:  AD  
-­‐  Apoptosis  gene:  p53  
-­‐  Gene  of  ductal  carcinoma:  p53  
-­‐  Enhance  apoptosis:  p53  
-­‐  Kidney  tumor  oncogen:  MET  in  hereditary  papillary  renal  cell  cancer,  and  ps53  
-­‐    Gene  for  copper  transport  ATPase:  
-­‐  IBD:  chromosome  16,  genes:  NOD2,  ATG16L1  
-­‐  DM  type  2:  chromosome  20  
-­‐  Gilbert:  chromosome  2,  q37,  UGT1A1  gene  
-­‐  High  cholesterol:  LDL  receptor  gene  
-­‐  X  linked  a  gamma  globulinemia:  CD19  and  CD  20  
-­‐  Problem  with  phagocytosis,  protein  defect  for  oxygenation:  Decrease  NADH  (chronic  
granulomatous  disease)  
-­‐  Angioedema:  SERPING1  Gene  
-­‐  Alzheimer  disease:  &gt;60  year:  21  chromosome  ,,  &lt;60:  19  chromosome  
-­‐  Xeroderma  pigmentosa:  defect  in  DNA  break  repair  gene  
-­‐  Alzheimer  disease:  &gt;60  year:  21  chromosome  ,,  &lt;60:  19  chromosome  
-­‐  Huntignton  disease:  chromosome  4,  CAG  trinucleotide  repeat  "3  M:  mood,  memory,  
movement"  
-­‐  SCA  and  thalasemia:  point  mutation  
-­‐  CML  and  ALL:  bcr  (on  chromosome  22)  and  abl1  (on  chromosome  9)  resulting  in  the  bcr-­‐  
abl1  fusion  gene  —&gt;  philadelphia  (ph)  chromosome  
-­‐    Starry  sky  biopsy  "burkett's  lymphoma":  C-­‐myc  gene,  chromosome  8  
   
   
   
 
   
   
   
   
   
   
   
   
 
 

You might also like